Vous êtes sur la page 1sur 602

PALAY, INC.

Vs.
CLAVE
G.R.No. L-56076
FACTS OF THE CASE
On March 28, 1965, petitioner Palay, Inc., through its President, Albert Onstott
executed in favor of private respondent, Nazario Dumpit, a Contract to sell a parcel of
Land in Antipolo, Rizal owned by said corporation. The sale price was P23, 300.00
with 9% interest per annum, payable. with a down payment of P4, 660.00 and monthly
installments of P246.42 until fully paid.
Paragraph 6 of the contract provided for automatic extrajudicial rescission upon
default in payment of any monthly installment after the lapse of 90 days from the
expiration of the grace period of one month, without need of notice and with forfeiture
of all installments paid. Respondent Dumpit paid the downpayment and several
installments amounting to P13, 722.50. The last payment was made on December 5,
1967 for installments up to September 1967.
On May 10, 1973, or almost six (6) years later, private respondent wrote
petitioner offering to update all his overdue accounts with interest, and seeking its
written consent to the assignment of his rights to a certain Lourdes Dizon. In response,
petitioners informed respondent that his Contract to Sell had long been rescinded
pursuant to paragraph 6 of the contract, and that the lot had already been resold.
A complaint was filed by the respondent with the NHA for conveyance with an
alternative prayer for refund. The NHA, in its resolution, ordered Palay, Inc. and
Alberto Onstott in his capacity as President of the corporation, jointly and severally, to
refund immediately to respondent the amount paid with 12% interest from the filing of
complaint. Respondent Presidential Executive Assistant Clave affirmed the NHA
resolution.
ISSUE
1. Whether the doctrine of piercing the veil of corporate fiction has application to the
case.
2. Whether petitioner On Stott can be held solidarity liable with petitioner Corporation
for the refund of the installment payments made by respondent Dumpit.
RULING
The doctrine of piercing the veil of corporate fiction has no application to the
case. Consequently, petitioner Onstott cannot be held solidarity liable with petitioner
Corporation for the refund of the installment payments made by respondent Dumpit.
A corporation is invested by law with a personality separate and distinct from
those of the persons composing it. As a general rule, a corporation may not be made
to answer for acts or liabilities of its stockholders or those of the legal entities to which
it may be connected and vice versa.
1

However, the veil of corporate fiction may be pierced when: it is used as a shield
to further an end subversive of justice; or for purposes that could not have been
intended by the law that created it; or to defeat public convenience, justify wrong,
protect fraud, or defend crime; or to perpetrate fraud or con fuse legitimate issues; or
to circumvent the law or perpetuate deception; or as an alter ego, adjunct or business
conduit for the sole benefit of the stockholders. In this case however, there are no
badges of fraud on the part of the petitioners. They had literally relied, although
mistakenly, on paragraph 6 of the contract with respondent when they rescinded the
contract to sell extra judicially.
Although Onstott appears to be the controlling stockholder, there being no fraud,
he cannot be made personally liable.

CRUZ
VS.
DALISAY
152 SCRA 482 (1987)
FACTS OF THE CASE
A sworn complaint was filed by Adelio Cruz charging Quiterio Dalisay, Senior
Deputy Sheriff of Manila, with malfeasance in office, corrupt practices and serious
irregularities who allegedly attached and/or levied the money belonging to complainant
Cruz when he was not himself the judgment debtor in the final judgment of an NLRC
case sought to be enforced but rather the company known as Qualitrans Limousine
Service, Inc.; and also caused the service of the alias writ of execution upon
complainant who is a resident of Pasay City, despite knowledge that his territorial
jurisdiction covers Manila only and does not extend to Pasay City.
Respondent, however, choose to pierce the veil of corporate entity usurping a
power belonging to the court and assumed improvidently that since the complainant is
the owner/president of Qualitrans Limousine Service, Inc., they are one and the same.
His reply explained that when he garnished complainants cash deposit at the Philtrust
bank he was merely performing a ministerial duty. And that while it is true that said writ
was addressed to Qualitrans Limousine Service, Inc., it is also a fact that complainant
had executed an affidavit before the Pasay City assistant fiscal stating that he is the
owner/ president of Qualitrans. Because of that declaration, the counsel for the plaintiff
in the labor case advised him to serve notice of garnishment on the Phil trust bank.
ISSUE
Whether the personal property of Cruz (complainant) can be levied or attached
being the owner/president of the corporation.
RULING
No. The mere fact that one is president of the corporation does not render the
property he owns or possesses the property of the corporation, since that president,
as an individual, and the corporation, are separate entities. It is a well settled doctrine
both in law and equity that as a legal entity, a corporation has a personality distinct
and separate from its individual stockholders or members.

REMO JR.
Vs.
INTERMEDIATE APPELATE COURT
G.R.No. L- 67626
FACTS OF THE CASE
The Board of Directors of Akron Customs Brokerage Corporation (Akron),
composed of Jose Remo, Jr., Ernesto Baares, Feliciano Coprada, Jemina Coprada,
and Dario Punzalan with Lucia Lacaste as Secretary, adopted a resolution authorizing
the purchase of 13 trucks for use in its business to be paid out of a loan the
corporation may secure from anyl ending institution. Feliciano Coprada, as President
and Chairman of Akron, purchased the trucks from E.B. Marcha Transport Company,
Inc.for P 525K as evidenced by a deed of absolute sale. The parties agreed on a
downpayment in the amount of P50K and that the balance of P 475K shall be paid
within 60 days from the date of the execution of the agreement. They also agreed that
until balance is fully paid, the down payment of P 50K shall accrue as rentals and
failure to pay the balance within 60 days, then the balance shall constitute as a chattel
mortgage lien covering the cargo trucks and the parties may allow an extension of 30
days and Marcha may ask for a revocation of the contract and the re-conveyance of all
trucks.
The obligation is further secured by a promissory note executed by Coprada in
favor of Akron. It is stated that the balance shall be paid from the proceeds of a loan
obtained from the Development Bank of the Philippines (DBP) within 60 days
After the lapse of 90 days, Marsha tried to collect from Coprada but the Coprada
promised to pay only upon the release of the DBP loan. Marsha found that no loan
application was ever filed by Akron with DBP.
In due time, Marsha filed a complaint for the recovery of P 525K or the return of
the 13 trucks with damages against Akron and its officers and directors. Remo Jr. sold
all his shares in Akron to Coprada. It also appears that Akron amended its articles of
incorporation thereby changing its name to Akron Transport International, Inc. which
assumed the liability of Akron to Marsha.
ISSUE
Whether Remo Jr. should be held personally liable together with Akron
Transport International, Inc.
RULING
No, the environmental facts of this case show that there is no cogent basis to
pierce the corporate veil of Akron and hold petitioner personally liable.
While it is true that in December, 1977 petitioner was still a member of the board of
directors of Akron and that he participated in the adoption of a resolution authorizing
the purchase of 13 trucks for the use in the brokerage business of Akron to be paid out
of a loan to be secured from a lending institution, it does not appear that said
resolution was intended to defraud anyone. The word "WE' in the said promissory note
must refer to the corporation which Coprada represented in the execution of the
note and not its stockholders or directors. Petitioner did not sign the said promissory
note so he cannot be personally bound thereby. It is his inherent right as a stockholder
4

to

dispose

of

his

shares

of

stock

anytime

he

desires.

PABALAN
Vs.
NATIONAL LABOR RELATIONS COMMISSION
G.R.NO. 89879
FACTS OF THE CASE
Eighty-four (84) workers of the Philippine Inter-Fashion, Inc. (PIF) filed a
complaint against the latter for illegal transfer simultaneous with illegal dismissal
without justifiable cause and in violation of the provision of the Labor Code on security
of tenure as well as the provisions of Batas Pambansa Bldg. 130. Complainants
demanded reinstatement with full back wages, living allowance, 13th month pay and
other benefits under existing laws and/or separation pay.
A decision was rendered by the labor arbiter ordering respondent Philippine
Inter-Fashion and its officers Mr. Jaime Pabalan and Mr. Eduardo Lagdameo to
reinstate the sixty two (62) complainants to their former or equivalent position without
loss of seniority rights and privileges and to pay, jointly and severally, their back wages
and other benefits from the time they were dismissed up to the time they are actually
reinstated.
Not satisfied therewith petitioners filed a motion for reconsideration in the First
Division of the public respondent, National Labor Relations Commission (NLRC),
which nevertheless, affirmed the appealed decision and dismissed the appeal for lack
of.
Hence, the herein petition for certiorari with prayer for the issuance of a
temporary restraining order.
ISSUE
Whether or not petitioners herein may be held jointly and severally liable with
Philippine Interfashion, Inc., to pay the judgment debt.
RULINGS
The settled rule is that the corporation is vested by law with a personality
separate and distinct from the persons composing it, including its officers as well as
from that of any other legal entity to which it may be related. Thus, a company
manager acting in good faith within the scope of his authority in terminating the
services of certain employees cannot be held personally liable for damages.
As a general rule, officers of a corporation are not personally liable for their
official acts unless it is shown that they have exceeded their authority. However, the
legal fiction that a corporation has a personality separate and distinct from
stockholders and members may be disregarded as when the notion of legal entity is
used as a means to perpetrate fraud or an illegal act or as a vehicle for the evasion of
an existing obligation, the circumvention of statutes, and or (to) confuse legitimate
issues the veil which protects the corporation will be lifted.
In this particular case complainants did not allege or show that petitioners, as
officers of the corporation deliberately and maliciously designed to evade the financial
obligation of the corporation to its employees, or used the transfer of the employees as
a means to perpetrate an illegal act or as a vehicle for the evasion of existing
obligations, the circumvention of statutes, or to confuse the legitimate issues.
Not one of the above circumstances has been shown to be present. Hence
petitioners cannot be held jointly and severally liable with the PIF Corporation under
the questioned decision and resolution of the public respondent.
Wherefore, the petition is granted and the questioned resolution of the public
respondent is modified by relieving petitioners of any liability as officers of the PIF and
holding that the liability shall be solely that of Philippine Inter-Fashion, Inc.
6

UMALI ET, AL.


VS.
COURT OF APPEALS
G.R. No. 89561 September 13, 1990
FACTS OF THE CASE
Santiago Rivera is the nephew of plaintiff Mauricia Meer Vda. de Castillo. The
Castillo family is the owners of a parcel of land located in Lucena City which was given
as security for a loan from the Development Banks of the Philippines. For their failure
to pay the amortization, foreclosure of the said property was about to be initiated. This
problem was made known to Santiago Rivera, who proposed to them the conversion
into subdivision of the four (4) parcels of land adjacent to the mortgaged property to
raise the necessary fund. The idea was accepted by the Castillo family and to carry
out the project, a Memorandum of Agreement was executed by and between Slobec
Realty and Development, Inc., represented by its President Santiago Rivera and the
Castillo family. In this agreement, Santiago Rivera obliged himself to pay the Castillo
family the sum of P70, 000.00 immediately after the execution of the agreement and to
pay the additional amount of P400, 000.00 after the property has been converted into
a subdivision. Rivera, armed with the agreement, approached Mr. Modesto Cervantes,
President of defendant Bormaheco, and proposed to purchase from Bormaheco two
(2) tractors Model D-7 and D-8. Subsequently, a Sales Agreement was executed on
December 28, 1970, which was accepted by the latter and executed Sales Agreement.
The balance of the consideration was secured by a surety bond from ICP (Insurance
Corporation of the Phil.) which was in turn secured by a mortagage, the properties of
the Castillos.
ISSUE
Whether the doctrine of piercing the veil of corporate fiction has application to
the case.
RULING
The doctrine of piercing the veil of corporate fiction has no application to the
case. Petitioners do not seek to impose a claim against the individual members of the
three corporations involved; on the contrary, it is these corporations which desire to
enforce an alleged right against petitioners. Assuming that petitioners were indeed
defrauded by private respondents in the foreclosure of the mortgaged properties, this
fact alone is not, under the circumstances, sufficient to justify the piercing of the
corporate fiction, since petitioners do not intend to hold the officers and/or members of
respondent corporations personally liable therefore. Petitioners are merely seeking the
declaration of the nullity of the foreclosure sale, which relief may be obtained without
having to disregard the aforesaid corporate fiction attaching to respondent
corporations. Secondly, petitioners failed to establish by clear and convincing
evidence that private respondents were purposely formed and operated, and
thereafter transacted with petitioners, with the sole intention of defrauding the latter.
The mere fact, therefore, that the businesses of two or more corporations are
interrelated is not a justification for disregarding their separate personalities, absent
sufficient showing that the corporate entity was purposely used as a shield to defraud
creditors and third persons of their rights.
7

INDOPHIL TEXTILE MILL WORKERS UNION PTGWO


VS.
CALICA
G.R. No. 124715
FACTS OF THE CASE
In April, 1987, petitioner Indophil Textile Mill Workers Union-PTGWO and private
respondent Indophil Textile Mills, Inc. executed a collective bargaining agreement
effective from April 1, 1987 to March 31, 1990.
Meanwhile, Indophil Acrylic Manufacturing Corporation was formed and
registered with the Securities and Exchange Commission. Subsequently, Acrylic
applied for registration with the Board of Investments for incentives under the 1987
Omnibus Investments Code. The application was approved on a preferred nonpioneer status.
In 1988, Acrylic became operational and hired workers according to its own
criteria and standards. Sometime in July, 1989, the workers of Acrylic unionized and a
duly certified collective bargaining agreement was executed.
In 1990 or a year after the workers of Acrylic have been unionized and a CBA
executed, the petitioner union claimed that the plant facilities built and set up by
Acrylic should be considered as an extension or expansion of the facilities of private
respondent Company pursuant to Section 1(c), Article I of the CBA, to wit,. c) This
Agreement shall apply to the Company's plant facilities and installations and to any
extension and expansion thereat.
In other words, it is the petitioner's contention that Acrylic is part of the Indophil
bargaining unit.
The petitioner's contention was opposed by private respondent which submits
that it is a juridical entity separate and distinct from Acrylic. Voluntary Arbitrator ruled in
favor of Indophil.
ISSUE
Whether Indophil Acrylic is a separate and distinct entity from respondent
company for purposes of union representation.
RULING
Yes.Under the doctrine of piercing the veil of corporate entity, when valid
grounds therefore exist, the legal fiction that a corporation is an entity with a juridical
personality separate and distinct from its members or stockholders may be
disregarded. In such cases, the corporation will be considered as a mere association
of persons. The members or stockholders of the corporation will be considered as the
corporation that is liability will attach directly to the officers and stockholders. The
doctrine applies when the corporate fiction is used to defeat public convenience, justify
wrong, protect fraud, or defend crime, or when it is made as a shield to confuse the
legitimate issues, or where a corporation is the mere alter ego or business conduit of a
person, or where the corporation is so organized and controlled and its affairs are so
conducted as to make it merely an instrumentality, agency, conduit or adjunct of
another corporation.

In the case at bar, petitioner seeks to pierce the veil of corporate entity of
Acrylic, alleging that the creation of the corporation is a devise to evade the application
of the CBA between petitioner Union and private respondent Company. While we do
not discount the possibility of the similarities of the businesses of private respondent
and Acrylic, neither are we inclined to apply the doctrine invoked by petitioner in
granting the relief sought. The fact that the businesses of private respondent and
Acrylic are related, that some of the employees of the private respondent are the same
persons manning and providing for auxiliary services to the units of Acrylic, and that
the physical plants, offices and facilities are situated in the same compound, it is our
considered opinion that these facts are not sufficient to justify the piercing of the
corporate veil of Acrylic.
Hence, the Acrylic not being an extension or expansion of private respondent,
the rank-and-file employees working at Acrylic should not be recognized as part of,
and/or within the scope of the petitioner, as the bargaining representative of private
respondent.

EPG CONSTRUCTION COMPANY, INC.


VS.
COURT OF APPEALS
G.R.NO. 103372
FACTS OF THE CASE
Petitioner EPG Construction Co., Inc. and the University of the Philippines,
herein private respondent, entered into a contract for the construction of the UP Law
Library Building for the stipulated price of P7, 545,000.00.
Sometime in July, 1983, the private respondent complained to the petitioner that
6 air-conditioning units on the third floor of the building were not cooling properly. After
inspection of the equipment, EPG agreed to shoulder the expenses for their repair,
including labor and materials, in the amount of P38.000.00.
For whatever reason, the repair was never undertaken. UP repeated its
complaints to EPG, which again sent its representatives to assess the defects. Finally,
it made UP a written offer to repair the system for P194, 000.00.
UP insisted that EPG was obligated to repair the defects at its own expense
under the guarantee provision in their contract. EPG demurred. UP then contracted
with another company, which repaired the defects for P190, 000.00.
The private respondent subsequently demanded from EPG reimbursement of
the said amount plus an equal sum as liquidated damages. When the demand was
rejected, UP sued EPG and its president, Emmanuel P. de Guzman, in the Regional
Trial Court of Quezon City. De Guzman moved to dismiss the complaint as to him for
lack of a cause of action, but the motion was denied.
After trial, judgment was rendered by Judge Antonio P. Solano requiring both
defendants jointly and severally to pay the plaintiff P190, 000.00 as actual damages,
P50, 000.00 as liquidated damages, P10, 000.00 as attorney's fees, and costs.
ISSUE
The petitioners appealed to the Court of Appeals, which sustained the trial
court. 1 They then came to this Court to fault the respondent court for not holding that:
1) UP was estopped by its certificate of acceptance from imputing liability to EPG for
the defects; 2) the defects were due to force majeure or fortuitous event; and 3)
Emmanuel de Guzman has a separate personality from that of EPG Construction Co.,
Inc.
RULING
WHEREFORE, the appealed decision is AFFIRMED but with the modification
that EPG Construction Co., Inc. shall be solely liable for the damages awarded in favor
of the University of the Philippines.
The final point of the petition is that Emmanuel P. de Guzman has a separate
legal personality from EPG Construction Co., Inc. and should not be held solidarity
liable with it. He stresses that the acts of the company are its own responsibility and
there is no reason why any liability arising from such acts should be ascribed to him.
Thus:
10

It is a doctrine well-established and obtains both at law and in equity that a


corporation is a distinct legal entity to be considered as separate and apart from the
individual stockholders or members who compose it, and is not affected by the
personal rights, obligations and transactions of its stockholders or members. 3
The trial court did not explain why Emmanuel de Guzman was held solidarity
liable with EPG Construction Co., Inc., and neither did the respondent court when it
affirmed the appealed decision, In its Comment on the present petition, UP also did
not refute the petitioners' argument and simply passed upon it sub silentio although
the matter was squarely raised and discussed in the petition.
Notably, when Emmanuel de Guzman moved to dismiss the complaint as to
him, UP said in its opposition to the motion that it was suing him "in his official capacity
and not in his personal capacity." His inclusion as President of the company was
therefore superfluous, as De Guzman correctly contended, because his acts as such
were corporate acts imputable to EPG itself as his principal. It is settled that;
A corporation is invested by law with a personality separate and distinct from
those of the persons composing it as well as from that of any other entity to which it
may be related. Mere ownership by a single stockholder or by another corporation of
all or nearly all of the capital stock of a corporation is not of itself sufficient ground for
disregarding the separate corporate personality. The general manager of a corporation
therefore should not be made personally answerable for the payment of the
employee's back wages unless he had acted maliciously or in bad faith in terminating
the services of the employee. 4
The exception noted is where the official "had acted maliciously or in bad faith,"
in which event he may be made personally liable for his own act. That exception is not
applicable in the case at bar, because it has not been proved that De Guzman acted
maliciously or in bad faith when, as President of EPG, he sought to protect its interests
and resisted UP's claims. Whatever damage was caused to UP as a result of his acts
is the sole responsibility of EPG even though De Guzman was its principal officer and
controlling stockholder.

11

BOYER - ROXAS
VS.
COURT OF APPEALS
211 SCRA 470 (1992)
FACTS OF THE CASE
When Eugenia V. Roxas died, her heirs formed a corporation under the name
and style of Heirs of Eugenia V. Roxas, Inc. using her estate as the capital of the
corporation, the private respondent herein. It was primarily engaged in agriculture
business, however it amended its purpose to enable it to engage in resort and
restaurant business. Petitioners are stockholders of the corporation and two of the heirs
of Eugenia. By tolerance, they were allowed to occupy some of the properties of the
corporation as their residence. However, the board of directors of the corporation
passed a resolution evicting the petitioners from the property of the corporation because
the same will be needed for expansion.
At the RTC, private respondent presented its evidence averring that the subject
premises are owned by the corporation. Petitioners failed to present their evidence due
to alleged negligence of their counsel. RTC handed a decision in favor of private
respondent.
Petitioners appealed to the Court of Appeals but the latter denied the petition and
affirmed the ruling of the RTC. Hence, they appealed to the Supreme Court. In their
appeal, petitioners argues that the CA made a mistake in upholding the decision of the
RTC, and that their occupancy of the subject premises should be respected because
they own an aliquot part of the corporation as stockholders, and that the veil of
corporate fiction must be pierced by virtue thereof.
ISSUE
1. Whether petitioners contention were correct as regards the piercing of the corporate
veil.
2. Whether petitioners were correct in their contention that they should be respected as
regards their occupancy since they own an aliquot part of the corporation.

RULING

Petitioners contention to pierce the veil of corporate fiction is untenable.


As aptly held by the court: ..The separate personality of a corporation may
ONLY be disregarded when the corporation is used as a cloak or cover for fraud
1.

12

SESBREO vs COURT OF APPEALS


G.R. No. 89252
May 24, 1993
FACTS OF THE CASE
Raul Sesbreo made a money market placement with the Philippine
Underwriters Finance Corporation ("Philfinance"). The placement, with a term of thirtytwo (32) days, would mature on 13 March 1981. In turn, Philfinance issued documents
which included posted checks, a Certificate of Confirmation of Sale with a Delta Motor
Corporation Promissory Note (DMC PN), and a Delivery Receipt indicating that the
DMC PN was in custodianship of Pilipinas Bank. The DMC PN had a face value of
P2.3m with Philfinance as "payee" and Delta as "maker" and that on face of the PN
was stamed "NON NEGOTIABLE".
The checks were dishonored for having insufficient funds. This prompted the
petitioner to hand a demand letter over to Pilipinas Bank asking for the physical
delivery of the promissory note. Pilipinas did not deliver the Note to petitioner.
Petitioner also made a written demand upon Delta for the partial satisfaction of DMC
PN. Delta denied any liability as the promissory note was not intended to be
negotiated.
As petitioner failed to collect his investment and interest thereon, he then filed a case
against Delta and Pilipinas. He contends that Phil finance and private respondents
Delta and Pilipinas should be treated as one corporate entity alleging that all three
companies have a common Director.
ISSUE
Whether or not Delta and Pilipinas can be held liable for the liability of
Philfinance to petitioner.
RULING
No. In the first place, as already noted, jurisdiction over the person of
Philfinance was never acquired either by the trial court or by the respondent Court of
Appeals. Petitioner similarly did not seek to implead Philfinance in the Petition before
us.
Secondly, it is not disputed that Philfinance and private respondents Delta and
Pilipinas have been organized as separate corporate entities. Petitioner asks us to
pierce their separate corporate entities, but has been able only to cite the presence of
a common Director Mr. Ricardo Silverio, Sr., sitting on the Board of Directors of all
three (3) companies. Petitioner has neither alleged nor proved that one or another of
the three (3) concededly related companies used the other two (2) as mere alter egos
or that the corporate affairs of the other two (2) were administered and managed for
the benefit of one. There is simply not enough evidence of record to justify
disregarding the separate corporate personalities of delta and Pilipinas and to hold
them liable for any assumed or undetermined liability of Philfinance to petitioner.

13

ROBLEDO
VS.
NLRC
(G.R. No. 110358, Nov. 9, 1994)
FACT OF THE CASE
Robledo ET. Al. filed a Petition for Review of the Decision of NLRC, setting
aside the decision of the Labor Arbiter, which held private respondents jointly and
severally liable to the petitioners for overtime and legal holiday pay.
Petitioners were former employees of Bacani Security and Protective Agency
(BPSA). They were employed as security guards at different times during the period
1969 to December 1989 when BPSA ceased to operate.
BPSA was a single proprietorship owned, managed, and operated by the late
Felipe Bacani.
On December 31, 1989, Felipe Bacani retired the business name and BSPA
ceased to operate effective on that day.
On Jan. 15, 1990 Felipe Bacani died. An intestate proceeding was instituted for
the settlement of his estate before Pasig-RTC.
Earlier, on Oct. 26, 1989, respondent Bacani Security and Allied Services Co.,
Inc. (BASEC) had been organized and registered as a corporation with SEC. Several
of the incorporator (3) surnamed Bacani, and that includes the daughter of the late
Felipe Bacani.
On July 5, 1990, the petitioners filed a complaint with the DOLE for
underpayment of wages and nonpayment of overtime pay and other accrued benefits,
and for the return of their cash bond, which they posted, with BPSA. Made
respondents were BSPA and BASEC.
On March 1, 1992, the Labor Arbiter rendered a decision upholding the right of
petitioners, finding the complainants entitled to their money claims to be paid by all the
respondents solidarily.
On appeal, the NLRC reversed the decision declaring that the Labor Arbiter is
without jurisdiction and instead suggested that petitioners file their claims with PasigRTC where an intestate proceeding of Bacanis estate was pending.
Petitioners moved for reconsideration but their motion was denied for lack of
merit.
The case was elevated to the SC and was treated as a special civil action of
certiorari to determine whether the NLRC committed a grave abuse of discretion in
reversing the Labor Arbiters decision.
ISSUE
Whether Bacani Security and Allied Services, Inc. (BASEC) can be held liable
for claims of petitioners against Bacani Security and Protective Agency (BSPA).

14

RULING
No. Petitioners contend that public respondent, NLRC, erred in setting aside the
Labor Arbiters judgment on the ground that BASEC is the same entity as BSPA the
latter being owned and controlled by one and the same family, the Bacani family. For
this reason they urge that corporate fiction should be disregarded and BASEC should
be held liable for the obligations of the defunct BSPA.
As correctly found by the NLRC, BASEC is an entity separate and distinct from
that of BSPA. BSPA is a single proprietorship owned and operated by Felipe Bacani.
Hence, its debts and obligations were the personal obligations of its owner.
Petitioners claims, which are based on these debts and personal obligations, did not
survive the death of Felipe Bacani on Jan. 15, 1990 and should have been filed
instead in the intestate proceedings involving his estate.

15

SOL LAGUIO
Vs.
NATIONAL LABOR RELATIONS COMMISSION
G.R. No. 108936
FACTS OF THE CASE
April Toy Inc. is a domestic corporation engaged in the manufacturing, exporting and
dealing at wholesale and retail, in stuffed toys. After a year of operation, April Toy
ceased operations because of its dire financial condition. Hence the petitioners filed a
complaint for illegal shutdown, retrenchment, dismissal and unfair labor practice.
Thereafter they amended their complaint to implead Well World Toy Inc. In their
complaint, petitioners alleged that they were originally probationary employees of Well
World Toy Inc. but were later laid off for starting to organize themselves into a union.
They applied and were hired by April Toy. They won as the exclusive bargaining agent
for the workers and when they submitted their CBA proposal, April Toy rejected in view
of its cessation of operations. The closure, petitioners alleged, is April Toy's clever
ploy to defeat their right to self-organization. Petitioners further allege that the
incorporators and principal officers of April Toy are also the incorporators of Well
World Toy, thus both should be treated as one corporation liable for their claims. The
Labor Arbiter and NLRC both recognized said corporations as two distinct corporations
and the closure of April Toy valid. Thus this petition.
ISSUE
Whether April Toy and Well World Toy should be treated as one corporation liable for
the grievances of the petitioners
RULING
No. It is basic that a corporation is invested by law with a personality separate and
distinct from those of the persons composing it as well as from that of any other legal
entity to which it may be related. Mere substantial identity of the incorporators of the
two corporations does not necessarily imply fraud nor warrant the piercing of the veil of
corporation fiction. In the absence of clear and convincing evidence that April and
Well Worlds corporate personalities were used to perpetuate fraud, or circumvent the
law said corporations were rightly treated as distinct and separate from each other. In
this case, with the facts and circumstances showing that the owners of April Toy are
different from those of Well-World, the management of one being different from the
other, and the office of April Toy is situated more than ten kilometers away from WellWorld, plus the fact that the closure of April Toy was for valid reasons, the Labor
Arbiter likewise correctly opined that the two corporations are separate and distinct
from each other, and that there is no basis for piercing the veil of corporate fiction.
Thus April Toy and Well World Toy should not be treated as one corporation liable for
the grievances of the petitioners.

16

MATUGUINA INTEGRATED WOOD PRODUCTS, INC.,


Vs.
COURT Of APPEALS
G.R. No. 98310
October 24, 1996
FACTS OF THE CASE
Milagros Matuguina, a sole proprietor, has a timber business named Matuguina
Logging Enterprises in Davao under the Provisional Timber License No. 30 for a
certain area. During the same time, Milagros became the majority stockholder of
MIWP (Matuguina Integrated Wood Products) by buying 70% of stock ownership.
Milagros requested to the director of Forest Development to transfer the management
of the timber license no. 30, granted for MLE, to MIWP.
Pending the approval of the transfer of the license, DAVENCOR (private
respondent) submitted a complaint regarding the encroachment of MLE in the
concession area of DAVENCOR. , The Minister of Natural Resources, Hon. Ernesto
M. Maceda rendered his decision against MLE for illegal logging operations on the
portion of the land under the concession of DAVENCOR. DAVENCOR then requested
the issuance of writ of execution for MLE and/or MIWP. MIWP, as a defense, filed for
prohibition, damages and injunction, with prayer for restraining order on the grounds
that they are a separate entity from MLE and, therefore, not a party to the complaint by
DAVENCOR. Trial Court granted the TRO. RTC ruled in favor of MIWP which was
reversed by the Court of Appeals; hence, this petition on certiorari.
ISSUE
WON the corporate veil of MIWP shall be pierced to be held liable for the acts of
MLE
RULING
NO, MIWP cannot be held liable. A corporation has a separate personality. . It
may not generally be held liable for that of the persons composing it unless when the
juridical personality of the corporation is used to defeat public convenience, justify
wrong, protect fraud or defend crime, the corporation shall be considered as a mere
association of persons. But for the separate juridical personality of a corporation to be
disregarded, the wrongdoing must be clearly and convincingly established. It cannot
be presumed.
It is likewise improper to state that the MIWPI is the privy or the successor-ininterest of MLE, as the liability for the encroachment over DAVENCOR's timber
concession is concerned, by reason of the transfer of interest in PTL No. 30 from MLE
to MIWPI. The transfer has never become effective. More importantly, even if it is
deemed that there was a valid change of name and transfer of interest in the PTL No.
30, this only signifies a transfer of authority, from MLE to MIWPI, to conduct logging
operations in the area covered by PTL No. 30. It does not show indubitable proof that
MIWPI was a mere conduit or successor of Milagros Matuguina/MLE, as far the latter's
liability for the encroachment upon DAVENCOR's concession is concerned.

17

REPUBLIC
VS
SANDIGANBAYAN
GR, NO 113420, MARCH 7, 1997
FACTS OF THE CASE
Republic of the Philippines is represented by the PCGG; SandiganBayan, Third
Division; PRIVIDENT INTERNATIONAL RESOURSES CORPORATION, and Phil.
Casino Operators Corp., respondents
On March 18,1986, pursuant to power vested upon it by the President (Aquino)
under E.O. # 1, the PCGG issued a writ of sequestration against the assets of
Provident International Resources Corp. and Philippine Casino Operators Corp.
(herein respondent Corp.)
On July 1987, petitioner RP through the Solicitor General filed before the
Sandiganbayan a complaint, a civil case against Edward T. Marcelo, Fabian C. Ver,
Ferdinand Marcos and Imelda Marcos for recovery of alleged ill-gotten wealth acquired
by them during the former Dictators regime.
Among the listed Corporations held and controlled by defendant Marcelo and
among the assets apparently acquired illegally by the defendants were respondent
Corporations.
On September 1991, respondent corporations filed before the Sandiganbayan a
petition for mandamus praying for lifting of the writ of Sequestration issued by PCGG.
On December 4, 1991 public respondent (Sandiganbayan) in its Resolution, granted
the respondent corporations prayer and lifted the writ of Sequestration issued against
them, and ordered the PCGG to restore to the petitioners all their assets, properties
records and documents subject of sequestration on the ground that:
ISSUE
1) For Failure of the respondent (PCGG) to file the proper judicial action against
them within the period fixed in Sec.26 Art XVIII of the 1987 Constitution : and
2) the writ of Sequestration was signed by only one PCGG Commission.
RULING
The petitioner (PCGG filed a petition for certiorari under rule 65 of the Rules of Court.
In terms of juridical personality and legal culpability from their erring members on
stockholders, said corporation are not themselves guilty of the sins of the latter. They
are simply the res in the actions for the recovery of illegally acquired wealth and there
is, in principle, no cause of action against them and no ground to implead them as
defendants.

18

TRADERS ROYAL BANK, petitioner,


Vs.
COURT OF APPEALS, FILRITERS GUARANTY ASSURANCE CORPORATION and
CENTRAL BANK of the PHILIPPINES, respondents.
G.R. No. 93397. March 3, 1997
FACTS OF THE CASE
Nature of the Case: Nature of the Case: Petition for Review on Certiorari.
CA affirmed the nulllity of the transfer of Central Bank Certificate of Indebtedness
(CBCI) No. D891,2 with a face value of Php 500,000.00 from the Philippine
Underwriters Finance Corporation (PhilFinance) to Petitioner Trader's Royal Bank
(TRB) under a Repurchase Agreement and a Detached Assignment.
Filriters Guaranty Assurance Corporation (Filriters) is the owner of the Central Bank
Certificate of Indebtednes (CBCI) No. D891 worth Php500,000.00 which was
transferred to Philippine Underwriters Finance Corporation (PhilFinance) through a
Deed of Assignment. Subsequently Philfinance transferred the said instrument (still
registered under the name of Filriters) to Traders Royal Bank (TRB). It was made
through a Repurchase Agreement. Philfinance defaulted in its obligation to TRB. It
then executed a Deed of Assignment to TRB.
TRB then notified the Central Bank (Security Servicing Department) to cause the
transfer and registration of the CBCI No. D891 under its name. It was however refused
to do so in lieu of an adverse claim filed by Filriters.
The Court of Appeals held that the CBCI is not a negotiable instrument. It is clearly
stated that it was payable to Filriters. The certificate lacked the words of negotiability
which serve as an expression of consent that the instrument may be transferred by
negotiation.
The assignment of Filriters to Philfinance was also null and void because it was made
without consideration. It also did not conform to the Central Bank Circular No. 769,
series of 1980 - Rules and Regulations Governing Central Bank Certificates of
Indebtedness. It provides that any assignment of registered certificates shall not be
valid unless made x x x by the registered owner thereof in person or by his
representative duly authorized in writing.
Alfredo O. Banaria (who signed the deed of assignment) did not have the necessary
written authorization from the Board of Directors of Filriters. For lack of such authority,
the assignment did not bind Filriters and violated the Central Bank Circular (No. 769)
which has the force and effect of a law.
For such violations, Philfinance acquired no title or rights under CBCI No. D891 which
it could assign or transfer to TRB, and which TRB can register with the Central Bank.
On petition, TRB argued that Philfinance owns 90% of Filriter's equity and the two
corporations have identical corporate officers, thus demanding the application of
piercing the veil of corporate fiction to give validity to the transfer of the CBCI from
Filriters to TRB.

19

ISSUE
Was the transfer of the CBCI from Filriters to PhilFinance and subsequently from
PhilFinance to TRB, in accordance with existing law, so as to entitle TRB to have the
CBCI registered in its name with the Central Bank?
RULING
Corporation Law; Piercing the Veil of Corporate Fiction; Piercing the veil of corporate
entity requires the court to see through the protective shroud which exempts its
stockholders from liabilities that ordinarily, they could be subject to, or distinguishes
one corporation from a seemingly separate one, were it not for the existing corporate
fiction.Petitioner cannot put up the excuse of piercing the veil of corporate entity, as
this is merely an equitable remedy, and may be awarded only in cases when the
corporate fiction is used to defeat public convenience, justify wrong, protect fraud or
defend crime or where a corporation is a mere alter ego or business conduit of a
person. Piercing the veil of corporate entity requires the court to see through the
protective shroud which exempts its stockholders from liabilities that ordinarily, they
could be subject to, or distinguishes one corporation from a seemingly separate one,
were it not for the existing corporate fiction. But to do this, the court must be sure that
the corporate fiction was misused, to such an extent that injustice, fraud, or crime was
committed upon another, disregarding, thus, his, her, or its rights. It is the protection of
the interests of innocent third persons dealing with the corporate entity which the law
aims to protect by this doctrine.
Filriters and PhilFinance remains separate.
Same; Same; Mere ownership by a single stockholder or by another corporation of all
or nearly all of the capital stock of a corporation is not of itself a sufficient reason for
disregarding the fiction of separate corporate personalities.Though it is true that
when valid reasons exist, the legal fiction that a corporation is an entity with a juridical
personality separate from its stockholders and from other corporations may be
disregarded, in the absence of such grounds, the general rule must be upheld. The
fact that Philfinance owns majority shares in Filriters is not by itself a ground to
disregard the independent corporate status of Filriters. In Liddel & Co., Inc. vs.
Collector of Internal Revenue, the mere ownership by a single stockholder or by
another corporation of all or nearly all of the capital stock of a corporation is not of
itself a sufficient reason for disregarding the fiction of separate corporate personalities.
TRB was not defrauded at all when it acquired the CBCI from PhilFinance.
Same; Same; An entity which deals with corporate agents within circumstances
showing that the agents are acting in excess of corporate authority may not hold the
corporation liable.Petitioner, being a commercial bank, cannot feign ignorance of
Central Bank Circular 769, and its requirements. An entity which deals with corporate
agents within circumstances showing that the agents are acting in excess of corporate
authority, may not hold the corporation liable. This is only fair, as everyone must, in the
exercise of his rights and in the performance of his duties, act with justice, give
everyone his due, and observe honesty and good faith.
TRB knew that PhilFinance is not the registered owner of CBCI No. D891. The fact
that a non-owner is disposing of the registered CBCI owned by another entity was a
good reason for the petitioner to verify or inquire as to the title of Philfinance to
20

dispose of the CBCI. Moreover the said instrument is governed by the rules and
regulations of the Central Bank.
Alfredo O. Banaria did not have the necessary authorization from the Board of
Directors of Filriters to bind it.
Lastly, Filriters acquired the CBCI to form part of its legal and capital reserves required
by law. Insurance companies are required to put up a legal reserve equivalent to 40
percent of the premiums receipt. The Insurance Commission requires this reserve to
be invested preferably in government securities or government bonds. Therefore, the
said CBCI cannot be taken out of the said fund, without violating the requirements of
the law. The unauthorized use or distribution of the same by a corporate officer of
Filriters, cannot bind the corporation, not without the approval of its Board of Directors,
and the maintenance of the required reserve fund.
Consequently, the title of Filriters over the subject certificate of indebtedness must be
upheld over the claimed interest of TRB.
DISPOSITION
Petition is DISMISSED and the decision appealed (Jan. 29, 1990) AFFIRMED.

21

a. PRESIDENTIAL COMMISSION ON GOOD GOVERNMENT


VS.
SANDIGANBAYAN
G.R. No. 125788
FACTS OF THE CASE
Petitioner PCGG filed in the Sandiganbayan a case for re-conveyance,
reversion, accounting, restitution and damages (Civil Case No. 0009) against Manuel
H. Nieto, Jose L. Africa, Roberto S. Benedicto, Potenciano Illusorio, Juan Ponce Enrile
and Ferdinand E. Marcos, Jr., alleging, in substance, that said defendants acted as
dummies of the late strongman and devised schemes and stratagems to monopolize
the telecommunications industry. Annexed to the complaint is a listing of the assets of
defendants Nieto and Africa, among which are their shares of stock in private
respondent Aerocom Investors and Managers, Inc. (Aerocom).
After a year, the PCGG sought to sequester Aerocom under a writ of
sequestration which was served and received under protest by Aerocoms president.
Then Aerocom filed a complaint against the PCGG urging the Sandiganbayan (Civil
Case No. 0044) to nullify the same on the ground that it was served on Aerocom
beyond the 18-month period from the ratification of the 1987 Constitution. Then,
Aerocom filed a Manifestation and Motion praying that the Sandiganbayan direct the
PCGG to release and distribute the dividends pertaining to the shares of Aerocom in
all corporations where it owns shares of stock. PCGG opposed the release of the
dividends on the argument that the fact that plaintiff Aerocom is mentioned in Annex
A of the complaint filed in Civil Case No. 0009 is a clear indication that the shares
thereof are likewise sequestered.
Sandiganbayan ordered the PCGG to release the dividends pertaining to
Aerocom except the dividends on the sequestered shares of stock registered in the
names of Nieto and Africa. This is on the ground that the complaint in Civil Case No.
0009 does not show that Aerocom was itself sequestered. Aerocom, being a
corporation, has a separate and distinct juridical personality from its stakeholders.
ISSUE
Whether Sandiganbayan erred in ordering the PCGG to release the dividends of
Aerocom, with exception as to the sequestered shares of stock of Nieto and Africa.
RULING
Sandiganbayan did not err in ordering the PCGG to release the dividends of
Aerocom. The PCGG failed to file the corresponding judicial action against Aerocom
within then period provided for in the Constitution. The fact that Aerocom was
mentioned in the complaint of the PCGG in Civil Case No. 0009 and in Annex A
thereof cannot justify the sequestration of the dividends pertaining to Aerocom, as it
was not impleaded as party-defendant.
There is no existing sequestration in this case, as the writ issued against
Aerocom is invalid. The suit in Civil Case No. 0009 against Nieto and Africa as
shareholders in Aerocom is not and cannot ipso facto be a suit against the
unimpleaded Aerocom itself without violating the fundamental principle that a
corporation has a legal personality distinct and separate from its stockholders.
22

Failure to implead these corporations as defendants and merely annexing a list


of such corporations to the complaints is a violation of their right to due process for it
would be in effect disregarding their distinct and separate personality without a
hearing.

23

b. PRESIDENTIAL COMMISSION ON GOOD GOVERNMENT VS.


SANDIGANBAYAN
G.R. Nos. 119609-10
FACTS OF THE CASE
The PCGG issued writs of sequestration against OWNI. Then, it sent Corporate
Secretary Africa of Ocean Wireless Network, Inc. (OWNI) a letter directing him to send
notices to all stockholders of record of OWNI for special stockholders meeting. He
was required to issue one qualifying share each to PCGG Commissioners Maceren
and Castro from the unissued shares and to record the transfer in the stock and
transfer book of OWNI. Failure to comply within 5 days from receipt thereof, Assistant
Solicitor General Desuasido would be designated as acting corporate secretary.
During the special stockholders meeting of OWNI, PCGG voted all the Class A
shares in the election of directors and elected to the board of directors Commissioners
Maceren, Parlade and Gutierrez representing the Class A shares, and Brooker and
Miller representing Class B and C shares. The new board of directors then elected
Maceren as Chairman of the Board, Gutierrez as President, ASG Desuasido as Acting
Corporate Secretary and Velasco as Acting Treasurer. None of the registered Class A
shareholders of OWNI was present in that special stockholders meeting.
Corporate Secretary Africa wrote the SEC questioning the election of the PCGG
nominees as directors of the OWNI board on the ground that they were not
stockholders of the OWNI. Then, a special stockholders meeting of OWNI took place,
were another election of directors for Class A shares were held. Thus, the PCGG
sought to enjoin the new directors from interfering with PCGGs management of OWNI
and/or representing themselves as directors. Sandiganbayan nullified the writs of
sequestration, stressing the need to file a separate action against OWNI.
ISSUE
Whether or not the PCGGs takeover of OWN is legal.
RULING
PCGGs takeover of OWNI is not legal.
It was previously ruled by the Court that the PCGG cannot exercise acts of
dominion over property sequestered, frozen or provisionally taken over.. the act of
sequestration.. does not import or bring about a divestment of title over said property;
does not make the PCGG the owner thereof.
Further, the writ of sequestration issued against OWNI is not valid because the
civil suit filed against its stockholders is not a suit against OWNI. This Court has held
that failure to implead these corporations as defendants and merely annexing a list of
such corporations to the complaints is a violation of their right to due process for it
would in effect be disregarding their distinct and separate personality without a
hearing.

24

UNION BANK OF THE PHILIPPINES


VS.
COURT OF APPEALS
290 SCRA 198 (1998)
FACT OF THE CASE
Private respondents EYCO Group of Companies (EYCO), Eulogio O.
Yutingco, Caroline Yutingco-Yao, and Theresa T. Lao (the Yutingcos), all of whom
are controlling stockholders of the aforementioned corporations, jointly filed with the
SEC a Petition for the Declaration of Suspension of Payments, Formation and
Appointment of Rehabilitation Receiver/Committee, Approval of Rehabilitation Plan
with Alternative Prayer for Liquidation and Dissolution of Corporations alleging that
the present combined financial condition of the petitioners clearly indicates that their
assets are more than enough to pay off the credits but that due to factors beyond the
control and anticipation of the management, the inability of the EYCO Group of
Companies to meet the obligations as they fall due on the schedule agreed with the
creditors has now become a stark reality. The Yutingcos justified their inclusion as copetitioners before the SEC on the ground that they had personally bound themselves
to EYCOs creditors under a J.S.S. Clause (Joint Several Solidary Guaranty) Cos.,
thereby in effect discarding the Veil of Corporate Fiction on their personal selves.
In connection with this petition, a the SEC Hearing Panel issued an order dated
September 19, 1997 setting its hearing on October 22, 1997 and directed the
suspension of all actions, claims and proceedings against private respondents
pending before any court, tribunal, office, board and/or commission.
Meanwhile, some of private respondents creditors, composed mainly of 22
domestic banks (the consortium) including herein petitioner Union Bank of the
Philippines Philippines, also convened on September 19, 1997 for the purpose of
deciding their options in the event that private Respondents invoke the provisions of
Presidential Decree No. 902-A, as amended.
Without notifying the members of the consortium, petitioner, however, decided
to break away from the group by suing private respondents in the regular courts. Aside
from commencing suits in the regular courts, petitioner also vehemently opposed
private respondents petition for suspension of payments in the SEC by filing a Motion
to Dismiss wherein it contended that the SEC was bereft of jurisdiction over such
petition on the ground that the inclusion of the Yutingcos in the petition cannot be
allowed since the authority and power of the Commission under the virtue of the law
applies only to corporations, partnerships and other forms of associations, and not to
individual petitioners who are not clearly covered by P.D. 902-A as amended.
Subsequently, a creditors meeting was again convened pursuant to SECs
order wherein the matter of creating a Mancom was submitted for resolution.
Apparently, only petitioner opposed the creation of said Mancom as it filed earlier with
the SEC its Motion to Dismiss.
The SEC Hearing Panel then issued an Omnibus Order directing this time the
creation of the Mancom and likewise granted an earlier Urgent Motion for
Reconsideration filed by creditor banks which sought to annotate the suspension order
on the titles of the properties of the private respondent corporations. This directive
expressly stated that the same was without prejudice to the resolution of petitioners
Motion to Dismiss.
Aggrieved, petitioner immediately took recourse to the Court of Appeals by filing
therewith a Petition for Certiorari with Prayer for the Issuance of a Temporary
Restraining Order and/or Writ of Preliminary Injunction. It imputed grave abuse of
25

discretion on the part of the SEC Hearing Panel in precipitately issuing the suspension
order and in prematurely directing the creation of the Mancom prior to the scheduled
hearing of its Motion to Dismiss. Petitioner lamented that these actions of the panel
deprived it of due process by effectively rendering moot and academic its Motion to
dismiss which allegedly presented a prejudicial question to the propriety of creating a
Mancom.
Meanwhile, members of the so-called steering committee of the consortium filed
with the appellate court an Urgent Motion for Intervention and a Consolidated
Intervention and Counter-Motion for Contempt and for the Imposition of Disciplinary
Measures Against Petitioners Counsel claiming that they were not impleaded at all by
petitioner in its petition before the appellate court when in fact they had actual,
material, direct and legal interest in the outcome of said case as owners of at least
eighty-five percent (85%) of private respondents obligations. Moreover, they opposed
said petition because of petitioners ostensible failure to exhaust administrative
remedies in the consortium and for being guilty of forum-shopping.
Series of Motions were filed and after several exchanges of pleadings finally
rendered its assailed decision granting the Motion for Intervention. Without moving for
reconsideration of the appellate courts decision, petitioner elevated the said matter to
this Court through Petition for Certiorari.
ISSUE
Whether suspension of payments with the SEC is the proper remedy on account
of the alleged insolvency of private respondents when they allegedly disposed of a
substantial portion of their properties in fraud of creditors.
RULING
Yes. The Supreme Court held that what determines the nature of an action, as
well as which court or body has jurisdiction over it, are the allegations of the complaint,
or a petition as in this case, and the character of the relief sought. that the petitioners
reasoning that the Yutingcos and the corporate entities making up the EYCO Group,
on the basis of the footnote that the former were filing the petition because they bound
themselves as surety to the corporate obligations, should be considered as mere
individuals who should file their petition for suspension of payments with the regular
courts pursuant to Section 2 of the Insolvency Law. The doctrine of piercing the veil of
corporate fiction heavily relied upon by petitioner is entirely misplaced, as said doctrine
only applies when such corporate fiction is used to defeat public convenience, justify
wrong, protect fraud or defend crime.

26

ASIONICS PHILIPPINES, INC.


Vs.
NATIONAL LABOR RELATIONS COMMISSION
G.R. No. 124950
FACTS OF THE CASE
API is a domestic corporation engaged in the business of assembling semiconductor chips and other electronic products mainly for export. Yolanda Boaquina
and Juana Gayola are working as material control clerk and as production operator.
API commenced negotiations with the duly recognized bargaining agent of its
employees, the Federation of Free Workers ("FFW"), for a Collective Bargaining
Agreement ("CBA"). A deadlock, however, ensued and the union decided to file a
notice of strike. This event prompted the two customers of API, Indala and CP Clare
Theta J, to thereupon refrain from sending to API additional kits or materials for
assembly. API, given the circumstance that its assembly line had to thereby grind to a
halt, was forced to suspend operations pursuant to Article 286 of the Labor
Code. Private respondents Boaquina and Gayola were among the employees asked
to take a leave from work.
Dissatisfied with their union (FFW), Boaquina and Gayola, together with some of
other co-employees, joined the Lakas ng Manggagawa sa Pilipinas Labor Union
("Lakas Union") where they eventually became members of its Board of Directors.
Lakas Union filed a notice of strike against API on the ground of unfair labor
practice.API filed for a petition for declaration of illegality of the strike.
Lakas Union countered that their strike was valid and staged as a measure of selfpreservation and as self-defense against the illegal dismissal of petitioners aimed at
union busting in the guise of a retrenchment program.
ISSUE
Whether a stockholder/director/officer of a corporation can be held liable for the
obligation of the corporation absent any proof and finding of bad faith.
RULING
No, APIs president and main stockholder Frank Yih cannot be held liable for the
obligation of the corporation to its employees. A corporation is a juridical entity with
legal personality separate and distinct from those acting for and in its behalf and, in
general, from the people comprising it. The rule is that obligations incurred by the
corporation, acting through its directors, officers and employees, are its sole
liabilities. Nevertheless, being a mere fiction of law, peculiar situations or valid
grounds can exist to warrant, albeit done sparingly, the disregard of its independent
being and the lifting of the corporate veil. As a rule, this situation might arise when a
corporation is used to evade a just and due obligation or to justify a wrong, to shield or
perpetrate fraud, to carry out similar unjustifiable aims or intentions, or as a subterfuge
to commit injustice and so circumvent the law.

27

FRANCISCO MOTORS CORPORATION


Vs.
COURT OF APPEALS
G.R. No. 100812
FACTS OF THE CASE
Petitioner filed a complaint against private respondents to recover an amount
representing the balance of the jeep body purchased by the Manuels from petitioner;
an additional sum representing the unpaid balance on the cost of repair of the vehicle;
and the cost of suit and attorney's fees. To the original balance on the price of jeep
body were added the costs of repair.
In their answer, private respondents interposed a counterclaim for unpaid legal
services by Gregorio Manuel which was not paid by the incorporators, directors and
officers of the petitioner. Private respondent alleged as an affirmative defense that,
while he was petitioner's Assistant Legal Officer, he represented members of the
Francisco family in the intestate estate proceedings of the late Benita Trinidad.
However, even after the termination of the proceedings, his services were not paid.
Said family members, he said, were also incorporators, directors and officers of
petitioner.
The trial court decided the case in favor of petitioner in regard to the petitioner's
claim for money, but also allowed the counter-claim of private respondents. Both
parties appealed. The Court of Appeals sustained the trial court's decision.
Hence, the present petition for review on certiorari under Rule 45 of the Rules of
Court.
ISSUE
Whether or not the personality of the corporation, vis-a-vis the individual
persons who hired the services of private respondent, is separate and distinct, in that
the liability of said individuals did not become an obligation chargeable against
petitioner.
RULING
Basic incorporation law is the principle that a corporation has a separate
personality distinct from its stockholders and from other corporations to which it may
be connected. However, under the doctrine of piercing the veil of corporate entity, the
corporation's separate juridical personality may be disregarded, for example, when the
corporate identity is used to defeat public convenience, justify wrong, protect fraud, or
defend crime. Also, where the corporation is a mere alter ego or business conduit of a
person, or where the corporation is so organized and controlled and its affairs are so
conducted as to make it merely an instrumentality, agency, conduit or adjunct of
another corporation, then its distinct personality may be ignored. In these
circumstances, the courts will treat the corporation as a mere aggrupation of persons
and the liability will directly attach to them. The legal fiction of a separate corporate
personality in those cited instances, for reasons of public policy and in the interest of
justice, will be justifiably set aside.
In our view, however, given the facts and circumstances of this case, the
doctrine of piercing the corporate veil has no relevant application here. Respondent
court erred in permitting the trial court's resort to this doctrine. The rationale behind
piercing a corporation's identity in a given case is to remove the barrier between the
corporation from the persons comprising it to thwart the fraudulent and illegal schemes
of those who use the corporate personality as a shield for undertaking certain
28

proscribed activities. However, in the case at bar, instead of holding certain individuals
or persons responsible for an alleged corporate act, the situation has been reversed. It
is the petitioner as a corporation which is being ordered to answer for the personal
liability of certain individual directors, officers and incorporators concerned. Hence, it
appears to us that the doctrine has been turned upside down because of its erroneous
invocation. Note that according to private respondent Gregorio Manuel his services
were solicited as counsel for members of the Francisco family to represent them in the
intestate proceedings over Benita Trinidad's estate. These estate proceedings did not
involve any business of petitioner.
Furthermore, considering the nature of the legal services involved, whatever
obligation said incorporators, directors and officers of the corporation had incurred, it
was incurred in their personal capacity. When directors and officers of a corporation
are unable to compensate a party for a personal obligation, it is far-fetched to allege
that the corporation is perpetuating fraud or promoting injustice, and be thereby held
liable therefore by piercing its corporate veil. While there are no hard and fast rules on
disregarding separate corporate identity, we must always be mindful of its function and
purpose. A court should be careful in assessing the milieu where the doctrine of
piercing the corporate veil may be applied. Otherwise an injustice, although
unintended, may result from its erroneous application.
The personality of the corporation and those of its incorporators, directors and
officers in their personal capacities ought to be kept separate in this case. The claim
for legal fees against the concerned individual incorporators, officers and directors
could not be properly directed against the corporation without violating basic principles
governing corporations. Moreover, every action including a counterclaim must be
prosecuted or defended in the name of the real party in interest. It is plainly an error to
lay the claim for legal fees of private respondent Gregorio Manuel at the door of
petitioner (FMC) rather than individual members of the Francisco family.
Wherefore, the petition is granted and the assailed decision is reversed insofar
only as it held Francisco Motors Corporation liable for the legal obligation owing to
private respondent Gregorio Manuel; but without prejudice to his filing the proper suit
against the concerned members of the Francisco family in their personal capacity.

29

COMPLEX ELECTRONICS EMPLOYEES ASSOCIATION


Vs.
NLRC, Et Al.
310 SCRA 403 (1999)
FACTS OF THE CASE
Complex informed its Lite-On personnel that a request from Lite on Philippines
to lower their selling price by 10% was not feasible as they were already incurring
losses at the present prices of their products. Under such circumstances, Complex
regretfully informed the employees that it was left with no alternative but to close down
the operations of the Lite-On Line. The Union, however, decried the decision and
voted to declare a strike. Labor unrest within the company eventually ensued.
In the evening of April 6, 1992, the machinery, equipment and materials being
used for production at Complex were pulled-out from the company premises and
transferred to the premises of Ionics Circuit, Inc. at Cabuyao, Laguna. The following
day, a total closure of company operation was effected at Complex.
A complaint was, thereafter, filed with the Labor Arbitration Branch of the NLRC
for unfair labor practice, illegal closure/illegal lockout, money claims for vacation leave,
sick leave, unpaid wages, 13th month pay, damages and attorney's fees. Ionics was
impleaded as a party defendant because the officers and management personnel of
Complex were also holding office at Ionics with Lawrence Qua as the President of
both companies.
Ionics contended that it was an entity separate and distinct from Complex and
had been in existence since July 5, 1984 or eight (8) years before the labor dispute
arose at Complex. Like Complex, it was also engaged in the semi-conductor business
where the machinery, equipment and materials were consigned to them by their
customers. While admitting that Lawrence Qua, the President of Complex was also
the President of Ionics, the latter denied having Qua as their owner since he had no
recorded subscription of P1,200,000.00 in Ionics as claimed by the Union.
ISSUE
Whether there is a clear ground to pierce the veil of corporate fiction and
whether Lawrence Qua should be held liable for the alleged illegal transfer of
machineries of Complex to Ionics.
RULING
It is settled that in the absence of malice or bad faith, a stockholder or an officer
of a corporation cannot be made personally liable for corporate liabilities. The fact that
the pull-out of the machinery, equipment and materials was effected during night-time
is not per se an indicia of bad faith on the part of respondent Qua since he had no
other recourse, and the same was dictated by the prevailing mood of unrest as the
laborers were already vandalizing the equipment, bent on picketing the company
premises and threats to lock out the company officers were being made. Such acts of
respondent qua were, in fact, made pursuant to the demands of Complex's customers
who were already alarmed by the pending labor dispute and imminent strike to be
stage by the laborers, to have their equipment, machinery and materials pull out of
30

Complex. As such, these acts were merely done pursuant to his official functions and
were not, in any way, made with evident bad faith.
As to the juridical personality of the corporations, Ionics may be engaged in the
same business as that of Complex, but this fact alone is not enough reason to pierce
the veil of corporate fiction of the corporation. Well-settled is the rule that a
corporation has a personality separate and distinct from that of its officers and
stockholders. Likewise, mere ownership by a single stockholder or by another
corporation of all or nearly all of the capital stock of a corporation is not of itself
sufficient ground for disregarding the separate corporate personality.

31

LIM
VS.
COURT OF APPEALS
G.R. No. 124715
FACTS OF THE CASE
Private respondents Auto Truck Corporation, Alliance Marketing Corporation,
Speed Distributing, Inc., Active Distributing, Inc. and Action Company are corporations
formed, organized and existing under Philippine laws and which owned real properties
covered under the Torrens system.
On 11 June 1994, Pastor Y. Lim died intestate. Herein petitioner Rufina Lim, as
surviving spouse and duly represented by her nephew George Luy, filed a joint
petition for the administration of the estate of Pastor Y. Lim before the Regional Trial
Court of Quezon City.
Private respondent corporations, whose properties were included in the
inventory of the estate of Pastor Y. Lim, then filed a motion for the lifting of lis
pendens and motion for exclusion of certain properties from the estate of the
decedent.
Rufina alleged that the assets of these corporations were owned wholly by
Pastor; that these corporations themselves are owned by Pastor and they are mere
dummies of Pastor. The corporations filed a motion for exclusion from the estate. They
presented proof (Torrens Titles) showing that the assets of the corporations are in their
respective names and titles. The probate court denied their motion. The Court of
Appeals reversed the decision of the probate court.
ISSUE
Whether the corporations and/or their assets should be included in
the inventory of the estate.
RULING
No.
As regards the assets, the corporations were able to present their respective
Torrens Titles over the disputed assets. It is true that a probate court may pass upon
the question ownership albeit in a provisional manner but still, a Torrens Title cannot
be attacked collaterally in a probate proceeding, it must be attacked directly in a
separate proceeding.
As regards the corporations, to include them in the inventory is tantamount to
the piercing of the veil of corporate fiction because the probate court effectively
adopted the theory of Rufina. This cannot be done. Firstly, the probate court is sitting
in a limited capacity. Secondly, Rufina was not able to present sufficient evidence that
indeed the corporations are mere conduits of Pastor. Mere ownership by a single
stockholder or by another corporation of all or nearly all of the capital stock of a
corporation is not of itself a sufficient reason for disregarding the fiction of separate
corporate personalities. The veil cant be pierced without any showing that indeed the
corporation is being used merely as a dummy. To disregard the separate juridical
personality of a corporation, the wrong-doing must be clearly and convincingly
established. It cannot be presumed.

32

MARUBENI CORPORATION
VS.
LIRAG, 362 SCRA 620 (2001)
G.R.NO. 130998
FACTS OF THE CASE
Petitioner Marubeni Corporation is a foreign corporation organized and existing
under the laws of Japan. It was doing business in the Philippines through its duly
licensed, wholly owned subsidiary companies.
On January 27, 1989, respondent Felix Lirag filed with the Regional Trial Court, Makati
a complaint for specific performance and damages claiming that petitioners owed him
the sum of P6, 000,000.00 representing commission pursuant to an oral consultancy
agreement with Marubeni.
The consultancy agreement was not reduced into writing because of the mutual
trust between Marubeni and the Lirag family. Their close business and personal
relationship dates back to 1960, when respondents family was engaged in the textile
fabric manufacturing business, in which Marubeni supplied the needed machinery,
equipment, spare parts and raw materials.
In compliance with the agreement, respondent Lirag made representations with
various government officials, arranged for meetings and conferences, relayed
pertinent information as well as submitted feasibility studies and project proposals,
including pertinent documents required by petitioners. As petitioners had been
impressed with respondents performance, six (6) additional projects were given to his
group under the same undertaking.
One of the projects handled by respondent Lirag, the Bureau of Post project,
amounting to P100, 000,000.00 was awarded to the Marubeni-Sanritsu
tandem. Despite respondents repeated formal verbal demands for payment of the
agreed consultancy fee, petitioners did not pay. In response to the first demand letter,
petitioners promised to reply within fifteen (15) days, but they did not do so.
On April 29, 1993, the trial court promulgated a decision and ruled that respondent
is entitled to a commission. Respondent was led to believe that there existed an oral
consultancy agreement. Hence, he performed his part of the agreement and helped
petitioners get the project.
The Court of Appeals relied on the doctrine of admission by silence in upholding
the existence of a consultancy agreement, noting that petitioner Tanakas reaction to
respondents September 26, 1988 demand letter was not consistent with their claim
that there was no consultancy agreement. On the contrary, it lent credence to
respondents claim that they had an existing consultancy agreement.
The Court of Appeals observed that if indeed there were no consultancy agreement, it
would have been easy for petitioners to simply deny respondents claim. Yet, they did
not do so. The conglomeration of these circumstances bolstered the existence of the
oral consultancy agreement.
ISSUE
In this appeal, petitioners raise the following issues: (1) whether or not there was a
consultancy agreement between petitioners and respondent; and corollary to this, (2)
whether or not respondent is entitled to receive a commission if there was, in fact, a
consultancy agreement
33

RULING
Wherefore, the petition is granted. The decision of the court of appeals is hereby
set aside. Civil Case No. 89-3037 filed before the Regional Trial Court, Branch 143,
Makati City is hereby dismissed.
No costs
An assiduous scrutiny of the testimonial and documentary evidence extant leads
us to the conclusion that the evidence could not support a solid conclusion that a
consultancy agreement, oral or written, was agreed between petitioners and
respondent. Respondent attempted to fortify his own testimony by presenting several
corroborative witnesses. However, what was apparent in the testimonies of these
witnesses was the fact that they learned about the existence of the consultancy
agreement only because that was what respondent told them.
In civil cases, he who alleges a fact has the burden of proving it; a mere allegation
is not evidence. He must establish his cause by a preponderance of evidence, which
respondent failed to establish in the instant case.
Any agreement entered into because of the actual or supposed influence which the
party has, engaging him to influence executive officials in the discharge of their duties,
which contemplates the use of personal influence and solicitation rather than an
appeal to the judgment of the official on the merits of the object sought is contrary to
public policy. Consequently, the agreement, assuming that the parties agreed to the
consultancy, is null and void as against public policy. Therefore, it is unenforceable
before a court of justice.
In light of the foregoing, we rule that the preponderance of evidence established no
consultancy agreement between petitioners and respondent from which the latter
could anchor his claim for a six percent (6%) consultancy fee on a project that was not
awarded to petitioners.

34

LAND BANK OF THE PHILIPPINES


VS.
COURT OF APPEAL
364 SCRA 375 (2001)
FACTS OF THE CASE
Land Bank of the Philippines extended credit to ECO Management Corporation
amounting to 26 million using the trust fund of Philippine Virginia Tobacco Corp. ECO
Management failed to pay the loan upon maturity so Land Bank filed a complaint for
collection of sum of money against ECO and its President Emmanuel C. Onate before
RTC branch 50.
RTC rendered a judgment in favor of LBP as regards the collection of the sum of
money, but absolved Onate as regards the solidary liability. LBP filed a motion for
reconsideration questioning the dismissal of the complaint against Onate. RTC
rendered an amended decision as regard the total mount but nevertheless absolved
Onate with the liability.
Upon appeal, the Court of Appeal affirmed in toto the decision of the lower court.
LBP filed a motion for reconsideration but to no avail. Hence, they appealed to SC. In
their appeal, LBP averred that there exists a justifiable and substantial ground to
incarcerate Onate and to pierce the doctrine of corporate fiction.
ISSUE
Whether LBPs contention is correct.
RULING
LBPs contention is not correct. The doctrine of corporate fiction may not be
pierced by the mere fact that Onate has the controlling interest when the loan was
contracted with them. It does not suffice that Onate was the president was the
president and chairman of ECO Management Corporation. The act of Onate selling all
his interest of the corporation does not justify fraud on his part. Fraud may not be
presumed, it must be proved by clear and convincing evidence.
A corporation upon coming into existence is invested by law with a personality
separate and distinct from those persons composing it as well as from any legal entity
to which it may be related. By this attribute, a stockholder may not, generally be made
to answer for acts or liabilities of said corporation and vice-versa. This separate and
distinct personality is, however, merely a fiction created by law for convenience and to
promote the ends of justice.

35

PHILIPPINE NATIONAL BANK


Vs.
RITRATTO GROUP INC.
G.R. No. 142616
July 31, 2001

FACTS OF THE CASE


On May 29, 1996, PNB International Finance Ltd. (PNB-IFL) a subsidiary
company of PNB, organized and doing business in Hong Kong, extended a letter of
credit in favor of the respondents secured by real estate mortgages constituted over
four (4) parcels of land in Makati City. Respondents made repayments of the loan
incurred by remitting those amounts to their loan account with PNB-IFL in Hong Kong.
However, when they defaulted, PNB-IFL, through its attorney-in-fact PNB,
notified the respondents of the foreclosure of all the real estate mortgages and that the
properties subject thereof were to be sold at a public auction on May 27, 1999 at the
Makati City Hall.
The respondents filed for a complaint for injunction. PNB filed a motion to
dismiss on the grounds of failure to state a cause of action and the absence of any
privity between the petitioner and respondents.
ISSUE
Whether or not PNB is privy to the loan contract between PNB-IFL and
respondents.
RULING
No. Respondents admit that petitioner is a mere attorney-in-fact for the PNB-IFL
with full power and authority to, inter alia, foreclose on the properties mortgaged to
secure their loan obligations with PNB-IFL. In other words, herein petitioner is an
agent with limited authority and specific duties under a special power of attorney
incorporated in the real estate mortgage. It is not privy to the loan contracts entered
into by respondents and PNB-IFL.
The general rule is that as a legal entity, a corporation has a personality distinct
and separate from its individual stockholders or members, and is not affected by the
personal rights, obligations and transactions of the latter. The mere fact that a
corporation owns all of the stocks of another corporation, taken alone is not sufficient
to justify their being treated as one entity. If used to perform legitimate functions, a
subsidiarys separate existence may be respected, and the liability of the parent
corporation as well as the subsidiary will be confined to those arising in their
respective business. The courts may in the exercise of judicial discretion step in to
prevent the abuses of separate entity privilege and pierce the veil of corporate entity.
In this jurisdiction, we have held that the doctrine of piercing the corporate veil is
an equitable doctrine developed to address situations where the separate corporate
personality of a corporation is abused or used for wrongful purposes. The doctrine
applies when the corporate fiction is used to defeat public convenience, justify wrong,
protect fraud or defend crime, or when it is made as a shield to confuse the legitimate
issues, or where a corporation is the mere alter ego or business conduit of a person,
or where the corporation is so organized and controlled and its affairs are so
conducted as to make it merely an instrumentality, agency, conduit or adjunct of
another corporation.
36

In any case, the parent-subsidiary relationship between PNB and PNB-IFL is not
the significant legal relationship involved in this case since the petitioner was not sued
because it is the parent company of PNB-IFL. Rather, the petitioner was sued
because it acted as an attorney-in-fact of PNB-IFL in initiating the foreclosure
proceedings. A suit against an agent cannot without compelling reasons be
considered a suit against the principal. Under the Rules of Court, every action must be
prosecuted or defended in the name of the real party-in-interest, unless otherwise
authorized by law or these Rules. In mandatory terms, the Rules require that partiesin-interest without whom no final determination can be had, an action shall be joined
either as plaintiffs or defendants. In the case at bar, the injunction suit is directed only
against the agent, not the principal.

37

Silverio
Vs.
Filipino Business Consultants, Inc.
(G.R. No. 143312, Aug. 12, 2005)
FACTS OF THE CASE
1. Petitioner Silverio, Jr. is the President of two corporations namely Esses Devt.
Corp. and Tristar Farms, Inc.
2. The above-mentioned corporations were in possession of the Calatagan
Property and registered in the names of Esses and Tristar.
3. On Sept. 22, 1995, Esses and Tristar executed a Deed of Sale with Assumption
of Mortgage in favor of Filipino Business Consultants, Inc. (FBCI). Esses and
Tristar failed to redeem the Calatagan Property.
4. On May 27, 1997, FBCI filed a Petition for Consolidation of Title of the
Calatagan Property with RTC-Balayan.
5. FBCI obtained a judgment by default. Subsequently, two land titles in the name
of Esses and Tristar were cancelled and new land title was issued in favor of
FBCI.
6. On April 20, 1998, RTC-Balayan issued a writ of possession in FBCIs favor.
The latter then entered the Calatagan Property.
7. When Silverio, Jr., Esses and Tristar learned of the judgment by default and writ
of possession, they filed a petition for relief from judgment and the recall of the
writ of possession. Silverio et. al. alleged that the judgment by default is void
because the RTC-Balayan did not acquire jurisdiction over them as a result of
forged service of summons on them.
8. On May 23, 2000, FBCI filed with RTC-Balayan an Urgent Ex-Parte Motion to
Suspend Enforcement of Writ of Possession. FBCI pointed out that it is now the
new owner of Esses and Tristar having purchased the substantial and
controlling shares of stocks of the two corporations.
ISSUE
Whether FBCIs acquisition of shares of stocks of Esses and Tristar
representing a controlling interest of the two corporations would also give FBCI a
proprietary right over the Calatagan Property owned by both Esses Corp. and Tristar.
RULING
No. FBCIs alleged controlling shareholdings in Esses and Tristar merely
represent a proportionate interest in the properties of the two corporations. Such
controlling shareholdings do not vest FBCI with any legal right or title to any of Esses
and Tristars corporate properties.
A corporation is a juridical person distinct from the members composing it.
Properties registered in the name of the corporation are owned by it as an entity
separate and distinct from its members.

38

JARDINE DAVIES, INC


Vs.
JRB REALTY, INC
G.R. No. 151438
FACTS OF THE CASE
Respondent JRB Realty built Blanco Center and contracted with Aircon &
Refrigeration Industries for the installation of aircon units with Fedders Air Conditions
USA compressors. Due to problems encountered after installation the two parties
agreed to replace the units. Thereafter the respondent learned that Maxim Industrial is
the new and exclusive licensed distributor of Fedders compressor. When Maxim
refused to honor the obligation of Aircon to replace the units JRB Realty filed an action
for specific performance with damages against Aircon. Considering that Aircon was it's
subsidiary, Jardine Davies was also impleaded. The RTC rendered its decision holding
Jardine Davies, Fedders USA and Maxim jointly and severally liable for the grievances
of JRB. Jardine Davies contends that it was not a party to the contract between JRB
and Aircon and that it had a personality distinct and separate from that of Aircon.
However the CA affirmed the ruling of the RTC. Thus this petition.

ISSUE
Whether Jardine Davies should be held liable for the alleged contractual breach of
Aircon solely because latter was formerly Jardine Davies subsidiary
RULING
No. A corporation is an artificial being with a personality separate and distinct from its
stockholders and from any other corporations to which it may be connected. While a
corporation may be allowed to exist solely for a lawful purpose, the law will regard it as
an association of persons or in case of two persons to merge it as one when this
corporate legal entity is used as a cloak for fraud or illegality. This is otherwise known
as the doctrine of piercing the veil of a corporate fiction. A subsidiary has an
independent and separate juridical personality from that of its parent company. Hence
any claim against the latter does not bind the former and vice versa. In applying this
doctrine of piercing the veil of a corporate fiction the following requisites must be
established: 1) control, not merely majority or complete stock control; 2) such control
must have been used by defendant to commit fraud or wrong, or to perpetuate the
violation of a statutory or other positive legal duty or dishonest acts in contravention of
plaintiffs legal rights; and 3) the aforesaid control and breach of duty is the
approximate cause of injury or unjust loss complained of. In this case, Aircon is a
subsidiary of Jardine Davies only because the latter acquired Aircons majority of
capital stock. However it does not exercise complete control over Aircon. No
management agreement exists between Jardine and Aircon. Thus Jardine Davies
should not be held liable being a separate and legal entity from that of Aircon

39

CHINA BANKING CORPORATION


Vs.
DYNE-SEM ELECTRONICS CORPORATION
G.R. No. 149237, June 11, 2006
FACTS OF THE CASE
Dynetics and Elpidio Ong (solidarily liable) obtained a P8,939,000 loan from
petitioner China Banking Corporation as evidenced by promissory notes. The
borrowers failed to pay the obligation. Dynetics has closed its operations and left Ong
as the sole responsible for the loan. Petitioner bank impleaded respondent Dyne-Sem
Electronic Corporation as according to petitioner bank, Dyne-sem is an alter ego of
Dynetics. The bases of the petitioner are as follows: 1) Dynetics and Dyne-Sem are
both engaged in the business of integrated circuits and semi conductor devices; 2)
The factory of Dynetics was used by Dyne-sem as its main office, 3) Dyne-Sem
acquired machineries from Dynetics, 4) Dyne-Sem retained some of the officers of
Dynetics. RTC ruled in favor of respondent to which Court of Appeals affirmed; hence,
this petition
ISSUE
WON Dyne-Sem is an alter-ego of Dynetics that would allow the court to pierce
the formers corporate veil.
RULING
Dyne-sem is not an alter-ego of Dynetics, and therefore, cannot be held liable
for the loan obtained by Dynetics. Petitioner failed to prove that Dyne-Sem was
organized and controlled, and its affairs conducted, in a manner that made it merely
an instrumentality, agency, conduit or adjunct of Dynetics, or that it was established to
defraud Dynetics creditors, including petitioner. The similarity of business of the two
corporations did not warrant a conclusion that respondent was but a conduit of
Dynetics. The acquisition of assets of Dyne-Sem was through a public bidding. What
took place was a sale of the assets of the former to the latter. Merger is legally distinct
from a sale of assets. Thus, where one corporation sells or otherwise transfers all its
assets to another corporation for value, the latter is not, by that fact alone, liable for
the debts and liabilities of the transferor. Even the overlapping of incorporators and
stockholders of two or more corporations will not necessarily lead to such inference
and justify the piercing of the veil of corporate fiction.

40

SPOUSES RAMON NISCHE


VS
EQUITABLE PCI BANK
GR.NO.167434. FEBRUARY 19, 2007
FACTS OF THE CASE
. (Hong Kong), which later renamed PCI Express Padala. Before the merger of
the PCIB and Equitable Bank, petitioner Natividad Nische has a dollar deposit account
with PCIB amounting to U$ 20,500.00. Later, Natividad Nische requested PCIB to
transfer U$ 20,000.00 of her account to PCI Capital. Meanwhile, PCIB and Equitable
Bank were Petitioners Ramon and Natividad Nische (Spouses) executed a Real
Estate Mortgage Contract in flavor of Equitable PCI Bank to secure their obligations
with the respondent bank.
PCI Bank, before it merged to equitable bank owns almost all of the stocks of
PCI-Capital Asia Ltdeventually merged.
For failure to pay their obligations despite several demands, respondent
Equitable PCI Bank as creditor-mortgagee filed a petition for extrajudicial foreclosure
of the properties subject to Real Estate Mortgage, before the Office of the Clerk of
Court, as Ex-officio Sheriff of RTC Makati City.
The spouses opposed the foreclosure, filed a motion for nullity of surety ship
agreement with prayer for injunctive relief against the bank and the Ex-officio Sheriff,
alleging that their obligation should have been offset by legal compensation to the
Extent of their account to the bank, since both parties are Creditors and Debtors with
respect to each other by the virtue of the fact that PCI Capital Asia (now PCI Express
padala) is a subsidiary of Equitable PCI Bank.
ISSUE
Whether or not a subsidiary corporation has a separate and district personality
from its parent company.
RULING
The fact that a corporation owns all of the stocks of another corporation is not
sufficient to justify their being treated as one entity. A corporation has a separate
personality distinct from its stockholder and other corporations to which it may be
connected.
The veil of corporate entity maybe lifted when the corporation is merely an
adjunct, a business conduit or an alter ego of another corporation, or where the
corporation is so organized and controlled and its affairs are so conducted as to make
it merely an instrumentality, agency conduit or adjunct of another corporation; or when
the corporation is used as cloak or cover for fraud or illegality, or to work injustice or
where necessary to achieve equity or for the protection of creditors.
Hence, the petition was denied by the SUPREME COURT.

41

PASRICHA
VS.
DON LUIS DIZON REALTY, INC.
G.R.NO.136409 March 14, 2008
FACTS OF THE CASE
Don Luis Dizon Reality Inc. Herein respondent and petitioners executed two
contracts of Lease involving several units of San Luis Building located at T.M Kalaw
Ermita, Manila owned by respondent being the Lessor.
While the contracts replaced by Roswinda Bautista as the President and
General Manager. At first petitioners continuously paid their rentals to Baustista, but
after few months of payment however, despite repeated demands, petitioners refuses
to continue payment of their rental obligations. Consequently, respondent were in
effect petitioners religiously paid monthly rentals to the respondent Realty thru its
President and General Manager Francis Pacheco. Thereafter, Pacheco was
constrained to refer the matter to its lawyer who in turn made a final demand, but to no
avail, her complaint for ejectment was filed by private respondent through its
representative Ms. Bautista before the METC of Manila.
The METC dismissed the complaint for ejectment due to alleged lack of
authority of Ms. Bautista to sue a behalf at the corporation.
On Appeal, the RTC reversed the METC decision in favor of the
respondent corporation and ordered the petitioner to pay the unpaid rentals and to
vacate the premises, hence petitioners debated the case to the court of appeals a
petitioner for review and certiorari, and the court of appeals affirmed the RTC decision.
A motion for reconsideration was filed but denied by the court of appeals for lack of
merit hence this petition for creational before the SUPREME COURT.
ISSUE
Whether or not Roswinda Bautista, by virtue of her subsequent designation by
the board of directors as the corporations attorney-in-fact had legal capacity to sue in
behalf of the corporation.
RULING
The Supreme Court upholds the capacity of Ms. Bautista to institute the
ejectment case in behalf of the corporation despite lack of proof of authority to
represent it.
A corporation has no powers except those expressly conferred on it by the
Corporation Code and those that are implied from or are incidental to its existence.
And in turn, a corporation exercise said powers thru its board of directors and/or its
duly authorized officers and agents. Thus any person suing on behalf of a corporation
must present proof of such authority.
Although Ms. Bautista initially failed to slow that she had the capacity to sign the
verification and institute the ejectment case on behalf of the corporation, when
confronted with such question, she immediately presented the Secretarys certificate
confirming her authority.
42

There is ample jurisprudence holding that subsequent and substantial


compliance may call for the relaxation of the rules of procedures in the interest of
justice.
Technical and procedural rules are intended to help secure, not suppress the
cause of justice, and deviation from the rigid enforcement of the rules may be allowed
to attain that prime objective.

43

YAMAMOTO
VS.
LEATHER INDUSTRIES AND IKUO NISHINO, INC.,
G.R. No. 15028329.
FACTS OF THE CASE
Nishino seeks to buy out the shares of Yamamoto in NLII (formerly WAKO
Enterprises) established by the latter. Nishinos counsel Atty. Doce presented a
memorandum in relation to the planned takeover of NLII by Nishino. Said
memorandum listed several machines of the corporation, which as indicated in the
document, Yamamoto may take out provided the value of the machines listed is
deducted from his and WAKOs capital contributions which will be paid to him. It is on
this basis that Yamamoto attempted to recover the machines. Having failed to do so,
he filed before RTC Makati a complaint against NLII and Nishino for replevin. RTC
Makati issued the same.
Respondents claim that the machines subject of replevin form part of
Yamamotos capital contributions in consideration of his equity in NLII and should thus
be treated as corporate property. They allege that the Atty. Doces letter was merely a
proposal yet to be authorized by the stockholders and Board of Directors of the NLII.
On the other hand, petitioner Yamamoto urged the Court to pierce the veil of
corporate fiction, arguing that the course of action of NLII depends on what Nishino
decides. It is a mere instrumentality of Nishino and his brother. Also, Yamamoto
alleges that the Company hardly holds board meetings and has an inactive board.
Only Nishino makes the decisions. He owns 70% of the shares of stock of the
corporation, with the additional 20% when Yamamotos shares decreased.
ISSUE
Whether the veil of corporate fiction should be pierced in the case at bar.
RULING
The veil of corporate fiction should not be pierced in this case. Although the veil
of separate corporate personality may be pierced when the corporation is merely an
adjunct, a business conduit or alter ego of a person, the mere ownership by a single
stockholder of even all or nearly all of the capital stocks of a corporation is not by itself
a sufficient ground to disregard the separate corporate personality.
The elements determinative of the applicability of the doctrine of piercing the veil
of corporate fiction are the following: control, not mere majority or complete stock
control, but complete domination so that the corporate entity as to this transaction had
at the time no separate mind, will or existence of its own; control must have been used
by the defendant to commit fraud or wrong in violation of plaintiffs legal rights; and
control and breach of duty must proximately cause the injury or unjust loss complained
of. In this case, there is no showing that Nishino used the separate personality of NLII
to unjustly act or do wrong to Yamamoto in contravention of his legal rights.
The machineries and equipment, which comprised Yamamotos investment in
NLII thus remained part of the capital property of the corporation. The property of a
corporation is not the property of its stockholders or members. Hence, the distribution
of corporate assets and property cannot be made to depend on the whims of its
44

stockholders, officers or directors unless indispensable conditions and procedures for


the protection of corporate creditors are followed, pursuant to the trust fund doctrine.

45

VIRGILIO S. DELIMA
VS.
SUSAN MERCAIDA GOIS
554 SCRA 731 (2008)
FACTS OF THE CASE
A case for illegal dismissal was filed by petitioner Virgilio S. Delima against
Golden Union Aquamarine Corporation (Golden), Prospero Gois and herein
respondent Susan Mercaida Gois. The Labor Arbiter rendered a decision ordering to
pay the back wages, separation pay and other fees amounting to P115, 561.05.
Golden failed to appeal the aforesaid decision; hence, it became final and executor. A
writ of execution was issued and an Isuzu Jeep with plate number PGE-531 was
attached. Thereafter, respondent Gois filed an Affidavit of Third Party Claim claiming
that the attachment of the vehicle was irregular because said vehicle was registered in
her name and not Goldens; and that she was not a party to the illegal dismissal case
filed by Delima against Golden which was denied in an Order issued by the Labor
Arbiter on grounds that respondent was named in the complaint as one of the
respondents; that summons were served upon her and Prospero Gois; that both
verified Goldens Position Paper and alleged therein that they are the respondents;
and that respondent is one of the incorporators/officers of the corporation.
Gois filed an appeal before the NLRC. At the same time, she filed a motion
before the Labor Arbiter to release the motor vehicle after substituting the same with a
cash bond in the amount of P115, 561.05. Meanwhile, the NLRC issued a Resolution
which dismissed respondents appeal for lack of merit. A Motion for Reconsideration
was filed which was also denied. The NLRC Resolution became final and executory;
subsequently, an Entry of Judgment was issued. Because of this, Gois filed a petition
for certiorari alleged that the NLRC committed grave abuse of discretion when it
dismissed her appeal. She claimed that by denying her third-party claim, she was in
effect condemned to pay a judgment debt issued against a corporation of which she is
neither a president nor a majority owner but merely a stockholder. She further argued
that her personality is separate and distinct from that of Golden; thus, the judgment
ordering the corporation to pay the petitioner could not be satisfied out of her personal
assets.
The Court of Appeals annulled and set aside the resolutions of the NLRC.
Hence, this petition.
ISSUE
Whether the NLRC is correct in attaching the subject vehicle owned by Gois to
answer for the liabilities of the corporation.
RULING
No. The Supreme Court held that since the Decision of the Labor Arbiter dated
April 29, 2005 directed only Golden to pay the petitioner the sum of P115,561.05 and
the same was not joint and solidary obligation with Gois, then the latter could not be
held personally liable since Golden has a separate and distinct personality of its own.
It remains undisputed that the subject vehicle was owned by Gois, hence it should not
be attached to answer for the liabilities of the corporation. Unless they have exceeded
their authority, corporate officers are, as a general rule, not personally liable for their
official acts, because a corporation, by legal fiction, has a personality separate and
distinct from its officers, stockholders and members. No evidence was presented to
show that the termination of the petitioner was done with malice or in bad faith for it to
hold the corporate officers, such as Gois, solidarily liable with the corporation.
A corporation has a personality distinct and separate from its individual
stockholders or members and from that of its officers who manage and run its affairs.
46

The rule is that obligations incurred by the corporation, acting through its directors,
officers and employees, are its sole liabilities. Thus, property belonging to a
corporation cannot be attached to satisfy the debt of a stockholder and vice versa, the
latter having only an indirect interest in the assets and business of the former.

47

SEAOIL PETROLEUM CORPORATION


Vs.
AUTOCORP GROUP
G.R. No.164326
FACTS OF THE CASE
Seaoil Petroleum Corporation purchased one unit of ROBEX 200 LC Excavator,
Model 1994 from plaintiff-appellee Autocorp Group. The original cost of the unit
was P2, 500,000.00 but was increased to P3, 112,519.94 because it was paid in 12
monthly installments up to September 30, 1995. The sales agreement was embodied
in the Vehicle Sales Invoice No. A-0209 and Vehicle Sales Confirmation,No.258.
Furthermore, it was agreed that despite delivery of the excavator, ownership thereof
was to remain with Autocorp until the obligation is fully settled. In this light, Seaoils
contractor, Romeo Valera, issued 12 postdated checks. However, Autocorp refused to
accept the checks because they were not under Seaoils name. Hence, Yu, on behalf
of Seaoil, signed and issued 12 postdated checks for P259, 376.62 each with
Autocorp as payee.
The relationship started to turn sour when the first check bounced. However, it
was remedied when Seaoil replaced it with a good check. The second check likewise
was also good when presented for payment. However, the remaining 10 checks were
not honored by the bank since Seaoil requested that payment be stopped. It was
downhill from there on. Despite repeated demands, Seaoil refused to pay the
remaining balance of P2, 593,766.20. Hence, on January 24, 1995, Autocorp filed a
complaint for recovery of personal property with damages and replevin in the Regional
Trial Court of Pasig. The trial court ruled for Autocorp. Hence, this appeal.
ISSUE
Whether the given the facts in evidence, the lower courts should have pierced
the corporate veil.
RULING
It is settled that a corporation has a personality separate and distinct from its
individual stockholders or members, and is not affected by the personal rights,
obligations and transactions of the latter. The corporation may not be held liable for
the obligations of the persons composing it, and neither can its stockholders be held
liable for its obligation.
Of course, this Court has recognized instances when the corporations separate
personality may be disregarded. However, we have also held that the same may only
be done in cases where the corporate vehicle is being used to defeat public
convenience, justify wrong, protect fraud, or defend crime.[23] Moreover, the
wrongdoing must be clearly and convincingly established. It cannot be presumed.
Moreover, Rodriguez, as stockholder and director of Uniline, cannot be held
personally liable for the debts of the corporation, which has a separate legal
personality of its own. While Section 31 of the Corporation Code lays down the
exceptions to the rule, the same does not apply in this case. Section 31 makes a
director personally liable for corporate debts if he willfully and knowingly votes for or
assents to patently unlawful acts of the corporation. Section 31 also makes a director
personally liable if he is guilty of gross negligence or bad faith in directing the affairs of
the corporation. The bad faith or wrongdoing of the director must be established
clearly and convincingly. Bad faith is never presumed.
48

EXCELLENT QUALITY APPAREL INC.


Vs.
WIN MULTI-RICH BUILDERS INC.,
G.R. No. 175048
FACTS OF THE CASE
Petitioner Excellent Quality Apparel, Inc. (petitioner) then represented by Max
L.F. Ying, Vice-President for Productions, and Alfiero R. Orden, Treasurer, entered
into a contract with Multi-Rich Builders (Multi-Rich) represented by Wilson G. Chua
(Chua), its President and General Manager, for the construction of a garment factory
within the Cavite Philippine Economic Zone Authority (CPEZ). Upon completion of the
construction of the factory building, respondent Win Multi-Rich Builders, Inc. (Win) was
incorporated with the Securities and Exchange Commission (SEC) with Chua as its
President and General Manager.
Respondent then filed a complaint for a sum of money against petitioner and Mr.
Ying and also prayed for the issuance of a writ of attachment claiming that Mr. Ying
was about to abscond and that petitioner was about to close. The RTC issued the Writ
of Attachment against the properties of petitioner. Petitioner issued a check to prevent
the Sheriff from taking possession of its properties.
In the hearing, the counsel of Win moved that its name in the case be changed
from "Win Multi-Rich Builders, Inc." to "Multi-Rich Builders, Inc." It was only then that
petitioner apparently became aware of the variance in the name of the plaintiff. In the
Reply filed by petitioner, it moved to dismiss the case since Win was not the contractor
and neither a party to the contract, thus it cannot institute the case. Petitioner obtained
a Certificate of Non-Registration of Corporation/Partnership from the SEC which
certified that the latter did not have any records of a "Multi-Rich Builders, Inc."
Moreover, Win in its Rejoinder did not oppose the allegations in the Reply. Win
admitted that it was only incorporated on 1997 while the construction contract was
executed on 1996. Likewise, it admitted that at the time of execution of the contract,
Multi-Rich was a registered sole proprietorship and was issued a business permit by
the Office of the Mayor of Manila.
Petitioner filed a petition for review on certiorari under Rule 65 before the Court
of Appeals, which questioned the jurisdiction of the RTC and challenged the orders
issued by the lower court. The Court of Appeals rendered its Decision annulling the 2
orders of the RTC and ruled that the RTC had jurisdiction over the case. Petitioner
filed a Motion for Reconsideration which was subsequently denied in a resolution.
Hence, this petition.
ISSUE
Whether or not Win have a legal personality to institute the present case.
RULING
We answer in the negative. Win admitted that the contract was executed
between Multi-Rich and petitioner. It further admitted that Multi-Rich was a sole
proprietorship with a business permit issued by the Office of the Mayor of Manila. A
sole proprietorship is the oldest, simplest, and most prevalent form of business
enterprise. It is an unorganized business owned by one person. The sole proprietor is
personally liable for all the debts and obligations of the business. In the case of
Mangila v. Court of Appeals, we held that in fact, there is no law authorizing sole
proprietorships to file a suit in court.

49

A sole proprietorship does not possess a juridical personality separate and


distinct from the personality of the owner of the enterprise. The law merely recognizes
the existence of a sole proprietorship as a form of business organization conducted for
profit by a single individual and requires its proprietor or owner to secure licenses and
permits, register its business name, and pay taxes to the national government. The
law does not vest a separate legal personality on the sole proprietorship or empower it
to file or defend an action in court.
The original petition was instituted by Win, which is a SEC-registered
corporation. It filed a collection of sum of money suit which involved a construction
contract entered into by petitioner and Multi-Rich, a sole proprietorship. The counsel of
Win wanted to change the name of the plaintiff in the suit to Multi-Rich. The change
cannot be countenanced. The plaintiff in the collection suit is a corporation. The name
cannot be changed to that of a sole proprietorship. Again, a sole proprietorship is not
vested with juridical personality to file or defend an action.
In order for a corporation to be able to file suit and claim the receivables of its
predecessor in business, in this case a sole proprietorship, it must show proof that the
corporation had acquired the assets and liabilities of the sole proprietorship. Win could
have easily presented or attached any document e.g., deed of assignment which will
show whether the assets, liabilities and receivables of Multi-Rich were acquired by
Win. Having been given the opportunity to rebut the allegations made by petitioner,
Win failed to use that opportunity. Thus, we cannot presume that Multi-Rich is the
predecessor-in-business of Win and hold that the latter has standing to institute the
collection suit.
Wherefore, the petition is granted.

50

NATIONAL POWER CORPORATION


VS.
PHILIPP BROTHERS OCEANIC, INC.
369 SCRA 629 (2001)

FACTS OF THE CASE


National Power Corporation (NAPOCOR) issued invitations to bid for the supply
and delivery of 120,000 metric tons of imported coal for its Batangas CoalFired Thermal Power Plant of which Philipp Brothers Oceanic, Inc. (PHIBRO) bidded
and was accepted. On July 10, 1987, PHIBRO told NAPOCOR that disputes might
soon plague Australia that will seriously hamper its ability to supply coal. On July 23 to
July 31, 1987, PHIBRO informed NAPOCOR that unless a "strike-free" clause is
incorporated in the charter party or the contract of carriage, the ship owners are
unwilling to load their cargo. In order to hasten the transfer of coal, they should share
the burden of the "strike-free" clause but NAPOCOR refused. PHIBRO effected its first
shipment only on November 17, 1987 which was supposed to be on the 30th day after
receipt of the letter of credit of which it received on August 6, 1987.
Consequently, In October 1987: NAPOCOR once more advertised for the
delivery of coal to its Calaca thermal plant of which PHIBRO applied but was rejected
since it was not able to satisfy the demand for damages on its delay. PHIBRO filed
for damages in the RTC alleging that the rejection was tainted with malice and bad
faith. After the trial, the trial court rendered a decision in favor of PHIBRO, ordering the
defendant NAPOCOR to reinstate PHIBRO in the defendant National Power
Corporations list of accredited bidders and indemnify the same actual, moral and
exemplary damages. On appeal, the CA affirmed in toto the decision of RTC.
ISSUE
Whether the Trial Court erred in awarding moral damages to PHIBRO.
RULING
The award of moral damages is improper. To reiterate, NAPOCOR did not act
in bad faith. Moreover, moral damages are not, as a general rule, granted to a
corporation. While it is true that besmirched reputation is included in moral damages, it
cannot cause mental anguish to a corporation, unlike in the case of a natural person,
for a corporation has no reputation in the sense that an individual has, and besides, it
is inherently impossible for a corporation to suffer mental anguish. In LBC Express,
Inc. v. Court of Appeals, we ruled:
Moral damages are granted in recompense for physical suffering, mental
anguish, fright, serious anxiety, besmirched reputation, wounded feelings, moral
shock, social humiliation, and similar injury. A corporation, being an artificial person
and having existence only in legal contemplation, has no feelings, no emotions, no
senses; therefore, it cannot experience physical suffering and mental anguish. Mental
suffering can be experienced only by one having a nervous system and it flows from
real ills, sorrows, and griefs of life all of which cannot be suffered by respondent
bank as an artificial person.

51

FILIPINAS BROADCASTING NETWORK


VS.
AGO MEDICAL AND EDUCATIONAL CENTER
G.R. No. 141994

FACTS OF THE CASE


Expos is a radio documentary program hosted by Carmelo Mel Rima
(Rima) and Hermogenes Jun Alegre (Alegre). Expos is aired every morning over
DZRC-AM which is owned by Filipinas Broadcasting Network, Inc. (FBNI). Expos
is heard over Legazpi City, the Albay municipalities and other Bicol areas.
Rima and Alegre exposed various alleged complaints from students, teachers
and parents against Ago Medical College of Medicine and its administrators. Claiming
that the broadcasts were defamatory, AMEC Angelita Ago as Dean of AMECs College
of Medicine filed a complaint for damages against FBNI, Rima and Alegre for the
libelous broadcast that AMEC is a dumping ground, garbage, not merely of moral and
physical misfits.
The trial court rendered a decision finding FBNI and Alegre liable for libel except
Rima. The trial court ruled that Rimas only participation was when he agreed with
Alegre. It is within the bounds of freedom of speech, expression and of the press.
Court of Appeals upheld the trial courts ruling.
ISSUE
Whether the broadcasts are libelous.
Whether AMEC is entitled to moral damages.
RULING
Yes.
There is no question that the broadcasts were made public and imputed to
AMEC defects or circumstances tending to cause it dishonor, discredit and
contempt. Every defamatory imputation is presumed malicious. Rima and Alegre failed
to show adequately their good intention and justifiable motive in airing the supposed
gripes of the students. As hosts of a documentary or public affairs program, Rima and
Alegre should have presented the public issues free from inaccurate and misleading
information. If the comments of Rima and Alegre were based on established facts, as
long as it might reasonably be inferred from the facts, the broadcasts are privileged
and not libelous. However, the comments of Rima and Alegre were not backed up by
facts.
Yes.
FBNI contends that AMEC is not entitled to moral damages because it is a
corporation.
A juridical person is generally not entitled to moral damages because, unlike a
natural person, it cannot experience physical suffering or such sentiments as wounded
feelings, serious anxiety, mental anguish or moral shock. Nevertheless, AMECs claim
for moral damages falls under item 7 of Article 2219 of the Civil Code. This provision
expressly authorizes the recovery of moral damages in cases of libel, slander or any
other form of defamation. Article 2219(7) does not qualify whether the plaintiff is a
natural or juridical person. Therefore, a juridical person such as a corporation can
validly complain for libel or any other form of defamation and claim for moral damages.

52

MERALCO
VS.
TEAM ELECTRONIC CORP
G.R. No. 131723
FACTS OF THE CASE
The law in force at the time material to this controversy was PD 401. It
penalized unauthorized installation of water, electrical, telephone connections and
such acts as the use of tampered electrical meters. PD 401 granted the electrical
companies the right to conduct inspections of electric meters and the criminal
prosecution or erring customers who were found to have tampered with their electrical
meters. It did not provide for more expedient remedies as the charging of differential
billing and immediate disconnection against erring customers. Thus, electric
companies found a creative way of availing themselves of such remedies by inserting
into the service contracts a provision for differential billing with the option of
disconnection upon non-payment by the erring customers. The Court has recognized
the validity of such stipulations. However, recourse to differential billing with
disconnection was subject to the prior requirement of a 48-hour written notice of
disconnection.
MERALCO, in the instant case, resorted to the remedy of disconnection without
prior notice. While it is true that MERALCO sent a demand letter to TEC for the
payment of differential billing, it did not include any notice that the electric supply
would be disconnected. In fine, it abused the remedies granted to it under PD 401 by
outright depriving TEC of electric services without first notifying it of the impending
disconnection.
ISSUE
Is TEC, a corporation not entitled to moral damages?
RULING
SC deems it proper to delete the award of moral damages. TEC's claim was
premised allegedly on the damage to its goodwill and reputation. as a rule, a
corporation is not entitled to moral damages because, not being a natural person, it
cannot experience physical suffering or sentiments like wounded feelings, serious
anxiety, mental anguish, and moral shock. The only EXCEPTION to this rule is when
the corporation has a reputation that is debased, resulting in its humiliation in the
business realm. But in such a case, it is imperative for the claimant to present proof to
justify the award. It is essential to prove the existence of the factual basis of the
damage and its causal relation to petitioner's acts. In the present case, the records are
bereft of any evidence that the name or reputation of TEC/TPC has been debased as
a result of petitioner's act. Besides, the trial court simply awarded moral damages in
the dispositive portion of its decision without stating the basis thereof

53

VILLA ELT TRANSIT


VS.
FERRER
25 SCRA 845 (1968)
FACTS OF THE CASE
Jose Villarama operates a bus line under the name Villa Rey Transit, pursuant
two (2) certificates of convenience issued by Public Service Commission. He sold the
two certificates to PANTRANCO with a restrictive clause that he, the seller, for a
period of 10 years will not apply for any TPU service identical or competing with that of
the buyer.
Three months after the sale, Jose Villarama organized a corporation named
Villa Rey Transit, Inc... In less than a month after its registration with SEC the
corporation bought five (5) certificated of public convenience from Valentin Fernando.
The parties thereafter applied for its approval with the PSC. PSC granted them a
provisional authority to operate during the pendency of their application. However,
before PSC approved their application, the sheriff of manila levied two of the
certificates of public convenience bought by Villarama from Fernando, pursuant to a
civil case in favor of Eusibio Ferrer.
The Sheriff auctioned the two certificates, Ferrer turned out to be the highest bidder
thus the same was issued in his name. Thereafter, Ferrer sold the two certificates to
PANTRANCO and both applied for approval from the PSC. PSC jointly heard the
application of PANTRANCO and Ferrer, and Villarama and Fernando. PSC ordered
that during the pendency of their application, PANTRANCO should provisionally
operate under the two certificates in question. The Corporation appealed the Supreme
Court which rendered judgment ordering that it is the corporation that should operate
under the two certificates.
Aggrieved, PANTRANCO filed a third-party complaint against Jose Villarama alleging
that Villarama and Villa Rey Transit Inc. are one and the same, and that Villarama is
therefore disqualified from operating the two certificates by virtue of the restriction
clause that enjoined him from engaging in the same business as that of PANTRANCO.
ISSUE
Whether Villarama and Villa Rey Transit Inc. are one and the same before the
law.
RULING
Based on the evidence and testimonies of the witnesses the court held that
Villarama and the corporation are one and the same. Villarama co-mingled his funds
with those of the corporation, he uses his personal checks for payment of the
equipments for the corporation, and he advanced and lent funds from the corporation
without a board resolution allowing it. These facts clearly showed that beyond doubt
the corporation is the alter-ego of Villarama. Consequently, the restrictive clause
entered into by Villarama and the PANTRANCO also binds the corporation. A seller
may not use a corporation as a means of evading the obligation of his covenant.
The doctrine that the corporation is a legal entity separate and distinct from its
members and stockholders composing it is recognized and respected in all cases
54

which are within the reason of the law. When the legal fiction is urged as a means of
perpetrating fraud or an illegal act, or as a vehicle for the evasion of an obligation,
circumvention of statutes, achievement or perfection of monopoly, perpetration of a
crime, the veil with which the law covers and isolates the corporation from members or
stockholders composing it will be lifted to allow its consideration merely as an
aggregation of individuals.

55

A.C. Ransom Labor Union-CCLU


Vs.
National Labor Relations Commission,
G.R.No. L-69494 May 29, 1987
FACTS OF THE CASE
In 1961, employees of A.C. Ransom Philippine Corporation (RANSOM) went on
strike. The said strike was lifted however 22 strikers were refused reinstatement. In
1972, the Court of Industrial Relations ordered RANSOM to reinstate the said strikers
with backwages. However RANSOM manifested that it did not have the necessary
funds for the employees earnings. In 1973, RANSOM filed an application for clearance
to cease operations and terminate employment. Their reason for clearance was the
financial difficulties.
It appears that although RANSOM had assumed financial difficulty, its officers
and principal stockholders had organized sometime in 1969, a new corporation, the
Rosario Industrial Corporation (ROSARIO). The closed corporation was engaged in
the same line of business as RANSOM with the same Hernandez family as owners,
the same officers, the same President, the same counsel, using the same equipment,
personnel, business stocks and the same place of business. RANSOM declared that
ROSARIO is a distinct and separate corporation, which was organized long before
these instant cases were decided adversely against RANSOM.
The UNION filed another case against the officers/agents of RANSOM and or their
states for payment of the backwages. The motion was granted by the Labor Arbiter but
was reversed by the NLRC.
ISSUE
Whether or not the ROSARIO and RANSOM are distinct and separate corporate
entities.
RULING
No. Sale of corporate assets to another corporation organized previously by the
same officers as the vendor and engaged in the same line of business, using the
machineries of the vendor in the same factory, is an instance where corporate veil
should be pierced, vis-a-vis, claim of laborers for backwages.
Aggravating RANSOM's clear evasion of payment of its financial obligations is
the organization of a "run-away corporation," ROSARIO, in 1969 at the time the unfair
labor practice case was pending before the CIR by the same persons who were the
officers and stockholders of RANSOM, engaged in the same line of business as
RANSOM, producing the same line of -products, occupying the same compound,
using the same machineries, buildings, laboratory, bodega and sales and accounts
departments used by RANSOM, and which is still in existence. Both corporations were
closed corporations owned and managed by members of the same family. Its
organization proved to be a convenience instrument to avoid payment of backwages
and the reinstatement of the 22 workers. This is another instance where the fiction of
separate and distinct corporate entities should be disregarded.

56

INDINO
VS.
NATIONAL LABORS RELATIONS COMMISSION
178 SCRA 168 (1989)
FACTS OF THE CASE
1. The petitioner, Benjamin Indino, joined the Phil. National Construction Corp.
(PNCC) as a project personnel officer on December 12, 1974.
2. On January 6, 1981, he was transferred to private respondent Dasmarinas
Industrial Steelworks Corp. (DISC), a sister corporation of PNCC.
3. On July 27, 1983, while the petitioner was on a paid vacation leave, he received
a letter memorandum from Roman Lopez, DISC personnel manager, informing
him that his services were no longer needed at the Philphos Project in Leyte.
4. Immediately after receipt of the letter-memorandum the petitioner filed with
NLRC a complaint for illegal dismissal against private respondent.
5. But before judgment could be rendered by NLRC, petitioner Indino and private
respondent DISC have reach an agreement to settle their differences and for the
petitioner to return to work with 50% payment of back wages, salaries and other
allowances.
6. On the basis of that agreement, the petitioner was reinstated on Oct. 1, 1983.
But barely two months after his reinstatement, the petitioner received another
better-memorandum from respondent DISC, again terminating his services.
7. Petitioner refused to accept his termination, he filed a complaint for illegal
dismissal, unpaid wages, moral and exemplary damages, and attorneys fees
against respondent DISC. Later, he amended his complaint and impleaded
PNCC as additional respondent.
8. The Labor Arbiter, to whom the case was assigned, dismissed the petitioners
complaint for lack of merit.
9. The petitioner appealed to the respondent NLRC. The latter, however, finding no
error in the appealed judgment, affirmed the decision of Labor Arbiter dismissing
the case. A motion for reconsideration filed by the petitioner was denied. Hence,
he filed a petition for certiorari under Rule 65 before the SC, alleging inter alia,
that the NLRC committed a grave abuse of discretion amounting to lack or
excess of jurisdiction.
ISSUE
Whether the PNCC, a sister corporation of DISC, with a separate and distinct
personality should also be held liable for the illegal dismissal case.
RULING
Yes. Considering that the petitioner stated his employment originally with the
PNCC, and the amount of his separation benefits only corresponds to the period of
employment with DISC and not with PNCC, the inclusion of PNCC as respondent in
their action is justified and proper.
The so-called separate and distinct personality of PNCC could be validly ignored
inasmuch as it would unjustly prejudice the petitioner vis--vis whatever benefits he
may receive by reason of his illegal dismissal.
It should always be borne in mind that the fiction of law that a corporation, as a
juridical entity, has a distinct and separate personality was designed for convenience
and to serve justice; therefore, it should not be used as a subterfuge to commit
injustice and circumvent labor laws.
57

SHOEMART, INC
Vs.
NATIONAL LABOR RELATIONS COMMISSION
G.R. No. 90795 225 SCRA 311 (1993)
FACTS OF THE CASE
One of the corporations involved is Moris Industries, Inc. (MORIS), a private
corporation engaged in the manufacture of leather products. In 1985 the Moris
Industries Union (UNION) affiliated itself with the Philippine Association of Free Labor
Unions (PAFLU). Thereafter the UNION, through PAFLU, sent a letter to MORIS
informing it of the UNION's existence, and inviting the latter to enter into negotiations
for a collective bargaining agreement (CBA). MORIS's reaction was as swift as it was
unexpected. Within two days, it suddenly closed shop and ceased operations, claiming
that such a closure had become inevitable because of business reverses. Thus the
UNION (PAFLU) filed a complaint for unfair labor practice against MORIS. A week
later, it commenced another case against MORIS, this time for recovery of wage
differentials and other monetary benefits. Shoemart, Inc., the other corporation
involved in these cases, was impleaded by the UNION in both cases, together with the
former's president, Mr. Henry Sy, on the stated theory that Shoemart, Inc.
(SHOEMART) and MORIS were one and the same juridical entity.The labor arbiter
held that both MORIS and SHOEMART "equally liable" to the complaining UNION.
The NLRC affirmed the decision of the labor arbiter. Thus this petition
ISSUE
Whether the veil of corporate fiction may be pierced in this case.
RULING
Yes. Records show the following facts established as evidence by the complainant:
1.

Mr. (Cresencio) Edic testified that he was first employed as sample maker, by
the people who owned SM. His job was to make samples to be displayed on the
window and only those which appealed to the customers were mass produced.
When he was promoted to over-all supervisor, the factory was transferred to its
present location and from then on, this production division was incorporated
separately and has undergone many changes in name, yet all throughout, the
known owners of the factory remain the same;

2.

Incorporation papers of SM Shoe Mart and Moris Manufacturing show (sic) that
except for Elizabeth Sy all other five (5) incorporators and directors of Morris
Industries are major stockholders of SM Shoe Mart as of July 20, 1985;

3.

The SM Shoe Mart is the exclusive buyer of all of Moris' products;

4.

Both are housed in one building and Moris for many years has been using the
payrolls of SM Shoe Mart. SM glibly excuses this fact by alleging that this was
done without its knowledge. We, however, considering the close relationship of
parties, find this incredible.

Indeed Moris Industries was but a conduit of SM Shoe Mart, Inc.," it appearing that the
"payrolls used by the former bear the letterhead of the latter," and that "Moris
Industries is a family corporation of the Sy's,the same family that owns and controls
SM Shoe Mart, Incorporated.
58

Manuel R. Dulay Enterprises, Inc.


Vs.
COURT of Appeals
G.R. No. 91889 August 27, 1993

FACTS OF THE CASE


Manuel Dulay Enterprises Inc is a domestic corporation with family relatives as
its members; Manuel as the president, treasurer, and general manager, Atty. Virgilio
as the vice president, Plaridel Jose as the secretary, Celia Dulay and Linda DulayMendoza as members. The corporation owns a 16-unit apartment in Pasay. Through a
Board Resolution, the corporation sold the property to respondents Veloso with a right
to repurchase. Without the knowledge of Manuel, Veloso mortgaged the apartment to
Torres. Veloso failed to fulfill the obligation which led to the foreclosure of the property
which was subsequently bought by mortgagee Torres as the highest bidder. Veloso
assigned the right to redeem to Manuel as a result of the extra judicial sale. Neither
Manuel nor Veloso was able to redeem the property so Torres applied for the
consolidation of ownership for the said property. Torres also filed for a writ of
possession against Veloso and Manuel. According to the lower court, since the
corporation had not given permission to the sale, the corporation should be included.
The corporation petitioned for the annulment of the sale of the apartment by Manuel to
Veloso arguing that not all its members had consented to the sale of the apartment.
RTC denied the petition of the corporation and ruled in favor of Torres to which CA
affirmed; hence, this petition
ISSUE
WON the respondent court erred in binding the petitioner corporation for the
transaction of Manuel Dulay by selling the apartment to Maria Veloso.
RULING
NO, Court of Appeals did not commit an error in its decision. In the instant case,
petitioner corporation is classified as a close corporation and consequently a board
resolution authorizing the sale or mortgage of the subject property is not necessary to
bind the corporation for the action of its president. At any rate, corporate action taken
at a board meeting without proper call or notice in a close corporation is deemed
ratified by the absent director unless the latter promptly files his written objection with
the secretary of the corporation after having knowledge of the meeting which, in his
case, petitioner Virgilio Dulay failed to do.
The privilege of being treated as an entity distinct and separate from its
stockholder or members is therefore confined to its legitimate uses and is subject to
certain limitations to prevent the commission of fraud or other illegal or unfair act.
When the corporation is used merely as an alter ego or business conduit of a person,
the law will regard the corporation as the act of that person. The Supreme Court had
repeatedly disregarded the separate personality of the corporation where the
corporate entity was used to annul a valid contract executed by one of its members.
Virgilio E. Dulay's protestations of complete innocence to the effect that he
never participated nor was even aware of any meeting or resolution authorizing the
mortgage or sale of the subject premises is difficult to believe. On the contrary, he is
very much privy to the transactions involved. The fact that petitioner Virgilio Dulay
executed an affidavit that he was a signatory witness to the execution of the postdated Deed of Absolute Sale of the subject property in favor of private respondent
Torres indicates that he was aware of the transaction executed between his Manuel
59

and private respondents and had, therefore, adequate knowledge about the sale of the
subject property to private respondents.
Consequently, Petitioner Corporation is liable for the act of Manuel Dulay and
the sale of the subject property to private respondents by Manuel Dulay is valid and
binding.

60

GUATSON INTERNATIONAL TRAVEL & TOUR, INC.


VS.
NATIONAL LABOR RELATIONS COMMISION
GR.NO.100322 MARCH 09, 1994
FACTS OF THE CASE
Petitioners Guatson Travel and Tours Inc., Philippine Integrated Labor
Assistance Corporation (PHILAC) and Mercury Express International Courier Services
Inc.(MEREX) assailed the decision rendered by National Labor Relations Commission
entitled Jolly M. Almoradie vs. Guatson Travel Company, PHILAC and MEREX
denying petitioners Motion for reconsideration and reversing the decision of the
Labor Arbiter, who finds that complainant Jolly Almoradie was not forced to resign in
said three sister companies.
From the records, it appears that Jolly Almoradie was first employed by Mercury
Express International Courier Services Inc. (MEREX) in October. 1983, as a
messenger. When MEREX close its operation, Almordie was absorbed by its sister
company Phil. Integrated Labor Assistance Corporation. (PHILAC), likewise as
messenger.
In September1986, Almoradie was transferred to Guatson Travel and Tours,
allegedly also a sister company of MEREX and PHILAC, as a Liaison officer.
Thereafter, he was promoted to the position of Sales Representative.
On May 1988, Almoradie was reverted to the position of messenger, yet
sometime in September 1988, he was again given the position of Account Executive,
the nature of work is similar to that of a sales representative.
In October 1988, Almondie was allegedly summoned by Henry Ocier, Guatson
Travel and Tours Vice President to his office and there forced by the latter to resign.
Ocier taunted Almoradie with threats that if he will not resign, he will file charges
against him which would adversely affect his chances of getting a job in the future,
hence Almoradie was forced to sign the resignation letter dictated by Mr. Ocier
himself.
Almoradie filed a complaint for illegal dismissal before the Labor Arbiter, which
the latter dismissed. On appeal, the NLRC reversed the decision of the Labor Arbiter,
and grant Almoradie back wages and separation pay equivalent to one-half month for
every year of service for seven (7) years.
ISSUE
Whether or not said sister companies are liable for the separation pay of the
private respondent illegally dismissed, the former having separate and distinct legal
personalities with that of the others.
RULING
The court upheld the decision of the National Labor Relations Commission. The
three companies are owned by one family, such that majority of the officers of the
companies are the same. The companies are located in one building and use the
same messengerial services. Moreover, there was no showing that private respondent
was paid separation pay when he was absorbed by PHILAC upon closure of MEREX;
61

nor was there evidence that he resigned from PHILAC when he transferred to Guatson
International Travel and Tours Inc. Hence, under the doctrine of piercing the veil of
corporate fiction when valid ground exist the legal fiction maybe disregarded, as in
the case of the three companies at bar, they may be regarded as one single entity in
so far a the respondent case in concern.

62

FIRST PHILIPPINE INTERNATIONAL BANK (Formerly Producers Bank of the


Philippines) and MERCURIO RIVERA, petitioners, vs. COURT OF APPEALS,
CARLOS EJERCITO, in substitution of DEMETRIO DEMETRIA, and JOSE
JANOLO, respondents
G.R. No. 115849 (1996)
FACTS OF THE CASE
Nature of the Case: PETITION for review on certiorari of a decision of the Court of
Appeals when it denied the motion for reconsideration. The decision of the lower court
is MODIFIED. It eliminated the damages awarded and reduced the award to Php
75,000 (assessed against defendant bank). In all other aspects the decision was
AFFIRMED.
The disposition portion of the lower court declared the existence of a perfected
contract to buy and sell the 6 parcels of land (Don Jose, Sta. Rosa, Laguna with an
area of 101 hectares) at an agreed price of Php 5.5 Million. The bank was also
ordered to execute a deed of absolute sale and to deliver the copied of the TCTs for it
to be registered under the name of the plaintiffs. An award for moral damages (Php
200,000), exemplary damages (Php 100,000), attorneys fees (Php 400,000), and
moderate damages (Php 20,000) was also ordered. The cost is against the
defendants.
FIRST PHILIPPINE INTERNATIONAL BANK (FPIPB) acquired 6 parcels of land (total
area: 101 hectares) located at Don Jose, Sta. Rosa, Laguna (TCT No. T-106932 and
T-106937). Demetrio Demetria and Jose O. Janolo wanted to purchase the property.
Carlos Ejercito (respondent Ejercito, for brevity) is the assignee of original plaintiffsappellees Demetrio Demetria and Jose Janolo.
Jose Fajardo, legal counsel of Demetria and Janolo sent formal purchase offer
amounting to Php 3.5 Million Pesos. The FPIPB counter-offered asking for a Php 5.5
Million price. The amended offer was 4.2 Million in CASH. No reply was then given by
the bank to this amended offer.
A meeting between plaintiffs and Louis Co (Senior Vice President of FPIPB), together
with Mauricio Rivera (Head-Manager of the Property Management Department of the
Bank) and Fajardo was held to discuss the sale.
The bank was then placed under conservatorship under Leonida T. Encarnacion.
The plaintiff demanded compliance by the bank on the alleged perfected contract of
sale, with the agreed price of Php 5.5 Million Pesos. The bank refused to comply.
ISSUES
In the absence of a formal deed of sale, may commitments given by bank officers in an
exchange of letters and/or in a meeting with the buyers constitute a perfected and
enforceable contract of sale over 101 hectares of land in Sta. Rosa, Laguna? Does the
doctrine of apparent authority apply in this case? If so, may the Central Bankappointed conservator of Producers Bank (now First Philippine International Bank)
repudiate such apparent authority after said contract has been deemed perfected?
During the pendency of a suit for specific performance, does the filing of a derivative
suit by the majority shareholders and directors of the distressed bank to prevent the
enforcement or implementation of the sale violate the ban against forum-shopping?
63

RULING

Actions; Pleadings and Practice; Forum-Shopping; Conflict of Laws; Principle of


Forum Non Conveniens; Forum-shopping originated as a concept in private
international law, where non-resident litigants are given the option to choose the
forum or place wherein to bring their suit for various reasons or excuses,
including to secure procedural advantages, to annoy and harass the defendant,
to avoid overcrowded dockets, or to select a more friendly venue.To begin
with, forum-shopping originated as a concept in private international law, where nonresident litigants are given the option to choose the forum or place wherein to bring
their suit for various reasons or excuses, including to secure procedural advantages,
to annoy and harass the defendant, to avoid overcrowded dockets, or to select a more
friendly venue. To combat these less than honorable excuses, the principle of forum
non conveniens was developed whereby a court, in conflicts of law cases, may refuse
impositions on its jurisdiction where it is not the most convenient or available forum
and the parties are not precluded from seeking remedies elsewhere.
Same; Same; Same; Same; Words and Phrases; Forum Shopping, Explained.
In this light, Blacks Law Dictionary says that forum shopping occurs when a party
attempts to have his action tried in a particular court or jurisdiction where he feels he
will receive the most favorable judgment or verdict. Hence, according to Words and
Phrases, a litigant is open to the charge of forum shopping whenever he chooses a
forum with slight connection to factual circumstances surrounding his suit, and litigants
should be encouraged to attempt to settle their differences without imposing undue
expense and vexatious situations on the courts.
Same; Same; Same; Same; In the Philippines, forum shopping has acquired a
connotation encompassing not only a choice of venues, as it was originally
understood in conflict of laws, but also to a choice of remedies.In the
Philippines, forum shopping has acquired a connotation encompassing not only a
choice of venues, as it was originally understood in conflicts of laws, but also to a
choice of remedies. As to the first (choice of venues), the Rules of Court, for example,
allow a plaintiff to commence personal actions where the defendant or any of the
defendants resides or may be found, or where the plaintiff or any of the plaintiffs
resides, at the election of the plaintiff (Rule 4, Sec. 2[b]). As to remedies, aggrieved
parties, for example, are given a choice of pursuing civil liabilities independently of the
criminal, arising from the same set of facts.
Same; Same; Same; To avoid or minimize this unethical practice of subverting
justice, the Supreme Court promulgated Circular 28-91.What therefore originally
started both in conflicts of laws and in our domestic law as a legitimate device for
solving problems has been abused and mis-used to assure scheming litigants of
dubious reliefs. To avoid or minimize this unethical practice of subverting justice, the
Supreme Court, as already mentioned, promulgated Circular 28-91. And even before
that, the Court had proscribed it in the Interim Rules and Guidelines issued on January
11, 1983 and had struck down in several cases the inveterate use of this insidious
malpractice.
Same; Same; Same; Words and Phrases; There is forum-shopping whenever, as
a result of an adverse opinion in one forum, a party seeks a favorable opinion
64

(other than by appeal or certiorari) in another.When does forum-shopping take


place? There is forum-shopping whenever, as a result of an adverse opinion in one
forum, a party seeks a favorable opinion (other than by appeal or certiorari) in another.
The principle applies not only with respect to suits filed in the courts but also in
connection with litigations commenced in the courts while an administrative
proceeding is pending, as in this case, in order to defeat administrative processes and
in anticipation of an unfavorable administrative ruling and a favorable court ruling. This
is specially so, as in this case, where the court in which the second suit was brought,
has no jurisdiction.
Same; Same; Same; Test to determine whether a party violated the rule against
forum shopping; Forum shopping exists where the elements of litis pendentia
are present or where a final judgment in one case will amount to res judicata in
the other.The test for determining whether a party violated the rule against forum
shopping has been laid down in the 1986 case of Buan vs. Lopez, also by Chief
Justice Narvasa, and that is, forum shopping exists where the elements of litis
pendentia are present or where a final judgment in one case will amount to res
judicata in the other.
Same; Same; Same; Where a litigant (or one representing the same interest or
person) sues the same party against whom another action or actions for the
alleged violation of the same right and the enforcement of the same relief is/are
still pending, the defense of litis pendentia in one case is a bar to the others,
and a final judgment in one would constitute res judicata and thus would cause
the dismissal of the restin either case forum shopping could be cited by the other
party as a ground to ask for summary dismissal of the two (or more) complaints or
petitions.Consequently, where a litigant (or one representing the same interest or
person) sues the same party against whom another action or actions for the alleged
violation of the same right and the enforcement of the same relief is/are still pending,
the defense of litis pendentia in one case is a bar to the others; and, a final judgment
in one would constitute res judicata and thus would cause the dismissal of the rest. In
either case, forum shopping could be cited by the other party as a ground to ask for
summary dismissal of the two (or more) complaints or petitions, and for the imposition
of the other sanctions, which are direct contempt of court, criminal prosecution, and
disciplinary action against the erring lawyer.
Same; Same; Same; There is forum shopping where the stockholders, in a
second case, and in representation of the Bank, seek to accomplish what the
Bank itself failed to do in the original casethe filing by a party of two apparently
different actions, but with the same objective, constitute forum shopping.Very simply
stated, the original complaint in the court a quo which gave rise to the instant petition
was filed by the buyer (herein private respondent and his predecessors-in-interest)
against the seller (herein petitioners) to enforce the alleged perfected sale of real
estate. On the other hand, the complaint in the Second Case seeks to declare such
purported sale involving the same real property as unenforceable as against the
Bank, which is the petitioner herein. In other words, in the Second Case, the majority
stockholders, in representation of the Bank, are seeking to accomplish what the Bank
itself failed to do in the original case in the trial court. In brief, the objective or the relief
being sought, though worded differently, is the same, namely, to enable the petitioner
Bank to escape from the obligation to sell the property to respondent. In Danville
65

Maritime, Inc. vs. Commission on Audit, this Court ruled that the filing by a party of two
apparently different actions, but with the same objective, constituted forum shopping.
Same; Same; Same; Corporations; Words and Phrases; Derivative Suits,
Explained.The allegations of the complaint in the Second Case show that the
stockholders are bringing a derivative suit. In the caption itself, petitioners claim to
have brought suit for and in behalf of the Producers Bank of the Philippines. Indeed,
this is the very essence of a derivative suit: An individual stockholder is permitted to
institute a derivative suit on behalf of the corporation wherein he holds stock in order
to protect or vindicate corporate rights, whenever the officials of the corporation refuse
to sue, or are the ones to be sued or hold the control of the corporation. In such
actions, the suing stockholder is regarded as a nominal party, with the corporation as
the real party in interest. (Gamboa v. Victoriano, 90 SCRA 40, 47 [1979]; italics
supplied).
Same; Same; Same; Same; Piercing the Veil of Corporate Fiction; When the
fiction is urged as a means of perpetrating a fraud or an illegal act or as a
vehicle for the evasion of an existing obligation, the circumvention of statutes,
the achievement or perfection of a monopoly or generally the perpetration of
knavery or crime, the veil with which the law covers and isolates the corporation
from the members or stockholders who compose it will be lifted to allow for its
consideration merely as an aggregation of individuals.Petitioner also tried to
seek refuge in the corporate fiction that the personality of the Bank is separate and
distinct from its shareholders. But the rulings of this Court are consistent: When the
fiction is urged as a means of perpetrating a fraud or an illegal act or as a vehicle for
the evasion of an existing obligation, the circumvention of statutes, the achievement or
perfection of a monopoly or generally the perpetration of knavery or crime, the veil with
which the law covers and isolates the corporation from the members or stockholders
who compose it will be lifted to allow for its consideration merely as an aggregation of
individuals.
Same; Same; Same; Same; Same; The corporate veil cannot be used to shield
an otherwise blatant violation of the prohibition against forum-shopping
shareholders, whether suing as the majority in direct actions or as the minority in a
derivative suit, cannot be allowed to trifle with court processes.In addition to the
many cases where the corporate fiction has been disregarded, we now add the instant
case, and declare herewith that the corporate veil cannot be used to shield an
otherwise blatant violation of the prohibition against forum-shopping. Shareholders,
whether suing as the majority in direct actions or as the minority in a derivative suit,
cannot be allowed to trifle with court processes, particularly where, as in this case, the
corporation itself has not been remiss in vigorously prosecuting or defending corporate
causes and in using and applying remedies available to it. To rule otherwise would be
to encourage corporate litigants to use their shareholders as fronts to circumvent the
stringent rules against forum shopping.
Same; Same; Same; Ultimately, what is truly important to consider in
determining whether forum-shopping exists or not is the vexation caused the
courts and parties-litigant by a party who asks different courts and/or
administrative agencies to rule on the same or related causes and/or to grant
the same or substantially the same reliefs, in the process creating the possibility
of conflicting decisions being rendered by the different fora upon the same
issue.Ultimately, what is truly important to consider in determining whether forum66

shopping exists or not is the vexation caused the courts and parties-litigant by a party
who asks different courts and/or administrative agencies to rule on the same or related
causes and/or to grant the same or substantially the same reliefs, in the process
creating the possibility of conflicting decisions being rendered by the different fora
upon the same issue. In this case, this is exactly the problem: a decision recognizing
the perfection and directing the enforcement of the contract of sale will directly conflict
with a possible decision in the Second Case barring the parties from enforcing or
implementing the said sale. Indeed, a final decision in one would constitute res
judicata in the other.
Contracts; Requisites of a Valid and Perfected Contract.Article 1318 of the Civil
Code enumerates the requisites of a valid and perfected contract as follows: (1)
Consent of the contracting parties; (2) Object certain which is the subject matter of the
contract; (3) Cause of the obligation which is established.
Same; Actions; Appeals; Petition for Review on Certiorari; In a petition under
Rule 45, errors of fact are, as a rule, not reviewable.Petitioners allege that there
is no counter-offer made by the Bank, and any supposed counter-offer which Rivera
(or Co) may have made is unauthorized. Since there was no counter-offer by the
Bank, there was nothing for Ejercito (in substitution of Demetria and Janolo) to
accept. They disputed the factual basis of the respondent Courts findings that there
was an offer made by Janolo for P3.5 million, to which the Bank counter-offered P5.5
million. We have perused the evidence but cannot find fault with the said Courts
findings of fact. Verily, in a petition under Rule 45 such as this, errors of factif there
be anyare, as a rule, not reviewable. The mere fact that respondent Court (and the
trial court as well) chose to believe the evidence presented by respondent more than
that presented by petitioners is not by itself a reversible error. In fact, such findings
merit serious consideration by this Court, particularly where, as in this case, said
courts carefully and meticulously discussed their findings. This is basic.
Same; Corporations; Banks; Agency; Doctrine of Apparent Authority; A
banking corporation is liable to innocent third persons where the representation
is made in the course of its business by an agent acting within the general
scope of his authority even though, in the particular case, the agent is secretly
abusing his authority and attempting to perpetrate a fraud upon his principal or
some other person, for his own ultimate benefit.The authority of a corporate
officer in dealing with third persons may be actual or apparent. The doctrine of
apparent authority, with special reference to banks, was laid out in Prudential Bank
vs. Court of Appeals, where it was held that: Conformably, we have declared in
countless decisions that the principal is liable for obligations contracted by the agent.
The agents apparent representation yields to the principals true representation and
the contract is considered as entered into between the principal and the third person
(citing National Food Authority vs. Intermediate Appellate Court, 184 SCRA 166). A
bank is liable for wrongful acts of its officers done in the interests of the bank or in the
course of dealings of the officers in their representative capacity but not for acts
outside the scope of their authority (9 C.J.S., p. 417). A bank holding out its officers
and agents as worthy of confidence will not be permitted to profit by the frauds they
may thus be enabled to perpetrate in the apparent scope of their employment; nor will
it be permitted to shirk its responsibility for such frauds, even though no benefit may
accrue to the bank therefrom (10 Am Jur 2d, p. 114). Accordingly, a banking
corporation is liable to innocent third persons where the representation is made in the
67

course of its business by an agent acting within the general scope of his authority
even though, in the particular case, the agent is secretly abusing his authority and
attempting to perpetrate a fraud upon his principal or some other person, for his own
ultimate benefit (McIntosh v. Dakota Trust Co., 52 ND 752, 204 NW 818, 40 ALR
1021).
Same; Same; Same; Same; Same; Evidence; Where the issue is apparent
authority, the existence of which is borne out by the Court of Appeals findings,
the evidence of actual authority is immaterial insofar as the liability of a
corporation is concerned.To be sure, petitioners attempted to repudiate Riveras
apparent authority through documents and testimony which seek to establish Riveras
actual authority. These pieces of evidence, however, are inherently weak as they
consist of Riveras self-serving testimony and various inter-office memoranda that
purport to show his limited actual authority, of which private respondent cannot be
charged with knowledge. In any event, since the issue is apparent authority, the
existence of which is borne out by the respondent Courts findings, the evidence of
actual authority is immaterial insofar as the liability of a corporation is concerned.
Same; There is a meeting of the minds where the acceptance of a revived offer
is absolute and unqualified.Hence, assuming arguendo that the counter-offer of
P4.25 million extinguished the offer of P5.5 million, Luis Cos reiteration of the said
P5.5 million price during the September 28, 1987 meeting revived the said offer. And
by virtue of the September 30, 1987 letter accepting this revived offer, there was a
meeting of the minds, as the acceptance in said letter was absolute and unqualified.
Same; Pleadings and Practice; Appeals; Points of law, theories, issues of fact
and arguments not adequately brought to the attention of the trial court need
not be, and ordinarily will not be, considered by a reviewing court, as they
cannot be raised for the first time on appeal.It also bears noting that this issue of
extinguishment of the Banks offer of P5.5 million was raised for the first time on
appeal and should thus be disregarded. This Court in several decisions has
repeatedly adhered to the principle that points of law, theories, issues of fact and
arguments not adequately brought to the attention of the trial court need not be, and
ordinarily will not be, considered by a reviewing court, as they cannot be raised for the
first time on appeal (Santos vs. IAC, No. 74243, November 14, 1986, 145 SCRA 592).
Same; Same; Statute of Frauds; Evidence; Contracts infringing the Statute of
Frauds are ratified by the failure to object to the presentation of oral evidence to
prove the same.But let it be assumed arguendo that the counter-offer during the
meeting on September 28, 1987 did constitute a new offer which was accepted by
Janolo on September 30, 1987. Still, the statute of frauds will not apply by reason of
the failure of petitioners to object to oral testimony proving petitioner Banks counteroffer of P5.5 million. Hence, petitionersby such utter failure to objectare deemed
to have waived any defects of the contract under the statute of frauds, pursuant to
Article 1405 of the Civil Code: Art. 1405. Contracts infringing the Statute of Frauds,
referred to in No. 2 of Article 1403, are ratified by the failure to object to the
presentation of oral evidence to prove the same, or by the acceptance of benefits
under them.
Same; Banks; Bank Conservator; Constitutional Law; Non-Impairment Clause;
The powers granted to the conservator of a bank, enormous and extensive as
they are, cannot extend to the post-facto repudiation of perfected transactions,
68

otherwise they would infringe against the non-impairment clause of the


Constitution.In the third place, while admittedly, the Central Bank law gives vast
and farreaching powers to the conservator of a bank, it must be pointed out that such
powers must be related to the (preservation of) the assets of the bank, (the
reorganization of) the management thereof and (the restoration of) its viability. Such
powers, enormous and extensive as they are, cannot extend to the post-facto
repudiation of perfected transactions, otherwise they would infringe against the nonimpairment clause of the Constitution. If the legislature itself cannot revoke an existing
valid contract, how can it delegate such nonexistent powers to the conservator under
Section 28-A of said law?
Same; Same; Same; Central Bank Law (R.A. 265); Section 28-A of R.A. 265
merely gives the conservator power to revoke contracts that are, under existing
law, deemed to be defectivethe conservator merely takes the place of a banks
board of directors, and what the said board cannot do, the conservator cannot do
either.Obviously, therefore, Section 28-A merely gives the conservator power to
revoke contracts that are, under existing law, deemed to be defectivei.e., void,
voidable, unenforceable or rescissible. Hence, the conservator merely takes the place
of a banks board of directors. What the said board cannot dosuch as repudiating a
contract validly entered into under the doctrine of implied authoritythe conservator
cannot do either. Ineluctably, his power is not unilateral and he cannot simply
repudiate valid obligations of the Bank. His authority would be only to bring court
actions to assail such contractsas he has already done so in the instant case. A
contrary understanding of the law would simply not be permitted by the Constitution.
Neither by common sense. To rule otherwise would be to enable a failing bank to
become solvent, at the expense of third parties, by simply getting the conservator to
unilaterally revoke all previous dealings which had one way or another come to be
considered unfavorable to the Bank, yielding nothing to perfected contractual rights
nor vested interests of the third parties who had dealt with the Bank.
Actions; Appeals; Petitions for Review on Certiorari; In petitions for review under Rule
45, findings of fact by the Court of Appeals are not reviewable by the Supreme
Court.Basic is the doctrine that in petitions for review under Rule 45 of the Rules of
Court, findings of fact by the Court of Appeals are not reviewable by the Supreme
Court.
Same; Evidence; Witnesses; Presumptions; Failure to present a witness who
would have been in the best position to establish a partys thesis gives rise to
the presumption that his testimony would have been adverse if produced.To
become credible and unequivocal, petitioners should have presented then
Conservator Rodolfo Romey to testify on their behalf, as he would have been in the
best position to establish their thesis. Under the rules on evidence, such suppression
gives rise to the presumption that his testimony would have been adverse, if produced.
Same; Same; Conclusions of fact of a trial judgeas affirmed by the Court of
Appealsare conclusive upon the Supreme Court, absent any serious abuse or
evident lack of basis of capriciousness of any kind.The best that can be said in
favor of petitioners on this point is that the factual findings of respondent Court did not
correspond to petitioners claims, but were closer to the evidence as presented in the
trial court by private respondent. But this alone is no reason to reverse or ignore such
factual findings, particularly where, as in this case, the trial court and the appellate
court were in common agreement thereon. Indeed, conclusions of fact of a trial
69

judgeas affirmed by the Court of Appealsare conclusive upon this Court, absent
any serious abuse or evident lack of basis or capriciousness of any kind, because the
trial court is in a better position to observe the demeanor of the witnesses and their
court-room manner as well as to examine the real evidence presented.
DISPOSITION
WHEREFORE, finding no reversible error in the questioned Decision and Resolution,
the Court hereby DENIES the petition. The assailed Decision is AFFIRMED.
Moreover, petitioner Bank is REPRIMANDED for engaging in forum-shopping and
WARNED that a repetition of the same or similar acts will be dealt with more severely.
Costs against petitioners.

70

TOMAS LAO CONSTRUCTION


Vs.
NATIONAL LABOR RELATIONS COMMISSION
G.R. No. 116781

FACTS OF THE CASE


Private respondents individually filed complaints for illegal dismissal against
petitioners with the NLRC. They alleged that they were hired for various periods as
construction workers in different capacities, alternately working for petitioners Tomas
Lao Corporation (TLC), Thomas and James Developers (T&J) and LVM Construction
Corporation (LVM), altogether informally referred to as the Lao Group of Companies,
the 3 entities comprising a business conglomerate exclusively controlled and managed
by members of the Lao family. With this arrangement, workers were transferred
whenever necessary to on-going projects of the same company or of the others, or
were rehired after the completion of the project or project phase to which they were
assigned.
In 1989, Andres Lao, Managing Director of LVM and President of T&J, issued a
memorandum requiring all workers and company personnel to sign employment
contract forms and clearances which were issued on July 1, 1989 but antedated
January 10, 1989. These were to be used allegedly for audit purposes pursuant to a
joint venture agreement between LVM and T&J. The contracts expressly described the
construction workers whose employments were for a definite period, i.e. upon the
expiration of the contract period or the completion of the project for which the workers
was hired.
Except for Gomez, all private respondents refused to sign, contending that this
scheme was designed to downgrade their status from regular employees to mere
project employees. As such, their salaries were withheld. Labor Arbiter Velasquez
dismissed the complaint finding private respondents to be project employees who can
be terminated upon project completion. NLRC reversed the decision, finding private
respondents to be regular employees who were dismissed without just cause and
denied due process. In granting monetary awards to complainants, the NLRC
disregarded the veil of corporate fiction and treated the three corporations as one on
the basis of petitioners admission that the three operated as one, intermingling and
commingling all its resources, including manpower facility.
ISSUE
Whether the NLRC erred when it pierced the veil of corporate personality of
petitioner-corporations.
RULING
The NLRC did not err in disregarding the veil of separate corporate personality
and holding petitioners jointly and severally liable for private respondents back wages
and separation pay. The records disclose that the three corporations were in fact
substantially owned and controlled by members of the Lao family. A majority of the
outstanding shares of stock in LVM and T&J is owned by the Lao family. The Lao
Group of Companies therefore is a closed corporation where the incorporators and
directors belong to a single family. Petitioners are engaged in the same line of
business under one management and use the same equipment including manpower
services. Where it appears that three business enterprises are owned, conducted and
controlled by the same parties, both law and equity will, when necessary to protect the
71

rights of third persons, disregard the legal fiction that the three corporations are
distinct entities and treat them as identical.
Hence, the Court disregards the separate personalities of the three corporations
and at the same time declare the members of the corporations jointly and severally
liable with the corporations for the monetary awards due to private respondents. The
fiction of law that a corporation as a separate juridical entity was envisaged for
convenience and to serve justice; therefore it should not be used as a subterfuge to
commit injustice and circumvent labor laws.

72

CONCEPT BUILDERS, INC.


VS.
NATIONAL LABOR RELATIONS COMMISSION,
257 SCRA 149 (1996)
FACTS OF THE CASE
Concept Builders, Inc., (CBI) a domestic corporation, with principal office at 355
Maysan Road, Valenzuela, Metro Manila, is engaged in the construction business
while Norberto Marabe; Rodolfo Raquel, Cristobal Riego, Manuel Gillego, Palcronio
Giducos, Pedro Aboigar, Norberto Comendador, Rogelio Salut, Emilio Garcia, Jr.,
Mariano Rio, Paulina Basea, Alfredo Albera, Paquito Salut, Domingo Guarino, Romeo
Galve, Dominador Sabina, Felipe Radiana, Gavino Sualibio, Moreno Escares,
Ferdinand Torres, Felipe Basilan, and Ruben Robalos (private respondents) were
employed by said company as laborers, carpenters and riggers. On November 1981,
private respondents were served individual written notices of termination of
employment by CBI, effective on 30 November 1981. It was stated in the individual
notices that their contracts of employment had expired and the project in which they
were hired had been completed. The NLRC found it to be, the fact, however, that at
the time of the termination of private respondents employment, the project in which
they were hired had not yet been finished and completed. CBI had to engage the
services of sub-contractors whose workers performed the functions of private
respondents. Aggrieved, they filed a complaint for illegal dismissal, unfair labor
practice and non-payment of their legal holiday pay, overtime pay and thirteenthmonth pay against CBI. On 19 December 1984, the Labor Arbiter rendered judgment
ordering CBI to reinstate the private respondents and to pay them back wages
equivalent to 1 year or 300 working days. On 27 November 1985, the NLRC dismissed
the motion for reconsideration filed by CBI on the ground that the said decision had
already become final and executory.
On 16 October 1986, the NLRC Research and Information Department made
the finding that private respondents back wages amounted to P199,800.00. On 29
October 1986, the Labor Arbiter issued a writ of execution directing the sheriff to
execute the Decision, dated 19 December 1984. The writ was partially satisfied
through garnishment of sums from CBI's debtor, the Metropolitan Waterworks and
Sewerage Authority, in the amount of P81,385.34. Said amount was turned over to the
cashier of the NLRC. On 1 February 1989, an Alias Writ of Execution was issued by
the Labor Arbiter directing the sheriff to collect from CBI the sum of P117,414.76,
representing the balance of the judgment award, and to reinstate the private
respondents to their former positions. On 13 July 1989, the sheriff issued a report
stating that he tried to serve the alias writ of execution on petitioner through the
security guard on duty but the service was refused on the ground that CBI no longer
occupied the premises. On 26 September 1986, upon motion of private respondents,
the Labor Arbiter issued a second alias writ of execution. The said writ had not been
enforced by the special sheriff because, as stated in his progress report dated 2
November 1989, that all the employees inside CBI's premises claimed that they were
employees of Hydro Pipes Philippines, Inc. (HPPI) and not by CBI; that levy was made
upon personal properties he found in the premises; and that security guards with high
powered guns prevented him from removing the properties he had levied upon. The
said special sheriff recommended that a "break-open order" be issued to enable him to
enter CBI's premises so that he could proceed with the public auction sale of the
73

aforesaid personal properties on 7 November 1989. On 6 November 1989, a certain


Dennis Cuyegkeng filed a third-party claim with the Labor Arbiter alleging that the
properties sought to be levied upon by the sheriff were owned by HPPI, of which he is
the Vice-President. On 23 November 1989, private respondents filed a "Motion for
Issuance of a Break Open Order," alleging that HPPI and CBI were owned by the
same incorporator/stockholders. They also alleged that petitioner temporarily
suspended its business operations in order to evade its legal obligations to them and
that private respondents were willing to post an indemnity bond to answer for any
damages which CBI and HPPI may suffer because of the issuance of the break-open
order. On 2 March 1990, the Labor Arbiter issued an Order which denied private
respondents motion for break-open order. They then appealed to the NLRC. On 23
April 1992, the NLRC set aside the order of the Labor Arbiter, issued a break-open
order and directed private respondents to file a bond. Thereafter, it directed the sheriff
to proceed with the auction sale of the properties already levied upon. It dismissed the
third-party claim for lack of merit. CBI moved for reconsideration but the motion was
denied by the NLRC in a Resolution, dated 3 December 1992. Hence, the petition.
ISSUE
Whether the NLRC was correct in issuing the break-open order to levy the
HPPI properties located at CBI and/or HPPIs premises at 355 Maysan Road,
Valenzuela, Metro Manila.
RULING
Yes, the NLRC was correct in issuing the break-open order. The Supreme Court
dismissed the petition and the assailed resolutions of the NLRC were affirmed.
It is a fundamental principle of corporation law that a corporation is an entity
separate and distinct from its stockholders and from other corporations to which it may
be connected. But, this separate and distinct personality of a corporation is merely a
fiction created by law for convenience and to promote justice. So, when the notion of
separate juridical personality is used to defeat public convenience, justify wrong,
protect fraud or defend crime, or is used as a device to defeat the labor laws, this
separate personality of the corporation may be disregarded or the veil of corporate
fiction pierced. This is true likewise when the corporation is merely an adjunct, a
business conduit or an alter ego of another corporation. The conditions under which
the juridical entity may be disregarded vary according to the peculiar facts and
circumstances of each case. No hard and fast rule can be accurately laid down, but
certainly, there are some probative factors of identity that will justify the application of
the doctrine of piercing the corporate veil, to wit: (1) Stock ownership by one or
common ownership of both corporations; (2) Identity of directors and officers; (3) The
manner of keeping corporate books and records; and (4) Methods of conducting the
business.
The test in determining the applicability of the doctrine of piercing the veil of
corporate fiction is as (1) Control, not mere majority or complete stock control, but
complete domination, not only of finances but of policy and business practice in
respect to the transaction attacked so that the corporate entity as to this transaction
had at the time no separate mind, will or existence of its own; (2) Such control must
have been used by the defendant to commit fraud or wrong, to perpetuate the violation
74

of a statutory or other positive legal duty or dishonest and unjust act in contravention
of plaintiff's legal rights; and (3) The aforesaid control and breach of duty must
proximately cause the injury or unjust loss complained of. The absence of any one of
these elements prevents "piercing the corporate veil." In applying the "instrumentality"
or "alter ego" doctrine, the courts are concerned with reality and not form, with how the
corporation operated and the individual defendant's relationship to that operation.
Thus the question of whether a corporation is a mere alter ego, a mere sheet or paper
corporation, a sham or a subterfuge is purely one of fact. In the case at bar, while CBI
claimed that it ceased its business operations on 29 April 1986, it filed an Information
Sheet with the SEC on 15 May 1987, stating that its office address is at 355 Maysan
Road, Valenzuela, Metro Manila. On the other hand, HPPI, the third-party claimant,
submitted on the same day, a similar information sheet stating that its office address is
at 355 Maysan Road, Valenzuela, Metro Manila. Further, both information sheets were
filed by the same Virgilio O. Casio as the corporate secretary of both corporations.
Both corporations had the same president, the same board of directors, the same
corporate officers, and substantially the same subscribers. From the foregoing, it
appears that, among other things, the CBI and the HPPI shared the same address
and/or premises. Under these circumstances, it cannot be said that the property levied
upon by the sheriff were not of CBI's. Clearly, CBI ceased its business operations in
order to evade the payment to private respondents of back wages and to bar their
reinstatement to their former positions. HPPI is obviously a business conduit of CBI
and its emergence was skillfully orchestrated to avoid the financial liability that already
attached to CBI.

75

REYNOSO IV
Vs.
COURT OF APPEALS
G.R. No.L-116124-25
FACTS OF THE CASE
Reynoso was the branch manager of Commercial Credit Corporation Quezon
City (CCC-QC), a branch of Commercial Credit Corporation (CCC). It was alleged that
Reynoso was opposed to certain questionable commercial practices being facilitated
by CCC which caused its branches, like CCC-QC, to rack up debts. Eventually,
Reynoso withdrew his own funds from CCC-QC. This prompted CCC-QC to file
criminal cases for estafa and qualified theft against Reynoso. The criminal cases were
dismissed and Reynoso was exonerated and at the same time CCC-QC was ordered
to pay Reynosos counterclaims which amounted to millions. A writ of execution was
issued against CCC-QC. The writ was opposed by CCC-QC as it now claims that it
has already closed and that its assets were taken over by the mother company, CCC.
Meanwhile, CCC changed its name to General Credit Corporation (GCC).
Reynoso then filed a petition for an alias writ of execution. GCC opposed the
writ as it argued that it is a separate and distinct corporation from CCC and CCC-QC,
in short, it raises the defense of corporate fiction.
ISSUE
Whether the court should pierce the corporate veil.
RULING
No. The veil of corporate fiction must be pierced. It is obvious that CCCs
change of name to GCC was made in order to avoid liability. CCC-QC willingly closed
down and transferred its assets to CCC and thereafter changed its name to GCC in
order to avoid its responsibilities from its creditors. GCC and CCC are one and the
same; they are engaged in the same line of business and single transaction process,
i.e. finance and investment. When the mother corporation and its subsidiary cease to
act in good faith and honest business judgment, when the corporate device is used by
the parent to avoid its liability for legitimate obligations of the subsidiary, and when the
corporate fiction is used to perpetrate fraud or promote injustice, the law steps in to
remedy the problem. When that happens, the corporate character is not necessarily
abrogated. It continues for legitimate objectives. However, it is pierced in order to
remedy injustice, such as that inflicted in this case.

76

LIPAT
vs.
PACIFIC BANKING CORP.
G.R. No. 142435
FACTS OF THE CASE
Petitioners, the spouses Alfredo Lipat and Estelita Burgos Lipat, owned "Bela's
Export Trading" (BET), a single proprietorship engaged in the manufacture of
garments for domestic and foreign consumption, which was managed by their
daughter Teresita B. Lipat. The spouses also owned the "Mystical Fashions" in the
United States, which sells goods imported from the Philippines through BET, managed
by Mrs. Lipat. In order to facilitate the convenient operation of BET, a special power of
attorney was executed appointing Teresita Lipat to obtain loans and other credit
accommodations from respondent Pacific Banking Corporation (Pacific Bank) and to
execute mortgage contracts on properties owned or co-owned by her as security for
the obligations. By virtue of the special power of attorney, a loan was secured for and
in behalf of Mrs. Lipat and BET, a Real Estate Mortgage was executed over their
property.
BET was then incorporated into a family corporation named Bela's Export
Corporation (BEC) engaged in the business of manufacturing and exportation of all
kinds of garments and utilized the same machineries and equipment previously used
by BET. Eventually, the loan was later restructured in the name of BEC and
subsequent loans were obtained with the corresponding promissory notes duly
executed by Teresita on behalf of the corporation. BEC defaulted in payments when it
became due and demandable. Consequently, the real estate mortgage was foreclosed
and was sold at public auction to respondent Eugenio D. Trinidad as the highest
bidder.
The spouses Lipat filed a complaint alleging, among others, that the promissory
notes, trust receipt, and export bills were all ultra vires acts of Teresita as they were
executed without the requisite board resolution of the Board of Directors of BEC. They
also averred that assuming said acts were valid and binding on BEC, the same were
the corporation's sole obligation, it having a personality distinct and separate from the
spouses.
The trial court ruled that there was convincing and conclusive evidence proving
that BEC was a family corporation of the Lipats. As such, it was a mere extension of
petitioners' personality and business and a mere alter ego or business conduit of the
Lipats established for their own benefit. The Lipats timely appealed which however,
was dismissed by the appellate court for lack of merit. Hence, this petition.
ISSUE
Whether or not the doctrine of piercing the veil of corporate fiction is applicable
in this case.
RULING
Petitioners' contentions fail to persuade this Court.
A careful reading of the judgment of the RTC and the resolution of the appellate
court show that in finding petitioners' mortgaged property liable for the obligations of
BEC, both courts below relied upon the alter ego doctrine or instrumentality rule, rather
than fraud in piercing the veil of corporate fiction. When the corporation is the mere
alter ego or business conduit of a person, the separate personality of the corporation
may be disregarded. This is commonly referred to as the "instrumentality rule" or the
77

alter ego doctrine, which the courts have applied in disregarding the separate juridical
personality of corporations.
We find that the evidence on record demolishes, rather than buttresses,
petitioners' contention that BET and BEC are separate business entities. Note that
Estelita Lipat admitted that she and her husband, Alfredo, were the owners of BET
and were two of the incorporators and majority stockholders of BEC. It is also
undisputed that Estelita Lipat executed a special power of attorney in favor of her
daughter, Teresita, to obtain loans and credit lines from Pacific Bank on her behalf.
Incidentally, Teresita was designated as executive-vice president and general
manager of both BET and BEC, respectively. We note further that: (1) Estelita and
Alfredo Lipat are the owners and majority shareholders of BET and BEC, respectively;
(2) both firms were managed by their daughter, Teresita; (3) both firms were engaged
in the garment business, supplying products to "Mystical Fashion," a U.S. firm
established by Estelita Lipat; (4) both firms held office in the same building owned by
the Lipats; (5) BEC is a family corporation with the Lipats as its majority stockholders;
(6) the business operations of the BEC were so merged with those of Mrs. Lipat such
that they were practically indistinguishable; (7) the corporate funds were held by
Estelita Lipat and the corporation itself had no visible assets; (8) the board of directors
of BEC was composed of the Burgos and Lipat family members; (9) Estelita had full
control over the activities of and decided business matters of the corporation; and that
(10) Estelita Lipat had benefited from the loans secured from Pacific Bank to finance
her business abroad and from the export bills secured by BEC for the account of
"Mystical Fashion." It could not have been coincidental that BET and BEC are so
intertwined with each other in terms of ownership, business purpose, and
management. Apparently, BET and BEC are one and the same and the latter is a
conduit of and merely succeeded the former. Petitioners' attempt to isolate themselves
from and hide behind the corporate personality of BEC so as to evade their liabilities to
Pacific Bank is precisely what the classical doctrine of piercing the veil of corporate
entity seeks to prevent and remedy. In our view, BEC is a mere continuation and
successor of BET and petitioners cannot evade their obligations in the mortgage
contract secured under the name of BEC on the pretext that it was signed for the
benefit and under the name of BET.
We are thus constrained to rule that the Court of Appeals did not err when it
applied the instrumentality doctrine in piercing the corporate veil of BEC.
Wherefore, the petition is denied.

78

MAVEST (U.S.A.), INC.


Vs.
SAMPAGUITA GARMENT CORPORATION
G.R. No. 127454 September 21, 2005
FACTS OF THE CASE
Petitioner MAVEST (U.S.A.), Inc. is a corporation duly organized and
existing under the laws of the United States of America but registered with the
Philippine Board of Investments while co-petitioner MAVEST Manila Liaison Office is
MAVEST U.S.A.s representative in the Philippines. On the other hand,
respondent Sampaguita Garment Corporation (Sampaguita) is a domestic corporation
engaged in the business of manufacturing and exporting garments. Sometime in July
and August 1989, petitioners Mavest U.S.A. and Mavest Manila Liaison Office entered
into a series of transactions with Sampaguita, whereby the former would furnish from
abroad raw materials to be manufactured by the latter into finished products, for
shipment to petitioners foreign buyers, Sears Roebuck and JC Penney. Each
transaction was embodied in a purchase order [PO] specifying the style and
description as well as the quantity, mode and date of delivery.
The orders of Sears Roebuck were duly paid in full by way of letter of credit.
The JC Penney orders consisting of 8,000 pcs. Cotton Woven Pants for a total of
$29,200.00 were not covered by a letter of credit. Despite shipment and receipt by JC
Penney of said orders, no payment was made, thus prompting [respondent] to send
demand letters which remained unheeded.
On April 27, 1990, respondent filed a complaint for collection of a sum of money
amounting to US$29,200.00 with damages before the Regional Trial Court at Makati
City against the herein petitioners and two (2) others, namely, MAVEST International
Co., LTD and Patrick Wang, former General Manager of MLO.
In their Answer with Counterclaim, petitioners and their two co-defendants
countered that plaintiff (Sampaguita) has already been paid by virtue of legal
compensation, and that it is plaintiff which owes defendants US5, 799.57 due to the
damages and losses it incurred as a result of the breaches committed in the previous
shipments to Sears Roebuck. The damages and losses refer to: i) failure to observe
specifications and quantity requirements; ii) delay in shipping out the garments; iii)
over declaration of value in Style No. 33303; iv) shortshipment of garments; v) failure
to return raw materials for the unshipped garments, amounting to US$34,999.57.
Moreover, petitioners and their co-defendants alleged that they also suffered losses on
account of delays in the JC Penney shipments.
After a protracted trial that lasted for four (4) years, the trial court rendered
judgment in favor of herein respondent Sampaguita, as plaintiff and against the
petitioners and their co-defendant Mavest. Therefrom, petitioners and their codefendants (Mavest) a quo appealed to the Court of Appeals (CA). And, in a decision
dated 10 December 1996, the appellate court modified that of the trial court in the
sense that petitioners co-defendants were in effect released from any liability and the
award of attorneys fees and costs of suit deleted. Petitioner submits that Mavest
Manila Liaison Office (MLO), being merely an agent of Mavest U.S.A, should not be
held solidarily liable with the principal.
79

ISSUE
Whether Mavest Liason Office may be held solidarily liable with Mavest USA.
RULING
Petitioners were two (2) of the original four (4) defendants impleaded in the
basic complaint, the other two (2) being Mavest International Co., Ltd. (MICL), a firm
organized under the laws of Taiwan, and Mr. Patrick Wang, a former manager of MICL
and MLO. Both MICL and Mr. Wang, while adjudged liable in solidum with the
petitioners by the trial court, were eventually absolved from any liability by the Court of
Appeals.
As it were, Mavest U.S.A. appears to have constituted MLO as its representative
and its fully subsidized extension office in the Philippines. As such, MLO can be
charged for the liabilities incurred by Mavest U.S.A. in the country. And if MLO can be
so charged, there is no rhyme or reason why it cannot be adjudged, as did the
appellate court, as solidarily liable with head office, Mavest U.S.A.

80

PAMPLONA PLANTATION COMPANY, INC.


Vs.
NLRC
G.R. No. 159121
FACTS OF THE CASE
Petitioner Pamplona Plantations Company, Inc. (company for brevity) was
organized for the purpose of taking over the operations of the coconut and sugar
plantation of Hacienda Pamplona located in Pamplona, Negros Oriental. It appears
that Hacienda Pamplona was formerly owned by a certain Mr. Bower who had in his
employ several agricultural workers. When the company took over the operation of
Hacienda Pamplona in 1993, it did not absorb all the workers of Hacienda Pamplona.
Some, however, were hired by the company during harvest season as coconut
hookers or sakador, coconut filers, coconut haulers, coconut scoopers or lugiteros,
and charcoal makers.
Sometime in 1995, Pamplona Plantation Leisure Corporation was established
for the purpose of engaging in the business of operating tourist resorts, hotels, and
inns, with complementary facilities, such as restaurants, bars, boutiques, service
shops, entertainment, golf courses, tennis courts, and other land and aquatic sports
and leisure facilities.
On 15 December 1996, the Pamplona Plantation Labor Independent Union
(PAPLIU) conducted an organizational meeting wherein several [respondents] who are
either union members or officers participated in said meeting.
Upon learning that some of the [respondents] attended the said meeting,
[Petitioner] Jose Luis Bondoc, manager of the company, did not allow [respondents] to
work anymore in the plantation.
Thereafter, on various dates, [respondents] filed their respective complaints with
the NLRC, Sub-Regional Arbitration Branch No. VII, Dumaguete City against
[petitioners] for unfair labor practice, illegal dismissal, underpayment, overtime pay,
premium pay for rest day and holidays, service incentive leave pay, damages,
attorneys fees and 13th month pay.
On 09 October 1997, [respondent] Carlito Tinghil amended his complaint to
implead Pamplona Plantation Leisure Corporation.
On 31 August 1998, Labor Arbiter Jose G. Gutierrez rendered a decision finding
[respondents], except Rufino Bacubac, Antonio Caolas and Felix Torres who were
complainants in another case, to be entitled to separation pay.
Petitioners appealed the Labor Arbiters decision to [the] NLRC. In the assailed
decision dated 19 July 2000, the NLRCs Fourth Division reversed the Labor Arbiter,
ruling that [respondents], except Carlito Tinghil, failed to implead Pamplona Plantation
Leisure Corporation, an indispensable party and that there exist no employeremployee relation between the parties.
The Court of Appeals held that respondents were employees of petitionercompany.

81

ISSUE
Whether the Court of Appeals has decided in a way not in accord with law in not
dismissing the respondents complaint for failure to implead Pamplona Plantation
Leisure Corp., which is an indispensable party to this case.
RULING
No.
The legal fiction of separate corporate entities cannot be invoked to further an
end subversive of justice. The principle requiring the piercing of the corporate veil
mandates the courts to see through the protective should that distinguishes one
corporation from seemingly separate one.
In the present case, the corporations have basically the same incorporators and
directors and are headed by the same official. Both use only one office and one payroll
and are under one management. In their individual affidavits, respondents allege that
they worked under the supervision and control of Petitioner Bondoc -- the common
managing director of both the petitioner-company and the leisure corporation. Some
of the laborers of the plantation also work in the golf course. Thus, the attempt to
make the two corporations appear as two separate entities, insofar as the workers are
concerned, should be viewed as a devious but obvious means to defeat the ends of
the law. Such a ploy should not be permitted to cloud the truth and perpetrate an
injustice.

82

TIMES TRANSPORTATION COMPANY, INC.


VS.
SANTOS SOTELO, ET AL.
G.R.NO. 163786
FACTS OF THE CASE
Times Transportation Company, Inc. (Times) is a corporation engaged in the
business of land transportation. Times Employees Union (TEU) was formed and
issued a certificate of union registration. Times challenged the legitimacy of TEU by
filing a petition for the cancellation of its union registration. TEU held a strike in
response to Times alleged attempt to form a rival union and its dismissal of the
employees identified to be active union members. The Labor Secretary assumed
jurisdiction over the case and referred the matter to the NLRC for compulsory
arbitration. A return-to-work order was likewise issued. In a certification election, TEU
was certified as the sole and exclusive collective bargaining agent in Times.
Consequently, TEUs president wrote the management of Times and requested for
collective bargaining. Times refused. TEU filed a Notice of Strike. Another
conciliation/mediation proceeding was conducted for the purpose of settling the
brewing dispute.
Times management implemented a retrenchment program and notices of
retrenchment were sent to some of its employees. TEU held a strike vote on grounds
of unfair labor practice on the part of Times. For alleged participation in an illegal
strike, Times terminated all the 123 striking employees. The DOLE Secretary issued
the second return-to-work order certifying the dispute to the NLRC. While the strike
was ended, the employees were no longer admitted back to work. Mencorp Transport
Systems, Inc. (Mencorp) had acquired ownership over Times Certificates of Public
Convenience and a number of its bus units by virtue of several deeds of sale. Mencorp
is controlled and operated by Mrs. Virginia Mendoza, daughter of Santiago Rondaris,
the majority stockholder of Times. Meanwhile, the NLRC rendered a decision declaring
the first strike LEGAL and the second ILLEGAL. Times and TEU both appealed the
decision of the NLRC, which CA affirmed. Upon denial of its motion for
reconsideration, Times filed a petition for review on certiorari.
After the closure of Times, the retrenched employees filed cases for illegal
dismissal, money claims and unfair labor practices against Times before the Regional
Arbitration Branch in San Fernando City, La Union. The employees withdrew their
complaints with leave of court and filed a new set of cases before the National Capital
Region Arbitration Branch, impleading Mencorp and the Spouses Mendoza. Times
sought the dismissal of these cases on the ground of lit is pendencia and forum
shopping.
The Labor Arbiter ruled that the dismissals of complainants Times, effected,
participated in, authorized or ratified by Santiago Rondaris constituted the prohibited
act of unfair labor practice and hence, illegal and that the sale of said respondent
company to respondents Mencorp Transport Systems Company (sic),Inc. and/or
Virginia Mendoza and Reynaldo Mendoza was simulated and/or effected in badfaith.
Times, Mencorp and the Spouses Mendoza submitted their respective memorandum
of appeal to the NLRC. NLRC rendered its decision remanding the records of the
consolidated cases to the Arbitration Branch of origin for disposition and for the
conduct of appropriate proceedings. NLRC denied the Motion for Reconsideration.
Thus, the employees appealed to the CA by way of a petition for certiorari, which
granted the petition and set aside the decision of the N LRC. Times, Mencorp and the
Spouses Mendoza filed Motions for Reconsideration, which were denied. Hence, this
petition for review on certiorari.

83

ISSUE
Whether or not piercing the corporate veil in this case was proper.
RULING
Yes. We have held that piercing the corporate veil is warranted only in cases
when the separate legal entity is used to defeat public convenience, justify wrong,
protect fraud, or defend crime, such that in the case of two corporations, the law will
regard the corporations as merged into one. It may be allowed only if the following
elements concur: (1) controlnot mere stock control, but complete dominationnot
only of finances, but of policy and business practice in respect to the transaction
attacked; (2) such control must have been used to commit a fraud or a wrong to
perpetuate the violation of a statutory or other positive legal duty, or a dishonest andan
unjust act in contravention of a legal right; and (3) the said control and breach of duty
musthave proximately caused the injury or unjust loss complained of.
In this case, the sale was transferred to a corporation controlled by V. Mendoza,
the daughter of S. Rondaris of Times where she is/was also a director. All of the
stockholders/incorporators of Mencorp are all relatives of S. Rondaris. The timing of
the sale evidently was to negate the employees/complainants/members right to
organization as it was effected when their union (TEU) was just organized/requesting
Times to bargain. Mencorp never obtained a franchise since its supposed
incorporation but at present, all the buses of Times are already being run/operated by
Mencorp, the franchise of Times having been transferred to it. The sale of Times
franchise as well as most of its bus units to a company owned by Rondaris daughter
and family members, right in the middle of a labor dispute, is highly suspicious. It is
evident that the transaction was made in order to remove Times remaining assets
from the reach of any judgment that may be rendered in the unfair labor practice cases
filed
against
it.
The
petition
was
DENIED.

84

APEX MINING INCORPORATED


VS.
COURT OF APPEALS
G.R. No. 133750. November 29, 1999
FACTS OF THE CASE
The case involves the Diwalwal Gold Rush Area (Diwalwal), a rich tract of
mineral land located inside the Agusan-Davao-Surigao Forest Reserve in Davao
del Norte and Davao Oriental. Since The early1980s, Diwalwal has been stormed
by conflicts brought about by numerous mining claims over it. On March10, 1986,
Marcopper Mining Corporation (MMC) was granted an Exploration Permit(EP 133)
by the Bureau of Mines and Geo-Sciences (BMG). A long battle ensued between
Apex and MMC with the latter seeking the cancellation of the mining claims of Apex
on the ground that such mining claims were within a forest reservation (AgusanDavao-Surigao Forest Reserve) and thus theacquisition on mining rights should
have been through an application for a permit to prospect with theBFD and
not through registration of a DOL with the BMG. When it reached the SC in 1991,
the Court ruled against Apex holding that the area is a forest reserve and thus it
should have applied for a permit to prospect with the BFD. On February 16 1994,
MMC assigned all its rights to EP 133 to Southeast Mindanao Gold Mining
Corporation (SEM), a domestic corporation which is alleged to be a 100%-owned
subsidiary of MMC. Subsequently, BMG registered SEMs Mineral Production
Sharing Agreement (MPSA) application and the Deed of Assignment. Several
oppositions were filed. The Panel of Arbitrators created by the DENR upheld the
validity of EP 133. During the pendency of the case, DENR AO No. 2002-18 was
issued declaring an emergency situation in the Diwalwal Gold Rush Area and
ordering the stoppage of all mining operations therein.
ISSUES
1. W/N EP 133 and its subsequent transfer to SEM are valid.
2.W/N the DENR Secretary has authority to issue DAO 66 declaring 729 hectares
of the areas covered by the Agusan-Davao-Surigao Forest Reserveas non-forest
lands and open to small-scale mining purposes.
3.Who (among petitioners Apex and Balite)has priority right over Diwalwal?
RULING
1. INVALID. One of the terms and Conditions of EP 133 is: That this permit shall
be for the exclusive use and benefit of the permittee or his duly authorized agents
85

and shall be used for mineral exploration purposes only and for no other purpose.
while it may be true that SEM is a100% subsidiary Corporation of MMC, there is no
showing that the former is the duly authorized agent of the latter. As such, the
assignment is null and void as it directly contravenes the terms and conditions of
the grant of EP 133.
a. The Deed of Assignment was a total abdication of MMCs rights over the permit.
It is not a mere grant of authority to SEM as agent.
b. Reason for the stipulation. Exploration permits are strictly granted to entities or
individuals possessing the resources and capability to undertake mining operations.
Without such condition, non-qualified entities or individuals could circumvent the
strict requirements under the law by the simple expediency of acquiring the permit
from the original permittee.
c. Separate personality. The fact that SEM is a 100% subsidiary of MMC does not
automatically make it an agent of MMC. A corporation is an artificial being invested
by law with a personality separate and distinct from persons composing it as well as
from that of another legal entity to which it may be related. Absent any clear proof
to the contrary, SEM is a separate and distinct entity from MMC.
d. Doctrine of piercing the corporate veil inapplicable. Only in cases where the
corporate fiction was used as a shield for fraud, illegality or inequity may the veil be
pierced and removed. The doctrine of piercing the corporate veil cannot therefore
be used as a vehicle to commit prohibited acts. The assignment of the permit in
favor of SEM is utilized to circumvent the condition of non-transferability of the
exploration permit. To allow SEM to avail itself of this doctrine and to approve the
validity of the assignment is tantamount to sanctioning an illegal act which is what
the doctrine precisely seeks to forestall.
e. PD 463 requires approval of Secretary of DENR. Also, PD 463 (Mineral
Resources Development Decree), which is the governing law when the assignment
was executed, explicitly requires that the transfer or assignment of mining rights,
including the right to explore a mining area, must be with the prior approval of the
Secretary of DENR. Such is not present in this case.
f. EP 133 expired by non-renewal. Although EP 133 was extended for 12 months
until July 6,1994,MMC never renewed its permit prior and after its expiration. With
the expiration of EP 133 on July 6, 1994,MMC lost any right to the Diwalwal Gold
Rush Area. SEM, on the other hand, has not acquired any right to the said area
because the transfer of EP 133 in its favor is invalid. Hence, both MMC and SEM
have not acquired any vested right over the area covered by EP 133.
86

2. NO. The DENR Secretary has no power to convert forest reserves into nonforest reserves. Such power is vested with the President. The DENR Secretary may
only recommend to the President which forest reservations are to be withdrawn
from the coverage thereof. Thus, DAO No. 66 is null and void for having been
issued in excess of the DENR Secretarys authority.

3. (Since its been held that neither MMC nor SEM has any right over Diwalwal, it is
thus necessary to make a determination of the existing right of the remaining
claimants, petitioners Apex and Balite, in the dispute.)The issue on who has priority
right over Diwalwal is deemed overtaken by the issuance of Proclamation 297and
DAO No. 2002-18, both being constitutionally-sanctioned acts of the Executive
Branch. Mining operations in the Diwalwal Mineral Reservation are now, therefore,
within the full control of the State through the executive branch. Pursuant to Sec. 5
of RA 7942, the State can either: (1) directly undertake the exploration,
development and utilization of the area or (2) opt to award mining operations in the
mineral reservation to private entities including petitioners Apex and Balite, if
it wishes. The exercise of this prerogative lies with the Executive Department over
which courts will not interfere.

87

GENERAL CREDIT CORPORATION


Vs.
ALSONS DEVELOPMENT and INVESTMENT CORPORATION and CCC
EQUITY CORPORATION
G.R. No. 154975, January 29, 2007
FACTS OF THE CASE
General Credit Corporation (GCC), then known as Commercial Credit
Corporation, established CCC franchise companies in different parts of the
country. In 1994, CCC Equity Corporation (EQUITY) was organized by GCC for
the purpose of taking over the operations and management of the various
franchise companies. Alsons Development and Investment Corporation
(ALSONS) and the Alcantara family, each owned shares in the aforesaid GCC
franchise companies.
In 1980, ALSONS and the Alcantara family sold their shareholdings to
EQUITY for a consideration of Two Million. In turn, EQUITY issued ALSONS et
al., a "bearer" promissory note. Later on, the Alcantara family assigned its rights
and interests on the note over to ALSONS. Before that assignment, Letters of
Demand for interest payment were already sent to EQUITY but its president
pleaded inability to pay as EQUITY no longer have assets or property to settle its
obligation nor being extended financial support by GCC.
Thus ALSONS filed a complaint against EQUITY and GCC. EQUITY
answered with a cross-claim against GCC. GCC answered the Cross-claim,
stressing that it is a distinct and separate entity from EQUITY. RTC rendered
judgment in favor of ALSONS finding that EQUITY was but an instrumentality or
adjunct of GCC, CA affirmed the decision. Hence this petition by GCC.
ISSUE
Whether or not EQUITY and GCC are distinct and separate corporate
entities.
RULING
No. A corporation is an artificial being vested by law with a personality
distinct and separate from those of the persons composing it as well as from that
of any other entity to which it may be related. The first consequence of the
doctrine of legal entity of the separate personality of the corporation is that a
corporation may not be made to answer for acts and liabilities of its stockholders
or those of legal entities to which it may be connected or vice versa. The notion
of separate personality, however, may be disregarded under the doctrine
piercing the veil of corporate fictionas in fact the court will often look at the
corporation as a mere collection of individuals or an aggregation of persons
undertaking business as a group, disregarding the separate juridical personality
of the corporation unifying the group. Another formulation of this doctrine is that
when two (2) business enterprises are owned, conducted and controlled by the
same parties, both law and equity will, when necessary to protect the rights of
third parties, disregard the legal fiction that two corporations are distinct entities
and treat them as identical or one and the same.
Authorities are agreed on at least three (3) basic areas where piercing the
veil, with which the law covers and isolates the corporation from any other legal
entity to which it may be related, is allowed. These are: 1) defeat of public
88

convenience, as when the corporate fiction is used as vehicle for the evasion of
an existing obligation; 2) fraud cases or when the corporate entity is used to
justify a wrong, protect fraud, or defend a crime; or 3) alter ego cases, where a
corporation is merely a farce since it is a mere alter ego or business conduit of a
person, or where the corporation is so organized and controlled and its affairs are
so conducted as to make it merely an instrumentality, agency, conduit or adjunct
of another corporation.
Verily, indeed, as the relationships binding herein [respondent EQUITY
and petitioner GCC] have been that of "parent-subsidiary corporations" the
foregoing principles and doctrines find suitable applicability in the case at bar.

89

STA. MONICA INDUSTRIAL DEVELOPMENT CORPORATION


Vs.
THE DEPARTMENT OF AGRARIAN REFOM REGIONAL DIRECTOR FOR
REGION III, ET AL.
555 SCRA 97 (2008)
FACTS OF THE CASE
1. Asuncion Trinidad is the owner of five parcels of land with a total area of
4.69 hectares in Calumpit, Bulacan. Private respondent Basilio De
Guzman is the agricultural leasehold tenant of Trinidad.
2. As an agricultural leasehold tenant, De Guzman was issued certificates of
Land Transfer on July 22, 1981.
3. De Guzman filed a petition for the issuance of patent under his name with
the Regional Office of DAR. The DAR sent notices to Trinidad requiring her
to comment. Instead of complying, Trinidad filed a motion for bill of
particulars.
4. After due proceedings, the Regional Director of DAR issued the order
granting Emancipation Patent in favor of De Guzman as qualified farmerbeneficiary of Agrarian Reform Program.
5. Trinidad filed a motion for reconsideration, but her motion was denied.
6. A year later, petitioner Sta. Monica filed a petition for certiorari and
prohibition with CA assailing the order of DAR. Sta. Monica claimed that
while it is true that Trinidad was the former owner of the disputed parcel of
land, the said landholding was sold on Jan. 27, 1986 in favor of Sta.
Monica.
7. Sta. Monica asserted that there was a denial of due process because it
was not furnished a notice of coverage under the CARP law.
8. The CA dismissed the petition of Sta. Monica for lack of merit. The CA held
that Sta. Monica is not a real party-in-interest because it cannot be
considered as an owner of the land it bought from Trinidad.
ISSUE
Whether Sta. Monica, a corporation with separate juridical personality has
been denied of the opportunity of notice and hearing when the DAR awarded
land ownership to an agrarian reform farmer-beneficiary, in the person of De
Guzman.

RULING
No. The corporation Sta. Monica was not denied of the opportunity of
notice and hearing. Trinidad is still deemed the owner of the agricultural land sold
to Sta. Monica; no need for separate notice of coverage under CARP law.
Buyer Sta. Monica is owned and controlled by Trinidad and her family of
which they own 98% of the outstanding capital stock. As owners of 98% of
outstanding capital stock, they are beneficial owners of all the assets of the
corporation including the agricultural land sold by Trinidad to Sta. Monica. At the
very last, the notice to her is already a notice to Sta. Monica because the
corporation acted as a mere conduit of Trinidad.
90

The sale of the land from Trinidad to Sta. Monica was a mere ploy to
evade the applicable provisions of the agrarian law. But it is a fiat that the
corporate vehicle cannot be used as a shield to protect fraud or justify wrong.
Thus, the veil of corporate fiction will be pierced when it is used to defeat public
convenience and subvert public policy.

91

SPS. PEDRO AND FLORENCIA VIOLAGO


Vs.
BA FINANCE CORPORATION and AVELINO VIOLAGO
G.R. No. 158262
FACTS OF THE CASE
In 1983, Avelino Violago, president of Violago Motor Sales Corporation (VMSC)
offered to sell a car to his cousin Pedro Violago and the latters wife. After
agreeing on the terms of the sale the spouses agreed to purchase a car. After
paying the downpayment, the spouses and Avelino signed a note binding them to
pay jointly and severally to VMSC the balance of the sale price. Said note was
endorsed without recourse by VMSC to BA Finance. A deed of assignment of its
rights and interests under the note and chattel mortgage was executed by VMSC
to BA Finance. Later it was found out that said car was already sold to
Esmeraldo Violago. Despite the spouses demand there was no delivery of the
vehicle so Pedro did not pay the monthly amortization to BA Finance. BA
Finance then filed a complaint for replevin with damages against the spouses.
The RTC rendered a decision in favor of BA Finance but against the Violago
spouses subject to indemnification by Avelino. On appeal, Avelino contended
that he should not be held liable since he was not a party to the transaction. The
CA set aside the order of the RTC, thus this petition.
ISSUE
Whether the veil of corporate entity may be invoked and sustained in this case
RULING
No. It is a fundamental principle of corporation law that a corporation is an entity
separate and distinct from its stockholders and from other corporations to which it
may be connected. But, this separate and distinct personality of a corporation is
merely a fiction created by law for convenience and to promote justice. So, when
the notion of separate juridical personality is used to defeat public convenience,
justify wrong, protect fraud or defend crime, or is used as a device to defeat the
labor laws, this separate personality of the corporation may be disregarded or the
veil of corporate fiction pierced. This is true likewise when the corporation is
merely an adjunct, a business conduit or an alter ego of another corporation. In
applying this doctrine of piercing the veil of a corporate fiction the following
requisites must be established: 1) control, not merely majority or complete stock
control; 2) such control must have been used by defendant to commit fraud or
wrong, or to perpetuate the violation of a statutory or other positive legal duty or
dishonest acts in contravention of plaintiffs legal rights; and 3) the aforesaid
control and breach of duty is the approximate cause of injury or unjust loss
complained of.
This case meets the foregoing test. VMSC is a family-owned corporation of
which Avelino was president. Avelino committed fraud in selling the vehicle to
petitioners, a vehicle that was previously sold to Avelinos other cousin,
Esmeraldo. Nowhere in the pleadings did Avelino refute the fact that the vehicle
in this case was already previously sold to Esmeraldo; he merely insisted that he
cannot be held liable because he was not a party to the transaction. The fact
that Avelino and Pedro are cousins, and that Avelino claimed to have a need to
increase the sales quota, was likely among the factors which motivated the
spouses to buy the car. Avelino, knowing fully well that the vehicle was already
sold, and with abuse of his relationship with the spouses, still preceded with the
92

sale and collected the down payment from petitioners. The trial court found that
the vehicle was not delivered to the spouses. Avellino clearly defrauded
petitioners. His actions were the proximate cause of petitioners loss. He cannot
now hide behind the separate corporate personality of VMSC to escape from
liability for the amount adjudged by the trial court in favor of petitioners.

93

CLAUDE P BAUTISTA
VS.
AUTO PLUS TRADERS INC,
G.R. No. 166405, August 6, 2008
FACTS OF THE CASE
Petitioner Claude P. Bautista, in his capacity as President and Presiding
Officer of Cruiser Bus Lines and Transport Corporation, purchased various spare
parts from private respondent Auto Plus Traders, Inc. and issued two postdated
checks to cover his purchases. The checks were subsequently dishonored.
Private respondent then executed an affidavit-complaint for violation of Batas
Pambansa Blg. 223 against petitioner. Petitioner was cleared of the the BP 22
case yet he is still ordered to pay for the value of the checks plus interest. CA
affirmed the said decision of the RTC.
ISSUE
WON the petitioner, as an officer of the corporation, is liable personally
and civilly to the respondent for the value of the two checks
RULING
NO, the petitioner should not be held liable for the checks presented to
Auto Plus Trader. Juridical entities have personalities separate and distinct from
its officers and the persons composing it. Generally, the stockholders and officers
are not personally liable for the obligations of the corporation except only when
the veil of corporate fiction is being used as a cloak or cover for fraud or illegality,
or to work injustice. These situations, however, do not exist in this case. The
evidence shows that it is Cruiser Bus Lines and Transport Corporation that has
obligations to Auto Plus Traders, Inc. for tires. There is no agreement that
petitioner shall be held liable for the corporation's obligations in his personal
capacity. Hence, he cannot be held liable for the value of the two checks issued
in payment for the corporation's obligation in the total amount of P248, 700.

94

LUCIA MAGALING, ET. AL.


VS.
PETER ONG
G.R.NO. 17333, AUGUST 13, 2008
FACTS OF THE CASE
Spouses Reynaldo Magaling and Lucia Magaling is the controlling stock
holders/owner of Thermo Loans and Credit Corporation. On December 1994,
defendant Reynaldo Magaling induce the Peter Ong at the latters store in Lipa
City to lend him and/or his company Thermo Loans and Credit Corporation
money amounting to P350,000.00 at the interest rate of 2 % per month. The
plaintiff, herein defendant extended the loan to the defendant, herein petitioner
based on the assurance of Reynaldo Magaling. Sometime in September 1997, or
three years thereafter the defendants issued and tendered to plaintiff series of
postdated checks for the payment of interest and principal of the loan.
Upon failure of Thermo Loans and Credit Corporation to pay its
outstanding loan despite demand from Mr. Ong, the latter filed a complaint with
the RTC for the collection of the loan with interest, attorneys fees and cost of
suit, with prayer for issuance of a writ of preliminary attachment against the
spouses Reynaldo Magaling and Lucia Magaling and Thermo Loans and Credit
Corporation.
The RTC dismissed the complaint of Peter Ong on the ground that the
subject obligation is the obligation of the defendant corporation in which the
stockholders and officers are not personally liable. Said corporation has a
personality separate and distinct from that of Reynaldo Magaling who happens to
be only a stockholder and president thereof at that time.
On appeal the appellate court reversed and set aside the decision of the
RTC. The Court of Appeals pierced the veil of corporate fiction and held the
spouses solidarity liable with Thermo Loans for the corporate obligations of the
latter. Hence, this petition for certioraris.
ISSUE
Whether or not the defendants being officers and stockholders of Thermo
Loans and Credit Corporation are solidarity liable to the financial liability of said
corporation.
RULING
The Supreme Court affirmed the ruling of the Court of Appeals. The
general rule is that obligations incurred by the corporation, acting through its
directors, officers and employees, are its sole liabilities, and vice versa. However,
there are exceptional circumstances warranting the disregard of a separate
personality and held such officers and directors solidarily liable with the
corporation:
1) When directors /trustees and officers of a corporation:
a) Vote or assent to patently unlawful acts of the corporation.;
95

b) Act in bad faith or with gross negligence in directing corporate affairs;

c) Are guilty of conflict of interest to the prejudice of the corporation, stock


holders or members, and other persons;
2) When a director or officer consented to the insurance of watered down stocks
or who, having knowledge, did not forthwith file with the corporate secretary his
written objection thereto; or
3) When a director, trustee or officer is made by specific provision of law,
personally liable for his corporate action.
In the instant case, Reynaldo Magalings very own testimony convincingly
displayed his gross negligence in the conduct of the affairs of Thermo Loans
without due regard to the plight of its investor. He resigned as President of
Thermo Loans in 1998 when the company already became insolvent. He also
admitted that no one or nobody took over as president of the corporation, when
he resigned. Neither was the investor in-formed about the bankruptcy thereof,
nor was any bankruptcy or involvers proceeding instituted to protect the assets of
the corporation and the interest of its investor. Reynaldo Magaling miserably
failed to exercise at most diligence expected from the highest officer of a
corporation in the conduct of its affairs. Hence, he should be held jointly and
severally liable for the corporate obligation of Thermo Loans to appellant Peter
Ong.

96

SIAIN ENTERPRISES, INC., petitioner,


Vs.
CUPERTINO REALTY CORP. and EDWIN R. CATACUTAN,
G.R. No. 170782 (2009)
FACTS OF THE CASE
Nature of the Case: PETITION for review on certiorari of a decision of the Court
of Appeals AFFIRMING the decision of the Regional Trial Court.
SIAIN Enterprises, Inc. (SIAIN) obtained a loan covered by promissory notes
signed by Cua Le Leng (President) and Wilfredo Lua from Cupertino Reality to
pay off the obligation of SIAIN with the Development Bank of the Philippines. It
was secured by a real-estate mortgage (2 parcels of land) and a chattel
mortgage (equipment and machineries). The promissory notes was amended to
include a 17% interest per anum on the Php 37M loan.
A 2nd promissory note was executed worth Php 160,000,000.00. Cua Le Leng
signed as co-maker; she is liable to Cupertino in her personal capacity.
SIAIN defaulted in its obligation which prompted Cupertino to extra-judicially
foreclose the mortgage.
SIAIN filed a complaint with a prayer for restraining order to enjoin Notary Public
Catacutan from proceeding with the Public Auction. The basis of the complaint:
Cupertino failed to give the Php 160,000.00 to SIAIN as consideration for the
real-estate mortgage.
The RTC dismissed the petitioners complaint and ordered it to Cupertino Php
100,000.00 each for actual and exemplary damages, and Php 500,000.00 as
attorneys fees.
The lower court and the CA upheld the validity of the amended real-estate
mortgage (REM). SIAIN failed to overcome and debunk Cupertinos evidence
that the emended REM had a consideration and it received the amount of Php
160,000.00
SIAIN bewails the lower courts application of piercing the veil of corporate fiction

ISSUES
Is the lower court correct in piercing the veil of corporate fiction?
RULING
In this case, Cupertino presented overwhelming evidence that petitioner and its
affiliate corporations had received the proceeds of the P160,000,000.00 loan
increase which was then made the consideration for the Amended Real Estate
Mortgage.
The checks, debit memos and the pledges of the jewelries, condominium units
and trucks were constituted not exclusively in the name of [petitioner] but also
97

either in the name of Yuyek Manufacturing Corporation, Siain Transport, Inc.,


Cua Leleng and Alberto Lim is of no moment. For the facts established in the
case at bar has convinced the Court of the propriety to apply the principle known
as piercing the veil of the corporate entity by virtue of which, the juridical
personalities of the various corporations involved are disregarded and the
ensuing liability of the corporation to attach directly to its responsible officers and
stockholders. x x x
xxxx
The conjunction of the identity of the [petitioner] corporation in relation to Siain
Transport, Inc. (Siain Transport), Yuyek Manufacturing Corp. (Yuyek), as well as
the individual personalities of Cua Leleng and Alberto Lim has been indubitably
shown in the instant case by the following established considerations, to wit:
1. Siain and Yuyek have [a] common set of [incorporators], stockholders and
board of directors;
2. They have the same internal bookkeeper and accountant in the person of
Rosemarie Ragodon;
3. They have the same office address at 306 Jose Rizal St., Mandaluyong City;
4. They have the same majority stockholder and president in the person of Cua
Le Leng; and
5. In relation to Siain Transport, Cua Le Leng had the unlimited authority by
and on herself, without authority from the Board of Directors, to use the funds of
Siain Trucking to pay the obligation incurred by the [petitioner] corporation.
Thus, it is crystal clear that [petitioner] corporation, Yuyek and Siain Transport
are characterized by oneness of operations vested in the person of their common
president, Cua Le Leng, and unity in the keeping and maintenance of their
corporate books and records through their common accountant and bookkeeper,
Rosemarie Ragodon. Consequently, these corporations are proven to be the
mere alter-ego of their president Cua Leleng, and considering that Cua Leleng
and Alberto Lim have been living together as common law spouses with three
children, this Court believes that while Alberto Lim does not appear to be an
officer of Siain and Yuyek, nonetheless, his receipt of certain checks and debit
memos from Willie Lua and Victoria Lua was actually for the account of his
common-law wife, Cua Leleng and her alter ego corporations. While this Court
agrees with Siain that a corporation has a personality separate and distinct from
its individual stockholders or members, this legal fiction cannot, however, be
applied to its benefit in this case where to do so would result to injustice and
evasion of a valid obligation, for well settled is the rule in this jurisdiction that the
veil of corporate fiction may be pierced when it is used as a shield to further an
end subversive of justice, or for purposes that could not have been intended by
the law that created it; or to justify wrong, or for evasion of an existing obligation.
Resultantly, the obligation incurred and/or the transactions entered into either by
Yuyek, or by Siain Trucking, or by Cua Leleng, or by Alberto Lim with Cupertino
are deemed to be that of the [petitioner] itself.

98

The same principle equally applies to Cupertino. Thus, while it appears that the
issuance of the checks and the debit memos as well as the pledges of the
condominium units, the jewelries, and the trucks had occurred prior to March 2,
1995, the date when Cupertino was incorporated, the same does not affect the
validity of the subject transactions because applying again the principle of
piercing the corporate veil, the transactions entered into by Cupertino Realty
Corporation, it being merely the alter ego of Wilfredo Lua, are deemed to be the
latters personal transactions and vice versa.
As can be viewed from the extant record of the instant case, Cua Leleng is the
majority stockholder of the three (3) corporations namely, Yuyek Manufacturing
Corporation, Siain Transport, Inc., and Siain Enterprises Inc., at the same time
the President thereof. Second. Being the majority stockholder and the president,
Cua Le leng has the unlimited power, control and authority without the approval
from the board of directors to obtain for and in behalf of the [petitioner]
corporation from [Cupertino] thereby mortgaging her jewelries, the condominiums
of her common law husband, Alberto Lim, the trucks registered in the name of
[petitioner] corporations sister company, Siain Transport Inc., the subject lots
registered in the name of [petitioner] corporation and her oil mill property at Iloilo
City. And, to apply the proceeds thereof in whatever way she wants, to the
prejudice of the public.
As such, [petitioner] corporation is now estopped from denying the above
apparent authorities of Cua Le Leng who holds herself to the public as
possessing the power to do those acts, against any person who dealt in good
faith as in the case of Cupertino.
DOCTRINE
Corporation Law; Piercing the Veil of Corporate Fiction; The general rule
that a corporation will be deemed a separate legal entity until sufficient
reason to the contrary appears, but the rule is not absolute.As a general
rule, a corporation will be deemed a separate legal entity until sufficient reason to
the contrary appears. But the rule is not absolute. A corporations separate and
distinct legal personality may be disregarded and the veil of corporate fiction
pierced when the notion of legal entity is used to defeat public convenience,
justify wrong, protect fraud, or defend crime.

99

GONZALES
Vs.
PHILIPPINE NATIONAL BANK
G.R.No. L-33320
FACTS OF THE CASE
Petitioner, in his capacity as taxpayer and stockholder of PNB, sued the
latter questioning the validity of its transactions relative to: the purchase of a
sugar central by the Southern Negros Development Corp. to be financed by
Japanese suppliers and financiers; and its financing of the Cebu-Mactan Bridge
to be constructed by V.C. Ponce Inc.; and the construction of the Passi Sugar
Mills in Iloilo. Hence, petitioner requests for mandamus to be filed against PNB to
allow him to look into the records on the aforementioned transactions.
Trial court denied petitioners prayer on the ground that the right of a
stockholder to inspect the record of the business transactions of a corporation
granted under Section 51 of the former Corporation Law (Act No. 1459, as
amended) is not absolute, but is limited to purposes reasonably related to the
interest of the stockholder, must be asked for in good faith for a specific and
honest purpose and not gratify curiousity or speculative or vicious purposes; that
such examination would violate the confidentiality of the records of PNB as
provided in Section 16 of its charter (R.A. NO. 1300 as amended); and that the
petitioner has not exhausted his administrative remedies.
Petitioner maintains that the lower court erred in ruling that his alleged
improper motive in asking for an examination of the books and records of PNB
disqualifies him to exercise the right of a stockholder to such inspection under
Section 51 of Act No. 1459.
ISSUE
Whether Petitioner is entitled to inspect the books and records of PNB in the
exercise of his right as a stockholder under Section 51 of Act No. 1459.
RULING
Petitioner is not entitled to inspect the books and records of PNB in the
exercise of his right as a stockholder under Section 51 of Act No. 1459.
Act No. 1459 has been replaced by Batas Pambansa Blg. 68 or the
Corporation Code of the Philippines. Accordingly, Section 51 of the former has
been retained with some modifications. Batas Pambansa Blg. 68, with respect to
the right of inspection, provided that the one requesting it must not have been
guilty of using improperly any information through a prior examination, and that
the person asking for such examination must be acting in good faith and for a
legitimate purpose in making his demand. The unqualified provision on the right
of inspection previously contained in Section 51 no longer holds true under the
provisions of the present law. Petitioner did not set forth the reasons and the
purposes for which he desires such inspection, except to satisfy himself as to the
truth of published reports regarding certain transactions and to inquire into their
validity. His purpose cannot be said to be germane to his interest as a
stockholder.
100

Also, the inspection sought to be exercised would be violative of PNBs


charter. PNB is not an ordinary corporation. Having a charter of its own, it is not
governed, as a rule, by the Corporation Code of the Philippines. Section 4 of the
Corporation Code provides that Corporations created by special laws or charters
shall be governed primarily by the provisions of the special law or charter
creating them or applicable to them, supplemented by the provisions of this Code
insofar as they are applicable. Under PNBs charter officers designated by law to
inspect or investigate the condition of the National Bank shall not reveal to any
person other than the President of the Philippines, Secretary of Finance and the
Board of Directors the details of the inspection or investigation, nor shall they
give any information relative to the funds in its custody, its current accounts or
deposits belonging to private individuals, corporations or any other entity, except
by order of a Court of competent jurisdiction.
Since the provision on the right to demand an inspection cannot be
reconciled with the provisions on the PNB charter, it is therefore not correct to
claim that the right of inspection may apply in a supplementary capacity to the
charter of respondent bank.

101

FELICIANO
VS.
COMMISSION ON AUDIT
464 PHIL. 439 (2004)
FACTS OF THE CASE
A Special Audit Team from Commission on Audit (COA) Regional Office No. VIII
audited the accounts of the Leyte Metropolitan Water District (LMWD).
Subsequently, LMWD received a letter from COA dated 19 July 1999 requesting
payment of auditing fees. As General Manager of LMWD, Engr. Ranulfo C.
Feliciano sent a reply dated 12 October 1999 informing COAs Regional Director
that the water district could not pay the auditing fees. Feliciano cited as basis for
his action Sections 6 and 20 of PD 198, as well as Section 18 of RA 6758. The
Regional Director referred Felicianos reply to the COA Chairman on 18 October
1999. On 19 October 1999, Feliciano wrote COA through the Regional Director
asking for refund of all auditing fees LMWD previously paid to COA. On 16 March
2000, Feliciano received COA Chairman Celso D. Gangans Resolution dated 3
January 2000 denying Felicianos request for COA to cease all audit services,
and to stop charging auditing fees, to LMWD. The COA also denied Felicianos
request for COA to refund all auditing fees previously paid by LMWD. Feliciano
filed a motion for reconsideration on 31 March 2000, which COA denied on 30
January 2001. On 13 March 2001, Felicaino filed the petition for certiorari.
ISSUE
Whether a Local Water District (LWD) is a government-owned or controlled
corporation.
RULING
Yes, a Local Water District (LWD) is a government-owned or controlled
corporation. The Supreme Court affirmed the Resolution of COA and denied the
petition.
The Court held that the Constitution recognizes two classes of corporations. The
first refers to private corporations created under a general law. The second refers
to government-owned or controlled corporations created by special charters. The
Constitution emphatically prohibits the creation of private corporations except by
a general law applicable to all citizens. The purpose of this constitutional
provision is to ban private corporations created by special charters, which
historically gave certain individuals, families or groups special privileges denied
to other citizens. In short, Congress cannot enact a law creating a private
corporation with a special charter. Such legislation would be unconstitutional.
Private corporations may exist only under a general law. If the corporation is
private, it must necessarily exist under a general law. Stated differently, only
corporations created under a general law can qualify as private corporations.
Under existing laws, that general law is the Corporation Code, except that the
Cooperative Code governs the incorporation of cooperatives. The Constitution
authorizes Congress to create government-owned or controlled corporations
through special charters. Since private corporations cannot have special
charters, it follows that Congress can create corporations with special charters
only if such corporations are government-owned or controlled. Obviously, LWDs
are not private corporations because they are not created under the Corporation
Code. LWDs are not registered with the SEC. LWDs have no articles of
incorporation, no incorporators and no stockholders or members. There are no
102

stockholders or members to elect the board directors of LWDs as in the case of


all corporations registered with the Securities and Exchange Commission. The
local mayor or the provincial governor appoints the directors of LWDs for a fixed
term of office.
LWDs exist by virtue of PD 198, which constitutes their special charter. Since
under the Constitution only government-owned or controlled corporations may
have special charters, LWDs can validly exist only if they are government-owned
or controlled. To claim that LWDs are private corporations with a special charter
is to admit that their existence is constitutionally infirm. Unlike private
corporations, which derive their legal existence and power from the Corporation
Code, LWDs derive their legal existence and power from PD 198.

103

ALBERT
Vs.
UNIVERSITY PUBLISHING CO.
G.R.NO. L-19118
FACTS OF THE CASE
Mariano Albert entered into a contract with the University Publishing
Company Inc. through Jose M. Aruego, its president.Whereby, they agreed that
the University would pay plaintiff for the exclusive right to publish his revised
Commentaries on the Revised Penal Code. The contract stipulated that failure
to pay on installment would render the rest of the payments due.When University
failed to pay the second installment, Albert sued for collection and won. However,
upon execution, it was found that University was not registered with the SEC.
Albert petitioned for a writ of execution against Jose M.Aruego as the real
defendant. University opposed, on the ground that Aruego was not a party to the
case.
ISSUE
Whether A r u e g o c a n b e h e l d p e r s o n a l l y l i a b l e t o t h e plaintiff.
RULING
Yes,the Supreme Court found that Aruegorepresented a nonexistent entity
and induced not onlyAlbert but the court to believe in such representation.Aruego
, acting as representative of such non-existentprincipal, was the real party to the
contract sued upon, and thus assumed such privileges and obligations and
became personally liable for the contract entered into or for other
acts performed as such agent. One who has induced
another to act upon his
willful misrepresentation that a corporation was duly organized and existing
under the law, cannot thereafter set up against his victim the principle
of corporation by estoppel The Supreme Court likewise held that the
doctrine of corporation by estoppel cannot be set up against Albert since it was
Aruego
who
had
induced
him
to
act
upon
his(Aruego's) willful representation that University had been duly org organized
and was existing under the law.

104

MSCI-NACUSIP LOCAL CHAPTER


Vs.
NATIONAL WAGES AND PRODUCTIVITY COMMISSION,
G.R. No. 125198
FACTS OF THE CASE
Asturias Sugar Central, Inc. (ASCI, for brevity), executed a Memorandum
of Agreement with Monomer Trading Industries, Inc. (MTII, for brevity), whereby
MTII shall acquire the assets of ASCI by way of a Deed of Assignment provided
that an entirely new organization in place of MTII shall be organized which shall
be the assignee of the assets of ASCI. By virtue of this Agreement, a new
corporation was organized and incorporated under the corporate name Monomer
Sugar Central, Inc. or MSCI, the private respondent herein.
Sometime in 1991, MSCI applied for exemption from the coverage of
Wage Order No. RO VI-01 issued by the Board on the ground that it is a
distressed employer. The petitioner herein MSCI-NACUSIP Local Chapter
(Union, for brevity), in opposition, maintained that MSCI is not distressed; that
respondent applicant has not complied with the requirements for exemption; and
that the financial statements submitted by MSCI do not reflect the true and valid
financial status of the company, and that the paid-up capital would have been
higher than P5 million and thus impairment would have been lower than 25% had
the pre-organization agreement between ASCI and MTII been complied with.
The Board denied MSCI's application for exemption based on the finding
that the applicant's losses constitute an impairment of only 5.25% of its paid-up
capital and cannot be said to be sufficient to meet the required 25% in order to
qualify for the exemption, as provided in NWPC Guidelines No. 01, Series of
1992 entitled "Revised Guidelines on Exemption from Compliance with the
Prescribed Wage/Cost of Living Allowance Increases Granted by the Regional
Tripartite Wages and Productivity Boards.
The motion for reconsideration filed by MSCI was denied by the Board. A
timely appeal was brought before the public respondent Commission. In its
decision, the Commission reversed and set aside the foregoing orders of the
Board, and granted MSCI's application for exemption from Wage Order No. RO
VI-01. Hence this petition for Certiorari under Rule 65.

ISSUE
(1) Whether or not the correct paid-up capital of MSCI for the pertinent period
covered by the application for exemption is P5 million or P64,688,528.00?;
(2) Whether or not respondent MSCI can qualify as a distressed employer and
thus be entitled to exemption from compliance with Wage Order No. RO VI-01.

105

RULING
We find no grave abuse of discretion on the part of the Commission in
setting aside the findings of the Board and granting full exemption to MSCI from
Wage Order No. R.VI-01.
NWPC Guidelines No. 01, Series of 1992 as well as the new NWPC
Guidelines No. 01, Series of 1996, define Capital as referring to paid-up capital at
the end of the last full accounting period, in the case of corporations or total
invested capital at the beginning of the period under review, in the case of
partnerships and single proprietorships. To have a clear understanding of what
paid-up capital is, however, a referral to Sections 12 and 13 of BP Blg. 68 or the
Corporation Code would be very helpful.
By express provision of Section 13, paid-up capital is that portion of the
authorized capital stock which has been both subscribed and paid. In the case
under consideration, there is no dispute, and the Board even mentioned in its
August 17, 1993 Decision, that MSCI was organized and incorporated on
February 15, 1990 with an authorized capital stock of P60 million, P20 million of
which was subscribed. Of the P20 million subscribed capital stock, P5 million
was paid-up. This fact is only too glaring for the Board to have been misled into
believing that MSCI'S paid-up capital stock was P64 million plus and not P5
million.

Henceforth, the paid-up capital stock of MSCI for the period covered by the
application for exemption still stood at P5 million. The losses, therefore,
amounting to P3,400,738.00 for the period February 15, 1990 to August 31, 1990
impaired MSCI's paid-up capital of P5 million by as much as 68%. Likewise, the
losses incurred by MSCI for the interim period from September 1, 1990 to
November 30, 1990, as found by the Commission, per MSCI's quarterly income
statements, amounting to P13,554,337.33 impaired the company's paid-up
capital of P5 million by a whopping 271.08%, more than enough to qualify MSCI
as a distressed employer. Respondent Commission thus acted well within its
jurisdiction in granting MSCI full exemption from Wage Order No. ROVI-01 as a
distressed employer.
Wherefore, the petition is dismissed.

106

DOCTORS ROSA P. ALFAFARA ET. Al


Vs.
ACEBEDO OPTICAL, CO., INC.
381 SCRA 293 (2002)
FACTS OF THE CASE
Petitioners are optometrists. They brought, in their own behalf and in
behalf of 80 other optometrists, who are members of the Samahan ng
Optometrists sa Pilipinas-Cebu Chapter, an injunctive suit in the Regional Trial
Court, Cebu City to enjoin respondent Acebedo Optical Co., Inc. and its agents,
representatives, and/or employees from practicing optometry in the province of
Cebu.
In their complaint, they alleged that respondent opened several optical
shops in Cebu and announced to the public, through leaflets, newspapers, and
other forms of advertisement, the availability of ready-to-wear eyeglasses for
sale at P60.00 each and free services by optometrists in such outlets. They
claimed that, through the licensed optometrists under its employ, respondent had
been engaging in the practice of optometry and that such acts of respondent
were done in violation of the Optometry Law (R.A. No. 1998) and the Code of
Ethics for Optometrists.
The TC granted their prayer for a writ of preliminary injunction and/or
restraining order. After hearing, judgment was rendered in favor of petitioners.
The trial court found that the hiring of licensed optometrists by the respondent
was unlawful because it resulted in the practice of the optometry profession by
respondent, a juridical person.
Respondent appealed to the Court of Appeals which reversed the decision
of the trial court and dismissed the complaint of petitioners. Citing the case
of Samahan ng Optometrists sa Pilipinas, Ilocos Sur-Abra Chapter v. Acebedo
International Corporation, the appeals court ruled that respondents hiring of
licensed optometrists did not constitute practice of optometry nor violate any law.
Hence, this petition alleging that the Court of Appeals erred in holding that
respondent Acebedo was not engaged in the practice of optometry.

ISSUE
Whether the respondent Acebedo was engaged in the practice of
optometry.
RULING
No. An optometrist is a person who has been certified by the Board of
Optometry and registered with the Professional Regulation Commission as
qualified to practice optometry in the Philippines. Thus, only natural persons can
engage in the practice of optometry and not corporations. Respondent, which is
not a natural person, cannot take the licensure examinations for optometrist and,
107

therefore, it cannot be registered as an optometrist under R.A. No. 1998. It is


noteworthy that, in Apacionado, the Court did not find Acebedo to be engaged in
the practice of optometry. The optometrists in that case were found guilty of
unprofessional conduct and their licenses were suspended for two (2) years for
having participated, in their capacities as optometrists, in the implementation of
the promotional advertisement of Acebedo. In contrast, in the case at bar,
respondent is merely engaged in the business of selling optical products, not in
the practice of optometry, whether directly or indirectly, through its hired
optometrists.

108

ISLAMIC DIRECTORATE OF THE PHILIPPINES


Vs.
COURT OF APPEALS
G.R. No. 117897
FACTS OF THE CASE
Petitioner IDP-Tamano Group alleges that sometime in 1971, Islamic
leaders of all Muslim major tribal groups in the Philippines headed by Dean
Cesar Adib Majul organized and incorporated the ISLAMIC DIRECTORATE OF
THE PHILIPPINES (IDP), the primary purpose of which is to establish an Islamic
Center in Quezon City for the construction of a Mosque (prayer place),
Madrasah (Arabic School), and other religious infrastructures so as to facilitate
the effective practice of Islamic faith in the area.
In the same year, the Libyan government donated money to the IDP to
purchase land at Culiat, Tandang Sora, and Quezon City, to be used as a Center
for the Islamic populace. After the purchase of the land by the Libyan
government in the name of IDP, Martial Law was declared by the late President
Ferdinand Marcos. Most of the members of the 1971 Board of Trustees like
Senators Mamintal Tamano, Salipada Pendatun, Ahmad Alonto, and
Congressman Al-Rashid Lucman flew to the Middle East to escape political
persecution.
Thereafter, two Muslim groups sprung, the Carpizo Group, headed by
Engineer Farouk Carpizo, and the Abbas Group, led by Mrs. Zorayda Tamano
and Atty. Firdaussi Abbas. Both groups claimed to be the legitimate
IDP. Significantly, on October 3, 1986, the SEC, in a suit between these two
contending groups, came out with a decision declaring the election of both the
Carpizo Group and the Abbas Group as IDP board members to be null and void.
Neither group, however, took the necessary steps prescribed by the SEC,
nor, thus, no valid election of the members of the Board of Trustees of IDP was
ever called. Although the Carpizo Group attempted to submit a set of by-laws,
the SEC found that, aside from Engineer Farouk Carpizo and Atty. Musib Buat,
those who prepared and adopted the by-laws were not bona fide members of the
IDP, thus rendering the adoption of the by-laws likewise null and void.
Without having been properly elected as new members of the Board of
Trustees of IDP, the Carpizo Group caused to be signed an alleged Board
Resolution of the IDP, authorizing the sale of the subject two parcels of land to
the private respondent INC. The petitioner 1971 IDP Board of Trustees headed
by former Senator Mamintal Tamano, or the Tamano Group, filed a petition
before the SEC, seeking to declare null and void the Deed of Absolute Sale since
the group was not legitimate Board of Trustees.
SEC ruled in favor of 1971 Board of Trustees. The CA reversed the
decision.
ISSUE
Whether the sale of 2 parcels of land between the IDP-Carpizo Group and
private respondent INC is null and void.

109

RULING
Yes.
There can be no question as to the authority of the SEC to pass upon the
issue as to who among the different contending groups is the legitimate Board of
Trustees of the IDP since this is a matter properly falling within the original and
exclusive jurisdiction of the SEC by virtue of Sections 3 and 5(c) of Presidential
Decree No. 902-A. If the SEC can declare who is the legitimate IDP Board, then
by parity of reasoning, it can also declare who is not the legitimate IDP
Board. This is precisely what the SEC did when it adjudged the election of the
Carpizo Group to the IDP Board of Trustees to be null and void. By this ruling,
the SEC in effect made the unequivocal finding that the IDP-Carpizo Group is a
bogus Board of Trustees. Consequently, the Carpizo Group is bereft of any
authority whatsoever to bind IDP in any kind of transaction including the sale or
disposition of IDP property.
The SEC already declared the election of the Carpizo Group as well as the
Abbas Group to the IDP Board as null and void for being violative of the Articles
of Incorporation. In this case, the IDP, owner of the subject parcels of land, never
gave its consent, thru a legitimate Board of Trustees, to the disputed Deed of
Absolute Sale executed in favor of INC. This is, therefore, a case not only of
vitiated consent, but one where consent on the part of one of the supposed
contracting parties is totally wanting. Ineluctably, the subject sale is void and
produces no effect whatsoever.

110

INTERNATIONAL EXPRESS TOUR


VS.
COURT Of APPEALS
G.R. No. 119002
FACTS OF THE CASE
In 1989, International Express Travel & Tour Services, Inc. (IETTI), offered to
the Philippine Football Federation (PFF) its travel services for the South East
Asian Games. PFF, through Henri Kahn, its president, agreed. IETTI then
delivered the plane tickets to PFF, PFF in turn made a down payment. However,
PFF was not able to complete the full payment in subsequent installments
despite repeated demands from IETTI. IETTI then sued PFF and Kahn was
impleaded as a co-defendant.
Kahn averred that he should not be impleaded because he merely acted as an
agent of PFF which he averred is a corporation with separate and distinct
personality from him. The trial court ruled against Kahn and held him personally
liable for the said obligation (PFF was declared in default for failing to file an
answer). The trial court ruled that Kahn failed to prove that PFF is a corporation.
The Court of Appealshowever reversed the decision of the trial court. The Court
of Appeals took judicial notice of the existence of PFF as a national sports
association; that as such, PFF is empowered to enter into contracts through its
agents; that PFF is therefore liable for the contract entered into by its agent
Kahn. The CA further ruled that IETTI is in estoppel; that it cannot now deny the
corporate existence of PFF because it had contracted and dealt with PFF in such
a manner as to recognize and in effect admit its existence.
ISSUES
Whether or not the Court of Appeals is correct.
RULINGS
No. PFF, upon its creation, is not automatically considered a national sports
association. It must first be recognized and accredited by the Philippine Amateur
Athletic Federation and the Department of Youth and Sports Development. This
fact was never substantiated by Kahn. As such, PFF is considered as an
unincorporated sports association. And under the law, any person acting or
purporting to act on behalf of a corporation which has no valid existence
assumes such privileges and becomes personally liable for contract entered into
or for other acts performed as such agent. Kahn is therefore personally liable for
the contract entered into by PFF with IETTI.
There is also no merit on the finding of the CA that IETTI is in estoppel. The
application of the doctrine of corporation by estoppel applies to a third party only
when he tries to escape liability on a contract from which he has benefited on the
irrelevant ground of defective incorporation. In the case at bar, IETTI is not trying
to escape liability from the contract but rather is the one claiming from the
contract.

111

Republic Planters Bank


Vs.
Court of Appeals,
216 SCRA 738 (1992)
FACT OF THE CASE
1. Yamaguchi and Canlas are officers of the Worldwide Garment
Manufacturing, which later changed its name to Pinch Manufacturing.
2. They were authorized to apply for credit facilities with the petitioner bank. The
two officers signed the promissory notes issued to secure the payment of
the obligations. Later, the bank instituted an action for collection of
money, impleading also the two officers. The trial court held the two officers
personally liable also.
ISSUE
Whether the change in corporate name would enable the corporation to
escape from corporate liability.
RULING
No. The respondent Court made a grave error in holding that an
amendment in a corporation's Articles of Incorporation effecting a change of
corporate name, in this case from Worldwide Garment manufacturing Inc to
Pinch Manufacturing Corporation extinguished the personality of the original
corporation.
The corporation, upon such change in its name, is in no sense a new
corporation, or the successor of the original corporation. It is the same
corporation with a different name, and its character is in no respect changed. 10
A change in the corporate name does not make a new corporation, and
whether affected by special act or under a general law, has no effect on the
identity of the corporation, or on its property, rights, or liabilities. 11
The corporation continues, as before, responsible in its new name for all
debts or other liabilities which it had previously contracted or incurred.

112

INDUSTRIAL REFRACTORIES CORPORATION OF THE PHILIPPINES


Vs.
COURT OF APPEALS, SECURITIES AND EXCHANGE COMMISSION and
REFRACTORIES CORPORATION OF THE PHILIPPINES,
G.R. No. 122174, October 3, 2002
FACTS OF THE CASE
Respondent Refractories Corporation of the Philippines (RCP) is a
corporation duly organized on October 13, 1976. Petitioner Industrial Refractories
Corporation of the Philippines (IRCP) was incorporated on August 23, 1979
originally under the name "Synclaire Manufacturing Corporation". It amended its
Articles of Incorporation on August 23, 1985 to change its corporate name to
IRCP. Both companies are the only local suppliers of monolithic gunning mix.
RCP filed with the Securities and Exchange Commission (SEC) a petition
to compel petitioner to change its corporate name on the ground that its
corporate name is confusingly similar with that of petitioner. The SEC decided in
favor of respondent RCP, directing petitioner to amend its AoI and change its
name. Petitioner appealed to the SEC En Banc and the decision was affirmed
with the modification that petitioner should delete or drop from its corporate name
only the word "Refractories". Hence, herein petition.
ISSUE
Whether or not RCP is not entitled to use the generic name "refractories".

RULING
No. Petitioners argument that there is no confusing or deceptive similarity
between petitioner and respondent RCPs corporate names is untenable. Section
18 of the Corporation Code expressly prohibits the use of a corporate name
which is identical or deceptively or confusingly similar to that of any existing
corporation or to any other name already protected by law or is patently
deceptive, confusing or contrary to existing laws. The policy behind the
foregoing prohibition is to avoid fraud upon the public that will have occasion to
deal with the entity concerned, the evasion of legal obligations and duties, and
the reduction of difficulties of administration and supervision over corporation.

As held in Philips Export B.V. vs. Court of Appeals,[28] to fall within the
prohibition of the law, two requisites must be proven, to wit:
(1)
That the complainant corporation acquired a prior right over the use of
such corporate name; and
(2)
the proposed name is either: (a) identical, or (b) deceptively or
confusingly similar to that of any existing corporation or to any other name
already protected by law; or (c) patently deceptive, confusing or contrary to
existing law.
As regards the first requisite, it has been held that the right to the exclusive
use of a corporate name with freedom from infringement by similarity is
113

determined by priority of adoption. In this case, respondent RCP was


incorporated on October 13, 1976 and since then has been using the corporate
name Refractories Corp. of the Philippines. Meanwhile, petitioner was
incorporated on August 23, 1979 originally under the name Synclaire
Manufacturing Corporation.
It only started using the name Industrial
Refractories Corp. of the Philippines when it amended its Articles of
Incorporation on August 23, 1985, or nine (9) years after respondent RCP started
using its name. Thus, being the prior registrant, respondent RCP has acquired
the right to use the word Refractories as part of its corporate name.
In determining the existence of confusing similarity in corporate names, the
test is whether the similarity is such as to mislead a person using ordinary care
and discrimination and the Court must look to the record as well as the names
themselves. Petitioners corporate name is Industrial Refractories Corp. of the
Phils., while respondents is Refractories Corp. of the Phils. Obviously, both
names contain the identical words Refractories, Corporation and Philippines.
The only word that distinguishes petitioner from respondent RCP is the word
Industrial which merely identifies a corporations general field of activities or
operations. We need not linger on these two corporate names to conclude that
they are patently similar that even with reasonable care and observation,
confusion might arise. It must be noted that both cater to the same clientele, i.e.
the steel industry. In fact, the SEC found that there were instances when
different steel companies were actually confused between the two, especially
since they also have similar product packaging. Such findings are accorded not
only great respect but even finality, and are binding upon this Court, unless it is
shown that it had arbitrarily disregarded or misapprehended evidence before it to
such an extent as to compel a contrary conclusion had such evidence been
properly appreciated. And even without such proof of actual confusion between
the two corporate names, it suffices that confusion is probable or likely to occur.
While the word refractories is a generic term, its usage is not widespread
and is limited merely to the industry/trade in which it is used, and its continuous
use by respondent RCP for a considerable period has made the term so closely
identified with it.

114

P.C. Javier & Sons, Inc.


Vs.
Court of Appeals
G.R. No. 129552
FACTS OF THE CASE
1. In February, 1981, Plaintiff P.C. Javier and Sons Services, Inc., Plaintiff
Corporation, for short, applied with First Summa Savings and Mortgage Bank,
later on renamed as PAIC Savings and Mortgage Bank, Defendant Bank, for
short, for a loan accommodation under the Industrial Guarantee Loan Fund
(IGLF) for P1.5 Million.
2. On March 21, 1981, Plaintiff Corporation through Plaintiff Pablo C. Javier,
Plaintiff Javier for short, was advised that its loan application was approved and
that the same shall be forwarded to the Central Bank (CB) for processing and
release.
3. The CB released the loan to Defendant Bank in two (2) tranches of P750,000
each. The first tranche was released to the Plaintiff Corporation on May 18, 1981
in the amount of P750,000.00 and the second tranche was released to Plaintiff
Corporation on November 21, 1981 in the amount of P750,000.00.
4. Plaintiff Corporation defaulted in the payment of its IGLF loan with Defendant
Bank hence Defendant Bank sent a demand letter dated November 22, 1983,
reminding Plaintiff Javier to make payments because their accounts have been
long overdue; that on May 2, 1984, Defendant Bank sent another demand letter
to Plaintiff spouses informing them that since they have defaulted in paying their
obligation, their mortgage will now be foreclosed; that when Plaintiffs still failed to
pay, Defendant Bank initiated extrajudicial foreclosure of the real estate
mortgage executed by Plaintiff spouses and accordingly the auction sale of the
property.
5. Petitioners argue that they are legally justified to withhold their amortized
payments to the respondent bank until such time they would have been properly
notified of the change in the corporate name of First Summa Savings and
Mortgage Bank.
ISSUE
Whether it is necessary for a bank to notify its debtors for the banks
change of corporate name.
RULING
No. After going over the Corporation Code and Banking Laws, as well as
the regulations and circulars of both the SEC and the Bangko Sentral ng Pilipinas
(BSP), the Court find that there is no such requirement. This being the case, this
Court cannot impose on a bank that changes its corporate name to notify a
debtor of such change absent any law, circular or regulation requiring it. Such
act would be judicial legislation. The formal notification is, therefore,
discretionary on the bank. Unless there is a law, regulation or circular from the
SEC or BSP requiring the formal notification of all debtors of banks of any
115

change in corporate name, such notification remains to be a mere internal policy


that banks may or may not adopt.

116

HYATT ELEVATORS AND ESCALATORS CORPORATION


vs
GOLDSTAR ELEVATORS PHILS INC.
G.R. No. 161026
FACTS OF THE CASE
Hyatt elevator is a domestic corporation engaged among others, in selling,
maintaining elevators and escalators with principal office located in Makati City,
as stated in its Articles of Incorporation. Hyatt filed a complaint for unfair trade
practices and damages against LG Industries and LG International alleging that it
was appointed by LG as the exclusive distributor of LG elevators and escalators
in the Philippines. During negotiations to change the exclusive distributorship
agency to that of a joint venture, LG terminated the exclusive distributorship
agency. As a consequence, Hyatt suffered damages. The complaint was filed in
Mandaluyong City where the new office of Hyatt is located. Subsequently Hyatt
amended its complaint to include Goldstar Elevator as party-defendant, alleging
that the latter is being utilized by LG in perpetuating its unlawful acts against
Hyatt. Goldstar is also a domestic corporation engaed in the similar business as
Hyatt with principal office in Makati City. Goldstar filed a motion to dismiss for
improper venue which was denied by the lower court. On appeal, the CA
reversed the ruling of the lower court. Thus this petition.
ISSUE
Whether venue was improper
RULING
Yes. Under the Rules of Court, personal action should be tried where the plaintiff
or any of the principal plaintiff resides or where the defendant or any of the
principal defendant resides, or in the case of non-resident defendant, where he
may be found, at the election of the plaintiff. Under the corporation Code, a
corporation's residence is the place where its principal office is located, as stated
in its Articles of Incorporation. In the case at bar, the principal office of Hyatt and
Goldstar is both located in Makati City, as stated in their respective Articles of
Incorporation. In this case the complaint was filed in Mandaluyong City which is
neither the residence of either the defendant or the plaintiff Thus the CA did not
err in dismissing the case for improper venue.

117

MARISSA R. UNCHUAN
Vs.
ANTONIO J.P. LOZADA,
G.R. No. 172671,
April 16, 2009
FACTS OF THE CASE
Two sisters, both residing in the United States, Anita Lozada Slaughter
and Peregrina Lozada Saribay were owners of the lots in controversy located in
Cebu City. The subject lots were sold to their nephew Antonio Lozada. The
payments were advanced by the sisters brother Dr. Lozada(also residing in the
US) for their nephew Antonio. The Deed of Sale was notarized and authenticated
in the Philippine Consuls Office. The sale was recorded in the registry of Cebu.
Pending registration of the deed, petitioner Marissa R. Unchuan caused the
annotation of an adverse claim on the lots. Marissa claimed that Anita donated
an undivided share in the lots to her under an unregistered Deed of Donation
dated February 4, 1987.
Antonio and Anita brought a case of quieting of title against Marissa. At the
trial, respondents presented a notarized and duly authenticated statement and a
videotape where Anita denied having donated land in favor of Marissa. Dr.
Lozada testified that he agreed to advance payment for Antonio in preparation for
their plan to form a corporation. The lots are to be eventually infused in the
capitalization of Damasa Corporation, where he and Antonio are to have 40%
and 60% stake, respectively.
Marissa contested that as a non-Filipino, Dr. Lozada cannot buy a lot in the
Philippines, thus invalidating the sale of the subject lots to Antonio Lozada.
ISSUE
WON the sale of the lot to Antonio, and the subsequent creation of a
corporation by Antonio and Dr. Lozada, would violate the prohibition on aliens
regarding corporations and land ownership
RULING
No. The advancement of payment by DR. Lozada did not viuolate any
constitutional provision on corporation of land ownership. In this case, we find
nothing to show that the sale between the sisters Lozada and their nephew
Antonio violated the public policy prohibiting aliens from owning lands in the
Philippines. Even as Dr. Lozada advanced the money for the payment of
Antonios share, at no point were the lots registered in Dr. Lozadas name. Nor
was it contemplated that the lots be under his control for they are actually to be
included as capital of Damasa Corporation. According to their agreement,
Antonio and Dr. Lozada are to hold 60% and 40% of the shares in said
corporation, respectively. Under Republic Act No. 7042, particularly Section 3, a
corporation organized under the laws of the Philippines of which at least 60% of
the capital stock outstanding and entitled to vote is owned and held by citizens of
the Philippines, is considered a Philippine National. As such, the corporation may
acquire disposable lands in the Philippines. Neither did petitioner present proof to
belie Antonios capacity to pay for the lots subjects of this case.

118

PIONEER SURETY & INSURANCE CORPORATION


VS.
COURT OF APPEALS
G.R.NO. 84197 JULY 28, 1989
FACTS OF THE CASE
This is a consolidated petitions of cases numbers G.R. No. 84197 July 28,
1989, entitled Pioneer Insurance and Surety Corporation vs. The Honorable
Court of Appeals, Borders Machinery and Heavy Equipment Inc.,
(BORMAHECO), Constancio M. Maglana and Jacob S. Lim and G.R. No. 84197
July 28 1989 entitled Jacob S. Lim vs. Court of Appeals, Pioneer Insurance and
Surety Corporation, Boarder Machinery and Heavy Equipment Co, Inc, Francisco
and Modesto Cervantes and Constacio Maglana.
In 1965 Jacob S. Lim was engaged in the airline business as the owner
operator of Southern Airlines (SAL), a single Proprietorship.
On May 17, 1965 at Tokyo, Japan, Japan Domestic Airlines (JDA) and Lim
entered into and executed a sales contract for the sale and purchase of two (2)
DC-3A type aircrafts and one (1) set of necessary spare parts for the total agreed
price of US$ 109,000 to be paid in installment.
Pioneer Insurance and Surety Corporation as surety executed and issued
its Surety Bond No-6639 in favor of JDA, in behalf of its principal, Jacob Lim, for
the balance price of the aircrafts spare parts.
Border Machinery and Heavy Equipment Company Inc. (BORMAHECO),
Francisco and Modesto Cervantes and Constancio Maglana contributed some
funds used in the purchased of the above aircrafts and spare parts. The funds
were supposed to be their contributions to a new corporation proposed by Lim to
expand his airline business. They executed two (2) separate indemnity
agreements in favor of pioneer, one signed by Maglana, and the other jointly
signed by Lim for SAL, BORMAHECO and the Cervantess. The indemnity
agreements stipulated that the indemnitors principally agree and binds
themselves jointly and severally to indemnify and hold and save harmless
Pioneer from and against any/all damages, losses, costs, taxes, penalties
charges and expenses of whatever kind and nature which Pioneer may incur on
consequence of having become surety upon the bond/note.
Lim, doing business under the name of SAL executed in favor of Pioneer a
deed of Chattel Mortgage as security for the latters surety ship in favor of the
former. It is stipulated that Lim transfer and convey to the surety the two aircrafts
in case he defaulted from the installment.
Lim defaulted on his subsequent installment payments prompting JDA to
request payments from the surety. Pioneer paid a total sum of 298,626.12
Pioneer then filed a petition for extrajudicial foreclosure of the said chattel
mortgage. The Cervantess and Maglana however, filed a third party claim
alleging that they are co-owners of the aircrafts.

119

Pioneer later filed an action for judicial foreclosure with an application for a
writ of preliminary attachment against Lim, the Cervantess, BORMAHECO and
Maglana.
In their answers, Maglana and Bormaheco and the Cervantess filed crossclaims against Lim alleging that they were not privies to the contract signed by
Lim, sought for damages and recovery of the survey of money they advanced to
Lim for the purchase of the aircrafts.
ISSUE
What regal rules govern the relationship among co-investors whose
agreement was to do business through the corporate vehicles but who failed to
incorporate the entity? How are loses to be treated under this situation? For
failure of respondents Bormaheco, Spouses, Cervantes, Maglana and petitioner
Lim to incorporate, whether or not a de facto partnership among them was
created.
RULING
The Supreme Court ruled that no de facto partnership was created among
the parties which would entitle petitioner Lim to a reimbursement of the supposed
loses of the proposed corporation. The record shows that the petitioner was
acting on his own and not in behalf of his other would-be incorporators in
transacting the sale of the aircrafts and spare parts. Petitioner Lim never had the
intention to form a corporation with the respondents and that they were induced
and lured by him to make contributions to a propose corporation which was never
formed because the petitioner reneged on their agreement.
It is ordinarily held that persons who attempt, but failed, to form a
corporation and who carry on business under the corporate name occupy the
position of partners (Lynch vs. Perryman) however, such a relation does not
necessary exist for ordinarily persons cannot be made to assume the relation of
partners as between themselves, when their purposed is that no partnership shall
exist.
Thus, one who takes no part except to subscribe for the stock in a
purposed corporation which is never legally formed does not become a partner
with other subscribers who engaged in business under the name of the
pretended corporation, so as to be liable a such in action for settlement of the
alleged partnership and contribution (Word vs. Brigham, 127 Mass. 24)

120

PEOPLE OF THE PHILIPPINES, plaintiff-appellee,


Vs.
ENGR. CARLOS GARCIA y PINEDA, PATRICIO BOTERO y VALES, LUISA
MIRAPLES (at large), accused, PATRICIO BOTERO y VALES, accusedappellant
G.R. No. 117010 (1997)
FACTS OF THE CASE
Nature of the Case: APPEAL from a decision of the Regional Trial Court of Pasig
City, Br. 158.
In 1993, Carlos Garcia, Patricio Botero, and Luisa Miraples were accused of
illegal recruitment. It was alleged that they represented themselves as the
incorporators and officers of Ricorn Philippine International Shipping Lines, Inc.;
that Ricorn is a recruitment agency for seamen; that Garcia is the president,
Botero is the vice-president, and Miraples (now at large) is the treasurer. It was
later discovered that Ricorn was never registered with the Securities and
Exchange Commission (SEC) and that it was never authorized to recruit by the
Philippine Overseas Employment Agency (POEA). Botero and Garcia were
convicted. Botero appealed.
In his defense, Botero averred that he was not an incorporator; that he was
merely an employee of Ricorn in charge of following up on their documents.
ISSUE
Whether Botero should be held liable with Ricorn for the crime of large scale
illegal recruitment?
RULING
For engaging in recruitment of workers without obtaining the necessary license
from the POEA, Botero should suffer the consequences of Ricorns illegal act for
(i)f the offender is a corporation, partnership, association or entity, the penalty
shall be imposed upon the officer or officers of the corporation, partnership,
association or entity responsible for violation; x x x. The evidence shows that
appellant Botero was one of the incorporators of Ricorn. For reasons that cannot
be discerned from the records, Ricorns incorporation was not consummated.
Even then, appellant cannot avoid his liabilities to the public as an incorporator of
Ricorn. He and his co-accused Garcia held themselves out to the public as
officers of Ricorn. They received money from applicants who availed of their
services. They are thus estopped from claiming that they are not liable as
corporate officials of Ricorn.31 Section 25 of the Corporation Code provides that
(a)ll persons who assume to act as a corporation knowing it to be without
authority to do so shall be liable as general partners for all the debts, liabilities
and damages incurred or arising as a result thereof: Provided, however, That
when any such ostensible corporation is sued on any transaction entered by it as
a corporation or on any tort committed by it as such, it shall not be allowed to use
as a defense its lack of corporate personality.
Appellant Botero is guilty of the crime of illegal recruitment in a large scale
considering it was proven that he, together with his cohorts, were able to defraud
121

the six complainant-witnesses in this case. Under Article 38 (b) of the Labor
Code, illegal recruitment in large scale is perpetrated if committed against three
(3) or more persons individually or as a group. And under Article 39 (a) of the
same Code, accused-appellants crime is punishable by life imprisonment and a
fine of one hundred thousand pesos (P100,000.00).
Finally, it is fruitless for appellant to deny he conspired with his co-accused to
commit the crime at bar. The fact that all the accused were co-conspirators in
defrauding the complainants could be inferred from their acts. They played
different roles in defrauding complainants: accused Garcia was the president,
appellant Botero was the vice-president and accused-at-large Miraples was the
treasurer of Ricorn.32 Each one played a part in the recruitment of complainants.
They were indispensable to each other.
DISPOSITION
IN VIEW WHEREOF, the decision of the Regional Trial Court convicting
accused-appellant Patricio Botero of the crime.
DOCTRINE
Same; Same; Same; Corporation Law; All persons who assume to act as a
corporation knowing it to be without authority to do so shall be liable as
general partners for all the debts, liabilities and damages incurred or
arising as a result thereof.For engaging in recruitment of workers without
obtaining the necessary license from the POEA, Botero should suffer the
consequences of Ricorns illegal act for (i)f the offender is a corporation,
partnership, association or entity, the penalty shall be imposed upon the officer or
officers of the corporation, partnership, association or entity responsible for
violation; x x x. The evidence shows that appellant Botero was one of the
incorporators of Ricorn. For reasons that cannot be discerned from the records,
Ricorns incorporation was not consummated. Even then, appellant cannot avoid
his liabilities to the public as an incorporator of Ricorn. He and his co-accused
Garcia held themselves out to the public as officers of Ricorn. They received
money from applicants who availed of their services. They are thus estopped
from claiming that they are not liable as corporate officials of Ricorn. Section 25
of the Corporation Code provides that (a)ll persons who assume to act as a
corporation knowing it to be without authority to do so shall be liable as general
partners for all the debts, liabilities and damages incurred or arising as a result
thereof: Provided, however, That when any such ostensible corporation is sued
on any transaction entered by it as a corporation or on any tort committed by it as
such, it shall not be allowed to use as a defense its lack of corporate personality.

122

LOZANO
VS.
DELOS SANTOS,
G.R.NO. 125221
FACTS OF THE CASE
Petitioner Lozane filed Civil Case No. 1214 for damages against
respondent Anda before MCTC. Petitioner and respondent was both president of
their respective jeepney drivers association. In August 1995, petitioner and
private respondent agreed to consolidate their respective associations and form
the Unified Mabalacat-Angeles Jeepney Operators and Drivers Association, Inc.
(UMAJODA). Petitioner and private respondent also agreed to elect one set of
officers who shall be given the sole authority to collect the daily dues from the
members of the consolidated association. Petitioner was elected to the position
of president, to which respondent objected. Respondent also refused to abide by
their agreement and continued collecting the dues from the members of his
association despite several demands to desist.
Private respondent moved to dismiss the complaint for lack of jurisdiction,
claiming that jurisdiction was lodged with the SEC. MCTC denied the motion.
RTC found the dispute to be intra corporate, hence subject to the jurisdiction of
the SEC, and ordered the MCTC to dismiss Civil Case No. 1214.
ISSUE
Whether the RTC erred in concluding that the SEC has jurisdiction over the case.
RULING
RTC erred in concluding that the SEC has jurisdiction over the case.
MCTC has the proper jurisdiction.
Jurisdiction of the SEC is determined by two elements: first, the status and
relationship of the parties, in that the controversy must arise out of intracorporate
or partnership relations; and second, the nature of the question that is the subject
of their controversy, which requires that the dispute among the parties be
intrinsically connected with the regulation of the corporation, partnerships and
associations with the end in view that investments in these entities may be
encouraged and protected, and their activities pursued for the promotion of
economic development.
There is no intra corporate or partnership relation between petitioner and
private respondent. The unified association was still a proposal, and had not
been approved by the SEC. There must be a certificate of consolidation issued
by the SEC, making the reorganization special. Afterwards, the new consolidated
corporation comes into existence and the constituent corporations dissolve and
cease to exist. There being no intracorporate nor does partnership relation
between the parties, the SEC have no jurisdiction over the case.
The doctrine of corporation by estoppel advanced by respondent cannot
override jurisdictional requirements. Jurisdiction is fixed by law and is not subject
to the agreement of the parties. Corporation by estoppels is founded on
123

principles of equity and is designed to prevent injustice and unfairness. It applies


when persons assume to form a corporation and exercise corporate functions
and enter into business relations with third persons. When there is no third
person involved and the conflict arises only among those assuming the form of a
corporation, who therefore know that it has not been registered, there is no
corporation by estoppel

124

LIM TONG LIM


VS.
PHILIPPINE FISHING GEAR INDUSTRIES INC.
317 SCRA 728 (1999)
FACTS OF THE CASE
This case is petition for review on Certiorari. On behalf of "Ocean Quest
Fishing Corporation," Antonio Chua and Peter Yao entered into a Contract dated
7 February 1990, for the purchase of fishing nets of various sizes from the
Philippine Fishing Gear Industries, Inc. (PFGI). They claimed that they were
engaged in a business venture with Lim Tong Lim, who however was not a
signatory to the agreement. The total price of the nets amounted to P532,045.
400 pieces of floats worth P68, 000 were also sold to the Corporation. The
buyers, however, failed to pay for the fishing nets and the floats; hence, PFGI
filed a collection suit against Chua, Yao and Lim Tong Lim with a prayer for a writ
of preliminary attachment. The suit was brought against the three in their
capacities as general partners, on the allegation that "Ocean Quest Fishing
Corporation" was a nonexistent corporation as shown by a Certification from the
Securities and Exchange Commission.
Instead of answering the Complaint, Chua filed a Manifestation admitting
his liability and requesting a reasonable time within which to pay. He also turned
over to PFGI some of the nets which were in his possession. Peter Yao filed an
Answer, after which he was deemed to have waived his right to cross-examine
witnesses and to present evidence on his behalf, because of his failure to appear
in subsequent hearings. Lim Tong Lim, on the other hand, filed an Answer with
Counterclaim and Cross claim and moved for the lifting of the Writ of Attachment.
The trial court maintained the Writ, and upon motion of PFGI, ordered the sale of
the fishing nets at a public auction. PFGI won the bidding and deposited with the
said court the sales proceeds of P900, 000. On 18 November 1992, the trial court
rendered its Decision, ruling in favor of PFGI and that Chua, Yao and Lim, as
general partners, were jointly liable to pay PFGI. Lim appealed to the Court of
Appeals (CA) which, affirmed the RTC. Hence, Lim filed the Petition for Review
on Certiorari arguing that under the doctrine of corporation by estoppel, liability
can be imputed only to Chua and Yao, and not to him.

ISSUE
Whether Lim should be held jointly liable with Chua and Yao under the Doctrine
of Corporation by estoppel.

RULING
Yes. The Supreme Court held that although technically, it is true that petitioner
did not directly act on behalf of the corporation. Still, a person who has reaped
the benefits of a contract entered into by persons with whom he previously had
an existing relationship is deemed to be part of said association and is covered
by the scope of the doctrine of corporation by estoppel.

125

INTERNATIONAL EXPRESS TRAVEL & TOUR SERVICES, INC.


Vs.
OF APPEALS
G.R.NO. 119002
FACTS OF THE CASE
In 1989, International Express Travel & Tour Services, Inc. (IETTI), offered
to the Philippine Football Federation (PFF) its travel services for the South East
Asian Games. PFF, through Henri Kahn, its president, agreed. IETTI then
delivered the plane tickets to PFF, PFF in turn made a down payment. However,
PFF was not able to complete the full payment in subsequent installments
despite repeated demands from IETTI. IETTI then sued PFF and Kahn was
impleaded as a co-defendant.
Kahn averred that he should not be impleaded because he merely acted
as an agent of PFF which he averred is a corporation with separate and distinct
personality from him. The trial court ruled against Kahn and held him personally
liable for the said obligation (PFF was declared in default for failing to file an
answer). The trial court ruled that Kahn failed to prove that PFF is a corporation.
The Court of Appeals however reversed the decision of the trial court. The Court
of Appeals took judicial notice of the existence of PFF as a national sports
association; that as such, PFF is empowered to enter into contracts through its
agents; that PFF is therefore liable for the contract entered into by its agent
Kahn. The CA further ruled that IETTI is in estoppel; that it cannot now deny the
corporate existence of PFF because it had contracted and dealt with PFF in such
a manner as to recognize and in effect admit its existence.
.
ISSUE
Whether or not the Court of Appeals is correct.
RULING
No. PFF, upon its creation, is not automatically considered a national
sports association. It must first be recognized and accredited by the Philippine
Amateur Athletic Federation and the Department of Youth and Sports
Development. This fact was never substantiated by Kahn. As such, PFF is
considered as an unincorporated sports association. And under the law, any
person acting or purporting to act on behalf of a corporation which has no valid
existence assumes such privileges and becomes personally liable for contract
entered into or for other acts performed as such agent. Kahn is therefore
personally liable for the contract entered into by PFF with IETTI.
There is also no merit on the finding of the CA that IETTI is in estoppel.
The application of the doctrine of corporation by estoppel applies to a third party
only when he tries to escape liability on a contract from which he has benefited
on the irrelevant ground of defective incorporation. In the case at bar, IETTI is not
trying to escape liability from the contract but rather is the one claiming from the
contract.

126

SAWADJAAN
Vs.
COURT OF APPEALS,
G.R. No. 141735
FACTS OF THE CASE
Petitioner Sappari K. Sawadjaan was among the first employees of the
Philippine Amanah Bank (PAB) when it was created by virtue of Presidential
Decree No. 264. Sometime in 1988, while still designated as
appraiser/investigator, he was assigned to inspect the properties offered as
collaterals by Compressed Air Machineries and Equipment Corporation
(CAMEC) for a credit line of Five Million Pesos (P5,000,000.00). The properties
consisted of two parcels of land covered by Transfer Certificates of Title (TCTs).
On the basis of his Inspection and Appraisal Report, the PAB granted the loan
application. When the loan matured, CAMEC requested an extension to repay
the loan.
In January 1990, Congress passed Republic Act 6848 creating the AIIBP
and repealing P.D. No. 264 (which created the PAB). All assets, liabilities and
capital accounts of the PAB were transferred to the AIIBP, and the existing
personnel of the PAB were to continue to discharge their functions unless
discharged. In the ensuing reorganization, Sawadjaan was among the personnel
retained by the AIIBP.
When CAMEC failed to pay despite the given extension, the bank, now
referred to as the AIIBP, discovered that one of the TCT was spurious, the
property described therein non-existent, and that the property covered by the
other TCT had a prior existing mortgage.
The Board of Directors of the AIIBP created an Investigating Committee to
look into the CAMEC transaction, which had cost the bank Six Million Pesos (P6,
000,000.00) in losses. The Board of Directors of AIIBP adopted Resolution No.
2309 finding petitioner guilty of Dishonesty in the Performance of Official Duties
and/or Conduct Prejudicial to the Best Interest of the Service and imposing the
penalty of Dismissal from the Service. On reconsideration, they adopted the
Resolution No. 2332 reducing the penalty imposed on petitioner from dismissal to
suspension for a period of six (6) months and one (1) day.
Petitioner filed a notice of appeal to the Merit System Protection Board
(MSPB). The CSC adopted Resolution No. 94-4483 dismissing the appeal for
lack of merit and affirming Resolution No. 2309 of the Board of Directors of
AIIBP. The CSC adopted Resolution No. 95-2574 denying petitioners Motion for
Reconsideration. Hence this petition for certiorari under Rule 65 of the Rules of
Court.
ISSUE
Whether or not the failure of AIIBP to file its by-laws within the period prescribed
results to a nullity of all actions and proceedings it has initiated.
RULING
Petitioners efforts are unavailing, and we deny his petition for its
procedural and substantive flaws. Petitioners recurrent argument, tenuous at its
127

very best, is premised on the fact that since respondent AIIBP failed to file its bylaws within the designated 60 days from the effectively of Rep. Act No. 6848, all
proceedings initiated by AIIBP and all actions resulting therefrom are a patent
nullity. Petitioner already raised the question of AIIBPs corporate existence and
lack of jurisdiction in his Motion for New Trial/Motion for Reconsideration and was
denied by the Court of Appeals. Despite the volume of pleadings he has
submitted thus far, he has added nothing substantial to his arguments.
The AIIBP was created by Rep. Act No. 6848. It has a main office where it
conducts business, has shareholders, corporate officers, a board of directors,
assets, and personnel. It is, in fact, here represented by the Office of the
Government Corporate Counsel, "the principal law office of government-owned
corporations, one of which is respondent bank." At the very least, by its failure to
submit its by-laws on time, the AIIBP may be considered a de facto corporation
whos right to exercise corporate powers may not be inquired into collaterally in
any private suit to which such corporations may be a party.
Moreover, a corporation which has failed to file its by-laws within the
prescribed period does not ipso facto lose its powers as such. The SEC Rules on
Suspension/Revocation of the Certificate of Registration of Corporations, details
the procedures and remedies that may be availed of before an order of
revocation can be issued. There is no showing that such a procedure has been
initiated in this case.
Wherefore, the petition is dismissed

128

GOKONGWEI
VS.
SECURITIES AND EXCHANGE COMMISSION
GR L-45911, 11 April 1979
FACTS OF THE CASE
SEC Case 1375 - On 22 October 1976, John Gokongwei Jr., as
stockholder of San Miguel Corporation, filed with the Securities and Exchange
Commission (SEC) a petition for "declaration of nullity of amended by-laws,
cancellation of certificate of filing of amended by-laws, injunction and damages
with prayer for a preliminary injunction" against the majority of the members of
the Board of Directors and San Miguel Corporation as an unwilling petitioner. As
a first cause of action, Gokongwei alleged that on 18 September 1976, Andres
Soriano, Jr., Jose M. Soriano, Enrique Zobel, Antonio Roxas, Emeterio Buao,
Walthrode B. Conde, Miguel Ortigas, and Antonio Prieto amended by bylaws of
the corporation, basing their authority to do so on a resolution of the stockholders
adopted on 13 March 1961, when the outstanding capital stock of the corporation
was only P70,139.740.00, divided into 5,513,974 common shares at P10.00 per
share and 150,000 preferred shares at P100.00 per share. At the time of the
amendment, the outstanding and paid up shares totaled 30,127,043, with a total
par value of P301, 270,430.00. It was contended that according to section 22 of
the Corporation Law and Article VIII of the by-laws of the corporation, the power
to amend, modify, repeal or adopt new by-laws may be delegated to the Board of
Directors only by the affirmative vote of stockholders representing not less than
2/3 of the subscribed and paid up capital stock of the corporation, which 2/3
should have been computed on the basis of the capitalization at the time of the
amendment. Since the amendment was based on the 1961 authorization,
Gokongwei contended that the Board acted without authority and in usurpation of
the power of the stockholders. As a second cause of action, it was alleged that
the authority granted in 1961 had already been exercised in 1962 and 1963, after
which the authority of the Board ceased to exist. As a third cause of action,
Gokongwei averred that the membership of the Board of Directors had changed
since the authority was given in 1961, there being 6 new directors. As a fourth
cause of action, it was claimed that prior to the questioned amendment,
Gokongwei had all the qualifications to be a director of the corporation, being a
substantial stockholder thereof; that as a stockholder, Gokongwei had acquired
rights inherent in stock ownership, such as the rights to vote and to be voted
upon in the election of directors; and that in amending the by-laws, Soriano, et.
Al. purposely provided for Gokongwei's disqualification and deprived him of his
vested right as afore-mentioned; hence the amended by-laws are null and void.
As additional causes of action, it was alleged that corporations have no inherent
power to disqualify a stockholder from being elected as a director and, therefore,
the questioned act is ultra vires and void; that Andres M. Soriano, Jr. and/or Jose
M. Soriano, while representing other corporations, entered into contracts
(specifically a management contract) with the corporation, which was avowed
because the questioned amendment gave the Board itself the prerogative of
determining whether they or other persons are engaged in competitive or
antagonistic business; that the portion of the amended by-laws which states that
129

in determining whether or not a person is engaged in competitive business, the


Board may consider such factors as business and family relationship, is
unreasonable and oppressive and, therefore, void; and that the portion of the
amended by-laws which requires that "all nominations for election of directors
shall be submitted in writing to the Board of Directors at least five (5) working
days before the date of the Annual Meeting" is likewise unreasonable and
oppressive. It was, therefore, prayed that the amended by-laws be declared null
and void and the certificate of filing thereof be cancelled, and that Soriano, et. al.
be made to pay damages, in specified amounts, to Gokongwei. On 28 October
1976, in connection with the same case, Gokongwei filed with the Securities and
Exchange Commission an "Urgent Motion for Production and Inspection of
Documents", alleging that the Secretary of the corporation refused to allow him to
inspect its records despite request made by Gokongwei for production of certain
documents enumerated in the request, and that the corporation had been
attempting to suppress information from its stockholders despite a negative reply
by the SEC to its query regarding their authority to do so. The motion was
opposed by Soriano, ET. Al. The Corporation, Soriano, et. al. filed their answer,
and their opposition to the petition, respectively. Meanwhile, on 10 December
1976, while the petition was yet to be heard, the corporation issued a notice of
special stockholders' meeting for the purpose of "ratification and confirmation of
the amendment to the By-laws", setting such meeting for 10 February 1977. This
prompted Gokongwei to ask the SEC for a summary judgment insofar as the first
cause of action is concerned, for the alleged reason that by calling a special
stockholders' meeting for the aforesaid purpose, Soriano, et. al. admitted the
invalidity of the amendments of 18 September 1976. The motion for summary
judgment was opposed by Soriano, ET. al. Pending action on the motion,
Gokongwei filed an "Urgent Motion for the Issuance of a Temporary Restraining
Order", praying that pending the determination of Gokongwei's application for the
issuance of a preliminary injunction and or Gokongwei's motion for summary
judgment, a temporary restraining order be issued, restraining Soriano, et. al.
from holding the special stockholders' meeting as scheduled. This motion was
duly opposed by Soriano, ET. Al. On 10 February 1977, Cremation issued an
order denying the motion for issuance of temporary restraining order. After
receipt of the order of denial, Soriano, et. al. conducted the special stockholders'
meeting wherein the amendments to the by-laws were ratified. On 14 February
1977, Gokongwei filed a consolidated motion for contempt and for nullification of
the special stockholders' meeting. A motion for reconsideration of the order
denying Gokongwei's motion for summary judgment was filed by Gokongwei
before the SEC on 10 March 1977.
SEC Case 1423 - Gokongwei alleged that, having discovered that the
corporation has been investing corporate funds in other corporations and
businesses outside of the primary purpose clause of the corporation, in violation
of section 17-1/2 of the Corporation Law, he filed with SEC, on 20 January 1977,
a petition seeking to have Andres M. Soriano, Jr. and Jose M. Soriano, as well as
the corporation declared guilty of such violation, and ordered to account for such
investments and to answer for damages. On 4 February 1977, motions to
dismiss were filed by Soriano, et. al., to which a consolidated motion to strike and
to declare Soriano, et. al. in default and an opposition ad abundantiorem
130

cautelam were filed by Gokongwei. Despite the fact that said motions were filed
as early as 4 February 1977, the Commission acted thereon only on 25 April
1977, when it denied Soriano, et. al.'s motions to dismiss and gave them two (2)
days within which to file their answer, and set the case for hearing on April 29
and May 3, 1977. Soriano, et. al. issued notices of the annual stockholders'
meeting, including in the Agenda thereof, the "reaffirmation of the authorization to
the Board of Directors by the stockholders at the meeting on 20 March 1972 to
invest corporate funds in other companies or businesses or for purposes other
than the main purpose for which the Corporation has been organized, and
ratification of the investments thereafter made pursuant thereto." By reason of
the foregoing, on 28 April 1977, Gokongwei filed with the SEC an urgent motion
for the issuance of a writ of preliminary injunction to restrain Soriano, et. al. from
taking up Item 6 of the Agenda at the annual stockholders' meeting, requesting
that the same be set for hearing on 3 May 1977, the date set for the second
hearing of the case on the merits. The SEC, however, cancelled the dates of
hearing originally scheduled and reset the same to May 16 and 17, 1977, or after
the scheduled annual stockholders' meeting. For the purpose of urging the
Commission to act, Gokongwei filed an urgent manifestation on 3 May 1977, but
this notwithstanding, no action has been taken up to the date of the filing of the
instant petition. Gokongwei filed a petition for petition for certiorari, mandamus
and injunction, with prayer for issuance of writ of preliminary injunction, with the
Supreme Court, alleging that there appears a deliberate and concerted inability
on the part of the SEC to act.
ISSUE
1. Whether the corporation has the power to provide for the (additional)
qualifications of its directors.
2. Whether the disqualification of a competitor from being elected to the Board of
Directors is a reasonable exercise of corporate authority.
3. Whether the SEC gravely abused its discretion in denying Gokongwei's
request for an examination of the records of San Miguel International, Inc., a fully
owned subsidiary of San Miguel Corporation.
4. Whether the SEC gravely abused its discretion in allowing the stockholders of
San Miguel Corporation to ratify the investment of corporate funds in a foreign
corporation.
RULING
First Issue: It is recognized by all authorities that "every corporation has the
inherent power to adopt by-laws 'for its internal government, and to regulate the
conduct and prescribe the rights and duties of its members towards itself and
among themselves in reference to the management of its affairs.'" In this
jurisdiction under section 21 of the Corporation Law, a corporation may prescribe
in its by-laws "the qualifications, duties and compensation of directors, officers
and employees." This must necessarily refer to a qualification in addition to that
specified by section 30 of the Corporation Law, which provides that "every
131

director must own in his right at least one share of the capital stock of the stock
corporation of which he is a director." Any person "who buys stock in a
corporation does so with the knowledge that its affairs are dominated by a
majority of the stockholders and that he impliedly contracts that the will of the
majority shall govern in all matters within the limits of the act of incorporation and
lawfully enacted by-laws and not forbidden by law." To this extent, therefore, the
stockholder may be considered to have "parted with his personal right or privilege
to regulate the disposition of his property which he has invested in the capital
stock of the corporation, and surrendered it to the will of the majority of his fellow
incorporators. It cannot therefore be justly said that the contract, express or
implied, between the corporation and the stockholders is infringed by any act of
the former which is authorized by a majority." Pursuant to section 18 of the
Corporation Law, any corporation may amend its articles of incorporation by a
vote or written assent of the stockholders representing at least two-thirds of the
subscribed capital stock of the corporation. If the amendment changes,
diminishes or restricts the rights of the existing shareholders, then the dissenting
minority has only one right, viz.: "to object thereto in writing and demand payment
for his share." Under section 22 of the same law, the owners of the majority of
the subscribed capital stock may amend or repeal any by-law or adopt new bylaws. It cannot be said; therefore, that Gokongwei has a vested right to be
elected director, in the face of the fact that the law at the time such right as
stockholder was acquired contained the prescription that the corporate charter
and the by-law shall be subject to amendment, alteration and modification.

Second Issue: Although in the strict and technical sense, directors of a private
corporation are not regarded as trustees, there cannot be any doubt that their
character is that of a fiduciary insofar as the corporation and the stockholders as
a body are concerned. As agents entrusted with the management of the
corporation for the collective benefit of the stockholders, "they occupy a fiduciary
relation, and in this sense the relation is one of trust." "The ordinary trust
relationship of directors of a corporation and stockholders is not a matter of
statutory or technical law. It springs from the fact that directors have the control
and guidance of corporate affairs and property and hence of the property
interests of the stockholders. Equity recognizes that stockholders are the
proprietors of the corporate interests and are ultimately the only beneficiaries
thereof." A director is a fiduciary. Their powers are powers in trust. He who is in
such fiduciary position cannot serve himself first and his cestuis second. He
cannot manipulate the affairs of his corporation to their detriment and in
disregard of the standards of common decency. He cannot by the intervention of
a corporate entity violate the ancient precept against serving two masters. He
cannot utilize his inside information and strategic position for his own preferment.
He cannot violate rules of fair play by doing indirectly through the corporation
what he could not do so directly. He cannot violate rules of fair play by doing
indirectly through the corporation what he could not do so directly. He cannot use
his power for his personal advantage and to the detriment of the stockholders
and creditors no matter how absolute in terms that power may be and no matter
how meticulous he is to satisfy technical requirements. For that power is at all
times subject to the equitable limitation that it may not be exercised for the
132

aggrandizement, preference, or advantage of the fiduciary to the exclusion or


detriment of the cestuis. The doctrine of "corporate opportunity" is precisely a
recognition by the courts that the fiduciary standards could not be upheld where
the fiduciary was acting for two entities with competing interests. This doctrine
rests fundamentally on the unfairness, in particular circumstances, of an officer or
director taking advantage of an opportunity for his own personal profit when the
interest of the corporation justly calls for protection. It is not denied that a
member of the Board of Directors of the San Miguel Corporation has access to
sensitive and highly confidential information, such as: (a) marketing strategies
and pricing structure; (b) budget for expansion and diversification; (c) research
and development; and (d) sources of funding, availability of personnel, proposals
of mergers or tie-ups with other firms. It is obviously to prevent the creation of an
opportunity for an officer or director of San Miguel Corporation, who is also the
officer or owner of a competing corporation, from taking advantage of the
information which he acquires as director to promote his individual or corporate
interests to the prejudice of San Miguel Corporation and its stockholders, that the
questioned amendment of the by-laws was made. Certainly, where two
corporations are competitive in a substantial sense, it would seem improbable, if
not impossible, for the director, if he were to discharge effectively his duty, to
satisfy his loyalty to both corporations and place the performance of his
corporation duties above his personal concerns. The offer and assurance of
Gokongwei that to avoid any possibility of his taking unfair advantage of his
position as director of San Miguel Corporation, he would absent himself from
meetings at which confidential matters would be discussed, would not detract
from the validity and reasonableness of the by-laws involved. Apart from the
impractical results that would ensue from such arrangement, it would be
inconsistent with Gokongwei's primary motive in running for board membership
which is to protect his investments in San Miguel Corporation. More important,
such a proposed norm of conduct would be against all accepted principles
underlying a director's duty of fidelity to the corporation, for the policy of the law is
to encourage and enforce responsible corporate management.
Third Issue: Pursuant to the second paragraph of section 51 of the Corporation
Law, "(t) he record of all business transactions of the corporation and minutes of
any meeting shall be open to the inspection of any director, member or
stockholder of the corporation at reasonable hours." The stockholder's right of
inspection of the corporation's books and records is based upon their ownership
of the assets and property of the corporation. It is, therefore, an incident of
ownership of the corporate property, whether this ownership or interest be
termed an equitable ownership, a beneficial ownership, or a quasi-ownership.
This right is predicated upon the necessity of self-protection. It is generally held
by majority of the courts that where the right is granted by statute to the
stockholder, it is given to him as such and must be exercised by him with respect
to his interest as a stockholder and for some purpose germane thereto or in the
interest of the corporation. In other words, the inspection has to be germane to
the petitioner's interest as a stockholder, and has to be proper and lawful in
character and not inimical to the interest of the corporation. The "general rule that
stockholders are entitled to full information as to the management of the
corporation and the manner of expenditure of its funds, and to inspection to
133

obtain such information, especially where it appears that the company is being
mismanaged or that it is being managed for the personal benefit of officers or
directors or certain of the stockholders to the exclusion of others." While the right
of a stockholder to examine the books and records of a corporation for a lawful
purpose is a matter of law, the right of such stockholder to examine the books
and records of a wholly owned subsidiary of the corporation in which he is a
stockholder is a different thing. Stockholders are entitled to inspect the books and
records of a corporation in order to investigate the conduct of the management,
determine the financial condition of the corporation, and generally take an
account of the stewardship of the officers and directors. herein, considering that
the foreign subsidiary is wholly owned by San Miguel Corporation and, therefore,
under Its control, it would be more in accord with equity, good faith and fair
dealing to construe the statutory right of petitioner as stockholder to inspect the
books and records of the corporation as extending to books and records of such
wholly owned subsidiary which are in the corporation's possession and control.
Fourth Issue: Section 17-1/2 of the Corporation Law allows a corporation to
"invest its funds in any other corporation or business or for any purpose other
than the main purpose for which it was organized" provided that its Board of
Directors has been so authorized by the affirmative vote of stockholders holding
shares entitling them to exercise at least two-thirds of the voting power. If the
investment is made in pursuance of the corporate purpose, it does not need the
approval of the stockholders. It is only when the purchase of shares is done
solely for investment and not to accomplish the purpose of its incorporation that
the vote of approval of the stockholders holding shares entitling them to exercise
at least two-thirds of the voting power is necessary. As stated by the corporation,
the purchase of beer manufacturing facilities by SMC was an investment in the
same business stated as its main purpose in its Articles of Incorporation, which is
to manufacture and market beer. It appears that the original investment was
made in 1947-1948, when SMC, then San Miguel Brewery, Inc., purchased a
beer brewery in Hong Kong (Hong Kong Brewery & Distillery, Ltd.) for the
manufacture and marketing of San Miguel beer thereat. Restructuring of the
investment was made in 1970-1971 thru the organization of SMI in Bermuda as
tax free reorganization. Assuming arguendo that the Board of Directors of SMC
had no authority to make the assailed investment, there is no question that a
corporation, like an individual, may ratify and thereby render binding upon it the
originally unauthorized acts of its officers or other agents. This is true because
the questioned investment is neither contrary to law, morals, public order or
public policy. It is a corporate transaction or contract which is within the corporate
powers, but which is defective from a purported failure to observe in its execution
the requirement of the law that the investment must be authorized by the
affirmative vote of the stockholders holding two-thirds of the voting power. This
requirement is for the benefit of the stockholders. The stockholders for whose
benefit the requirement was enacted may, therefore, ratify the investment and its
ratification by said stockholders obliterates any defect which it may have had at
the outset. Besides, the investment was for the purchase of beer manufacturing
and marketing facilities which is apparently relevant to the corporate purpose.
The mere fact that the corporation submitted the assailed investment to the
stockholders for ratification at the annual meeting of 10 May 1977 cannot be
134

construed as an admission that the corporation had committed an ultra vires act,
considering the common practice of corporations of periodically submitting for the
ratification of their stockholders the acts of their directors, officers and managers.

135

SALES
Vs.
SECURITIES AND EXCHANGE COMMISSION
G.R.No. L-54330
FACTS OF THE CASE
Sipalay Mining Exploration Corporation (SMEC) sold 200M common
shares of its capital stock in the amount of 2.6M to State Investment House Inc.
(SIHI) under a Sales Agreement providing that the sale shall be only up to 5M
shares per buyer. SIHI requested for the transfer of the 200M shares to Anselmo
Trinidad Co. (ATCO) to which SMEC complied. During the time that ATCO held
the shares, it voted them in the stockholders meetings of SMEC. ATCO in turn
sold 198,500,000 of the shares to respondent Vulcan Industrial and Mining Corp.
(VIMC). SMEC was requested by ATCO to transfer the 198,500,000 shares to
the name of VIMC. By resolution of the Board of Directors of Sipalay Mining, its
President was directed to sign the certificate of stock that would effect the
transfer. Eight days prior to the scheduled annual stockholders' meeting of SMEC
on July 18,1979, petitioners filed before the SEC a petition to nullify the sale of
the shares to VIMC, with a prayer for the issuance of a writ of preliminary
injunction to enjoin VIMC from voting the shares. VIMC was temporarily
restrained and the meeting was held without participation of VIMCs shares and
Board of Directors was elected only from the group of petitioners.
In VIMCs answer, it questioned the said election. SEC denied the petition
as well as motion to dismiss and lifted the Restraining it issued earlier and
allowed the shares of VIMC to be counted in determining the quorum of the 1980
annual stockholders meeting, which was already near, and the same shares
were allowed to vote and be voted for. Before the Supreme Court, petitioners
contended that the SEC gravely abuse its discretion in not enjoining the
participation of VIMC in the 1980 election considering that the sale of the shares
to VIMC was null and void as it was done in violation of the Sales Agreement on
the limit of shares to be sold to each buyer and that VIMCs ownership of the
shares is contrary to Sec. 13 (5-A) of the old corporation law.

ISSUE

Whether SEC acted with grave abuse of discretion in not permanently


enjoining VIMC in voting.

RULING
No.
The court found no grave abuse of discretion on the part of the SEC in
note restraining VIMC. It adopted the SEC resolution stating that the sale of the
shares of stock had long been perfected and is presumed valid until declared
otherwise. As against this presumption, petitioners prayer for injunction cannot
prevail as the issue of the validity of the sale is still to be resolved by the SEC.
Considering that the shares constitute the majority, it is more equitable that
the same be allowed to vote rather than be enjoined. As it has been ruled the
removal of a majority stockholder from the management of the corporation nad/or
136

dissolution of a corporation in a suit filed by a minority stockholder is a drastic


measure. It should be resorted to only when the necessity is clear. With more
reason, the court will not deprive a stockholder of his right to vote his shares in
the annual stockholders meeting, except upon a clear showing of its lawful
denial under the articles of incorporation or by-laws of the corporation, as it is a
right inherent in stock ownership.

137

PENA
VS
COURT OF APPEALS
G.R. No. 91478
FACTS OF THE CASE
PAMPANGA BUS CO., INC. (PAMBUSCO) is the owner of the three lots in
dispute. PAMBUSCO mortgaged the lots to the Development Bank of the
Philippines
(DBP),
which
were
later
on
foreclosed.
Rosita Pea was awarded the lots in a foreclosure sale for being the highest
bidder. The certificate of sale was later issued to her and registered in her name.
Subsequently, the Board of Directors of PAMBUSCO, through three out of its five
directors, issued a resolution to assign its right of redemption over the lots in
favor of any interested party. The right of redemption was later on assigned to
Marcelino
Enriquez,
who
redeemed
the
property.
Enriquez then sold the lots to spouses Rising T. Yap and Catalina Lugue-Yap.
Meanwhile, a case involving the validity of the sale to the spouses Yap was
pending, and despite the protestations of Pea as to validity of the PAMBUSCO's
assignment of the right of redemption, the lots were somehow registered in the
name of spouses Yap. Despite the registration of the lots to spouses Yap, Pea
retained
possession
of
the
property.
Spouses Yap sought to recover the possession of the lots from Pea. The latter
countered that she is now the legitimate owner of the subject lands for having
purchased the same in a foreclosure proceeding instituted by the DBP against
PAMBUSCO and no valid redemption having been effected within the period
provided
by
law.
The defense was that since the deed of assignment executed by PAMBUSCO in
favor of Enriquez was void ab initio for being an ultra vires act of its board of
directors and for being without any valuable consideration, it could not have had
any
legal
effect.
(It should be noted that the by-laws of PAMBUSCO provide that four out of five
directors must be present in a special meeting of the board to constitute a
quorum, and that the corporation has already ceased to operate.)
CFI ruled in favor of Petitioner Pea, but the same was overturned by the CA.
ISSUE
Whether Pea is entitled to the lots.

RULING
The by-laws of a corporation are its own private laws, which substantially have
the same effect as the laws of the corporation. They are in effect, written, into the
charter. In this sense they become part of the fundamental law of the corporation
with which the corporation and its directors and officers must comply.
138

Apparently, only three (3) out of five (5) members of the board of directors of
respondent PAMBUSCO convened by virtue of a prior notice of a special
meeting. There was no quorum to validly transact business since it is required
under its by-laws that at least four (4) members must be present to constitute a
quorum
in
a
special
meeting
of
the
board
of
directors.
Under Section 25 of the Corporation Code of the Philippines, the articles of
incorporation or by-laws of the corporation may fix a greater number than the
majority of the number of board members to constitute the quorum necessary for
the valid transaction of business. Any number less than the number provided in
the articles or by-laws therein cannot constitute a quorum and any act therein
would not bind the corporation; all that the attending directors could do is to
adjourn.
Moreover, the records show that respondent PAMBUSCO ceased to operate for
about 25 years prior to the board meeting. Being a dormant corporation for
several years, it was highly irregular, for a group of three (3) individuals
representing themselves to be the directors of respondent PAMBUSCO to pass a
resolution disposing of the only remaining asset of the corporation in favor of a
former
corporate
officer.
As a matter of fact, the three (3) alleged directors who attended the special
meeting on November 19, 1974 were not listed as directors of respondent
PAMBUSCO in the latest general information sheet. Similarly, the latest list of
stockholders of respondent PAMBUSCO on file with the SEC doeight of
respondent PAMBUSCO by virtue of the questioned resolution was not approved
by the required number of stockholders, the said resolution, as well as the
subsequent
assignment
and
sale,
were
null
and
void.
Lastly, for lack of consideration, the assignment should be construed as a
donation. Under Article 725 of the Civil Code, in order to be valid, such a
donation must be made in a public document and the acceptance must be made
in the same or in a separate instrument. In the latter case, the donor shall be
notified of the acceptance in an authentic form and such step must be noted in
both instruments. Since assignment to Enriquez shows that there was no
acceptance of the donation in the same and in a separate document, the said
deed of assignment is thus void ab initios not show that the said alleged directors
were among the stockholders of respondent PAMBUSCO, in contravention of the
rule requiring a director to own one (1) share in their to qualify as director of a
corporation.
Further, under the Corporation Law, the sale or disposition of any and/or
substantially all properties of the corporation requires, in addition to a proper
board resolution, the affirmative votes of the stockholders holding at least twothirds (2/3) of the voting power in the corporation in a meeting duly called for that
purpose. This was not complied with in the case at bar.
At the time of the passage of the questioned resolution, respondent PAMBUSCO
was insolvent and its only remaining asset was its right of redemption over the
subject properties. Since the disposition of said redemption are. Right of
respondent PAMBUSCO by virtue of the questioned resolution was not approved
by the required number of stockholders, the said resolution, as well as the
subsequent
assignment
and
sale,
were
null
and
void.
Lastly, for lack of consideration, the assignment should be construed as a
139

donation. Under Article 725 of the Civil Code, in order to be valid, such a
donation must be made in a public document and the acceptance must be made
in the same or in a separate instrument. In the latter case, the donor shall be
notified of the acceptance in an authentic form and such step must be noted in
both instruments. Since assignment to Enriquez shows that there was no
acceptance of the donation in the same and in a separate document, the said
deed of assignment is thus void as initio.

140

VISAYAN
VS.
NATIONAL LABOR RELATIONS COMMISSION
196 SCRA 410
GR.NO.69999. APRIL 30, 1991
FACTS OF THE CASE
Aquilino Rivera organized a corporation he named FUJIYAMA HOTEL
ANG RESTAURANT. Thereafter, he opened a Japanese establishment bearing
the name of the corporation, and hired Isamu Kawasaki as its chef and
supervisor.
Lourdes Jureideni and Milagros Tsuchiya allegedly pretending to be
stockholders of the corporation filed a case with the RTC of manila for control of
the corporation. RTC issued a mandatory preliminary injunction transferring all
the possession and control of the corporation to both Jureideni and Tsuchiya.
Thereafter, the two replaced almost all of the employees of the corporation.
Upon appeal by Rivera and Akasako to the Sc, the Court reversed the decision
of the RTC. Subsequently, the management and control of the corporation was
regained by Rivera and Akasako. They dismissed the employees hired by
Jureideni and Tsuchiya, and reinstated all their original employees. The
dismissed employees filed a case for illegal dismissal against the corporation at
the NLRC. The labor arbiter rendered a decision ordering the corporation to
reinstate the dismissed employees and pay their back wages. The corporation
then appealed the decision of the arbiter to the commissioner of NLRC who
reversed the said decision. The NLRC ruled that the real employers of the
dismissed employees were Jureideni and Tsuchiya, thereby absolving the
corporation from any liability. Hence, the dismissed employees appealed to the
SC.
ISSUE
Whether there is privity between the corporation and the dismissed
employees as a basis to establish an employer-employee relationship.
RULING
There is no privity between the corporation and the petitioners- the
dismissed employees, to establish as basis for an employer-employee
relationship which in turn could be a basis to bind the corporation for any liability
arising from the dismissal.
Section 23 of the Corporation Code expressly provides that the corporation
can only act through its board of directors. It is settles that contracts entered into
between a corporation and a third party must be made by or under the authority
of the board of directors and not just the stockholders. The action of the
stockholders in such cases are only advisory and not in any way binding to the
corporation.
A corporation through its board of directors may legally delegate some of it
functions and powers to its officers, committees or agents appointed by it. In the
141

absence of authority from the board, no person, not even the officers of the
corporation, may validly bind the corporation.
The acts of Jureideni and Tsuchiya for and in behalf of the corporation as
alleged stockholders, and without the authority from the board, can never bind
the corporation in any way.

142

LEE
Vs
CA, SACOBA MANUFACTURING CORP. GONZALES, JR. and GONZALES,
G.R. No. 93695,
February 4, 1992
FACTS OF THE CASE
In 1985, a complaint for a sum of money was filed by the International
Corporate Bank, Inc. (ICB) against the private respondents who, in turn, filed a
third party complaint against ALFA and the petitioners. The petitioners filed a
motion to dismiss informing the court that the summons for ALFA was
erroneously served upon them considering that the management of ALFA had
been transferred to the DBP. In a manifestation dated July 22, 1988, the DBP
claimed that it was not authorized to receive summons on behalf of ALFA since
the DBP had not taken over the company which has a separate and distinct
corporate personality and existence. Petitioners submit that Rule 14, section 13
of the Revised Rules of Court is not applicable since they were no longer officers
of ALFA and that the private respondents should have availed of another mode
of service under Rule 14, Section 16 of the said Rules, i.e., through publication to
effect proper service upon ALFA. However, private respondents argued that the
voting trust agreement dated March 11, 1981 did not divest the petitioners of
their positions as president and executive vice-president of ALFA so that service
of summons upon ALFA through the petitioners as corporate officers was proper.
In 1989, the trial court upheld the validity of the service of summons on
ALFA through the petitioners. Upon motion for reconsideration, the trial court
reversed itself by setting aside its previous Order. The private respondents filed a
petition for certiorari in which the Court of Appeals granted and set aside the
orders of the trial court.
ISSUE
Whether or not the execution of the voting trust agreement deprives the
stockholders of their positions in the corporation, thus no longer authorized to
receive service of summons for and behalf of the private domestic corporation.

RULING
Yes. Every director must own at least one (1) share of the capital stock of
the corporation of which he is a director which share shall stand in his name on
the books of the corporation. Any director who ceases to be the owner of at least
one (1) share of the capital stock of the corporation of which he is a director shall
thereby cease to be a director.
Under the old Corporation Code, the eligibility of a director, strictly
speaking, cannot be adversely affected by the simple act of such director being a
party to a voting trust agreement inasmuch as he remains owner (although
beneficial or equitable only) of the shares subject of the voting trust agreement
pursuant to which a transfer of the stockholder's shares in favor of the trustee is
required (section 36 of the old Corporation Code). No disqualification arises by
virtue of the phrase "in his own right" provided under the old Corporation Code.
With the omission of the phrase "in his own right" the election of trustees and
other persons who in fact are not the beneficial owners of the shares registered
in their names on the books of the corporation becomes formally legalized (see
143

Campos and Lopez-Campos, supra, p. 296) Hence, this is a clear indication that
in order to be eligible as a director, what is material is the legal title to, not
beneficial ownership of, the stock as appearing on the books of the corporation
(2 Fletcher, Cyclopedia of the Law of Private Corporations, section 300, p. 92
[1969] citing People v. Lihme, 269111. 351, 109 N.E. 1051)
voting trust agreement results in the separation of the voting rights of a
stockholder from his other rights such as the right to receive dividends and other
rights to which a stockholder may be entitled until the liquidation of the
corporation.
There can be no reliance on the inference that the five-year period of the
voting trust agreement in question had lapsed in 1986 so that the legal title to the
stocks covered by the said voting trust agreement ipso facto reverted to the
petitioners as beneficial owners pursuant to the 6th paragraph of section 59 of
the new Corporation Code which reads: "Unless expressly renewed, all rights
granted in a voting trust agreement shall automatically expire at the end of the
agreed period, and the voting trust certificates as well as the certificates of stock
in the name of the trustee or trustees shall thereby be deemed cancelled and
new certificates of stock shall be reissued in the name of the transferors." On the
contrary, it is manifestly clear from the terms of the voting trust agreement
between ALFA and the DBP that the duration of the agreement is contingent
upon the fulfillment of certain obligations of ALFA with the DBP.
Had the five-year period of the voting trust agreement expired in 1986, the
DBP would not have transferred all its rights, titles and interests in ALFA
"effective June 30, 1986" to the national government through the Asset
Privatization Trust (APT) as attested to in a Certification dated January 24, 1989
of the Vice President of the DBP's Special Accounts Department II. In the same
certification, it is stated that the DBP, from 1987 until 1989, had handled APT's
account which included ALFA's assets pursuant to a management agreement by
and between the DBP and APT (CA Rollo, p. 142) Hence, there is evidence on
record that at the time of the service of summons on ALFA through the
petitioners on August 21, 1987, the voting trust agreement in question was not
yet terminated so that the legal title to the stocks of ALFA, then, still belonged to
the DBP.

144

Citibank, N.A.
Vs.
Chua
FACTS OF THE CASE
1. Petitioner Citibank, N.A. is a foreign commercial banking corporation duly
licensed to do business in the Philippines. Private respondents spouses
Cresencio and Zenaida Velez, were good clients of petitioner banks
branch in Cebu until March 14, 1986 when they filed a complaint for
specific performance and damages against it before the RTC of Cebu.
2. During the date of the pre-trial conference, counsel for
petitioner bank appeared, presenting a special power of
attorney executed by Citibank o fficer Florencia Tarriela in favor of
petitioner banks counsel, J.P. Garcia and Associates, to represent and
bind petitioner bank at the pre -trial conference of the case at
bar. Inspite of this special power of attorney, counsel for private
respondents orally moved to declare petitioner bank as in default on the
ground that the special power of attorney was not executed by
the Board of Directors of Citibank.
ISSUE
Whether or not a resolution of the Board of Directors of a
corporation is always necessary for granting authority to an agent to
represent the corporation in court cases.
RULING
No. Just as a natural person may authorize another to do certain acts in
his behalf, so may the board of directors of a corporation validly delegate some
of its functions to individual officers or agents appointed by it.
Corporate powers may be directly conferred upon corporate officers or
agents by statute, the articles of incorporation, the by-laws or by resolution or
other act of the board of directors. In addition, an officer who is not a director may
also appoint other agents when so authorized by the by-laws or by the board of
directors.

145

GRACE CHRISTIAN HIGH SCHOOL


Vs.
THE COURT OF APPEALS
G.R. No. 108905
FACTS OF THE CASE
Petitioner is an educational institution at the Grace Village in Quezon City.
Respondent Grace Village Association Inc is the homeowners association of
Grace Village. It has existing by-laws which was already in effect in 1968.
Sometime in 1975, The BOD made a draft amending the by-laws providing that
the representative of Grace Christian High School shall have a permanent seat in
the board. Such draft however was never presented to the general membership
for approval. Nevertheless, the representative of Grace Christian High School
has been recognized as a permanent director of the board for 15 years from
1975 1989 preceding the decision of the present board members to reexamine
such practice. When the board denied petitioners request to be allowed
representation without election, petitioner brought an action for mandamus in the
Home Insurance and Guaranty Corporation, contending that it has acquired a
vested right as a permanent director of the association by virtue of the fact that
such practice continued for 15 years without any member of the association
registering any desire to remove petitioner from its automatic inclusion in the
board. HGIC ruled that allowing the automatic inclusion of a member
representative of petitioner as permanent director [was] contrary to law and the
registered by-laws of respondent association. The decision of HGIC was
subsequently affirmed by the appeals board. Petitioner appealed to the Court of
Appeals contending that the provisions of Sec 92 of the Corporation Code
provides only for the manner of election of the members of the board of trustees
of non-stock corporations which may be more than fifteen in number and which
manner of election is even subject to what is provided in the articles of
incorporation or by-laws of the association thus showing that said provisions [are]
not even mandatory. That allowing a member to occupy a permanent seat in the
board is not prohibited as there is no showing of prohibition in the code with
respect to such practice provided it is stipulated in the by-laws or Articles of
Incorporation. The appellate court affirmed the decision of the appeals board.
Thus this petition for review.
ISSUE
Whether the practice of tolerating the automatic inclusion of petitioner as a
permanent member of the Board of Directors of the Association without the
benefit of election is allowed under the law
RULING
No. The present Corporation Code (B.P. Blg. 68), which took effect on May 1,
1980,[12] similarly provides:
Section 23. The Board of Directors or Trustees. - Unless otherwise provided in
this Code, the corporate powers of all corporations formed under this Code shall
be exercised, all business conducted and all property of such corporations
controlled and held by the board of directors or trustees to be elected from
among the holders of stocks, or where there is no stock, from among the
members of the corporation, who shall hold office for one (1) year and until their
successors are elected and qualified. (Emphasis added)

146

This provision of the corporation law leave no room for doubt as to their meaning:
the board of directors of corporations must be elected from among the
stockholders or members.
Since the provision in question is contrary to law, the fact that for fifteen years it
has not been questioned or challenged but, on the contrary, appears to have
been implemented by the members of the association cannot forestall a later
challenge to its validity. Neither can it attain validity through acquiescence
because, if it is contrary to law, it is beyond the power of the members of the
association to waive its invalidity. For that matter the members of the association
may have formally adopted the provision in question, but their action would be of
no avail because no provision of the by-laws can be adopted if it is contrary to
law

147

NAGUIAT
Vs
NATIONAL LABOR RELATIONS COMMISSION
G.R. No. 116123 March 13, 1997
FACTS OF THE CASE
Private respondents, Leonardo Galang et al, were previously employed by
petitioner Clark Field Taxi, Inc (CFTI). CFTI held a concessionaire contract with
the Army Air Force Exchange Services (AAFES) for taxi services in the Clark Air
Base. Sergio Naguiat was CFTIs president, while Anton Naguiat was the vicepresident. CFTI is a family owned corporation. When the US military base was
phased out, the services of CFTI was also terminated. The drivers union of the
taxi services bargained for a severance pay to which they were granted P500 for
every year of service. Private respondents refused to accept their severance pay.
Through a new labor organization National Organization of Workingmen
the private respondents disaffiliated themselves from the drivers' union, filed a
complaint against Sergio F. Naguiat doing business under the name and style
Sergio F. Naguiat Enterprises, Inc. and CFTI with Antolin T. Naguiat as vice
president and general manager, as party respondent., for payment of separation
pay due to termination/phase-out.
Private respondents alleged that they were regular employees of Naguiat
Enterprises, although their individual applications for employment were approved
by CFTI. They claimed to have been assigned to Naguiat Enterprises after
having been hired by CFTI and that the former thence managed, controlled and
supervised their employment. They averred further that they were entitled to
separation pay based on their latest daily earnings of US$15.00 for working
sixteen (16) days a month.
The petitioners averred that the business was closed because of great
financial losses resulting from the eruption of Mt. Pinatubo. They admitted that
CFTI had agreed with the drivers' union, through its President Eduardo Castillo
who claimed to have had blanket authority to negotiate with CFTI in behalf of
union members, to grant its taxi driver-employees separation pay equivalent to
P500.00 for every year of service.
The labor arbiter found the drivers as regular employees of CFTI and ordered the
latter to pay them P1, 200 for every year of services for humanitarian
consideration. The decision was appealed to NLRC which granted the
respondents a right to severance pay from CFTI and made petitioners, Naguiat
Enterprises, Sergio Naguiat and Antonio Naguiat as solidary liable.
ISSUE
WON Naguiat Enterprises, Sergio and Antonio should be liable for the
severance pay of CFTIs taxi drivers.
RULING
Naguiat Enterprises shall not be liable. Private respondents failed to
substantiate their claim that Naguiat Enterprises managed, supervised and
controlled their employment. It appears that they were confused on the
personalities of Sergio F. Naguiat as an individual who was the president of
CFTI, and Sergio F. Naguiat Enterprises, Inc., as a separate corporate entity with
a separate business. They presumed that Sergio F. Naguiat, who was at the
same time a stockholder and director of Sergio F. Naguiat Enterprises, Inc. was
148

managing and controlling the taxi business on behalf of the latter. A closer
scrutiny and analysis of the records, however, evince the truth of the matter: that
Sergio F. Naguiat, in supervising the taxi drivers and determining their
employment terms, was rather carrying out his responsibilities as president of
CFTI. Hence, Naguiat Enterprises as a separate corporation does not appear to
be involved at all in the taxi business.
Sergio Naguiat shall be liable. Sergio F. Naguiat, admittedly, was the
president of CFTI who actively managed the business. Thus, applying the ruling
in A.C. Ransom, he falls within the meaning of an "employer" as contemplated by
the Labor Code, who may be held jointly and severally liable for the obligations of
the corporation to its dismissed employees.
Moreover, petitioners also conceded that both CFTI and Naguiat Enterprises
were "close family corporations" owned by the Naguiat family. Section 100,
paragraph 5, (under Title XII on Close Corporations) of the Corporation Code,
states:
(5) To the extent that the stockholders are actively engaged in the
management or operation of the business and affairs of a close corporation, the
stockholders shall be held to strict fiduciary duties to each other and among
themselves. Said stockholders shall be personally liable for corporate torts
unless the corporation has obtained reasonably adequate liability insurance.
The fifth paragraph of Section 100 of the Corporation Code specifically
imposes personal liability upon the stockholder actively managing or operating
the business and affairs of the close corporation.
Antonio Naguiat is not liable. Antolin was the vice president of the CFTI.
Although he carried the title of "general manager" as well, it had not been shown
that he had acted in such capacity. Furthermore, no evidence on the extent of his
participation in the management or operation of the business was preferred. In
this light, he cannot be held solidarily liable for the obligations of CFTI and Sergio
Naguiat to the private respondents.

149

YAO KA SIN TRADING


VS.
COURT OF APPEALS
GR.NO L-53820, JUNE 15, 1992
FACTS OF THE CASE
Constancio B. Maglana, President and Chairman of the Board of private
respondent Prime White Cement Corporation (PWCC) executed a letter-offer to
sell forty-five thousand (45,000) bags of Prime White Cement at 94lbs net per
bag for 24.30/ bag to Yao Ka Sin Trading ( YKS), a single Proprietorship
represented by its Manager, Mr. Henry Yao. A down payment of Two Hundred
Forty-three Thousand, (P 243,000.00) payable upon signing the contract and the
balance to be paid upon presenting corresponding shipping documents. The offer
was accepted by Mr. Henry Yao and signed the contract on June 7, 1973,
hereinafter referred to as Exhibit A.
In its meeting on June 30, 1973, the Board of Directors of PWCC
disapproved the contract (Exhibit A) and notified YKS thru letter informing it of
the disapproval of the contract, and instead issued Delivery Order for 10, 0000
bags of cement and Official Receipt for the payment of the same in amount of
243,000. The Board of Directors of PWCC intended the sale of the 10,000 bags
as a new and separate contract with that of the first.
YKS accepted without protest both the Delivery and Official Receipt. Later,
YKS denied having received the letter from PWCC informing it of the disapproval
of the first contract between Maglana and Henry Yao, and insisted for the
delivery of the 45,000 bags of white cement enforcing the validity of the first
contract. YKS demanded full compliance of the terms of the first contract. PWCC
ignored the demands, reiterated the unenforceability of the first contract on the
ground that it was signed by Mr. Maglana without the proper approval of the
Board.
YKS filed with the then CFI of Leyte a complaint for specific performance
with damages against PWCC. PWCC filed a Counterclaim for damages. The CFI
upheld the validity and enforceability of the first contract. On appeal, the Court of
Appeals reversed the decision of the CFI.
Issue
Whether or not a contract entered into by the President and at the same
time Chairman of the Board of a Corporation, signed by him without the approval
of the Board binds the Corporation.
Ruling
The Supreme Court affirmed the decision of the Court of Appeals
upholding the unenforceability of the first contract.
A corporate officer or agent may represent and bind the corporation in
transactions with third persons to the extent that authority to do so has been
conferred upon him, and also such powers as, in the usual course of business,
are incidental to, or may be implied from the power intentionally conferred, and
150

such apparent powers as the corporation has caused persons dealing with the
officer or agent to belief that it has conferred.
The respondent corporations By-Laws do not in any way confer upon the
President the authority to enter into contracts for the Corporation, independent of
the Board of Directors. That power is exclusively lodged in the Board. Under its
By-Laws to facilitate or expedite the execution of the contract, only the
President-not all the members of the Board-shall sign it for the corporation, and
the power of the Chairman to execute and sign for and in behalf of the
corporation all contracts and agreements which the corporation may entered into
presuppose a prior act of the corporation exercised through the Board of
Directors.

151

BENGUET ELECTRIC COOPERATIVE, INC.,


vs.
NATIONAL LABOR RELATIONS COMMISSION,
G.R. No. 89070 (1992)
FACTS OF THE CASE
Nature of the Case: PETITION for certiorari to review the decision of the National
Labor Relations Commission.
Peter Cosalan was the General Manager of Benguet Electric Cooperative, Inc.
(BANECO).
BANECO was audited by COA. This Audit Report noted and enumerated
irregularities in the utilization of funds amounting to P37 Million released by NEA
to Beneco, and recommended that appropriate remedial action be taken.
Cosalan implemented the remedial measures recommended by COA.
The Board of Beneco reacted by issuing a series of Board Resolutions: a)
abolishing the housing allowing of Cosalan; b) reducing his salary and
representation and commutable allowance; c) directing him to hold in abeyance
all pending personnel disciplinary actions; d) and struck his out as a principal
signatory to transactions of BANECO.
As a result of these adversarial Board Resolutions, Cosalan was ousted from his
position as General Manager and was excluded from the performance of his
regular duties. His salary was also withheld.
Cosalan continued to work as General Manager of BANECO in the belief that he
could only be suspended or removed by the duly authorized officials of NEA.
Cosalan requested from the Board the release of his compensation, but was
denied.
Cosalan eventually filed a complaint with the NLRC against the members of the
board as Directors of BANECO and in their personal capacities. He also
challenged the legality of the board resolutions which ordered his suspension
and termination from service. He also demanded payment of his salaries and
allowances.
The labor arbiter reinstated Cosalan. It also ordered BANECO together with its
Board-as jointly and severally liable- to pay of his backwages and allowances
amounting to Php 344,000.000. The board was also ordered to pay jointly and
severally moral damages of Php 50,000.00 plus attorneys fees of 10% of the
wages and allowances awarded.
On appeal, NLRC modified the award rendered by the labor arbiter by declaring
BANECO alone and not the respondent board liable for Cosalans Backwages
and allowances. There was no basis fpr the award of moral damages and
attorneys fees.
ISSUE

152

Had the NLRC acted with grave abuse of discretion amounting to lack of
jurisdiction in holding petitioner alone liable for payment of the backwages and
allowances due to Cosalan and releasing respondent Board members from
liability?
RULING
The applicable general rule is clear enough. The Board members and officers of
a corporation who purport to act for and in behalf of the corporation, keep within
the lawful scope of their authority in so acting, and act in good faith, do not
become liable, whether civilly or otherwise, for the consequences of their acts.
Those acts, when they are such a nature and are done under such
circumstances, are properly attributed to the corporation alone and no personal
liability is incurred by such officers and Board members.
As noted earlier, the respondent Board members responded to the efforts of
Cosalan to take seriously and implement the Audit Memoranda issued by the
COA explicitly addressed to the petitioner Beneco, first by stripping Cosalan of
the privileges and perquisites attached to his position as General Manager, then
by suspending indefinitely and finally dismissing Cosalan from such position. As
also noted earlier, respondent Board members offered no suggestion at all of any
just or lawful cause that could sustain the suspension and dismissal of Cosalan.
They obviously wanted to get rid of Cosalan and so acted, in the words of the
NLRC itself, "with indecent haste" in removing him from his position and denying
him substantive and procedural due process. Thus, the record showed strong
indications that respondent Board members had illegally suspended and
dismissed Cosalan precisely because he was trying to remedy the financial
irregularities and violations of NEA regulations which the COA had brought to the
attention of Beneco. The conclusion reached by the NLRC that "the records do
not disclose that the individual Board members were motivated by malice or bad
faith" flew in the face of the evidence of record. At the very least, a strong
presumption had arisen, which it was incumbent upon respondent Board
members to disprove, that they had acted in reprisal against respondent Cosalan
and in an effort to suppress knowledge about and remedial measures against the
financial irregularities the COA Audits had unearthed. That burden respondent
Board members did not discharge.
Under Section 31 of the Corporation Code which reads as follows:
"Sec. 31. Liability of directors, trustees or officers.Directors or trustees who
willfully and knowingly vote for or assent to patently unlawful acts of the
corporation or who are guilty of gross negligence or bad faith in directing the
affairs of the corporation or acquire any personal or pecuniary interest in conflict
with their duty as such directors or trustees shall be jointly liable and severally for
all damages resulting therefrom suffered by the corporation, its stockholders or
members and other persons x x x." (Italics supplied)
We agree with the Solicitor General, firstly, that Section 31 of the Corporation
Code is applicable in respect of Beneco and other electric cooperatives similarly
situated. Section 4 of the Corporation Code renders the provisions of that Code
applicable in a supplementary manner to all corporations, including those with
special or individual charters so long as those provisions are not inconsistent with
153

such charters. We find no provision in P.D. No. 269, as amended, that would
exclude expressly or by necessary implication the applicability of Section 31 of
the Corporation Code in respect of members of the boards of directors of electric
cooperatives. Indeed, P.D. No. 269 expressly describes these cooperatives as
"corporations:"
"Sec. 15. Organization and Purpose.Cooperative non-stock, non-profit
membership corporations may be organized, and electric co-operative
corporations heretofore formed or registered under the Philippine nonAgricultural Co-operative Act may as hereinafter provided be converted, under
this Decree for the purpose of supplying, and of promoting and encouraging the
fullest use of, service on an area coverage basis at the lowest cost consistent
with sound economy and the prudent management of the business of such
corporations." (Italics supplied)
10 See also Section 17 of P.D. No. 269, as amended, which requires the
members of electric cooperatives to include the abbreviation "Inc." (in the name
of the cooperative). Section 18 refers to the organizers of a cooperative as
"Incorporators." Sections 19-27 of the same statute refer to "Articles of
Incorporation of a Cooperative." Section 37 expressly incorporates the provision
of limited liability of members (but not of directors or other officers) which is the
hallmark of corporations:
"No member shall be liable or responsible for any debts of the cooperative and
the property of the members shall not be subject to execution therefor."
The legislative intent to make applicable to directors and officers of cooperatives
generally (i.e., electric cooperatives, agricultural cooperatives etc.) the provisions
of Section 31 of the Corporation Code, was confirmed by Article 46 of the
Philippine Cooperative Code (R.A. No. 6938, approved 10 March 1990). Article
46 of the Cooperative Code reads as follows:
"Article 46. Liability of Directors, Officers and Committee Members.Directors,
officers and committee members, who willfully and knowingly vote for or assent
to patently unlawful acts or who are guilty of gross negligence or bad faith in
directing the affairs of the cooperative or acquire any personal or pecuniary
interest in conflict with their duty as such directors, officers or committee
members shall be liable jointly and severally for all damages or profits resulting
therefrom to the cooperative, members and other persons.
When a director, officer or committee member attempts to acquire or acquires, in
violation of his duty, any interest or equity adverse to the cooperative in respect
to any matter which has been reposed in him in confidence, he shall, as a trustee
for the cooperative, be liable for damages and for double the profits which
otherwise would have accrued to the cooperative."
Article 122 of the Cooperative Code states that "[e]lectric cooperatives shall be
covered by this Code. x x x." Upon the other hand, Article 127 of the same Code
provides that electric cooperatives which qualify under this Code "shall fall under
the coverage of [P.D. No. 269 as amended]." The Cooperative Code is
substantially a reproduction of the general provisions of the Corporation Code.
154

The respondent Board members were guilty of "gross negligence or bad faith in
directing the affairs of the corporation" in enacting the series of resolutions noted
earlier indefinitely suspending and dismissing respondent Cosalan from the
position of General Manager of Beneco. Respondent Board members, in doing
so, acted beyond the scope of their authority as such Board members. The
dismissal of an officer or employee in bad faith, without lawful cause and without
procedural due process, is an act that is contra legem. It cannot be supposed
that members of boards of directors derive any authority to violate the express
mandates of law or the clear legal rights of their officers and employees by
simply purporting to act for the corporation they control.
We believe and so hold, further, that not only are Beneco and respondent Board
members properly held solidarily liable for the awards made by the Labor Arbiter,
but also that petitioner Beneco which was controlled by and which could act only
through respondent Board members, has a right to be reimbursed for any
amounts that Beneco may be compelled to pay to respondent Cosalan. Such
right of reimbursement is essential if the innocent members of Beneco are not to
be penalized for the acts of respondent Board members which were both done in
bad faith and ultra vires. The liability-generating acts here are the personal and
individual acts of respondent Board members, and are not properly attributed to
Beneco itself.
DISPOSITION
WHEREFORE, the Petition for Certiorari is GIVEN DUE COURSE, the comment
filed by respondent Board members is TREATED as their answer, and the
decision of the National Labor Relations Commission dated 21 November 1988
in NLRC Case No. RAB-1-0313-84 is hereby SET ASIDE and the decision dated
5 April 1988 of Labor Arbiter Amado T. Adquilen hereby REINSTATED in toto. In
addition, respondent Board members are hereby ORDERED to reimburse
petitioner Beneco any amounts that it may be compelled to pay to respondent
Cosalan by virtue of the decision of Labor Arbiter Amado T. Adquilen. No
pronouncement as to costs.
DOCTRINE
Corporation Law; Damages; The Board Members and Officers of a
corporation who purport to act for and in behalf of the corporation, keep
within the lawful scope of their authority in so acting and act in good faith,
do not become liable whether civilly or otherwise for the consequences of
their acts.The Board members and officers of a corporation who purport to act
for and in behalf of the corporation, keep within the lawful scope of their authority
in so acting, and act in good faith, do not become liable, whether civilly or
otherwise, for the consequences of their acts. Those acts, when they are such a
nature and are done under such circumstances, are properly attributed to the
corporation alone and no personal liability is incurred by such officers and Board
memb

155

LOUIS VUITTON S.A.


Vs.
VILLANUEVA, A.M.No.MTJ-92-643
GR.NO.151413,
FACTS OF THE CASE
Louis Vuitton, S.A. filed a criminal case of unfair competition against Jose
V. Rosario, an owner/proprietor of Manila COD Department Store for unlawfully
selling bags, wallets and other similar goods made of leather with the labels,
trademarks and logo of LOUIS VUIITTON and LV which are exclusive
trademarks owned and registered with the Philippine Patent Office in the name of
private complainant Vuitton. Trial Court acquitted the accused, finding that the
prosecution failed to prove the essential elements of unfair competition. Further,
trial court also noted that the prosecution filed the case against accused Rosario
in his personal capacity and not as an officer of the Manila COD Department
Store, which is a corporation, and has a separate legal capacity.
Unsatisfied, petitioner Vuitton filed a case against respondent Judge for
knowingly rendering a manifestly unjust judgment, in violation of Article 204 of
the Revised Penal Code.
ISSUE
Whether Judge Villanueva violated Article 204 of the Revised Penal Code by
knowingly rendering a manifestly unjust judgment.
RULING
Respondent Judge is not guilty of violation of Article 204 of the Revised
Penal Code. Complainant failed to substantiate its claims. One of the
requirements of the law is that the judgment was unjust and maliciously
rendered. A judgment is said to be unjust when it is contrary to the standards of
conduct prescribed by the law. The test to determine whether an order or
judgment is unjust may be inferred from the circumstances that it is contrary to
law or is not supported by evidence.
In this case, there was no unfair competition because the elements of the
crime were not sufficiently proven. Second, Jose V. Rosario, the accused, was
not sufficiently charged as owner/proprietor of COD as his personality is distinct
from that of the latters.
As to the second ground, Section 23 of the Corporation Code provides that the
corporate powers of all corporations formed under this code shall be exercised,
all business conducted, and all property of such corporations controlled and held
by the Board of Directors. COD is not a single proprietorship but one that is run
and owned by a corporation, Rosario Bros., of which accused is a stockholder
and Executive Vice President. A stockholder generally does not have a hand in
the management of the corporate affairs. On the Other hand, the Vice-President
had no inherent power to bind the corporation. As general rule, his duties must
be specified in the by-laws. In this case, the information did not specify his duties
as Executive Vice-President. The trial court had no basis for holding that as such,
the accused entered into a contract with the concessionaire thereby giving the
156

latter an opportunity to practice unfair competition. The board has the sole power
and responsibility to decide whether a corporation should enter into any contract
or perform any act. The amendment of the charge, as proposed by the private
prosecutor, would not in any way affect the application of the doctrine that the
corporation has a personality distinct from that of its owners.

157

METROPOLITAN BANK & TRUST CO.


VS.
QUILTS & ALL, INC.
222 SCRA 480 (1993)
[SEE ALSO LEE
VS.
COURT OF APPEALS, 345 SCRA 579 (2000)]
FACTS OF THE CASE
On April 7, 1987, Relita P. de los Santos (de los Santos) then Corporate
Secretary issued a Secretarys Certificate which certified that in a special meeting
of the Board of Directors of Quilts and All, Inc. (Quilts) its President, Mr. Senen B.
Dizon (Dizon) was authorized and empowered to mortgage in favor of Metro
bank, a property belonging to Quilts. On the basis of this Secretarys Certificate,
Metro bank restructured Dizons existing personal loan in the amount of P700,
000.00, secured by his house and lot at Angeles City and the property owned by
Quilts covered by TCT No. 74172. Aside from the mortgage lien, the Secretarys
Certificate was likewise annotated on TCT No. 74172 on April 10, 1977. On July
7, 1988, Metro bank received a letter from Atty. Cesar Villanueva, Quilts counsel
offering the amount of P200, 000.00 for the cancellation of the mortgage on the
property owned by Quilts because, allegedly, Mr. & Mrs. Senen Dizon had left
the Philippines, leaving several creditors. Metro bank refused the offer since the
amount offered did not approximate the appraised value of the mortgaged
property. On October 4, 1988, Atty. Ranel L. Trinidad, Quilts new counsel wrote
Metro bank. (Reiterating the mortgage cancellation. In addition, counsel claimed
that the alleged April 7, 1987 special meeting could not have taken place for lack
of the requisite number of directors present to constitute a quorum since the
Chairman and 2 other members of the Board of Directors were abroad on that
date.
On October 20, 1988, Quilts filed a complaint against Metro bank, Dizon and de
los Santos for annulment and cancellation of mortgage. Metro bank moved to
dismiss the complaint based on 1) lack of jurisdiction and 2) failure to state a
cause of action. Judge Reynaldo B. Daway granted the motion on February 9,
1989 but on August 4, 1989, upon Quilts motion, Judge Daway issued an Order
reconsidering and setting aside the dismissal order because the grounds relied
upon by Metro bank did not appear to be indubitable, and deferred the
determination of the motion until the trial.
Metro bank filed an original petition for certiorari, prohibition or mandamus,
contesting the reinstatement of the complaint and in the process reiterating as
grounds lack of jurisdiction on the part of the trial court and failure of Quilts
complaint to state a cause of action. The Court of Appeals upheld the jurisdiction
of the lower Court only with respect to Metro bank. It dismissed the case against
Dizon and de los Santos, since the issue of whether or not these two persons
had committed ultra vires acts is an intra-corporate matter which falls within the
original and exclusive jurisdiction of the Securities and Exchange Commission
(SEC) Hence, this petition.
ISSUE
Whether the certification given by the Corporate Secretary could be given
probative value despite the fact that there was no quorum of the Board of
Directors of Quilts to conduct the special meeting.
158

RULING
The Supreme Court decided in favor of the petitioner on the premise that Metro
bank cannot be faulted for relying on the Secretarys Certificate. It did so in good
faith, unaware of any flaw and on the presumption that the ordinary course of
business had been followed and that the Corporate Secretary had regularly
performed her duties. It was held the complaint does not contain allegations that
Metro bank had prior knowledge of, or could have known with the exercise of due
diligence, that the recitals in the Secretarys Certificate were false. The complaint
does not even allege specific overt acts which show that Metrobank acted in
conspiracy with its codefendants to defraud Quilts.

159

LOPEZ REALTY, INC. AND ASUNCION LOPEZ GONZALES,


Vs.
FONTECHA ET AL. and NLRC
G.R. No. 76801 August 11, 1995
FACTS OF THE CASE
Lopez Realty, Inc., is a corporation engaged in real estate business, while
petitioner Asuncion Lopez Gonzales is one of its majority shareholders.
Sometime in 1978, Arturo Lopez, one of the shareholders, submitted a
proposal relative to the distribution of certain assets of the corporation including
the reduction of employees with provision for their gratuity pay. It was approved
in a special meeting of the board of directors. Lopez Realty approved two
resolutions, both passed in 1980, providing for the gratuity pay of its employees.
In 1981, except for Asuncion Lopez Gonzales who was then abroad, the
remaining members of the Board of Directors passed another resolution on how
the gratuity of the employees will be given.
Private respondents were the retained employees of petitioner corporation.
They requested for the full payment of their gratuity pay. This was granted in a
special meeting but petitioner Asuncion was still abroad at that time. She sent a
cablegram to the corporation objecting to certain matters taken up by the board
in her absence. Upon her return, she filed a derivative suit with the SEC against
Arturo Lopez.
Notwithstanding the "corporate squabble" between petitioner Asuncion
Lopez Gonzales and Arturo Lopez, the first two (2) installments of the gratuity
pay were paid by petitioner corporation but the rest of the cash vouchers and
checks were cancelled by petitioner Asuncion. Despite private respondents'
repeated demands for their gratuity pay, corporation refused to pay the same.
ISSUE
Whether or not the corporation is bound to give the full gratuity pay
considering the lack of notice to one of the board diractors during the resolution
that granted it.

RULING
Yes. The general rule is that a corporation, through its board of directors,
should act in the manner and within the formalities, if any, prescribed by its
charter or by the general law. Thus, directors must act as a body in a meeting
called pursuant to the law or the corporations by-laws, otherwise, any action
taken therein may be questioned by any objecting director or shareholder.
Be that as it may, jurisprudence tells us that an action of the board of
directors during a meeting, which was illegal for lack of notice, may be ratified
either expressly, by the action of the directors in subsequent legal meeting, or
impliedly, by the corporations subsequent course of conduct. Thus, despite lack
of notice at that time the assailed resolutions were passed, Asuncion is now
precluded from questioning the validity since she acquiesced thereto by signing
the vouchers of the gratuity pay.
160

Assuming, arguendo, that there was no notice given to Asuncion Lopez


Gonzales during the special meetings held on August 17, 1981 and September
1, 1981, it is erroneous to state that the resolutions passed by the board during
the said meetings were ultra vires. In legal parlance, ultra vires act refers to one
which is not within the corporate powers conferred by the Corporation Code or
articles of incorporation or not necessary or incidental in the exercise of the
powers so conferred.
The assailed resolutions before us cover a subject which concerns the
benefit and welfare of the companys employees. To stress, providing gratuity
pay for its employees is one of the express powers of the corporation under the
Corporation Code, hence, petitioners cannot invoke the doctrine of ultra vires to
avoid any liability arising from the issuance of the subject resolutions.
Petitioners try to convince us that the subject resolutions had no force and
effect in view of the non-approval thereof during the Annual Stockholders
Meeting held on March 1, 1982. To strengthen their position, petitioners cite
section 28 1/2 of the Corporation Law (Section 40 of the Corporation Code). We
are not persuaded. The cited provision is not applicable to the case at bench as it
refers to the sale, lease, exchange or disposition of all or substantially all of the
corporations assets, including its goodwill. In such a case, the action taken by
the board of directors requires the authorization of the stockholders on record.
It will be observed that, except for Arturo Lopez, the stockholders of
Petitioner Corporation also sit as members of the board of directors. Under the
circumstances in field, it will be illogical and superfluous to require the
stockholders approval of the subject resolutions. Thus, even without the
stockholders approval of the subject resolutions, petitioners are still liable to pay
private respondents gratuity pay.

161

PREMIUM MARBLE RESOURCES, INC.


Vs.
COURT OF APPEALS,
G.R. No. 96551
FACTS OF THE CASE
Ayala Investment and Development Corporation issued three checks
payable to petitioner and drawn against Citibank, but former officers of petitioner
headed by Saturnino G. Belen, Jr., without any authority whatsoever deposited
the abovementioned checks to the current account of his conduit corporation,
Intervest Merchant Finance (Intervest, for brevity) which the latter maintained
with the defendant. Although the checks were clearly payable to petitioner and
crossed on their face and for payee's account only, defendant bank accepted the
checks to be deposited to the current account of Intervest and thereafter
presented the same for collection from the drawee bank which subsequently
cleared the same thus allowing Intervest to make use of the funds. Petitioner
herein, assisted by Atty. Arnulfo Dumadag as counsel, filed an action for
damages against International Corporate Bank. Printline Corporation, a sister
company of Premium also filed an action for damages against International
Corporate Bank. Thereafter, both civil cases were consolidated. In its Answer
International Corporate Bank alleged, inter alia, that Premium has no
capacity/personality/authority to sue in this instance and the complaint should,
therefore, be dismissed for failure to state a cause of action.
Meantime, the same corporation, i.e., Premium, but this time represented
by Siguion Reyna, Montecillio and Ongsiako Law Office as counsel, filed a
motion to dismiss on the ground that the filing of the case was without authority
from its duly constituted board of directors as shown by the excerpt of the
minutes of the Premium's board of directors' meeting.
In its opposition to the motion to dismiss, Premium thru Atty. Dumadag
contended that the persons who signed the board resolution namely Belen, Jr.,
Nograles & Reyes, are not directors of the corporation and were allegedly former
officers and stockholders of Premium who were dismissed for various
irregularities and fraudulent acts; that Siguion Reyna Law office is the lawyer of
Belen and Nograles and not of Premium and that the Articles of Incorporation of
Premium shows that Belen, Nograles and Reyes are not majority stockholders.
On the other hand, Siguion Reyna Law firm as counsel of Premium in a
rejoinder, asserted that it is the general information sheet filed with the Securities
and Exchange Commission, among others, that is the best evidence that would
show who are the stockholders of a corporation and not the Articles of
Incorporation since the latter does not keep track of the many changes that take
place after new stockholders subscribe to corporate shares of stocks.
The lower court concluded that the officers represented by Atty. Dumadag
do not as yet have the legal capacity to sue for and in behalf of the plaintiff
corporation and/or the filing of the present action. On appeal, the Court of
Appeals affirmed the trial court's Order. Hence, this petition.
ISSUE
Whether or not the filing of the case for damages against private
respondent was authorized by a duly constituted Board of Directors of the
petitioner corporation.
162

RULING
We find the petition without merit.
We agree with the finding of public respondent Court of Appeals, that "in
the absence of /any board resolution from its board of directors the authority to
act for and in behalf of the corporation, the present action must necessarily fail.
The power of the corporation to sue and be sued in any court is lodged with the
board of directors that exercises its corporate powers. Thus, the issue of
authority and the invalidity of plaintiff-appellant's subscription which is still
pending is a matter that is also addressed, considering the premises, to the
sound judgment of the Securities & Exchange Commission."
By the express mandate of the Corporation Code (Section 26), all
corporations duly organized pursuant thereto are required to submit within the
period therein stated (30 days) to the Securities and Exchange Commission the
names, nationalities and residences of the directors, trustees and officers
elected.
Evidently, the objective sought to be achieved by Section 26 is to give the
public information, under sanction of oath of responsible officers, of the nature of
business, financial condition and operational status of the company together with
information on its key officers or managers so that those dealing with it and those
who intend to do business with it may know or have the means of knowing facts
concerning the corporation's financial resources and business responsibility.
The claim, therefore, of petitioners as represented by Atty. Dumadag, that
Zaballa, et al., are the incumbent officers of Premium has not been fully
substantiated. In the absence of an authority from the board of directors, no
person, not even the officers of the corporation, can validly bind the corporation.
We find no reversible error in the decision sought to be reviewed.
Accordingly, for lack of merit, the petition is denied.

163

JULIETA V. ESGUERRA
Vs.
COURT OF APPEALS
G.R. No. 119310 February 3, 1997
FACTS OF THE CASE
Julieta Esguerra filed a complaint for administration of conjugal partnership
or separation of property against her husband Vicente Esguerra, Jr. and V.
Esguerra Construction Co., Inc. (VECCI) and other family corporations as
defendants before the trial court.
The parties entered into a compromise agreement. By virtue of said
agreement, Esguerra Bldg. I was sold and the net proceeds distributed according
to the agreement. The controversy arose with respect to Esguerra Building II.
Herein petitioner started claiming one-half of the rentals of the said building
which VECCI refused. Thus, petitioner filed a motion with respondent court
praying that VECCI be ordered to remit one-half of the rentals to her. The trial
court ruled in favour of petitioner.
Meanwhile, Esguerra Bldg. II was sold to private respondent Sureste
Properties. Inc. for P150, 000,000.00 prompting Julieta V. Esguerra to file a
motion seeking the nullification of the sale on the ground that VECCI is not the
lawful and absolute owner thereof and that she has not been notified nor
consulted as to the terms and conditions of the sale. The trial court ruled that the
sale to Sureste was valid.
ISSUE
Whether the sale of Esguerra Building II is a valid exercise of corporate
power.

RULING
Yes. VECCI's sale of all the properties mentioned in the judicially-approved
compromise agreement was done on the basis of its Corporate Secretary's
Certification of these two resolutions. The partial decision did not require any
further board or stockholder resolutions to make VECCI's sale of these properties
valid. Being regular on its face, the Secretary's Certification was sufficient for
private respondent Sureste Properties, Inc. to rely on. It did not have to
investigate the truth of the facts contained in such certification. Otherwise,
business transactions of corporations would become tortuously slow and
unnecessarily hampered. Ineluctably, VECCI's sale of Esguerra Building II to
private respondent was not ultra vires but a valid execution of the trial court's
partial decision.
Based on the foregoing, the sale is also deemed to have satisfied the
requirements of Section 40 of the Corporation Code.

164

TRADERS ROYAL BANK


Vs.
COURT OF APPEALS
G.R. No. 93397
FACTS OF THE CASE
Filriters Guaranty Assurance Corporation (FGAC) is the owner of several
Central Bank Certificates of Indebtedness (CBCI). These certificates are actually
proof that FGAC has the required reserve investment with the Central Bank to
operate as an insurer and to protect third persons from whatever liabilities FGAC
may incur. In 1979, FGAC agreed to assign said CBCI to Philippine Underwriters
Finance Corporation (PUFC). Later, PUFC sold said CBCI to Traders Royal Bank
(TRB). Said sale with TRB comes with a right to repurchase on a date certain.
However, when the day to repurchase arrived, PUFC failed to repurchase said
CBCI hence TRB requested the Central Bank to have said CBCI be registered in
TRBs name. Central Bank refused as it alleged that the CBCI are not negotiable;
that as such, the transfer from FGAC to PUFC is not valid; that since it was
invalid, PUFC acquired no valid title over the CBCI; that the subsequent transfer
from PUFC to TRB is likewise invalid.
TRB then filed a petition for mandamus to compel the Central Bank to
register said CBCI in TRBs name. TRB averred that PUFC is the alter ego of
FGAC; that PUFC owns 90% of FGAC; that the two corporations have identical
sets of directors; that payment of said CBCI to PUFC is like a payment to FGAC
hence the sale between PUFC and TRB is valid. In short, TRB avers that that the
veil of corporate fiction, between PUFC and FGAC, should be pierced because
the two corporations allegedly used their separate identity to defraud TRD into
buying said CBCI.
ISSUE
Whether or not Traders Royal Bank is correct.
RULING
No.
Traders Royal Bank failed to show that the corporate fiction is used by the
two corporations to defeat public convenience, justify wrong, protect fraud or
defend crime or where a corporation is a mere alter ego or business conduit of a
person. TRB merely showed that PUFC owns 90% of FGAC and that their
directors are the same. The identity of PUFC cant be maintained as that of
FGAC because of this mere fact; there is nothing else which could lead the court
under the circumstance to disregard their corporate personalities. Further, TRB
cant argue that it was defrauded into buying those certificates. In the first place,
TRB as a banking institution is not ignorant about these types of transactions. It
should know for a fact that a certificate of indebtedness is not negotiable
because the payee therein is inscribed specifically and that the Central Bank is
obliged to pay the named payee only and no one else.

165

WESTERN INSTITUTE OF TECHNOLOGY


VS.
SALAS
G.R. No. 113032
FACTS OF THE CASE
Private respondents Ricardo T. Salas, Salvador T. Salas, Soledad SalasTubilleja, Antonio S. Salas, and Richard S. Salas, belonging to the same family,
are the majority and controlling members of the Board of Trustees of Western
Institute of Technology, Inc. (WIT, for short), a stock corporation engaged in the
operation, among others, of an educational institution. According to petitioners,
the minority stockholders of WIT, sometime on June 1, 1986 in the principal
office of WIT at La Paz, Iloilo City, a Special Board meeting was held. In
attendance were other members of the Board including one of the petitioners
Reginald Villasis. Prior to aforesaid Special Board Meeting, copies of notice
thereof, dated May 24, 1986, were distributed to all Board Members. The notice
allegedly indicated that the meeting to be held on June 1, 1986 included item No.
6 which states:
"Possible implementation of Art. III, Sec. 6 of the Amended By-Laws of
Western Institute of Technology, Inc. on compensation of all officers of the
corporation." [1]
In said meeting, the Board of Trustees passed Resolution No. 48, s. 1986,
granting monthly compensation to the private respondents as corporate
officers retroactive June 1, 1985, viz.:
Resolution No. 48 s. 1986
On the motion of Mr. Richard Salas (accused), duly seconded by
Mrs. Soledad Tubilleja (accused), it was unanimously resolved that:
The Officers of the Corporation be granted monthly
compensation for services rendered as follows: Chairman P9,000.00/month, Vice-Chairman - P3,500.00/month,
Corporate Treasurer - P3,500.00/month and Corporate
Secretary - P3,500.00/month, retroactive June 1, 1985 and the
ten per centum of the net profits shall be distributed equally
among the ten members of the Board of Trustees. This shall
amend and superceed(sic) any previous resolution.
There were no other business.
The Chairman declared the meeting adjourned at 5:11 P.M.
This is to certify that the foregoing minutes of the regular
meeting of the Board of Trustees of Western Institute of
Technology, Inc. held on March 30, 1986 is true and correct to
the best of my knowledge and belief.
(Sgd) ANTONIO S. SALAS
Corporate Secretary
A few years later, that is, on March 13, 1991, petitioners Homero Villasis,
Preston Villasis, Reginald Villasis and Dimas Enriquez filed an affidavit-complaint
against private respondents before the Office of the City Prosecutor of Iloilo, as a
result of which two (2) separate criminal informations, one for falsification of a
public document under Article 171 of the Revised Penal Code and the other for
estafa under Article 315, par. 1(b) of the RPC, were filed before Branch 33 of the
Regional Trial Court of Iloilo City. The charge for falsification of public document
was anchored on the private respondents submission of WITs income statement
for the fiscal year 1985-1986 with the Securities and Exchange Commission
(SEC) reflecting therein the disbursement of corporate funds for the
compensation of private respondents based on Resolution No. 4, series of 1986,
166

making it appear that the same was passed by the board on March 30, 1986,
when in truth, the same was actually passed on June 1, 1986, a date not covered
by the corporations fiscal year 1985-1986 (beginning May 1, 1985 and ending
April 30, 1986).
Thereafter, trial for the two criminal cases, docketed as Criminal Cases
Nos. 37097 and 37098, was consolidated. After a full-blown hearing, Judge
Porfirio Parian handed down a verdict of acquittal on both counts dated
September 6, 1993 without imposing any civil liability against the accused
therein.
Petitioners filed a Motion for Reconsideration of the civil aspect of the RTC
Decision which was, however, denied in an Order dated November 23, 1993.
Hence, the instant petition.
Significantly on December 8, 1994, a Motion for Intervention, dated
December 2, 1994, was filed before this Court by Western Institute of
Technology, Inc., supposedly one of the petitioners herein, disowning its
inclusion in the petition and submitting that Atty. Tranquilino R. Gale, counsel for
the other petitioners, had no authority whatsoever to represent the corporation in
filing the petition. Intervenor likewise prayed for the dismissal of the petition for
being utterly without merit. The Motion for Intervention was granted on January
16, 1995.[8]
Petitioners would like us to hold private respondents civilly liable despite their
acquittal in Criminal Cases Nos. 37097 and 37098. They base their claim on the
alleged illegal issuance by private respondents of Resolution No. 48, series of
1986 ordering the disbursement of corporate funds in the amount of P186,470.70
representing the retroactive compensation as of June 1, 1985 in favor of private
respondents, board members of WIT, plus P1,453,970.79 for the subsequent
collective salaries of private respondent every 15th and 30th of the month until the
filing of the criminal complaints against them on March 1991. Petitioners maintain
that this grant of compensation to private respondents is proscribed under
Section 30 of the Corporation Code. Thus, private respondents are obliged to
return these amounts to the corporation with interest.

ISSUE
Are the private respondents civilly liable despite their acquittal and is the instant
case a derivative suit?

RULING
The pertinent section of the Corporation Code provides:
Sec.30. Compensation of directors.--- In the absence of any provision in
the by-laws fixing their compensation, the directors shall not receive any
compensation, as such
directors, except
for
reasonable
per
diems: Provided, however, That any such compensation (other than per
diems) may be granted to directors by the vote of the stockholders
representing at least a majority of the outstanding capital stock at a regular
or special stockholders meeting. In no case shall the total yearly
compensation of directors, as such directors, exceed ten (10%) percent of
the net income before income tax of the corporation during the preceding
year. [Underscoring ours]
167

There is no argument that directors or trustees, as the case may be, are
not entitled to salary or other compensation when they perform nothing
more than the usual and ordinary duties of their office. This rule is founded
upon a presumption that directors /trustees render service gratuitously and
that the return upon their shares adequately furnishes the motives for
service, without compensation. Under the foregoing section, there are only
two (2) ways by which members of the board can be granted
compensation apart from reasonable per diems: (1) when there is a
provision in the by-laws fixing their compensation; and (2) when the
stockholders representing a majority of the outstanding capital stock at a
regular or special stockholders meeting agree to give it to them.
This proscription, however, against granting compensation to
directors/trustees of a corporation is not a sweeping rule. Worthy of note is the
clear phraseology of Section 30 which states: xxx [T]he directors shall not
receive any compensation, as such directors, xxx. The phrase as such
directors is not without significance for it delimits the scope of the prohibition to
compensation given to them for services performed purely in their capacity as
directors or trustees. The unambiguous implication is that members of the board
may receive compensation, in addition to reasonable per diems, when they
render services to the corporation in a capacity other than as directors/trustees.
Clearly, therefore, the prohibition with respect to granting compensation to
corporate directors/trustees as such under Section 30 is not violated in this
particular case. Consequently, the last sentence of Section 30 which provides:
xxx xxx. In no case shall the total yearly compensation of directors, as
such directors, exceed ten (10%) percent of the net income before income
tax of the corporation during the preceding year.
does not likewise find application in this case since the compensation is being
given to private respondents in their capacity as officers of WIT and not as board
members.
A derivative suit is an action brought by minority shareholders in the name
of the corporation to redress wrongs committed against it, for which the directors
refuse to sue. It is a remedy designed by equity and has been the principal
defense of the minority shareholders against abuses by the majority. Here,
however, the case is not a derivative suit but is merely an appeal on the civil
aspect of Criminal Cases Nos. 37097 and 37098 filed with the RTC of Iloilo
for estafa and falsification of public document. Among the basic requirements for
a derivative suit to prosper is that the minority shareholder who is suing for and
on behalf of the corporation must allege his complaint before the proper forum
that he is suing on a derivative cause of action on behalf of the corporation and
all other shareholders similarly situated who wish to join. This is necessary to
vest jurisdiction upon the tribunal in line with the rule that it is the allegations in
the complaint that vests jurisdiction upon the court or quasi-judicial body
concerned over the subject matter and nature of the action. This was not
complied with by the petitioners either in their complaint before the court a
quo nor in the instant petition which, in part, merely states that this is a petition
for review on certiorari on pure questions of law to set aside a portion of the RTC
decision in Criminal Cases Nos. 37097 and 37098 since the trial courts
judgment of acquittal failed to impose any civil liability against the private
respondents. By no amount of equity considerations, if at all deserved, can a
mere appeal on the civil aspect of a criminal case be treated as a derivative suit.

168

Granting, for purposes of discussion, that this is a derivative suit as


insisted by petitioners, which it is not, the same is outrightly dismissible for
having been wrongfully filed in the regular court devoid of any jurisdiction to
entertain the complaint. The case should have been filed with the Securities and
Exchange Commission (SEC) which exercises original and exclusive jurisdiction
over derivative suits, they being intra-corporate disputes, per Section 5(b) of P.D.
No. 902-A:
In addition to the regulatory and adjudicative functions of the Securities
and Exchange Commission over corporations, partnerships and other
forms of associations registered with it as expressly granted under existing
laws and decrees, it shall have original and exclusive jurisdiction to hear
and decide cases involving:
xxx xxx
xxx
b) Controversies
arising
out
of
intra-corporate or
partnership relations, between and among stockholders, members, or
associates; between any or all of them and the corporation, partnership or
association of which they are stockholders, members or associates,
respectively; and between such corporation, partnership or association and
the State insofar as it concerns their individual franchise or right to exist as
such entity;
xxx xxx
xxx.
Once the case is decided by the SEC, the losing party may file a petition
for review before the Court of Appeals raising questions of fact, of law, or mixed
questions of fact and law. It is only after the case has ran this course, and not
earlier, can it be brought to us via a petition for review on certiorari under Rule 45
raising only pure questions of law. Petitioners, in pleading that we treat the
instant petition as a derivative suit, are trying to short-circuit the entire process
which we cannot here sanction.
The acquittal in Criminal Cases Nos. 37097 and 37098 is not merely based
on reasonable doubt but rather on a finding that the accused-private respondents
did not commit the criminal acts complained of. Thus, pursuant to the above rule
and settled jurisprudence, any civil action ex delicto cannot prosper. Acquittal in a
criminal action bars the civil action arising therefrom where the judgment of
acquittal holds that the accused did not commit the criminal acts imputed to
them.
WHEREFORE, the instant petition is hereby DENIED with costs against
petitioners.

169

BITONG
VS.
COURT OF APPEALS
292 SCRA 503
GR.NO.123553. JULY 13, 1998
FACT OF THE CASE
Nora Bitong filed a derivative suit before the SEC for the benefit of Mr. &
Ms. Publishing Co., Inc. against respondent espouses Eugenia Apostol and Jose
Apostol. Bitong imputes fraud, disloyalty, bad faith, and mismanagement to
respondents.
Bitong claimed that she was the treasurer and a member of the BOD of Mr.
& Ms. From the time it was incorporated in 1976 until 1989, and an owner of
1,000 shares of the company. She complained of thee irregularities done by
Apostol as the president and chairman of the corporation. She claimed that all
the acts of Apostol are done without authority from the Board of Directors.
Bitong further claimed that she was a bona fide stockholder of the
company by virtue of a Deed of Sale executed by JAKA, in 1983, in her favor for
the total 1,000 shares registered in the name of JAKA, under Certificate of
Stocks No. 008. She claimed that Senator Enrile decided to divest itself of its
holdings in Mr.. & Ms. resulting in the sale to her of JAKAs interest and holding
in the company. She claims that she is a holder of a formal certificate of stock
and that the transfer to her name of such stocks was recorded in the STB of the
corporation. She invokes Section 63 of the corporation code which provides that
no transfer shall be valid except as between the parties until the transfer is
recorded in the books of the corporation, and upon its recording the the legal
personality to institute the derivative suit being only a holder-in-trust of JAKA
Investments Corporation. According to them, Mr. & Ms. Started out as Ex Libris
in 1976 as a weekly magazine, the original stockholders of which are spouses
Senator Juan Ponce Enrile and Christina Enrile through JAKA, and themselves.
It was only when Ex Libris faced financial difficulty that it admitted another
investors and changed the name to Mr. & Mrs. It was agreed among them, the
stockholders, that management of the company would be that of a partnership or
a close corporation. Bitong only sits in the board as a representative and agent of
JAKAcorporation is bound by it and is stopped from denying the fat of transfer of
said shares.
Respondents refute the allegations of Bitong, arguing that the latter doesnt
have.
ISSUE
Whether Bitong has a legal personality to institute stockholders derivative
suit for and behalf of the corporation.
RULING
Bitong has NO legal personality to institute a derivative suit for and in
behalf of the corporation. There is overwhelming evidence that despite what
170

appears on the certificate of stock and the stock and transfer books of the
corporation, Bitong was not a bona fide stockholder of Mr. & Ms. before March
1989 or at the time the complained acts were committed to qualify her to institute
a stockholders derivative suit.
The evidence presented by the parties categorically showed that the real
party-in-interest was not Bitong but JAKA and or Senator Enrile. The contentions
of Bitong are bereft of any legal basis. She only represents the interest and
holdings of JAKA in MR. & Ms.
Bitong is not a holder of a formal certificate of stock, contrary to her claim.
Section 63 of the corporation code provides that a formal certificate of stock can
only be issued n compliance with the aid provision, that is:
FIRST, it must be signed by the president or the vice president, it must be
countersigned by the corporate secretary, and it must be sealed with the
seal of the corporation. A mere typewritten statement advising a
stockholder of the extent of his ownership in a corporation without
qualification and/or authentication cannot be considered as a formal
certificate of stock.
SECOND, the certificate of stock must be delivered to the stockholder to
constitute issuance. Delivery is an essential element of issuance. There is
no issuance of a certificate where it is never detached from the stock
books, although blanks there in are properly filled up if the person whose
name is inserted therein has no control over the books of the company.
THIRD, that there must be full payment of the par value, if it I a par value
share, or the payment of the full subscription if it is a no par value share.
FOURTH, that there must be a surrender of the original certificate if the
person requesting the issuance is a transferee of stockholder.
The certificate of stock itself once issued is a continuing affirmation or
representation that the stock described therein is valid and genuine and is a
prima facie evidence that it was legally issued in the absence of evidence to the
contrary. Similarly, the books and records of the corporation which include even
the STB are generally admissible in evidence in favor of or against the
corporation and its members to prove the corporate acts, its financial status and
other matters. However, books and records are not conclusive evidence even
against the corporation but are prima facie evidence only. Parol evidence may be
admitted to SUPPLY omissions in the record, EXPLAIN ambiguities, or SHOW
what transpired where no records were kept, on in some cases where such
record where controverted.
As regards Certificate of Stock No. 008, it was admitted by Bitong that the
same was only signed by Apostol in 1989 but the same was issued by the
corporate secretary in 1983. There is no truth to the statement written in
Certificate of Stock No. 008 that the same was issued and signed in 1983 by duly
authorized officers because the actual signing thereof was only in 1989. A formal
certificate of stock could not be considered issued in contemplation of law unless

171

signed by the president or vice-president and countersigned by the secretary or


assistant secretary.
As regards the sale of 1,000 shares to Bitong, the Court said that the same is
without a valid basis in law and in facts. It was admitted by Bitong herself that
said shares was actually transferred to Apostol as a trustee because Senator
Enrile does not want to be involved in the company anymore since it would just
be prejudicial and disadvantageous to then. Bitong claimed that few days after
said transfer, Senator Enrile sold the said shares to her. This sale, the Court
said, is legally impossible. When the shares were transferred to Apostol by virtue
of a trust, the Certificate of Stock No. 001 which represents said shares were
cancelled and a new certificate of stock (certificate of Stock 007) was issued in
the name of Apostol. If there was a sale of the same, the sale should have taken
place between the trustee Apostol and Bitong, and not between Enrile and the
same alleged transferee. There was no evidence that the trustee assigned or
endorsed said shares to Bitong. It is settle rule that the trustee should first
endorse the stock certificate o validate the cancellation of her share and to have
the transfer recorded in the books of the corporation

172

TAM WING TAK


Vs.
MAKASIAR and DE GUIA,
G.R. No. 122452, January 29, 2001
FACTS OF THE CASE
On November 11, 1992, petitioner, in his capacity as director of ConcordWorld Properties, Inc., (Concord for brevity), a domestic corporation, filed a
complaint charging Vic Ang Siong with violation of B.P. Blg. 22. The complaint
alleged that a check for the amount of P83, 550,000.00, issued by Vic Ang Siong
in favor of Concord, was dishonored when presented for encashment.
Vic Ang Siong sought the dismissal of the case on two grounds: First, that
petitioner had no authority to file the case on behalf of Concord, the payee of the
dishonored check, since the firms board of directors had not empowered him to
act on its behalf.
The City Prosecutor dismissed the complaint.
ISSUE
Whether or not petitioner had no standing to file the criminal complaint
since he was neither the payee nor holder of the bad check.
RULING
Yes. It is not disputed in the instant case that Concord, a domestic
corporation, was the payee of the bum check, not petitioner. Therefore, it is
Concord, as payee of the bounced check, which is the injured party. Since
petitioner was neither a payee nor a holder of the bad check, he had neither the
personality to sue nor a cause of action against Vic Ang Siong. Under Section
36 of the Corporation Code, read in relation to Section 23, it is clear that where a
corporation is an injured party, its power to sue is lodged with its board of
directors or trustees. Note that petitioner failed to show any proof that he was
authorized or deputized or granted specific powers by Concords board of
director to sue Victor Ang Siong for and on behalf of the firm. Clearly, petitioner
as a minority stockholder and member of the board of directors had no such
power or authority to sue on Concords behalf. Nor can we uphold his act as a
derivative suit. For a derivative suit to prosper, it is required that the minority
stockholder suing for and on behalf of the corporation must allege in his
complaint that he is suing on a derivative cause of action on behalf of the
corporation and all other stockholders similarly situated who may wish to join him
in the suit. There is no showing that petitioner has complied with the foregoing
requisites. It is obvious that petitioner has not shown any clear legal right which
would warrant the overturning of the decision of public respondents to dismiss
the complaint against Vic Ang Siong. A public prosecutor, by the nature of his
office, is under no compulsion to file criminal information where no clear legal
justification has been shown, and no sufficient evidence of guilt nor prima facie
case has been presented by the petitioner. No reversible error may be attributed
to the court a quo when it dismissed petitioners special civil action for
mandamus.

173

DBP v. CA
(G.R. No. 126200)
FACTS OF THE CASE
1. For failure of Marinduque Mining to settle its loan obligations, PNB and
DBP instituted sometime on July and August 1984 extrajudicial foreclosure
proceedings over the mortgaged properties.
2. At the public auction sale conducted on September 18, 1984 on the
foreclosed personal properties of MMIC, the same were sold to PNB and
DBP as the highest bidder in the sum of P678,772,000.00.
3. PNB and DBP thereafter thru a Deed of Transfer dated August 31, 1984,
purposely, in order to ensure the continued operation of the Nickel refinery
plant and to prevent the deterioration of the assets foreclosed, assigned
and transferred to Nonoc Mining and Industrial Corporation all their rights,
interest and participation over the foreclosed properties of MMIC located at
Nonoc Island, Surigao del Norte for an initial consideration of P14,
361,000,000.00.
4. In the meantime, between July 16, 1982 to October 4, 1983, Marinduque
Mining purchased and caused to be delivered construction materials and
other merchandise from Remington Industrial Sales Corporation
(Remington) worth P921, 755.95. The purchases remained unpaid as of
August 1, 1984 when Remington filed a complaint for a sum of money and
damages against Marinduque Mining for the value of the unpaid
construction materials and other merchandise purchased by Marinduque
Mining, as well as interest, attorneys fees and the costs of suit.
5. Remington in this case alleged that, co-defendants PNB, DBP NMIC,
Maricalum and Island Cement being all corporations created by DBP in the
pursuit of business ventures should not be allowed to ignore the financial
obligations of MMIC whose operations co-defendants PNB and DBP had
highly financed before the alleged extrajudicial foreclosure of defendant
MMICs assets, machineries and equipment to the extent that major
policies of co-defendant MMIC were being decided upon by co-defendants
PNB and DBP as major financiers who were represented in its board of
directors forming part of the majority thereof which through the alleged
extrajudicial foreclosure culminated in a complete take-over by codefendants PNB and DBP bringing about the organization of their codefendants NMIC, Maricalum and Island Cement to which were transferred
all the assets, machineries and pieces of equipment of co-defendant MMIC
to the prejudice of creditors of co-defendant MMIC such as plaintiff
Remington Industrial Sales Corporation.
ISSUE
Whether the existence of interlocking directors between the creditor, DBP,
and the debtor, MMIC, prejudiced the interest of another creditor, Remington.
RULING
No. Two principles in corporation law were mentioned in this case. The first
pertains to transactions between corporations with interlocking directors resulting
in the prejudice to one of the corporations. This rule does not apply in this case,
174

however, since the corporation allegedly prejudiced (Remington) is a third party,


not one of the corporations with interlocking directors (Marinduque Mining and
DBP).
The second principle invoked by respondent court involves directors who
are creditors which is also inapplicable herein. Here, the creditor of Marinduque
Mining is DBP, not the directors of Marinduque Mining.
Neither do we discern any bad faith on the part of DBP by its creation of
Nonoc Mining, Maricalum and Island Cement. As Remington itself concedes,
DBP is not authorized by its charter to engage in the mining business.[13] The
creation of the three corporations was necessary to manage and operate the
assets acquired in the foreclosure sale lest they deteriorate from non-use and
lose their value. In the absence of any entity willing to purchase these assets
from the bank, what else would it do with these properties in the
meantime? Sound business practice required that they be utilized for the
purposes for which they were intended.

175

AF REALTY & DEVELOPMENT, INC.


Vs.
DIESELMAN FREIGHT SERVICES, CO.
G.R. No. 111448
FACTS OF THE CASE
Dieselman Freight Services Co., (Diesel man for brevity) is a domestic
corporation and the registered owner of a commercial lot in Brgy. Ugong, Pasig
City. Manuel Cruz Jr, a director of Diesel man issued a letter denominated as
Authority to Sell Real Estate to Cristeta Polintan, a real estate broker, to sell
and negotiate said lot for P3,000.00/sqm. Polintan then authorized Felicisima
Noble to sell the same lot. Noble offered the lot to petitioner who accepted the
offer. Manuel Cruz Sr, president of Diesel man acknowledged receipt of P300,
000.00 as earnest money from petitioner and required the latter to finalize the
sale at P4,000.00/sqm which petitioner agreed with. Manuel Cruz Sr, thereafter,
terminated the offer prompting petitioner to file a complaint for specific
performance against Dieselman and Manuel Cruz Jr., to execute and deliver a
final deed of sale in their favor. In its answer, Dieselman alleged that there was
no meeting of the minds between the parties in the sale of the lot and that it did
not authorize any person to enter into such transaction on its behalf. The trial
court ruled in favor of petitioner but its decision was reversed by the Court of
Appeals. Thus this petition.
ISSUE
Whether the purported contract of sale is binding on Dieselman
RULING
No. The present Corporation Code (B.P. Blg. 68), which took effect on May 1,
1980, provides:
Section 23. The Board of Directors or Trustees. - Unless otherwise provided in
this Code, the corporate powers of all corporations formed under this Code shall
be exercised, all business conducted and all property of such corporations
controlled and held by the board of directors or trustees xxx
As such, the board may validly delegate some of its functions to individual
officers or corporate agent duly appointed by it. Contracts or acts of a corporation
must be made either through the board or its authorized corporate agent. Absent
such valid delegation or authorization, the rule is the acts of individual directors
or officers of the corporation which are not in the course of or in the performance
of their specific function are held not binding on the corporation.
Here, it is undisputed that Manuel Cruz Jr. is not authorized by the Dieselman
board to sell or offer the subject lot, much less appoint persons for the same
purpose. Since there is no valid delegation or authorization from the board, then
the alleged contract of sale is not binding on Dieselman.

176

FILIPINAS PORT SERVIVES, INC.


Vs
GO ET AI,
GR 161886, March 16, 2007
FACTS OF THE CASE
Elidoro Cruz is Filipinas Port Services Inc. from 1968 until he lost his bid
for reelection as the Filports president during a meeting in 1991. He then wrote a
letter to the corporations Board of Directors questioning the Boards creation of
some executive positions with the remunerations of P13,050 monthly each. Cruz
requested the board to take the necessary actions to recover the salaries that the
questioned officers received. The corporation did not take actions on the request
of Cruz. With the inaction, Cruz represented Filport and its stockholders in a
derivative suit against the respondents, the incumbent board of directors during
that time for acts of mismanagement detrimental to the interest of the corporation
and its shareholders at large for the creation of the questioned positions.
RTC ruled in favor of Cruz ruling that Filipinas Port Services is not a big
corporation requiring multiple executive positions" and that said positions "were
just created for accommodation" ordering the respondent to refund the salaries to
the corporation. This decision, however, was reversed by CA; hence, this
petition.
ISSUE
WON the Board of Directors of Filport committed acts of mismanagements
in creating executive positions questioned
RULING
No. Filports Board of Directors did not mismanage the corporation by it
creating the contested executive positions. With the exception only of some
powers expressly granted by law to stockholders (or members, in case of nonstock corporations), the board of directors (or trustees, in case of non-stock
corporations) has the sole authority to determine policies, enter into contracts,
and conduct the ordinary business of the corporation within the scope of its
charter.
In the present case, the boards creation of the positions of Assistant Vice
Presidents for Corporate Planning, Operations, Finance and Administration, and
those of the Special Assistants to the President and the Board Chairman, was in
accordance with the regular business operations of Filport as it is authorized to
do so by the corporations by-laws, pursuant to the Corporation Code.
The election of officers of a corporation is provided for under Section 25 of
the Corporation Code to which Filport faithfully followed in its bylaws
Officers of the corporation, as provided for by the by-laws, shall be elected by
the board of directors at their first meeting after the election of Directors.
The officers of the corporation shall be a Chairman of the Board,
President, a Vice-President, a Secretary, a Treasurer, a General Manager and
such other officers as the Board of Directors may from time to time provide, and
these officers shall be elected to hold office until their successors are elected and
qualified.

177

Likewise, the fixing of the corresponding remuneration for the positions in


question is provided for in the same by-laws of the corporation.
Notwithstanding the silence of Filports bylaws on the matter, the court cannot
rule that the creation of the executive committee by the board of directors is
illegal or unlawful. One reason is the absence of a showing as to the true nature
and functions of said executive committee considering that the "executive
committee," referred to in Section 35 of the Corporation Code which is as
powerful as the board of directors and in effect acting for the board itself, should
be distinguished from other committees which are within the competency of the
board to create at any time and whose actions require ratification and
confirmation by the board. Another reason is that, ratiocinated by both the two (2)
courts below, the Board of Directors has the power to create positions not
provided for in Filports bylaws since the board is the corporations governing
body, clearly upholding the power of its board to exercise its prerogatives in
managing the business affairs of the corporation.
It must also be brought upon that it was during the incumbency of Cruz
that the executive positions he questions were created, and that he was even the
one who moved for the creation of the positions of the AVPs for Operations,
Finance and Administration. By his acquiescence and/or ratification of the
creation of the aforesaid offices, Cruz is virtually precluded from suing to declare
such acts of the board as invalid or illegal. And it makes no difference that he
sues in behalf of himself and of the other stockholders. Indeed, as his voice was
not heard in protest when he was still Filports president, raising a hue and cry
only now leads to the inevitable conclusion that he did so out of spite and
resentment for his non-reelection as president of the corporation

178

ASSOSIATED BANK
VS.
SPS.RAFAEL AND MONALIZA PRONSTROLLER
GR.NO.148444, JULY 14, 2008
FACTS OF THE CASE
On April 21, 1988, Spouses Eduardo and Ma. Pilar Voca executed a Real
Estate Mortgage in favor of the Associated Bank over their parcel of residential
land and house. Due to failure to pay its obligation, Associated Bank won its
bidding in the public auction and was issued the title thereto. Spouses Voca
commenced on action for the nullification of the REM and the foreclosure sale.
The Court of Appeals favored Associated Bank. During the pendency of the
cases, Associated Bank advertised the subject property for sale. Rafael and
Monaliza Pronstroller bought it for P7.5 million with 10% down payment.
On March 18, 1993, Associated Bank though Atty. Soluta, and the
Pronstrollers, executed a Letter-Agreement. In view of the pendency of the case,
the Pronstrollers requested that the balance be payable upon service on them on
the final decision affirming Associated Banks right to posses the property. Atty.
Soluta referred the respondents proposal to Associated Banks Assent Recovery
and Remedial Management Committee (ARRMC), who deferred the action. After
a month, Atty. Soluta executed another Letter-Agreement allowing the request of
the respondent. Meanwhile, Associated Bank reorganized its management.
Atty. Braulio Dayday became Asst. VP and Head of the Documentation
Section, while Atty. Soluta was relieved of his responsibilities. Atty. Dayday
discovered that respondents failed to deposit the balance of the purchase price.
Associated Bank rescinded and cancelled the contract of sale with the
respondent due to breach of contract. The Pronstrollers showed to Atty. Dayday
the second Letter of Agreement signed by Atty. Soluta granting them extension
to pay the balance. Atty. Dayday told them that Atty. Soluta was not authorized to
give them such extension.
In the Vaca case, the Court of Appeals upheld Associated Banks right
over the property. Respondents filed a complaint for specific performance against
Associated Bank. The trial court favored respondents declaring the rescission
null and void. On appeal, the Court of Appeals affirmed the decision of the trial
Court.
ISSUE
Whether or not the Letter- Agreement signed by Atty. Soluta, granting
respondents for an extension to pay the balance binds the bank even without
express authority from the Board of Directors.
RULING
The Letter-Agreement signed by Atty. Soluta is valid and binding upon the
Bank. General Rule is that in the absence of authority from the Board of
Directors, no person, not even its officers, can validly bind a corporation.

179

Exception, The Board may validly delegate some of its functions and
powers to officers, committees and agents.
In previously allowing Atty. Soluta to enter into the First Letter-Agreement
without a Board Resolution expressly authorizing him, petitioner had clothed him
with apparent authority to modify the same via the second Letter-Agreement.

180

CEBU COUNTRY CLUB, INC. ET AL.,


VS
ELIZAGAQUE
G.R. No. 160273. January 18, 2008
FACTS OF THE CASE
Cebu Country Club, Inc. (CCCI), petitioner, is a domestic corporation operating
as a non-profit and non-stock private membership club, having its principal place
of business in Banilad, Cebu City. Petitioners herein are members of its Board of
Directors.
Sometime in 1987, San Miguel Corporation, a special company proprietary
member of CCCI, designated respondent Ricardo F. Elizagaque, its Senior Vice
President and Operations Manager for the Visayas and Mindanao, as a special
non-proprietary member. The designation was thereafter approved by the CCCIs
Board of Directors.
In 1996, respondent filed with CCCI an application for proprietary membership.
The application was indorsed by CCCIs two (2) proprietary members, namely:
Edmundo T. Misa and Silvano Ludo.
As the price of a proprietary share was around the P5 million range, Benito
Unchuan, then president of CCCI, offered to sell respondent a share for only
P3.5 million. Respondent, however, purchased the share of a certain Dr. Butalid
for only P3 million. Consequently, on September 6, 1996, CCCI issued
Proprietary Ownership Certificate No. 1446 to respondent.
Elaziques application for proprietary membership was deferred during the
meetings of the CCCI Board of Directors dated April 4, 1997 and May 30, 1997.
In a Board Meeting dated July 30, 1997 Mr. Elizaques application was voted
upon. He was disapproved. Mr. Elizaque received a letter from the corporate
secretary Julius Z. Neri informing the status of his application.
On August 6, 1997, Edmundo T. Misa, on behalf of respondent, wrote CCCI a
letter of reconsideration. As CCCI did not answer, respondent, on October 7,
1997, wrote another letter of reconsideration. Still, CCCI kept silent. On
November 5, 1997, respondent again sent CCCI a letter inquiring whether any
member of the Board objected to his application. Again, CCCI did not reply.
Consequently, on December 23, 1998, respondent filed with the Regional Trial
Court (RTC), Branch 71, Pasig City a complaint for damages against petitioners,
docketed as Civil Case No. 67190.
ISSUE
Whether in disapproving respondents application for proprietary
membership with CCCI, petitioners are liable to respondent for damages,
and if so, whether their liability is joint and several.
RULING
Human Relations; Principle of Abuse of Rights; Damages; Corporation
Law; While the Board of Directors, under the Articles of Incorporation of a
non-profit and non-stock membership club, may have the right to approve
or disapprove an application for proprietary membership, right should not
181

be exercised arbitrarily.As shown by the records, the Board adopted a secret


balloting known as the black ball system of voting wherein each member will
drop a ball in the ballot box. A white ball represents conformity to the admission
of an applicant, while a black ball means disapproval. Pursuant to Section 3(c),
as amended, cited above, a unanimous vote of the directors is required. When
respondents application for proprietary membership was voted upon during the
Board meeting on July 30, 1997, the ballot box contained one (1) black ball.
Thus, for lack of unanimity, his application was disapproved. Obviously, the CCCI
Board of Directors, under its Articles of Incorporation, has the right to approve or
disapprove an application for proprietary membership. But such right should not
be exercised arbitrarily. Articles 19 and 21 of the Civil Code on the Chapter on
Human Relations provide restrictions, thus: Article 19. Every person must, in the
exercise of his rights and in the performance of his duties, act with justice, give
everyone his due, and observe honesty and good faith. Article 21. Any person
who willfully causes loss or injury to another in a manner that is contrary to
morals, good customs or public policy shall compensate the latter for the
damage.

Same; Same; Same; Same; A right, though by itself legal because


recognized or granted by law as such, may nevertheless become the
source of some illegalitywhen a right is exercised in a manner which does
not conform with the norms enshrined in Article 19 and results in damage to
another, a legal wrong is thereby committed for which the wrongdoer must be
held responsible.In GF Equity, Inc. v. Valenzona, 462 SCRA 466 (2005), we
expounded Article 19 and correlated it with Article 21, thus: This article, known to
contain what is commonly referred to as the principle of abuse of rights, sets
certain standards which must be observed not only in the exercise of ones rights
but also in the performance of ones duties. These standards are the following: to
act with justice; to give everyone his due; and to observe honesty and good faith.
The law, therefore, recognizes a primordial limitation on all rights; that in their
exercise, the norms of human conduct set forth in Article 19 must be observed. A
right, though by itself legal because recognized or granted by law as such, may
nevertheless become the source of some illegality. When a right is exercised in a
manner which does not conform with the norms enshrined in Article 19 and
results in damage to another, a legal wrong is thereby committed for which the
wrongdoer must be held responsible. But while Article 19 lays down a rule of
conduct for the government of human relations and for the maintenance of social
order, it does not provide a remedy for its violation. Generally, an action for
damages under either Article 20 or Article 21 would be proper. (Emphasis in the
original)

Same; Same; Same; Same; The Court cannot fathom why a prestigious and
exclusive golf country club, like the petitioner Cebu Country Club, Inc.,
whose members are all affluent, did not have enough money to cause the
printing of an updated application form.It bears stressing that the
amendment to Section 3(c) of CCCIs Amended By-Laws requiring the
unanimous vote of the directors present at a special or regular meeting was not
182

printed on the application form respondent filled and submitted to CCCI. What
was printed thereon was the original provision of Section 3(c) which was silent on
the required number of votes needed for admission of an applicant as a
proprietary member. Petitioners explained that the amendment was not printed
on the application form due to economic reasons. We find this excuse flimsy and
unconvincing. Such amendment, aside from being extremely significant, was
introduced way back in 1978 or almost twenty (20) years before respondent filed
his application. We cannot fathom why such a prestigious and exclusive golf
country club, like the CCCI, whose members are all affluent, did not have enough
money to cause the printing of an updated application form.

Same; Same; Same; Principle of Damnum Absque Injuria; The principle of


damnum absque injuria does not apply when there is an abuse of a
persons right.As to petitioners reliance on the principle of damnum absque
injuria or damage without injury, suffice it to state that the same is misplaced. In
Amonoy v. Gutierrez, 351 SCRA 731 (2001), we held that this principle does not
apply when there is an abuse of a persons right, as in this case.

Same; Same; Same; While there is no hard-and-fast rule in determining


what would be a fair and reasonable amount of moral damages, the same
should not be palpably and scandalously excessive.As to the appellate
courts award to respondent of moral damages, we find the same in order. Under
Article 2219 of the New Civil Code, moral damages may be recovered, among
others, in acts and actions referred to in Article 21. We believe respondents
testimony that he suffered mental anguish, social humiliation and wounded
feelings as a result of the arbitrary denial of his application. However, the amount
of P2,000,000.00 is excessive. While there is no hard-and-fast rule in
determining what would be a fair and reasonable amount of moral damages, the
same should not be palpably and scandalously excessive. Moral damages are
not intended to impose a penalty to the wrongdoer, neither to enrich the claimant
at the expense of the defendant. Taking into consideration the attending
circumstances here, we hold that an award to respondent of P50,000.00, instead
of P2,000,000.00, as moral damages is reasonable.

Same; Same; Same; Corporation Law; Joint and Solidary Liability;


Directors or trustees who willfully and knowingly vote for or assent to
patently unlawful acts of the corporation or who are guilty of gross
negligence or bad faith in directing the affairs of the corporation or acquire
any personal or pecuniary interest in conflict with their duty as such
directors, or trustees shall be liable jointly and severally for all damages
resulting therefrom suffered by the corporation, its stockholders or
members and other persons.Petitioners argument that they could not be
held jointly and severally liable for damages because only one (1) voted for the
disapproval of respondents application lacks merit. Section 31 of the Corporation
Code provides: SEC. 31. Liability of directors, trustees or officers.Directors or
trustees who willfully and knowingly vote for or assent to patently unlawful acts of
183

the corporation or who are guilty of gross negligence or bad faith in directing the
affairs of the corporation or acquire any personal or pecuniary interest in conflict
with their duty as such directors, or trustees shall be liable jointly and severally
for all damages resulting therefrom suffered by the corporation, its stockholders
or members and other persons. (Emphasis ours)
DISPOSITION
WHEREFORE, we DENY the petition. The challenged Decision and Resolution
of the Court of Appeals in CA-G.R. CV No. 71506 are AFFIRMED with
modification in the sense that (a) the award of moral damages is reduced from
P2,000,000.00 to P50,000.00; (b) the award of exemplary damages is reduced
from P1,000,000.00 to P25,000.00; and (c) the award of attorneys fees and
litigation expenses is reduced from P500,000.00 and P50,000.00 to P50,000.00
and P25,000.00, respectively.
Costs against petitioners.
SO ORDERED.

184

PEOPLE
Vs.
DUMLAO y CASTILIANO and LAOO y GONZALES
G.R. No. 168918
FACTS OF THE CASE
An Amended Information was filed before the Sandiganbayan against
respondents Dumlao and Lao, together with Clave, Cruz and Ver, with violation
of Section 3(g) of R.A. No. 3019 otherwise known as the Anti-Graft and Corrupt
Practices Act. Respondents were sued as members of the Board of Trustees of
the GSIS, a government corporation, hence they are public officers (except Lao)
who allegedly entered into a contract of lease-purchase with Lao, a private
person. Under this contract. The GSIS agreed to sell to Lao a GSIS acquired
property known as the Government Counsel Center, granting Lao the right to
sublease the ground floor for his own account during the period of the lease, from
which he collected yearly rentals in excess of the yearly amortization which
contract is manifestly and grossly advantageous to the government.
Respondent Dumlao filed a Motion to Dismiss/Quash on the ground that
the facts charged do not constitute an offense. He stressed that the prosecutions
main thrust against him was the alleged approval by the GSIS Board of Trustees
of which he was a member of the Lease-Purchase Agreement entered into by
and among the GSIS, the Government Corporate Counsel (OGCC) and
Respondent Lao. He argued that the allegedly approved Board Resolution was
not in fact approved by the GSIS Board of Trustees, contrary to the allegations in
the information. Since the signatures of the officers did not appear in the minutes
of the meeting, he said that it was safe to conclude that these people did not
participate in the alleged approval of the Lease-Purchase Agreement. Thus,
there was no quorum of the board to approve the supposed resolution
authorizing the sale of the GSIS property. There being no approval by the
majority of the Board of Trustees, there can be no resolution approving the
Lease-Purchase Agreement. The unapproved resolution, he added, proved his
innocence.
Sandiganbayan held that the minutes of the meeting of the GSIS Board of
Trustees shows that the Board failed to approve the Lease-Purchase Agreement
in question, and the Resolution was not validly passed since it was only signed
by three (3) members of the Board. Thus, the prosecution has no cause of action
against Dumlao.

ISSUE
Whether or not the signatures of the majority of the GSIS Board of
Trustees are necessary on the minutes of the meeting to give force and effect to
resolution approving the proposed agreement by and among the GSIS, the
OGCC and Respondent Lao.
RULING
The signatures on the minutes of the meeting are not necessary to give
force and effect to resolution approving the proposed agreement by and among
the GSIS, the OGCC and Respondent Lao.

185

A resolution and distinct and different from the minutes of the meeting. A
board resolution is a formal action by a corporate board of directors or other
corporate body authorizing a particular act, transaction or appointment. It is
ordinarily special and limited in its operation, applying usually to some single
specific act or affair of the corporation; or to some specific person, situation or
occasion. On the other hand, minutes are a brief statement not only of what
transpired at a meeting, usually of stockholders/members or directors/trustees,
but also at a meeting of an executive committee. The minutes are usually kept in
a book specially designed for that purpose, but they may also be kept in the form
of memoranda or in any other manner in which they can be identified as minutes
of the meeting.
The Sandiganbayan erred in concluding that since only three members out
of seven signed the minutes of the meeting, the resolution approving the LeasePurchase Agreement was not passed by the GSIS Board of Trustees. The nonsigning by the majority of the members of the GSIS Board of Trustees of the said
minutes does not necessarily mean that the supposed resolution was not
approved by the board. The signing of the minutes by all the members of the
board is not required. There is no provision in the Corporation Code of the
Philippines that the minutes of the meeting should be signed by all members of
the board. It is only the signature of the corporate secretary that gives the
minutes of the meeting probative value and credibility, as the proper custodian of
the books, minutes and official records of the corporation.

186

WESTMONT BANK
VS.
INLAND CONSTRUCTION AND DEVELOPMENT CORPORATION
582 SCRA 230 (2009)
FACTS OF THE CASE
This case is a petition for review on certiorari. Inland Construction and
Development Corp. (Inland) obtained various loans and other credit
accommodations from petitioner, then known as Associated Citizens Bank which
later became United Overseas Bank, Phils., and still later Westmost Bank in
1977 and to secure the payment of its obligations, Inland executed real estate
mortgages over three real properties in Pasig City and likewise issued
promissory note in favor of the bank. When the first and second promissory notes
fell due, Inland defaulted in its payments but authorized the bank to debit
P350,000 from its savings account to partially satisfy its obligations.
Through Deed of Assignment, Conveyance and Release dated May 2,
1978, Felix Aranda, President of Inland, assigned and conveyed all his rights and
interests at Hanil-Gonzales Construction & Development (Phils.) Corporation
(Hanil-Gonzales Corporation) in favor of Horacio Abrantes (Abrantes), Executive
Vice-President and General Manager of Hanil-Gonzales Corporation which
included the assumption of the obligation covered by PN# BD-2884 [-77]
amounting to P800,000. The banks Account Officer, Lionel Calo Jr. (Calo),
signed for its conformity to the deed.
On December 14, 1979, Inland was served a Notice of Sheriffs Sale
foreclosing the real estate mortgages over its real properties, prompting it to file a
complaint for injunction against the bank and the Provincial Sheriff of Rizal at the
RTC of Pasig City. The bank, as response to the complaint, contended that it
had no knowledge, much less did it give its conformity to the alleged assignment
of the obligation covered by PN# BD-2884 [-77]. The RTC found that the bank
ratified the act of its account officer Calo and that the defendant Bank ratified the
act of Calo when its Executive Committee failed to repudiate the assignment
within a reasonable time and even approved the request for a restructuring of
Liberty Const. & Dev. Corp./Hanil- Gonzales Construction & Development
Corp.s obligations. It rendered judgment in favor of Inland. The bank appealed
the trial courts decision to the Court of Appeals which affirmed the decision of
the RTC only insofar as it finds appellant Associated Bank to have ratified the
Deed of Assignment but reversed in all other respects, and judgment is
accordingly rendered ordering the plaintiff-appellee Inland Construction and
Development Corporation to pay defendant-appellant Associated Bank. The bank
moved for partial reconsideration of the appellate courts decision on the aspect
of its ratification of the Deed of Assignment but the same was denied by
Resolution. Hence, this petition.
ISSUE
Whether the RTC erred in finding that the petitioner had ratified the deed of
assignment.
RULING
187

No. Both the RTC and CAs inferences and conclusion that petitioner
ratified its account officers act are thus rationally based on evidence and
circumstances duly highlighted in their respective decisions.
The general rule remains that, in the absence of authority from the board of
directors, no person, not even its officers, can validly bind a corporation. If a
corporation, however, consciously lets one of its officers, or any other agent, to
act within the scope of an apparent authority, it will be estopped from denying
such officers authority. The records show that Calo was the one assigned to
transact on petitioners behalf respecting the loan transactions and arrangements
of Inland as well as those of Hanil-Gonzales and Abrantes. Since it conducted
business through Calo, who is an Account Officer, it is presumed that he had
authority to sign for the bank in the Deed of Assignment.

188

BOARD OF LIQUIDATORS Vs KALAW


G.R. No.L-18805
FACTS OF THE CASE
The National Coconut Corporation (NACOCO, for short) was chartered as
a non-profit governmental organization on avowedly for the protection,
preservation and development of the coconut industry in the Philippines. On
August 1, 1946, NACOCO's charter was amended [Republic Act 5] to grant that
corporation the express power to buy and sell copra. The charter amendment
was enacted to stabilize copra prices, to serve coconut producers by securing
advantageous prices for them, to cut down to a minimum, if not altogether
eliminate, the margin of middlemen, mostly aliens. General manager and board
chairman was Maximo M. Kalaw; defendants Juan Bocar and Casimiro Garcia
were members of the Board; defendant Leonor Moll became director only on
December 22, 1947. NACOCO, after the passage of Republic Act 5, embarked
on
copra
trading
activities.
An unhappy chain of events conspired to deter NACOCO from fulfilling the
contracts it entered into. Nature supervened. Four devastating typhoons visited
the Philippines in 1947. When it became clear that the contracts would be
unprofitable, Kalaw submitted them to the board for approval. It was not until
December 22, 1947 when the membership was completed. Defendant Moll took
her oath on that date. A meeting was then held. Kalaw made a full disclosure of
the situation, apprised the board of the impending heavy losses. No action was
first taken on the contracts but not long thereafter, that is, on January 30, 1948,
the board met again with Kalaw, Bocar, Garcia and Moll in attendanceIn this suit
started in February, 1949, NACOCO seeks to recover the above sum of P1,
343,274.52 from general manager and board chairman Maximo M. Kalaw, and
directors Juan Bocar, Casimiro Garcia and Leonor Moll. It charges Kalaw with
negligence under Article 1902 of the old Civil Code (now Article 2176, new Civil
Code); and defendant board members, including Kalaw, with bad faith and/or
breach of trust for having approved the contracts. By Executive Order 372, dated
November 24, 1950, NACOCO, together with other government-owned
corporations, was abolished, and the Board of Liquidators was entrusted with the
function of settling and closing its affairs.
ISSUE
Whether plaintiff Board of Liquidators has lost its legal personality to
continue with this suit since the three year period has elapsed, the Board of
Liquidators may not now continue with, and prosecute, the present case to its
conclusion
RULING
No, the provision should be read not as an isolated provision but in
conjunction with the whole. So reading, it will be readily observed that no time
limit has been tacked to the existence of the Board of Liquidators and its function
of closing the affairs of the various government owned corporations, including
NACOCO. The President thought it best to do away with the boards of directors
of the defunct corporations; at the same time, however, the President had
chosen to see to it that the Board of Liquidators step into the vacuum. And
nowhere in the executive order was there any mention of the lifespan of the
Board of Liquidators.
189

DE TAVERA
vs.
PHILIPPINE TUBERCULOSIS SOCIETY INC.
G.R.No. L-48928
FACTS OF THE CASE
Petitioner Tavera filed a complaint alleging that she is a doctor of Medicine
by profession and a recognized specialist in the treatment of tuberculosis, having
been in the continuous practice of her profession since 1945; that she is a
member of the Board of Directors of the respondent Society, in representation of
the Philippine Charity Sweepstakes Office and that she was duly appointed as its
Executive Secretary; that sometime in 1974, the past Board of Directors removed
her summarily from her position, the lawful cause of which she was not informed,
through the simple expedient of declaring her position vacant and that
immediately thereafter, respondent Alberto Romulo was appointed to the position
by an affirmative vote of seven directors; and that four of whom, not being
members of defendant Society when they were elevated to the position of
members of the Board of Directors, are not qualified to be elected as such and
hence, all their acts in said meeting are null and void.
The respondents filed their answer specifically denying that plaintiff was
illegally removed from her position as Executive Secretary and averring that
under the Code of By-Laws of the Society, said position is held at the pleasure of
the Board of Directors and when the pleasure is exercised, it only means that the
incumbent has to vacate the same because her term has expired.
The court a quo rendered a decision holding that petitioner was not illegally
rendered or usurped from her position as Executive Secretary in The Society
since plaintiff as holding an appointment all the pleasure of the appointing power
and hence her appointment in essence was temporary in nature, terminable at a
moment's notice without need to show that the termination was for cause.
Petitioner filed a Motion for Reconsideration which the court a quo denied.
Dissatisfied with the decision, petitioner made an appeal to which the Court of
Appeals issued a resolution certifying the case to this Court considering that the
appeal raises no factual issues and involves only issues of law.

ISSUE
Whether or not petitioner was illegally removed from her position as
Executive Secretary.
RULING
Contrary to her claim, petitioner was not illegally removed or from her
position as Executive Secretary in violation of Code of By-laws of the Society, the
New Civil Code and the pertinent provisions of the Constitution.
In the organizational meeting of the Society, the minutes of the meeting
reveal that the Chairman mentioned the need of appointing a permanent
Executive Secretary and stated that the former Executive Secretary, Dr. Jose Y.
Buktaw, tendered his application for optional retirement, and while on terminal
leave, Dr. Mita Pardo de Tavera was appointed Acting Executive Secretary. In
view thereof, Don Francisco Ortigas, Jr. moved, duly seconded, that Dr. Mita
Pardo de Tavera be appointed Executive Secretary of the Philippine
Tuberculosis Society, Inc. The motion was unanimously approved.
190

Although the minutes of the organizational meeting show that the


Chairman mentioned the need of appointing a "permanent" Executive Secretary,
such statement alone cannot characterize the appointment of petitioner without a
contract of employment definitely fixing her term because of the specific provision
of Section 7.02 of the Code of By-Laws that: "The Executive Secretary, the
Auditor, and all other officers and employees of the Society shall hold office at
the pleasure of the Board of Directors, unless their term of employment shall
have been fixed in their contract of employment. Besides the word permanent"
could have been used to distinguish the appointment from acting capacity". The
absence of a fixed term in the letter addressed to petitioner informing her of her
appointment as Executive Secretary is very significant. This could have no other
implication than that petitioner held an appointment at the pleasure of the
appointing power.
An appointment held at the pleasure of the appointing power is in essence
temporary in nature. It is co-extensive with the desire of the Board of Directors.
Hence, when the Board opts to replace the incumbent, technically there is no
removal but only expiration of term and in an expiration of term, there is no need
of prior notice, due hearing or sufficient grounds before the incumbent can be
separated from office. The protection afforded by Section 7.04 of the Code of ByLaws on Removal of Officers and Employees, therefore, cannot be claimed by
petitioner.
Moreover, the act of the Board in declaring her position as vacant is not
only in accordance with the Code of By-Laws of the Society but also meets the
exacting standards of honesty and good faith. The meeting, at which petitioner's
position was declared vacant, was caged specifically to take up the unfinished
business of the Reorganizational Meeting of the Board. Hence, and act cannot
be said to impart a dishonest purpose or some moral obliquity and conscious
doing to wrong but rather emanates from the desire of the Board to reorganize
itself.
Wherefore, premises considered, the decision of the lower court holding
that petitioner was not illegally removed or ousted from her position as Executive
Secretary of the Philippine Tuberculosis Society, Inc., is affirmed.

191

JOSE SIA
Vs.
PEOPLE OF THE PHILIPPINES
GR L-30896, 28 April 1983
FACTS OF THE CASE
Jose O. Sia sometime prior to 24 May, 1963, was General Manager of the
Metal Manufacturing Company of the Philippines, Inc. engaged in the
manufacture of steel office equipment. On 31 May, 1963, because his company
was in need of raw materials to be imported from abroad, he applied for a letter
of credit to import steel sheets from Mitsui Bussan Kaisha, Ltd. of Tokyo, Japan,
the application being directed to the Continental Bank, herein complainant, and
his application having been approved, the letter of credit was opened on 5 June,
1963 in the amount of $18,300. The goods arrived sometime in July, 1963
according to accused himself, now from here on there is some debate on the
evidence; according to Complainant Bank, there was permitted delivery of the
steel sheets only upon execution of a trust receipt, while according to the
accused, the goods were delivered to him sometime before he executed that
trust receipt in fact they had already been converted into steel office equipment
by the time he signed said trust receipt. But there is no question - and this is not
debated that the bill of exchange issued for the purpose of collecting the
unpaid account thereon having fallen due neither accused nor his company
having made payment thereon notwithstanding demands, and the accounts
having reached the sum in pesos of P46, 818.68 after deducting his deposit
valued at P28,736.47.
ISSUE
Whether petitioner Jose O. Sia, having only acted for and in behalf of the
Metal Manufacturing Company of the Philippines as President thereof in dealing
with the complainant, the Continental Bank, may be held liable for the crime
charged.
RULING
No. The bank is transacting with Metal Manufacturing and not with him.
The case cited by the Court of Appeals in support of its stand - Tan Boon Kong
case, supra - may however not be squarely applicable to the instant case in that
the corporation was directly required by law to do an act in a given manner, and
the same law makes the person who fails to perform the act in the prescribed
manner expressly liable criminally. The performance of the act is an obligation
directly imposed by the law on the corporation. Since it is a responsible officer or
officers of the corporation who actually perform the act for the corporation, they
must of necessity be the ones to assume the criminal liability; otherwise this
liability as created by the law would be illusory, and the deterrent effect of the
law, negated.
In the present case, a distinction is to be found with the Tan Boon Kong
case in that the act alleged to be a crime is not in the performance of an act
directly ordained by law to be performed by the corporation. The act is imposed
by agreement of parties, as a practice observed in the usual pursuit of a business
or a commercial transaction. The offense may arise, if at all, from the peculiar
terms and condition agreed upon by the parties to the transaction, not by direct
provision of the law. The intention of the parties, therefore, is a factor determinant
of whether a crime was committed or whether a civil obligation alone intended by
192

the parties. With this explanation, the distinction adverted to between the Tan
Boon Kong case and the case at bar should come out clear and meaningful. In
the absence of an express provision of law making the petitioner liable for the
criminal offense committed by the corporation of which he is a president as in
fact there is no such provisions in the Revised Penal Code under which petitioner
is being prosecuted, the existence of a criminal liability on his part may not be
said to be beyond any doubt. In all criminal prosecutions, the existence of
criminal liability for which the accused is made answerable must be clear and
certain. The maxim that all doubts must be resolved in favour of the accused is
always of compelling force in the prosecution of offenses.

193

VILLANUEVA
Vs.
ADRE
G.R. No. 80863
FACTS OF THE CASE
A recovery of unpaid 13th month pay was filed by the Sarangani Marine
and General Workers Union-ALU and 37 South Cotabato Integrated Port
Services, Inc. (SCIPSI) employees with the Department of Labor against SCIPSI,
a Philippine corporation.
Labor arbiter Villanueva after hearing, ordered a dismissal. On appeal,
however, the NLRC, reversed and accordingly, ordered the private respondents,
SCIPSI and its president and general, Lucio Velayo, to pay the 13th month pays
demanded.
Thereafter, the parties, on orders of the labor arbiter, were made to appear
before a corporate auditing examiner to determine the private respondents exact
liability. The corporate auditing examiner submitted an accounting and found the
private respondents liable in the total sum of 1.1M. Thereupon, the private
respondents interposed an objection and prayed for a revision. It appears,
however, that the private respondents never pursued their exceptions.
The union moved for execution and pursuant thereto, the labor arbiter
issued a writ of execution. As a result, the sheriff levied on 2 parcels of land, both
registered in Lucio Velayos name.
Velayo alone filed a petition with the respondent court on a cause of action
based on an alleged irregular execution, on the ground that he was never a party
to the labor case and that a corporation (that is, SCIPSI) has a separate and
distinct personality from its incorporators, stockholders and officers.
ISSUE
Whether Velayo was no party to the controversy.
RULING
No.
The fact that he was never mentioned in the pleadings before the
petitioner-labor arbiter is of no moment. The fact is that he himself had
questioned the findings of the corporate auditor and this is enough evidence that
he admits personal liability, although he does not agree with the amount
supposedly due from him.
But other than estoppel, the law itself stands as a formidable obstacle to
Velayos claims. In AC Ransom Labor Union-CCLU v. NLRC, the SC held that in
case of corporations. It is the president who responds personally for violation of
the labor pay laws.
Accordingly, Velayo cannot be excused from payment of SCIPSIs liability
by mere reason of SCIPSIs separate corporate existence. The theory of
corporate entity, in the first place, was not meant to promote unfair objectives or
otherwise, to shield them. This court has not hesitated in penetrating the veil of
corporate fiction when it would defeat the ends envisaged by law, not to mention
194

the clear decree of the Labor Code. And if Velayo truly had a valid objection (to
the levy on his properties), he could have raised it at the earliest hour, and in the
course of the labor proceedings themselves. He is not only estopped, litis
pendencia is a bar to such a separate action.

195

PRIME WHITE CEMENT CORPORATION


VS.
HONORABLE INTERMEDIATE APPELLATE COURT
G.R.NO. L-68555
FACTS OF THE CASE
On or about the 16th day of July, 1969, plaintiff and defendant corporation
thru its President, Mr. Zosimo Falcon and Justo C. Trazo, as Chairman of the
Board, entered into a dealership agreement (Exhibit A) whereby said plaintiff was
obligated to act as the exclusive dealer and/or distributor of the said defendant
corporation of its cement products in the entire Mindanao area for a term of five
(5) years and proving (sic) among others that:
a. The corporation shall, commencing September, 1970,
sell to and supply the plaintiff, as dealer with 20,000
bags (94 lbs/bag) of white cement per month;
b. The plaintiff shall pay the defendant corporation
P9.70, Philippine Currency, per bag of white cement,
FOB Davao and Cagayan de Oro ports;
c. The plaintiff shall, every time the defendant
corporation is ready to deliver the good, open with any
bank or banking institution a confirmed, unconditional,
and irrevocable letter of credit in favor of the corporation
and that upon certification by the boat captain on the bill
of lading that the goods have been loaded on board the
vessel bound for Davao the said bank or banking
institution shall release the corresponding amount as
payment of the goods so shipped.
Right after the plaintiff entered into the aforesaid dealership agreement, he
placed an advertisement in a national, circulating newspaper the fact of his being
the exclusive dealer of the defendant corporation's white cement products in
Mindanao area, more particularly, in the Manila Chronicle dated August 16, 1969
(Exhibits R and R-1) and was even congratulated by his business associates, so
much so, he was asked by some of his businessmen friends and close
associates if they can be his sub-dealer in the Mindanao area.
Relying heavily on the dealership agreement, plaintiff sometime in the
months of September, October, and December, 1969, entered into a written
agreement with several hardware stores dealing in buying and selling white
cement in the Cities of Davao and Cagayan de Oro which would thus enable him
to sell his allocation of 20,000 bags regular supply of the said commodity, by
September, 1970 (Exhibits O, O-1, O-2, P, P-1, P-2, Q, Q-1 and Q-2). After the
plaintiff was assured by his supposed buyer that his allocation of 20,000 bags of
white cement can be disposed of, he informed the defendant corporation in his
letter dated August 18, 1970 that he is making the necessary preparation for the
opening of the requisite letter of credit to cover the price of the due initial delivery
for the month of September, 1970 (Exhibit B), looking forward to the defendant
corporation's duty to comply with the dealership agreement. In reply to the
aforesaid letter of the plaintiff, the defendant corporation thru its corporate
secretary, replied that the board of directors of the said defendant decided to
impose the following conditions:
a. Delivery of white cement shall commence at the end
of November, 1970;
b. Only 8,000 bags of white cement per month for only a
period of three (3) months will be delivered;
196

c. The price of white cement was priced at P13.30 per


bag;
d. The price of white cement is subject to readjustment
unilaterally on the part of the defendant;
e. The place of delivery of white cement shall be
Austurias (sic);
f. The letter of credit may be opened only with the
Prudential Bank, Makati Branch;
g. Payment of white cement shall be made in advance
and which payment shall be used by the defendant as
guaranty in the opening of a foreign letter of credit to
cover costs and expenses in the procurement of
materials in the manufacture of white cement. (Exhibit
C).
Several demands to comply with the dealership agreement (Exhibits D, E,
G, I, R, L, and N) were made by the plaintiff to the defendant, however,
defendant refused to comply with the same, and plaintiff by force of
circumstances was constrained to cancel his agreement for the supply of white
cement with third parties, which were concluded in anticipation of, and pursuant
to the said dealership agreement.
Notwithstanding that the dealership agreement between the plaintiff and
defendant was in force and subsisting, the defendant corporation, in violation of,
and with evident intention not to be bound by the terms and conditions thereof,
entered into an exclusive dealership agreement with a certain Napoleon Co for
the marketing of white cement in Mindanao (Exhibit T) hence, this suit.

ISSUE
Whether or not the "dealership agreement" referred by the President and
Chairman of the Board of petitioner corporation is a valid and enforceable
contract.
RULING
Under the Corporation Law, which was then in force at the time this case
arose, as well as under the present Corporation Code, all corporate powers shall
be exercised by the Board of Directors, except as otherwise provided by
law. Although it cannot completely abdicate its power and responsibility to act for
the juridical entity, the Board may expressly delegate specific powers to its
President or any of its officers. In the absence of such express delegation, a
contract entered into by its President, on behalf of the corporation, may still bind
the corporation if the board should ratify the same expressly or impliedly. Implied
ratification may take various forms like silence or acquiescence; by acts
showing approval or adoption of the contract; or by acceptance and retention of
benefits flowing therefrom. Furthermore, even in the absence of express or
implied authority by ratification, the President as such may, as a general rule,
bind the corporation by a contract in the ordinary course of business, provided
the same is reasonable under the circumstances. These rules are basic, but are
all general and thus quite flexible. They apply where the President or other
officer, purportedly acting for the corporation, is dealing with a third person, i. e.,
a person outside the corporation.
The contract with Henry Wee was on September 15, 1969, and that with
Gaudencio Galang, on October 13, 1967. A similar contract with Prudencio Lim
was made on December 29, 1969. All of these contracts were entered into soon
197

after his "dealership agreement" with Petitioner Corporation, and in each one of
them he protected himself from any increase in the market price of white cement.
Yet, except for the contract with Henry Wee, the contracts were for only two
years from October, 1970. Why did he not protect the corporation in the same
manner when he entered into the "dealership agreement"? For that matter, why
did the President and the Chairman of the Board not do so either? As director,
especially since he was the other party in interest, respondent Te's bounden duty
was to act in such manner as not to unduly prejudice the corporation. In the light
of the circumstances of this case, it is quite clear that he was guilty of disloyalty
to the corporation; he was attempting in effect, to enrich himself at the expense
of the corporation. There is no showing that the stockholders ratified the
"dealership agreement" or that they were fully aware of its provisions. The
contract was therefore not valid and this Court cannot allow him to reap the fruits
of his disloyalty.
As a result of this action which has been proven to be without legal basis,
petitioner corporation's reputation and goodwill have been prejudiced. However,
there can be no award for moral damages under Article 2217 and succeeding
articles on Section 1 of Chapter 3 of Title XVIII of the Civil Code in favor of a
corporation.
In view of the foregoing, the Decision and Resolution of the Intermediate
Appellate Court dated March 30, 1984 and August 6, 1984, respectively, are
hereby SET ASIDE. Private respondent Alejandro Te is hereby ordered to pay
Petitioner Corporation the sum of P20, 000.00 for attorney's fees, plus the cost of
suit and expenses of litigation.

198

TRAMAT MERCANTILE VS. COURT OF APPEALS


283 SCRA 14 (1994)
VITUG, J.:
FACTS
David Ong, president of Tramat Mercantile Inc, bought a hinamoto tractor
form Melchor De La Cuesta. As payment for the said tractor, Ong issued a check
worth 33,500. Tramat sold the said tractor to MWSS (NAWASA) for 67,500
together with a lawn mower attached to it fabricated by tramat. Ong causes a
stop payment to dela cuesta after NAWASA refused to pay the tractor when it
discovered some defects and that the engine was reconditioned unit.
De la cuesta filed an action for collection against Ong and Tramat for the
unpaid balance, Ong opposed the action averring that dela cuesta does not have
the cause of action against him in his personal capacity. The trial court rendered
a decision ordering Ong and tramat to pay solidarily the unpaid balance with
legal interest.
ISSUE
Whether Ong should be held solidarily liable with tramat when he was only
acting as an officer of the corporation
RULING
Ong should not be held solidarily liable with the corporation, since he was
only acting not in his personal capacity, but as an officer of the corporation, who
has a separate and distinct personality. As such, it should only be tramat that
should be held liable.
Personal liability of a corporate director, trustee, or officer, may only be
validly attached when the following requisites are present:
1. if the officer assents to a (i.) patently unlawful act of a corporation,
(ii.) or for bad faith/gross negligence in directing the corporate affairs, (iii.) or for
conflict of interest resulting to damages to the corporation.
2. If the officer consents to the issuance of watered stocks or who,
having knowledge thereof, does not immediately file with the corporate secretary
his objection.
3. If he agrees to hold himself personally and solidarily liable.
4. If he is made by specific provision of the law, to be personally
liable for his corporate actions

There is no indication that Ong could be personally liable under any of the
aforementioned conditions.

199

DE GUZMAN
VS.
NLRC, et al,
G.R. No. 90856, July 23, 1992
FACTS OF THE CASE
Petitioner was the general manager of the Manila Office of Affiliated
Machineries Agency, Ltd. (AMAL) and among the respondents in a complaint for
illegal dismissal and non-payment of statutory benefits filed by herein
respondents who were former employees of AMAL.
Respondent employees initiated the complaint following AMALs refusal to
pay the formers monetary claims after AMAL decided to cease its operations in
1986. Petitioner was impleaded for allegedly selling part of AMALs assets and
applying the proceeds of the same, as well as the remaining assets, to satisfy his
own claims against the company. He also formed a new company named
Susarco, Inc. and engaged in the same line of business with the former clients of
AMAL.
In 1987, the Labor Arbiter rendered judgment and held petitioner jointly
and severally liable with AMAL for respondent employees claims. The NLRC
affirmed the decision. Petitioner proceeded to the Supreme Court on certiorari.
ISSUE
Whether or not petitioner should be held jointly and severally liable for the
respondent employees' claims
RULING
No. A mere general manager cannot be held solidarily liable with the
corporation for unpaid labor claims. The petitioner, while admittedly the highest
ranking local representative of AMAL in the Philippines, is nevertheless not a
stockholder and much less a member of the board of directors or an officer
thereof.
As such, the petitioner cannot be held directly responsible for the decision
to close the business that resulted in his separation and that of the private
respondents. That decision came directly and exclusively from AMAL. The
petitioners participation was limited to the enforcement of this decision in line
with his duties as general manager of the company. Even in a normal situation, in
fact, he would not be liable, as a managerial employee of AMAL, for the
monetary claims of its employees. There should be no question that the private
respondents recourse for such claims cannot be against the petitioner but
against AMAL and AMAL alone.
However, a general manager who appropriated to himself the assets of his
employer-corporation to pay his claims against the latter after it folded-up without
reserving a portion to pay the claims of other employees acts in bad faith and
liable for damages. The petitioner in the case at bar had his own claims against
AMAL and consequently had some proportionate right over its assets. However,
this right ceased to exist when, knowing fully well that the private respondents
200

had similarly valid claims, he took advantage of his position as general manager
and applied AMALs assets in payment exclusively of his own clai

201

NEW DURAWOOD COMPANY INC.


Vs.
COURT OF APPEALS
G.R. No. 111732
FACTS OF THE CASE
On February 14, 1990, a Petition for Judicial Reconstitution of the Lost
Owners Duplicate Certificates of TCT Nos. 140486; 156454 and 1404855. was
filed in the Regional Trial Court, Branch LXXI, Antipolo, Rizal by petitionercorporation, represented by its Branch Manager, Wilson M. Gaw x x x. Attached
to said petition was an Affidavit of Loss dated December 31, 19906 of
respondent Orlando S. Bongat, one of the stockholders of petitioner-corporation.
Finding the petition to be sufficient in form and in substance, respondent Judge
set the case for hearing on March 18, 1991. On April 16, 1991, respondent Judge
issued the questioned order.
Sometime in May, 1991, petitioner discovered that the original TCT Nos.
N-140485, N-140486 and 156454 on file with the Register of Deeds of Rizal had
been cancelled and, in lieu thereof, TCT Nos. 200100, 200101 and 200102 had
been issued in the name of respondent Durawood Construction and Lumber
Supply, Inc. Surprised by this cancellation, petitioner - after investigation - found
out about the reconstitution proceeding in the respondent trial court. So, on July
17, 1991, petitioner filed suit7 in the Court of Appeals docketed as CAG.R.25434 praying for the annulment of the assailed order in LRC Case No. 91924 penned by respondent Judge. It also prayed for the cancellation of the new
certificates (TCT Nos. 200100, 200101 and 200102). On May 31, 1993, the
respondent Court of Appeals rendered the assailed Decision and on August 30,
1993, the Resolution denying the motion for reconsideration. Hence, the present
recourse to the Supreme Court.
ISSUE
Whether a board resolution issued in the absence of a quorum during the
board meeting is valid.
RULING
No. The appellate court explained that while there may not have been a
quorum during the board meeting of petitioner-corporation on May 10, 1984
when a resolution authorizing Gaw to sue on its behalf was allegedly passed, this
did not mean however, that New Durawood Co., Inc. cannot be bound by Gaws
action because no howl of protest, complaint or denial came from (said
corporation), and that said corporation in fact had taken advantage of the
benefits therefrom. Hence, petitioner is estopped from questioning Gaws acts.
The appellate Court was of the belief that petitioner-corporation ratified Gaw s
authority by acquiescence to his acts. The respondent Court thus concluded
that petitioner-corporations claim of being a victim of extrinsic fraud is
baseless.
We are appalled by this rather novel interpretation of corporate law. It is
clear that, there having been no quorum present during the meeting in question,
the board of directors could not have validly given Gaw any express authority to
file the petition. Upon the other hand, the doctrine of apparent authority cannot
apply as to Gaw because, being a mere branch manager, he could not be looked
upon as acorporate officer clothed with the implied or apparent power to file suit
202

for and in behalf of a corporation.11 Neither will estoppel prevent the corporation
from questioning Gaws acts. Precisely, these acts were hidden from the
company and its top officers. How then can estoppel attach?

203

PURIFICACION G. TABANG
Vs.
NATIONAL LABOR RELATIONS COMMISSION and PAMANA GOLDEN
CARE MEDICAL CENTER FOUNDATION INC.
G.R. No. 121143
FACTS OF THE CASE
Petitioner was a member of the Board of Trustees and corporate secretary
of respondent Pamana Golden Care Medical Center Foundation Inc (Pamana for
brevity). She was appointed by the board as Medical Director and Hospital
Administrator in its medical center in Calamba, Laguna. Thereafter she was
relieved of her position through a resolution during a special meeting by the
board prompting her to file a complaint for illegal dismissal and non-payment of
wages, allowances and 13th month pay before the labor arbiter. Pamana moved
for the dismissal of the complaint for lack of jurisdiction contending that the case
is an intra-corporate controversy which falls under the exclusive jurisdiction of the
SEC. Petitioner opposed the motion to dismiss, contending that her position as
Medical Director and Hospital Administrator was separate and distinct from her
position as member of the Board of Trustees. She claimed that there is no intracorporate controversy involved since she filed the complaint in her capacity as
Medical Director and Hospital Administrator, or as an employee of private
respondent. The labor arbiter dismissed her complaint for lack of jurisdiction
which was affirmed by the NLRC.
ISSUE
Whether the petitioner is a corporate officer which would render her
dismissal an intra-corporate controversy
RULING
Yes. An office is created by the charter of the corporation and the officer
is elected by the directors or stockholders. On the other hand, an employee
usually occupies no office and generally is employed not by action of the
directors or stockholders but by the managing officer of the corporation who also
determines the compensation to be paid to such employee.
The president, vice-president, secretary and treasurer are commonly
regarded as the principal or executive officers of a corporation, and modern
corporation statutes usually designate them as the officers of the
corporation. However, other offices are sometimes created by the charter or bylaws of a corporation or the board of directors may be empowered under the bylaws of a corporation to create additional offices as may be necessary.
Under the by-laws of Respondent Corporation a medical director and a
hospital administrator are considered as corporate officers.
Section 2(i), Article I thereof states that one of the powers of the Board of
Trustees is (t)o appoint a Medical Director, Comptroller/Administrator, Chiefs of
Services and such other officers as it may deem necessary and prescribe their
powers and duties
In the case at bar, considering that herein petitioner, unlike an ordinary
employee, was appointed by Respondent Corporations Board of Trustees in its
memorandum of October 30, 1990, she is deemed an officer of the corporation
204

A corporate officers dismissal is always a corporate act, or an intracorporate controversy, and the nature is not altered by the reason or wisdom with
which the Board of Directors may have in taking such action. Also, an intracorporate controversy is one which arises between a stockholder and the
corporation. There is no distinction, qualification, nor any exemption
whatsoever. The provision is broad and covers all kinds of controversies
between stockholders and corporations.

205

NAGUIAT
Vs
NATIONAL LABOR RELATIONS COMMISSION
G.R. No. 116123
March 13, 1997
FACTS OF THE CASE
Private respondents, Leonardo Galang et al, were previously employed by
petitioner Clark Field Taxi, Inc (CFTI). CFTI held a concessionaire contract with
the Army Air Force Exchange Services (AAFES) for taxi services in the Clark Air
Base. Sergio Naguiat was CFTIs president, while Anton Naguiat was the vicepresident. CFTI is a family owned corporation. When the US military base was
phased out, the services of CFTI was also terminated. The drivers union of the
taxi services bargained for a severance pay to which they were granted P500 for
every year of service. Private respondents refused to accept their severance pay.
Through a new labor organization National Organization of Workingmen
the private respondents disaffiliated themselves from the drivers' union, filed a
complaint against Sergio F. Naguiat doing business under the name and style
Sergio F. Naguiat Enterprises, Inc. and CFTI with Antolin T. Naguiat as vice
president and general manager, as party respondent., for payment of separation
pay due to termination/phase-out.
Private respondents alleged that they were regular employees of Naguiat
Enterprises, although their individual applications for employment were approved
by CFTI. They claimed to have been assigned to Naguiat Enterprises after
having been hired by CFTI and that the former thence managed, controlled and
supervised their employment. They averred further that they were entitled to
separation pay based on their latest daily earnings of US$15.00 for working
sixteen (16) days a month.
The petitioners averred that the business was closed because of great
financial losses resulting from the eruption of Mt. Pinatubo. They admitted that
CFTI had agreed with the drivers' union, through its President Eduardo Castillo
who claimed to have had blanket authority to negotiate with CFTI in behalf of
union members, to grant its taxi driver-employees separation pay equivalent to
P500.00 for every year of service.
The labor arbiter found the drivers as regular employees of CFTI and ordered the
latter to pay them P1, 200 for every year of services for humanitarian
consideration. The decision was appealed to NLRC which granted the
respondents a right to severance pay from CFTI and made petitioners, Naguiat
Enterprises, Sergio Naguiat and Antonio Naguiat as solidary liable.
ISSUE
WON Naguiat Enterprises, Sergio and Antonio should be liable for the
severance pay of CFTIs taxi drivers.
RULING
Naguiat Enterprises shall not be liable. Private respondents failed to
substantiate their claim that Naguiat Enterprises managed, supervised and
controlled their employment. It appears that they were confused on the
personalities of Sergio F. Naguiat as an individual who was the president of
CFTI, and Sergio F. Naguiat Enterprises, Inc., as a separate corporate entity with
a separate business. They presumed that Sergio F. Naguiat, who was at the
206

same time a stockholder and director of Sergio F. Naguiat Enterprises, Inc. was
managing and controlling the taxi business on behalf of the latter. A closer
scrutiny and analysis of the records, however, evince the truth of the matter: that
Sergio F. Naguiat, in supervising the taxi drivers and determining their
employment terms, was rather carrying out his responsibilities as president of
CFTI. Hence, Naguiat Enterprises as a separate corporation does not appear to
be involved at all in the taxi business.
Sergio Naguiat shall be liable. Sergio F. Naguiat, admittedly, was the
president of CFTI who actively managed the business. Thus, applying the ruling
in A.C. Ransom, he falls within the meaning of an "employer" as contemplated by
the Labor Code, who may be held jointly and severally liable for the obligations of
the corporation to its dismissed employees.
Moreover, petitioners also conceded that both CFTI and Naguiat Enterprises
were "close family corporations" owned by the Naguiat family. Section 100,
paragraph 5, (under Title XII on Close Corporations) of the Corporation Code,
states:
(5) To the extent that the stockholders are actively engaged in the
management or operation of the business and affairs of a close corporation, the
stockholders shall be held to strict fiduciary duties to each other and among
themselves. Said stockholders shall be personally liable for corporate torts
unless the corporation has obtained reasonably adequate liability insurance.
The fifth paragraph of Section 100 of the Corporation Code specifically
imposes personal liability upon the stockholder actively managing or operating
the business and affairs of the close corporation.
Antonio Naguiat is not liable. Antolin was the vice president of the CFTI.
Although he carried the title of "general manager" as well, it had not been shown
that he had acted in such capacity. Furthermore, no evidence on the extent of his
participation in the management or operation of the business was preferred. In
this light, he cannot be held solidarily liable for the obligations of CFTI and Sergio
Naguiat to the private respondents.

207

LLAMADO VS COURT OF APPEALS


GR.NO.99032 MARCH 26, 1997
FACTS OF THE CASE
Ricardo Llamado and his co-accused Jacinto Pascual were the Treasure
and President, respectively of the Pan Asia Finance Corporation. Both of them
were charged with violation of B.P. 22. Jacinto Pascual remained at larged
hence; trial on the merits was conducted against Ricardo Llamado only.
Private complainant Leon GAW delivered to accused Ricardo Llamado the
amount P180, 000.00 in cash with the assurance that it will be re paid on
November 4, 1983, plus interest thereon of 12% plus a share in the profits of the
corporation.
Accused Jacinto Pascual and Ricardo Llamado then signed Philippine
Trust Company Check No. 047809, postdated November 4, 1983 in the amount
of P186.500, issued to private complainant, in payment of the cash money
delivered, plus interest thereon for sixty (60) days in the amount of P6,500.00.
On November 4, 1983 complainant deposited the Check in his current
account with Equitable Bank, which later dishonored by said bank because
payment was stopped, and that the check was drawn against insufficient funds.
Private complainant went to accused Ricardo Llamado to inform him of the
dishonor of the check. Accused offered to pay 10% of the principal amount and
the balance to be rolled over for 90 days. The offer was accepted by the
complainant.
Accused, however failed to pay the aforesaid 10% and to roll over the
balance of the money. Complainant demanded full payment of the principal plus
interest, but the accused offered to pay only 30% of his money which was
refused by the letter. Hence, a complaint for violation of B.P. 22 was filed by the
private complainant.
The trial court rendered the accused guilty of Violation of B.P. 22. On
appeal, the Court of Appeals affirmed the decision of the trial court.
ISSUE
Whether or not the accused is liable for the amount of the check which he
merely signed the same on behalf of Pan Asia Finance Corporation, in his
capacity as Treasurer of said corporation?
RULING
The accused should be held liable to the amount of the check. Third
paragraph of Sec. 1 of B.P. 22 states:
Where the check is drawn by a corporation, company or entity, the person
or persons who actually signed the check in behalf of such drawer shall be liable
under this Act.
The Supreme Court affirmed the decision of the Court of Appeals.
208

ELENA F. UICHICO, SAMUEL FLORO, VICTORIA F. BASILIO, petitioners,


vs. NATIONAL LABOR RELATIONS COMMISSION, LUZVIMINDA SANTOS,
SHIRLEY POR-RAS, CARMEN ELIZARDE, ET AL., respondents.
G.R. No. 121434 (June 2, 1997)
FACTS OF THE CASE
Nature of the Case: Special Civil Action for Certiorari and Prohibition against the
Resolution of the NLRC holding petitioners liable for illegal dismissal.
Private respondents were employed by Crispa, Inc. for many years and their
employment was terminated by Crispa for retrenchment on the ground of serious
business losses.
Complaints for illegal dismissal were filed against Crispa and Valeriano Floro
who was a major stockholder, incorporator and director of Crispa, and the
petitioners who were high ranking officers and directors of the company.
The Labor Arbiter initially rendered a decision dismissing the complaint for illegal
dismissal but ordered Crispa, Floro and the Petitioners to pay the employees
separation pay equivalent to 17 days for every year of service.
Dissatisfied, the employees appealed and the Second Division of the NLRC
found the petitioners also liable for illegal dismissal and modified the separation
pay to 1 month pay for every year of service. Retrenchment to be valid, the
NLRC said that the company must present clear and convincing evidence that it
is suffering from substantial losses.
Petitioners filed this petition after a denial of their Motion for Reconsideration.
The Petitioners are assailing the decision of the NLRC holding them solidarily
liable with Crispa to pay the salaries and backwages of the dismissed
employees.
ISSUE
WHETHER THE AWARD OF BACKWAGES AND SEPARATION PAY IS A
CORPORATE OBLIGATION WHICH SHOULD ONLY BE SHOULDERED BY
CRISPA ALONE?
RULING
The Court denied the petition. The general rule is that obligations incurred by the
corporation acting through its directors, officers and employees are it soles
liabilities. However solidarity liability may be incurred but only through
exceptional circumstances:
1. When directors and trustees or, in appropriate cases, the officers of a
corporation a) vote for or assent to patently unlawful acts of the
corporation, its stockholders or members, and other persons b) gross
negligence in directing the corporate affairs c) are guilty of conflict of
interest to the prejudice of the corporation, its stockholders, or
members, and other persons

209

2. When a director or officer has consented to the issuance of watered


stocks or who, having knowledge thereof, did not forthwith file with the
corporate secretary his written objection thereto;
3. When a director, trustee, or officer has contractually agreed or
stipulated to hold himself personally and solidariy liable with the
corporation or
4. When a director, trustee or officer is made, by specific provision of law,
personally liable for his corporate action.
In labor cases, corporate directors and officers are solidarily liable with the
corporation for the termination of employment of corporate employees done with
malice or in bad faith. Petitioners in this case have a direct hand in the illegal
dismissal of the respondent employees. They were the ones who signed the
Board Resolution retrenching the private respondents on the feigned ground of
serious business losses that has no basis apart from an unsigned and unaudited
Profit and Loss Statement which had no evidentiary value.

210

TORRES
Vs.
COURT OF APEALS
G.R. No. 120138
FACTS OF THE CASE
Judge Torres was the majority stockholder of Tormil Realty & Development
Corporation while private respondents who are the children of Judge Torres
deceas Brother Antonio A. Torres, constituted the minority stockholders. In 1987,
the annual stockholders meeting and election of directors of Tormil was
scheduled, in compliance with its by-laws. Pursuant to this, Judge Torres
assigned from his own shares, one share each to petitioners Tobias, Jocson,
Jurisprudential, Azura and Pabalan. These assigned shares were in the nature of
qualifying shares for the sole purpose of meeting the legal requirement to be
able to elect them to the Board of Directors as Torres nominees.
During the election, the corporate secretary refused to write down the
names of nominees, prompting Azura to initiate the appointment of Jocson as
Acting Secretary. Judge Torres nominated his companions and at this point,
private respondent Milagros Torres told them to leave. Then, Judge Torres
together with his companions went to the residence of Ma. Jacinta Torres. They
represented the majority of the outstanding capital stock and still constituted a
quorum. The meeting was resumed and the nominees were elected as directors
of Tormil.
Private respondents instituted a complaint with the SEC, praying that the
election of the petitioners to the Board of Directors be annulled. They alleged that
petitioners-nominees were not legitimate stockholders of Tormil because the
assignment of shares to them violated the minority stockholders right of preemption as provided in the corporations articles and by-laws. The SEC rendered
a decision declaring the election null and void. SEC en banc affirmed the
decision. The Court of Appeals also affirmed the SEC en banc decision.
ISSUE
Whether or not the CA erred in affirming the declaration of nullity of the
said election
RULING
The CA did not err in affirming the declaration of nullity of the election.
In the absence of any provision to the contrary, the corporate secretary is
the custodian of the corporate records. Corollarily, he keeps the stock and
transfer book and makes proper and necessary entries therein. Contrary to the
generally accepted corporate practice, the stock and transfer book of Tormil was
not kept by the corporate secretary by Torres, the President and Chairman of the
Board of Directors of Tormil. Thus, any entry made in the stock and transfer book
by Judge Torres of an alleged transfer of nominal shares cannot be given any
valid effect. Therefore, the elected officers cannot be considered stockholders of
record of Tormil. Because they are not stockholders, they cannot be elected as
directors of Tormil. To rule otherwise would not only encourage violation of the
clear mandate of Sec. 74 of the Corporation Code that stock and transfer book
shall be kept in the principal office of the corporation but would likewise open the
flood gates of confusion in the corporation as to who has the proper custody of
211

the stock and transfer book and who are the real stockholders of records of a
certain corporation as any holder of the stock and transfer book, though not the
corporate secretary, at pleasure would make entries therein. The fact that Judge
Torres holds 81.28% of the outstanding capital stock of Tormil is of no moment
and is not a license for him to arrogate unto himself a duty of the corporate
secretary.

212

SAN JUAN STRUCTURE AND STEEL FABRICATORS INC.


VS.
COURT OF APPEALS
296 SCRA 631 (1998)
FACTS OF THE CASE
This case is a petition for review on certiorari. On 14 February 1989, San
Juan Structural and Steel Fabricators, Inc. (SJSSFI) entered into an agreement
with Motorich Sales Corporation (MSC) for the transfer to it of a parcel of land in
the District of Murphy, Quezon City, Metro Manila, containing an area of 414
square meters, covered by TCT (362909) 2876 which was still registered in the
name of ACL Development Corporation (ADC) at that time. As stipulated in the
Agreement, SJSSFI paid the downpayment in the sum of P100,000.00, the
balance to be paid on or before 2 March 1989. On 1 March 1989, Mr. Andres T.
Co, SJSSFI president, wrote a letter to MSC requesting for a computation of the
balance to be paid. Linda Aduca, MSC's broker, wrote the computation of the
balance. On 2 March 1989, SJSSFI was ready with the amount corresponding to
the balance, covered by Metrobank Cashier's Check 004223, payable to MSC.
SJSSFI and MSC were supposed to meet in the office of SJSSFI but MSC's
treasurer, Nenita Lee Gruenberg, did not appear. MSC, despite repeated
demands and in utter disregard of its commitments had refused to execute the
Transfer of Rights/Deed of Assignment which is necessary to transfer the
certificate of title. On 6 April 1989, ADC and MSC entered into a Deed of
Absolute Sale whereby the former transferred to the latter the subject property.
By reason of said transfer, the Registry of Deeds of Quezon City issued a new
title in the name of MSC, represented by Nenita Lee Gruenberg and Reynaldo L
Gruenberg, under Transfer Certificate of Title 3571. SJSSFI filed the complaint
for damages against MSC, and Nenita Lee Gruenberg, as a result of the latters
alleged bad faith in refusing to execute a formal Transfer of Rights/Deed of
Assignment. It impleaded ADC and JNM Realty & Development Corp. (JRDC) as
necessary parties, since Transfer Certificate of Title (362909) 2876 was in the
name of ADC, and that JRDC is the transferor of right in favor of MDC. In its
answer, MSC and Nenita Lee Gruenberg interposed as affirmative defense that
the President and Chairman of Motorich did not sign the agreement adverted to;
that Mrs. Gruenberg's signature on the agreement is inadequate to bind MSC as
the other signature, that of Mr. Reynaldo Gruenberg, President and Chairman of
MSC, is required; that SJSSFI knew this from the very beginning as it was
presented a copy of the Transfer of Rights at the time the Agreement was
signed; that SJSSFI itself drafted the Agreement and insisted that Mrs.
Gruenberg accept the P100,000.00 as earnest money; that granting, without
admitting, the enforceability of the agreement, SJSSFI nonetheless failed to pay
in legal tender within the stipulated period (up to 2 March 1989); that it was the
understanding between Mrs. Gruenberg and SJSSFI that the Transfer of
Rights/Deed of Assignment will be signed only upon receipt of cash payment;
thus they agreed that if the payment be in check, they will meet at a bank
designated by SJSSFI where they will en cash the check and sign the Transfer of
Rights/Deed, but that SJSSFI informed Mrs. Gruenberg of the alleged availability
of the check, by phone, only after banking hours. On the basis of the evidence,
and on 18 June 1994, the Regional Trial Court of Makati, Metro Manila, Branch
213

63 rendered judgment, dismissing SJSSFI's complaint, finding that Nenita Lee


Gutenberg was not authorized by the corporation to dispose of the property as
such disposition is governed by the requirements of Section 40, Corporation
Code; and that Nenita Lee Gutenberg did not in any way misrepresent herself to
be authorized by the corporation to sell the property to SJSSFI. The trial court
also dismissed the counterclaim. SJSSFI appealed.
On 18 March 1997, the Court of Appeals modified the decision of the trial
court by ordering Nenita Lee Gutenberg to refund or return to SJSSFI the down
payment of P100, 000.00 which she received from the latter. SJSSFI moved for
reconsideration, which was denied by the appellate court. Hence, this petition.
ISSUE
Whether a corporate treasurer, by herself and without any authorization
from the board of directors, validly sell a parcel of land owned by the corporation.
RULING
No. Nenita Gruenberg is the treasurer of Motorich does not free petitioner
from the responsibility of ascertaining the extent of her authority to represent the
corporation. Petitioner cannot assume that she, by virtue of her position, was
authorized to sell the property of the corporation. Selling is obviously foreign to a
corporate treasurers function, which generally has been described as to receive
and keep the funds of the corporation, and to disburse them in accordance with
the authority given him by the board or the properly authorized officers.
As a general rule, the acts of corporate officers within the scope of their
authority are binding on the corporation. But when these officers exceed their
authority, their actions cannot bind the corporation, unless it has ratified such
acts or is estopped from disclaiming them.

214

PEOPLE AIR CAGRO AND WAREHOUSING COMPANY INC.


Vs.
COURT OF APPEALS
G.R. No.117847
FACTS OF THE CASE
Petitioner is a domestic corporation organized in 1986 to operate a
customs bonded warehouse at the old Manila International Airport (MIA).
To obtain a license from the Bureau of Customs, Antonio Punsalan, Jr., the
corporation president, solicited a proposal from private respondent Stefani Sano
for the preparation of a feasibility study. Sano submitted a letter proposal dated
October 17, 1986 (First Contract) to Punsalan regarding his request for
professional engineering consultancy services which services are offered in the
amount of P350, 000.00. Initially, Cheng Yang, the majority stockholder of
petitioner, objected to say offer as another company can provide for the same
service at a lower price. However, Punsalan preferred Sanos services because
of latter are membership in the task force, which task force was supervising the
transition of the Bureau from the Marcos to the Aquino government. Petitioner,
through Punsalan, thereafter confirmed the contract.
On December 4, 1986, upon Punsalans request, private respondentsent
petitioner another letter-proposal (Second Contract) which offers the same
service already at P400, 000.00 instead of the previous P350,000.00 offer. On
January 10, 1987, Andy Villaceren, vice-president of petitioner, received the
operations manual prepared by Sano and which manual operations were
submitted by petitioner to the Bureau in compliance for its application to operate
abandoned warehouse. Thereafter, in May 1987, the Bureau issued to it a
license to operate. Private respondent also conducted in the third week of
January 1987 in the warehouse of petitioner, a three-day training seminar for the
petitioners employees.
On February 9, 1988, private respondent filed a collection suit against
petitioner. He alleged that he had prepared an operationsmanual for petitioner,
conducted a seminar-workshop for its employees and delivered to it a computer
program but that despite demand, petitioner refused to pay him for his services.
Petitioner, on its part, denied that Sano had prepared such manual operations
and at the same time alleged that the letter-agreement was signed by Punsalan
without authority and as such unenforceable. It alleges that the disputed contract
was not authorized by its board of directors.

ISSUE
Whether the Second Contract signed by Punsalan is enforceable and
binding against petitioner.
RULING
Being a juridical entity, a corporation may act through its board of directors,
which exercises almost all corporate powers, lays down all corporate business
policies and is responsible for the efficiency of management, as provided in
Section 23 of the Corporation Code.
However, it is familiar doctrine that if a corporation knowingly permits one
of its officers, or any other agent, to act within the scope of an apparent authority,
215

it holds him out to the public as possessing the power to do those acts and thus,
the corporation will, as against anyone who has in good faith dealt with it through
such agent, be estopped from denying the agents authority. Thus private
respondent shall not be faulted for believing that Punsalans conformity to the
contract in dispute was also binding on petitioner. In the case at bar, petitioner,
through its president Antonio Punsalan Jr., entered into the First Contract without
first securing board approval. Despite such lack of board approval, petitioner did
not object to or repudiate said contract, thus "clothing" its president with the
power to bind the corporation. The grant of apparent authority to Punsalan is
evident in the testimony of Yong senior vice president, treasurer and major
stockholder of petitioner. Furthermore, private respondent prepared an
operations manual and conducted a seminar for the employees of petitioner in
accordance with their contract. Petitioner accepted the operations manual,
submitted it to the Bureau of Customs and allowed the seminar for its employees.
As a result of its aforementioned actions, petitioner was given by the Bureau of
Customs a license to operate a bonded warehouse. Granting arguendo then that
the Second Contract was outside the usual powers of the president, petitioner's
ratification of said contract and acceptance of benefits have made it binding,
nonetheless. The enforceability of contracts under Article 1403(2) is ratified "by
the acceptance of benefits under them" under Article 1405.

216

RESTAURANTE LAS CONCHAS


Vs.
LLEGO
G.R. No. 119085
FACTS OF THE CASE
Private respondents were employees of petitioner Restaurante Las
Conchas which was allegedly operated by the Restaurant Services Corporation
and by petitioners David Gonzales and Elizabeth Anne Gonzales who are
members of the board of directors and officers of the corporation. While private
respondents were being employed by petitioners, the Restaurant Services
Corporation got involved in a legal battle with the Ayala Land, Inc. over the land
allegedly being occupied by petitioners for their restaurant. Ayala Land, Inc.
obtained a favorable judgment in the case filed against Restaurant Services
Corporation for unlawful detainer and the latter were ordered to vacate the
premises. The case was appealed to the Court of Appeals and ultimately to this
Court which affirmed the decision of the trial court.
Petitioners attempted to look for a suitable place for their restaurant
business at the Ortigas Center but to no avail, thus they shut down their
business. This resulted in the termination of employment of private respondents.
Private respondents filed a complaint with the Labor Arbiter for payment of
separation pay and 13th month pay. This was, however, dismissed by the Labor
Arbiter prompting the private respondents to appeal the case to the respondent
NLRC. The NLRC rendered a decision in favor of private respondents. A motion
for reconsideration was filed by petitioners but this was denied by the respondent
NLRC. Hence this petition for Certiorari.
ISSUE
Whether or not petitioners David Gonzales and Elizabeth Ann Gonzales
were mere officers and members of the board of directors of Petitioner
Corporation cannot be held to answer for the corporations liabilities.
RULING
We are not persuaded.
Assuming that indeed, the Restaurant Services Corporation was the owner
of the Restaurant Las Conchas and the employer of private respondents, this will
not absolve petitioners David Gonzales and Elizabeth Anne Gonzales from their
liability as corporate officers. Although as a rule, the officers and members of a
corporation are not personally liable for acts done in the performance of their
duties, this rule admits of exceptions, one of which is when the employer
corporation is no longer existing and is unable to satisfy the judgment in favor of
the employee, the officers should be held liable for acting on behalf of the
corporation. Here, the corporation does not appear to exist anymore.
In the present case, the employees can no longer claim their separation
benefits and 13th month pay from the corporation because it has already ceased
operation. To require them to do so would render illusory the separation and 13th
month pay awarded to them by the NLRC. Their only recourse is to satisfy their
claim from the officers of the corporation who were, in effect, acting in behalf of
the corporation. It would appear that, originally, Restaurant Las Conchas was a
single proprietorship put up by the parents of Elizabeth Anne Gonzales, who
together with her husband, petitioner David Gonzales, later took over its
217

management. Private respondents claim, and rightly so, that the former were the
real owners of the restaurant. The conclusion is bolstered by the fact that
petitioners never revealed who were the other officers of the Restaurant Services
Corporation, if only to pinpoint responsibility in the closure of the restaurant that
resulted in the dismissal of the private respondents from employment. Petitioners
David Gonzales and Elizabeth Anne Gonzales are, therefore, personally liable for
the payment of the separation and 13th month pay due to their former
employees.
Wherefore, premises considered, the petition is hereby dismissed and the
decision of the respondent National Labor Relations Commission is affirmed in
Toto.

218

CONSOLIDATED FOOD CORPORATION


VS.
NATIONAL LABOR RELATIONS
315 SCRA 129 (1999)
FACTS OF THE CASE
Wilfredo Baron, acting section manager of Petitioner Company for North
Luzon covering Baguio City, La Trinidad and Benguet, was subjected to several
audits after a killer earthquake hit Baguio City on July 16, 1990. The audits
disclosed that he reported a higher quantity of damaged stocks than the total
quantity of damaged stocks confirmed by customers listing, over declared
excess bad orders in the amount of P64, 740.56 and failed to account for his
expense allowance. He was required to temporarily report in Pasig pending result
of the investigation. However, his salaries during that period were withheld from
him. He discontinued reporting for work and filed a complaint for constructive
dismissal for his assignment in Pasig and non-payment of salaries, among
others. Despite the report of the final investigation conducted on respondent
affirming the irregularities committed by him, the Labor Arbiter ruled that he was
dismissed constructively. On appeal, the NLRC upheld petitioners prerogative to
investigate and reassign private respondent, and affirmed the findings of the
Labor Arbiter. Hence, this petition.
ISSUE
Whether a transfer of work assignment pending investigation of
irregularities allegedly committed by an employee is a valid exercise of
management.
RULING
This Court has defined a valid exercise of management prerogative as one
which covers hiring, work assignment, working methods, time, place and manner
of work, tools to be used, processes to be followed, supervision of workers,
working regulations, transfer of employees, work supervision, lay-off of workers
and the discipline, dismissal and recall of workers. Except as provided for or
limited by special laws, employers are free to regulate, according to their own
discretion and judgment, all aspects of employment. Re-assignments made by
management pending investigation of irregularities allegedly committed by an
employee fall within the ambit of management prerogative. The purpose of
reassignments is no different from that of preventive suspension which
management could validly impose as a disciplinary measure for the protection of
the companys property pending investigation of any alleged malfeasance or
misfeasance committed by the employee.

219

a. RURAL BANK OF MILAOR (CAMARINES SUR)


VS.
OCFEMIA
G.R. No. 137686
FACTS OF THE CASE
The spouses Felicisimo Ocfemia and Juanita Arellano Ocfemia were not
able to redeem the mortgaged properties consisting of 7 parcels of land and so
the mortgage was foreclosed and thereafter ownership thereof was transferred to
the [petitioner] bank. Out of the 7 parcels that were foreclosed, 5 of them are in
the possession of the respondents because these 5 parcels of land described in
paragraph 6 of the petition were sold by the [petitioner] bank to the parents of
Marife O. Nio as evidenced by a Deed of Sale executed in January 1988.
Marife O. Nio went to the Register of Deeds of Camarines Sur with the
Deed of Sale in order to have the same registered. The Register of Deeds,
however, informed her that the document of sale cannot be registered without a
board resolution of the petitioner Bank. Marife Nio then went to the bank,
showed to if the Deed of Sale, the tax declaration and receipt of tax payments
and requested the petitioner for a board resolution so that the property can be
transferred to the name of Renato Ocfemia the husband of petitioner Francisca
Ocfemia and the father of the other [respondents] having died already.
The bank, after requiring so many requirements and making so many alibis
to Marife, refused to issue the board resolution. It claims that its bank manager
Fe Tena did not have authority to sell the properties to the Ocfemias therefore
rendering the deed of sale invalid.
ISSUE
Whether the bank manager has authority to act on behalf of the bank.
RULING
Yes.
The bank acknowledged, by its own acts or failure to act, the authority of
Fe S. Tena to enter into binding contracts. After the execution of the Deed of
Sale, respondents occupied the properties in dispute and paid the real estate
taxes due thereon. If the bank management believed that it had title to the
property, it should have taken some measures to prevent the infringement or
invasion of its title thereto and possession thereof.
Likewise, Tena had previously transacted business on behalf of the bank,
and the latter had acknowledged her authority. A bank is liable to innocent third
persons where representation is made in the course of its normal business by an
agent like Manager Tena, even though such agent is abusing her
authority. Clearly, persons dealing with her could not be blamed for believing that
she was authorized to transact business for and on behalf of the bank.
In this light, the bank is estopped from questioning the authority of the
bank manager to enter into the contract of sale. If a corporation knowingly
permits one of its officers or any other agent to act within the scope of an
apparent authority, it holds the agent out to the public as possessing the power to
do those acts; thus, the corporation will, as against anyone who has in good faith
dealt with it through such agent, be estopped from denying the agent's authority.
220

More so, the bank is in default for failing to answer the complaint of the Ocfemias
within the reglamentary period without any justifiable excuse.
b. SOLER
VS.
COURT OF APPEALS
G.R. No. 123892

FACTS OF THE CASE


Petitioner Jazmin Soler is a well-known licensed professional interior
designer. Her friend Rosario Pardo asked her to talk to Nida Lopez, who was
manager of the COMBANK Ermita Branch for they were planning to renovate the
branch offices. During their meeting, petitioner was hesitant to accept the job
because of her many out of town commitments, and also considering that Ms.
Lopez was asking that the designs be submitted by December 1986, which was
such a short notice. Ms. Lopez insisted, however, because she really wanted
petitioner to do the design for renovation. Petitioner acceded to the
request. Petitioner even told Ms. Lopez that her professional fee was P10,
000.00, to which Ms. Lopez acceded.
After a few days, petitioner requested for the blueprint of the building so
that the proper design, plans and specifications could be given to Ms. Lopez in
time for the board meeting in December 1986. So come December 1986, the lay
out and the design were submitted to Ms. Lopez. She even told petitioner that
she liked the designs.
Subsequently, petitioner repeatedly demanded payment for her services
but Ms. Lopez just ignored the demands. To settle the controversy, petitioner
referred the matter to her lawyers, who wrote Ms. Lopez, demanding payment for
her professional fees in the amount of P10, 000.00 which Ms. Lopez
ignored. Hence, the lawyers wrote Ms. Lopez once again demanding the return
of the blueprint copies petitioner submitted which Ms. Lopez refused to return.
Petitioner filed at the RTC Pasig, a complaint against COMBANK and Ms.
Lopez for collection of professional fees and damages.
In its answer, COMBANK stated that there was no contract between
COMBANK and petitioner; that Ms. Lopez merely invited petitioner to participate
in a bid for the renovation of the COMBANK Ermita Branch; that any proposal
was still subject to the approval of the COMBANKs head office.
RTC ruled in favor of Soler. The Court of Appeals reversed the decision.
ISSUE
Whether Nida Lopez, the manager of the bank branch, had authority to
bind the bank in the transaction.
RULING
Yes.
It is familiar doctrine that if a corporation knowingly permits one of its
officers, or any other agent, to act within the scope of an apparent authority, it
221

holds him out to the public as possessing the power to do those acts; and thus,
the corporation will, as against anyone who has in good faith dealt with it through
such agent, be estopped from denying the agents authority.
Petitioner believed that once she submitted the designs she would be paid
her professional fees. Ms. Lopez assured petitioner that she would be paid.
The discussion between petitioner and Ms. Lopez was to the effect that
she had authority to engage the services of petitioner. During their meeting, she
even gave petitioner specifications as to what was to be renovated in the branch
premises and when petitioners requested for the blueprints of the building, Ms.
Lopez supplied the same.
Ms. Lopez even insisted that the designs be rushed in time for
presentation to the bank. With all these discussion and transactions, it was
apparent to petitioner that Ms. Lopez indeed had authority to engage the services
of petitioner.

222

LAO
VS.
COURT OF APPEALS
G.R. No. 60647
FACTS OF THE CASE
On April 6, 1965, The Associated Anglo-American Tobacco Corporation
(Corporation for brevity) entered into a "Contract of Sales Agent" with Andres
Lao. Under the contract, Lao agreed to sell cigarettes manufactured and shipped
by the Corporation to his business address in Tacloban City. Lao would in turn
remit the sales proceeds to the Corporation. For his services, Lao would receive
commission depending on the kind of cigarettes sold, fixed monthly salary, and
operational allowance. As a guarantee to Laos compliance with his contractual
obligations, his brother Jose and his father Tomas executed a deed of
mortgage[1] in favor of the Corporation in the amount of P200,000.00
In compliance with the contract, Lao regularly remitted the proceeds of his
sales to the Corporation, generating, in the process, a great deal of business.
Thus, the Corporation awarded him trophies and plaques in recognition of his
outstanding performance from 1966 to 1968. However, in February 1968 and
until about seven (7) months later, Lao failed to accomplish his monthly sales
report. In a conference in Cebu, Ching Kiat Kam, the President of the
Corporation, reminded Lao of his enormous accounts and the difficulty of
obtaining a tally thereon despite Laos avowal of regular remittances of his
collections.
Sometime in August and September 1969, Esteban Co, the vice-president
and general manager of the Corporation, summoned Lao to Pasay City for an
accounting. It was then and there established that Laos liability amounted to
P525, 053.47. And so, Lao and his brother Lao Y Ka enlisted the services of the
Sycip Gorres and Velayo Accounting Firm (SGV) to check and reconcile the
accounts. Esmmis
Ching Kiat Kam allowed Lao to continue with the sales agency provided
Lao would reduce his accountability to P200, 000.00, the amount secured by the
mortgage. The Corporation thereafter credited in favour of Lao the amount of
P325, 053.47 representing partial payments he had made but without prejudice
to the result of the audit of accounts. However, the SGV personnel Lao had
employed failed to conclude their services because the Corporation did not honor
its commitment to assign two of its accountants to assist them. Neither did the
Corporation allow the SGV men access to its records.
Subsequently, the Corporation discovered that Lao was engaging in the
construction business so much so that it suspected that Lao was diverting the
proceeds of his sales to finance his business. In the demand letter of April 15,
1979,[2] counsel for the Corporation sought payment of the obligations of Lao,
warning him of the intention of the Corporation to foreclose the mortgage.
Attached to said letter was a statement of account indicating that Laos total
obligations duly supported by receipts amounted to P248, 990.82.
Since Lao appeared to encounter difficulties in complying with his
obligations under the contract of agency, the Corporation sent Ngo Kheng to
supervise Laos sales operations in Leyte and Samar. Ngo Kheng discovered
that, contrary to Laos allegation that he still had huge collectibles from his
customers, nothing was due the Corporation from Laos clients. From then on,
Lao no longer received shipments from the Corporation which transferred its
223

vehicles to another compound controlled by Ngo Kheng. Shipments of cigarettes


and the corresponding invoices were also placed in the name of Ngo Kheng.
On May 21, 1970, Andres, Jose and Tomas Lao brought a complaint for
accounting and damages with writ of preliminary injunction[3] against the
Corporation
ISSUE
Whether Ngo Keng should be impleaded in the case as an agent, acting
within his authority granted by the Board of Directors of Associated AngloAmerican Tobacco Corporation.
RULING
A perusal of his affidavit-complaint reveals that at the time he filed the
same on June 24, 1974, petitioner Co was the vice-president of the Corporation.
As a corporate officer, his power to bind the Corporation as its agent must be
sought from statute, charter, by-laws, a delegation of authority to a corporate
officer, or from the acts of the board of directors formally expressed or implied
from a habit or custom of doing business.[46] In this case, no such sources of
petitioners authority from which to deduce whether or not he was acting beyond
the scope of his responsibilities as corporate vice-president are mentioned, much
less proven. It is thus logical to conclude that the board of directors or by laws- of
the corporation vested petitioner Co with certain executive duties[47]one of which
is a case for the Corporation.
That petitioner Co was authorized to institute the estafa case is buttressed by the
fact that the Corporation failed to make an issue out of his authority to file said
case. Upon well-established principles of pleading, lack of authority of an officer
of a corporation to bind it by contract executed by him in its name, is a defense
which should have been specially pleaded by the Corporation.[48] The
Corporations failure to interpose such a defense could only mean that the filing
of the affidavit-complaint by petitioner Co was with the consent and authority of
the Corporation. In the same vein, petitioner Co may not be held personally liable
for acts performed in pursuance of an authority and therefore, holding him
solidarily liable with the Corporation for the damages awarded to respondent Lao
does accord with law and jurisprudence

224

BASAVINGS BANK
VS.
SIA
GR.NO.131214. JULY 27, 2000
FACTS OF THE CASE
The Court of Appeals denied the petition for certiorari filed by BA savings
bank on the ground that the same was not signed by duly authorized
representative of BA, as required by Supreme Court Circular. BA filed a motion
for reconsideration attaching to it a secretarys certificate showing that the BOD
of BA approved a resolution authorizing its lawyers to represent in any
proceeding before any court, tribunal, or agency, and to execute a certificate of
non-forum shopping. The motion for reconsideration was denied on the ground
that the certification for non-forum shopping should be signed by the petitioner
who shall certify under oath.
ISSUE
Whether BAs lawyers are authorized to execute and sign the certificate of
non-forum shopping in its behalf.
RULING
BAs lawyers are authorized to sign and executed a certificate of nonforum shopping by virtue of the resolution approved by the board.
A corporation has no power except those expressly conferred by law and
those that are implied or incidental to its existence. A corporation exercises said
powers through its board of directors, and or its duly authorized officers or
agents. Physical acts like signing of documents can be performed only by natural
persons duly authorized for such purpose by the corporate by-laws, or by specific
acts of the board of directors. All acts within the powers of a corporation may be
performed by agents of its selection, and except so far as limitation or restrictions
as may be imposed by special charter, by-law, or statutory provisions, the same
general principles of law which govern the relation of agency for natural persons
governs the officer of agents of a corporation.

225

SECURITY BANK AND TRUST COMPANY INC.


VS.
CUENCA,
G.R. No. 138544, October 3, 2000
FACTS OF THE CASE
Sta. Ines Melale is a corporation engaged in logging operations. On 10
November 1980, Security Bank and Trust Co. [SBTC] granted Sta. Ines Melale
Corporation [SIMC] a credit line in the amount of P8,000,000.00 to assist the
latter in meeting the additional capitalization requirements of its logging
operations. The Memorandum expressly stated that the P8M Credit Loan Facility
shall be effective until 30 November 1981. To secure the payment of the credit
line, SIMC executed a Chattle Mortgage over some of its machinery and
equipment in favor of SBTC and as additional security, respondent Rodolfo
Cuenca executed an Indemnity Agreement in favor of SBTC whereby he
solidarily bound himself with SIMC.
Sometime in 1985, Cuenca resiged as President and Chairman of the
Board of Directors of SIMC. Subsequently, the shareholdings of Cuenca in SIMC
were sold at a public auction and said shares were bought by Adolfo Angala.
Meanwhile, SIMC repeatedly availed of its credit line and obtained loans to
the aggregate amount of more than P12M. SIMC requested for a complete
restructuring of its indebtedness in which SBTC accommodated the request and
although some loans were incurred after the expiration of the credit line, without
notice to or the prior consent of Cuenca, all loans were consolidated in the 1989
Loan Agreement restructure. SIBC defaulted in the payment of its loan
obligations to SBTC despite demands made upon SIMC and Cuenca. Thus,
SBTC filed a complaint for collection of sum of money.
The CA released Cuenca from liability ruling that the 1989 Loan
Agreement had novated the 1980 credit accommodation granted by the bank to
Sta. Ines. Hence, this recourse to this court.
ISSUE
Whether or not Cuenca should be released from his liability as surety.

RULING
Yes. Novation of a contract is never presumed. It has been held that [i]n
the absence of an express agreement, novation takes place only when the old
and the new obligations are incompatible on every point. Indeed, the following
requisites must be established: (1) there is a previous valid obligation; (2) the
parties concerned agree to a new contract; (3) the old contract is extinguished;
and (4) there is a valid new contract. Clearly, the requisites of novation are
present in this case. The 1989 Loan Agreement extinguished the obligation
obtained under the 1980 credit accommodation.
To begin with, the 1989 Loan Agreement expressly stipulated that its
purpose was to liquidate, not to renew or extend, the outstanding indebtedness.
226

Moreover, respondent did not sign or consent to the 1989 Loan Agreement,
which had allegedly extended the original P8 million credit facility. Hence, his
obligation as a surety should be deemed extinguished.
While respondent held him liable for the credit accommodation or any
modification thereof, and the November 30, 1981 term. It did not give the bank or
Sta. Ines any license to modify the nature and scope of the original credit
accommodation, without informing or getting the consent of respondent who was
solidarily liable. Taking the banks submission to the extreme, respondent (or his
successors) would be liable for loans even amounting to, say, P100 billion
obtained 100 years after the expiration of the credit accommodation, on the
ground that he consented to all alterations and extensions thereof.
It is a common banking practice to require the JSS (joint and solidary
signature) of a major stockholder or corporate officer, as an additional security
for loans granted to corporations. There are at least two reasons for this. First, in
case of default, the creditors recourse, which is normally limited to the corporate
properties under the veil of separate corporate personality, would extend to the
personal assets of the surety. Second, such surety would be compelled to ensure
that the loan would be used for the purpose agreed upon, and that it would be
paid by the corporation.
Following this practice, it was therefore logical and reasonable for the bank
to have required the JSS of respondent, who was the chairman and president of
Sta. Ines in 1980 when the credit accommodation was granted. There was no
reason or logic, however, for the bank or Sta. Ines to assume that he would still
agree to act as surety in the 1989 Loan Agreement, because at that time, he was
no longer an officer or a stockholder of the debtor-corporation. Verily, he was not
in a position then to ensure the payment of the obligation. Neither did he have
any reason to bind himself further to a bigger and more onerous obligation.

227

Atrium Management Corporation


Vs.
Court of Appeals
G.R. No. 109491
FACTS OF THE CASE
On January 3, 1983, Atrium Management Corporation filed with the
Regional Trial Court, Manila an action for collection of the proceeds of four
postdated checks in the total amount of P2 million. Hi-Cement Corporation
through its corporate signatories, petitioner Lourdes M. de Leon, treasurer, and
the late Antonio de las Alas, Chairman, issued checks in favor of E.T. Henry and
Co. Inc., as payee. E.T. Henry and Co., Inc., in turn, endorsed the four checks to
petitioner Atrium Management Corporation for valuable consideration.
Upon presentment for payment, the drawee bank dishonored all four
checks for the common reason payment stopped. Atrium, thus, instituted this
action after its demand for payment of the value of the checks was denied.
After due proceedings, on July 20, 1989, the trial court rendered a decision
ordering Lourdes M. de Leon, her husband Rafael de Leon, E.T. Henry and Co.,
Inc. and Hi-Cement Corporation to pay petitioner Atrium, jointly and severally, the
amount of P2 million corresponding to the value of the four checks, plus interest
and attorneys fees.
On appeal to the Court of Appeals, on March 17, 1993, the Court of
Appeals promulgated its decision modifying the decision of the trial court,
absolving Hi-Cement Corporation from liability and dismissing the complaint as
against it. The appellate court ruled that: Lourdes M. de Leon was not
authorized to issue the subject checks in favor of E.T. Henry, Inc.
ISSUE
Whether Lourdes de Leon and Antonio delas Alas should be held
personally liable for the checks issued as corporate officers and authorized
signatories of the check.

RULING
"Personal liability of a corporate director, trustee or officer along (although
not necessarily) with the corporation may so validly attach, as a rule, only when:
1. He assents (a) to a patently unlawful act of the corporation, or (b) for bad faith
or gross negligence in directing its affairs, or (c) for conflict of interest, resulting in
damages to the corporation, its stockholders or other persons;
2. He consents to the issuance of watered down stocks or who, having
knowledge thereof, does not forthwith file with the corporate secretary his written
objection thereto;
3. He agrees to hold himself personally and solidarily liable with the corporation;
or
4. He is made, by a specific provision of law, to personally answer for his
corporate action.
In the case at bar, Lourdes M. de Leon and Antonio de las Alas as
treasurer and Chairman of Hi-Cement were authorized to issue the
checks. However, Ms. de Leon was negligent when she signed the confirmation
228

letter requested by Mr. Yap of Atrium and Mr. Henry of E.T. Henry for the
rediscounting of the crossed checks issued in favor of E.T. Henry. She was
aware that the checks were strictly endorsed for deposit only to the payees
account and not to be further negotiated. What is more, the confirmation letter
contained a clause that was not true, that is, that the checks issued to E.T.
Henry were in payment of Hydro oil bought by Hi-Cement from E.T. Henry. Her
negligence resulted in damage to the corporation. Hence, Ms. de Leon may be
held personally liable therefor.

229

SAFIC ALCAN & CIE


Vs.
IMPERIAL VEGETABLE OIL CO., INC
G.R. No. 126751
FACTS OF THE CASE
Petitioner Safic Alcan & Cie (hereinafter, Safic) is a French corporation
engaged in the international purchase, sale and trading of coconut oil. It filed
with the RTC of Manila a complaint against respondent Imperial Vegetable Oil
Co., Inc. (hereinafter, IVO) In 1985, Safic entered into an agreement with IVO
whereby the latter shall deliver tones of coconut oil to Safic. Both parties
complied. IVO was represented by its president, Dominador Monteverde. In
1986, Safic again entered into several agreements with IVO but this time it was
agreed that IVO shall deliver the coconut oil 8 months from the agreement or
sometime in 1987. This time, IVO failed to deliver and Safic sued IVO. IVO in its
defense aver that Monteverde was acting beyond his power as president when
he made the 1986 agreement with Safic; that Monteverde is acting beyond his
power because the 1986 contracts were speculative in nature and speculative
contracts are prohibited by the by-laws of IVO. The lower court rendered
judgment dismissing the complaint of Safic without prejudice to any action it
might subsequently institute against Dominador Monteverde. The CA affirmed
the decision of the lower court. Thus this petition for review.
ISSUE
Whether Monteverdes act in entering into the 1986 contracts is ultra vires.
RULING
Article III, Section 3 [g] of the By-Laws of IVO provides, among others, that
Section 3. Powers and Duties of the President. The President shall be elected
by the Board of Directors from their own number.
He shall have the following duties:
xxx

xxx

xxx

[g] Have direct and active management of the business and operation of the
corporation, conducting the same according to the orders, resolutions and
instruction of the Board of Directors and according to his own discretion
whenever and wherever the same is not expressly limited by such orders,
resolutions and instructions.
It can be clearly seen from the foregoing provision of IVOs By-laws that
Monteverde had no blanket authority to bind IVO to any contract. He must act
according to the instructions of the Board of Directors. Even in instances when
he was authorized to act according to his discretion that discretion must not be in
conflict with prior Board orders, resolutions and instructions. The evidence
shows that the IVO Board knew nothing of the 1986 contracts and that it did not
authorize Monteverde to enter into speculative contracts. In fact, Monteverde had
earlier proposed that the company engage in such transactions but the IVO
Board rejected his proposal. Since the 1986 contracts marked a sharp departure
from past IVO transactions, Safic should have obtained from Monteverde the
prior authorization of the IVO Board. It must be pointed out that the Board of
230

Directors, not Monteverde, exercises corporate power. Clearly, Monteverdes act


of entering into speculative contracts with Safic is ultra vires and Safic cannot
therefore enforce those contracts against IVO.

231

NAKPIL
Vs
INTERCONENTAL BROADCAING CORPOTRATION
GR 144167, March 21, 2002
FACTS OF THE CASE
Petitioner Dily Dany Nacpil states that he was Asst. General Manager for
Finance/Administration and comptroller for International Broadcasting
Corporation from 1996 until 1997. When Emiliano Templo replaced the current
president of the corporation in March of 97, he told the Board of Directors that he
will terminate the services of the petitioner as he blames the latter for
mismanagement. Templo allegedly harassed the petitioner until the latter retired.
Despite the termination, the petitioners benefits was put on hold as he has not
yet secured clearance from Presidential Commission on Good Governance and
the Commission on Audit. In addition, Templo did not recognize the employment
of the petitioner as the latter only usurped the position. This led the petitioner to
file a complaint for illegal dismissal and non-payment of benefits with the NLRC.
IBC, in its position paper, contended that the case is outside the
jurisdiction of NLRC as the petitioner is a corporate officer duly elected by the
Board of Directors, therefore qualifies the case as intra-corporate dispute falling
within the jurisdiction of the Securities and Exchange Commission. NLRC denied
the position and the Labor Arbiter rendered a decision stating that the petitioner
has been illegally dismissed. CA reversed the said decision, hence, this petition.
ISSUE
WON the petitioner is a corporate officer
RULING
Yes. The petitioner is a corporate officer; therefore, the issue is within the
jurisdiction of SEC and not of NLRC.
Under Presidential Decree No. 902-A (the Revised Securities Act), the law in
force when the complaint for illegal dismissal was instituted by petitioner in 1997,
the following cases fall under the exclusive of the SEC:
c) Controversies in the election or appointment of directors, trustees, officers, or
managers of such corporations, partnerships or associations
Petitioner argues that he is not a corporate officer of the IBC but an employee
thereof since he had not been elected nor appointed as Comptroller and
Assistant Manager by the IBC's Board of Directors. He points out that he had
actually been appointed as such on January 11, 1995 by the IBC's General
Manager, Ceferino Basilio. In support of his argument, petitioner underscores the
fact that the IBC's By-Laws does not even include the position of comptroller in
its roster of corporate officers. He therefore contends that his dismissal is a
controversy falling within the jurisdiction of the labor courts.
Petitioners argument is untenable. Even assuming that he was in fact appointed
by the General Manager, such appointment was subsequently approved by the
Board of Directors of the IBC. That the position of Comptroller is not expressly
mentioned among the officers of the IBC in the By-Laws is of no moment,
because the IBC's Board of Directors is empowered under Section 25 of the
Corporation Code and under the corporation's By-Laws to appoint such other
officers as it may deem necessary.

232

An "office" has been defined as a creation of the charter of a corporation,


while an "officer" as a person elected by the directors or stockholders. On the
other hand, an "employee" occupies no office and is generally employed not by
action of the directors and stockholders but by the managing officer of the
corporation who also determines the compensation to be paid to such employee.
As petitioner's appointment as comptroller required the approval and formal
action of the IBC's Board of Directors to become valid, it is clear therefore holds
that petitioner is a corporate officer whose dismissal may be the subject of a
controversy cognizable by the SEC under Section 5(c) of P.D. 902-A which
includes controversies involving both election and appointment of corporate
directors, trustees, officers, and managers. Had petitioner been an ordinary
employee, such board action would not have been required.

233

GREAT ASIAN SALES CENTER CORPORATION


VS.
COURT OF APPEALS
381 SCRA 557 (2002)
FACTS OF THE CASE
Great Asian is engaged in the business of buying and selling general
merchandise in particular home appliances. On March 17, 1981, a board
resolution was approved authorizing its Treasurer and General Manager, Arsenio
Lim Piat, Jr., to secure a loan from Bancasia in an amount not to exceed P1.0
Million. The Board resolution also authorized Arsenio to sign all papers,
documents or promissory notes necessary to secure the loan. On February 10,
1982, the board also approved a second resolution authorizing Great Asian to
secure a discounting line with Bancasia in an amount not to exceed P2.0 Million.
The second board resolution also designated Arsenio as the authorized signatory
to sign all instruments, documents and checks necessary to secure the
discounting line.
Great Asian through its Treasurer and General Manager, Arsenio signed
four (4) Deeds of Assignment of Receivables, assigning to Bancasia fifteen (15)
postdated checks. Various customers of Great Asia issued these postdate
checks in payment for appliances and other merchandize. Great Asian assigned
the postdated checks to Bancasia at a discount of less than 24% of the face
value of the checks. Arsenio endorsed all the fifteen checks by signing his name
at the back of checks. The drawee banks dishonored all the fifteen checks on
maturity when deposited for collection by Bancasia with the reasons as either
account closed, payment stopped, account under garnishment, and
insufficiency of funds. The total amount of the fifteen checks dishonored is P1,
042,005.00.
Bancasia, thru its counsel sent a letter to Tan Chong Lin, a surety in favor
of Bancasia, notifying him of the dishonor and demanding payment from him.
Neither Great Asian nor Tan Chong Lim paid Bancasia the dishonored checks.
On June 23, 1982 Bancasia filed a complaint for collection of a sum of
money against Great Asian and Tan Chong Lin. Great Asian raised the alleged
lack of authority of Arsenio to sign the Deeds of Assignment.
The RTC held the two defendants jointly and severally liable to pay the
petitioner Bancasia. On appeal the Court Of Appeals sustained the decision of
the lower court.
ISSUE
Whether the corporations Treasurer and General Manager Arsenio Lim
Piat, Jr., had authority to execute the deeds of assignment and thus bind Great
Asian.
RULING
The Corporation Code of the Philippines vest in the Board of Directors the
exercise of the corporate power of the corporation, save in those instances
234

where the code requires stockholders approval for certain specific Acts. In the
ordinary course of business, a corporation can borrow funds or dispose of assets
of the corporation only on authority of the board of directors. The board of the
directors normally designates one or more corporate officers to sign loan
documents or deed of assignments for the corporation.
Arsenio had all the proper and necessary authority from the board of
directors of the Great Asian to sign the deeds of Assignment and endorsed the
fifteen postdate checks. Arsenio signed the Deeds of Assignment as agent and
authorized signatory of Great Asian under an authority expressly granted by its
board of Directors. The signature of Arsenio to the Deeds of Assignment is
effectively also the signature of the board of directors of Great Asian, binding
upon the corporation. The SC held Arsenio as having been duly authorized by
the board to discount its receivables.

235

DEVELOPMENT BANK OF THE PHILIPPINES, petitioner,


Vs.
SPOUSES FRANCISCO ONG and LETICIA ONG, respondents
G.R. Nos. 144661 and 144797 (2005)
FACTS OF THE CASE
Nature of the Case: Petitions for review on certiorari of the resolutions of the
Court of Appeals.
Spouses Francisco Ong and Leticia Ong offered to buy a property on a
negotiated sale basis. They paid the downpayment of Php 14,000.00. The
balance of Php 122,000 payable upon ejectment of the occupants. In the letter
offer they executed, it is expressly stipulated that the downpayment does not
bind the DBP to the offer until until after the approval by the higher authorities of
the bank. Should the bank receive an offer from a third-party buyer higher by
more than 5% or at more advantageous term accompanied by a deposit of at
least 10% of the offered price, or a higher offer from the former-owner for at least
the updated Total Claim of the Bank accompanied by a minimum deposit of 20%
of the purchase price, the Bank may favorably consider the higher offer and
thereafter refund my/our deposit within three (3) working days after the
determination of the most advantageous offer.
It was NOTED by the branch manager, Lagrito.
Another interested third-party buyer offered to buy the same property at the same
price and term, but better and more advantageous to the Bank considering that
the buyer will assume the responsibility at her expense for the ejectment of
present occupants in the said property. The spouses were given 3 days to
match the offer. Hence, the Spouses Ong matched the said offer and offered to
assume the expenses of the ejectment of squatters/occupants, if any, on the
property.
Later on, the bank offered to sell the property through a public bidding. The
Spouses Ong then filed for a suit for breach of contract and specific performance
with prayer for preliminary injuction and damages against the DBP.
The Spouses Ong averred that they have transacted with the bank clerk who
assured them that the papers are just mere formalities and the subject property is
to be sold to them.
ISSUE
Whether the verbal representations of the bank clerk are sufficient to bind DBP to
perfect the contract of sale between Sps. Ong and DBP?
RULING
The transaction between the respondents and the petitioner was limited to
Palasan, one of the clerks of petitioners branch in Cagayan de Oro City. Lagrito,
the branch manager, had no personal or direct communication with respondents
to express his alleged consent to the sale transaction.

236

Palasan, a mere bank clerk, and not the branch manager himself who assured
respondents that theirs was a closed deal.
As already observed, it is familiar doctrine that if a corporation knowingly permits
one of its officers, or any other agent, to do acts within the scope of an apparent
authority, and thus holds him out to the public as possessing power to do those
acts, the corporation will, as against any one who has in good faith dealt with the
corporation through such agent, be estopped from denying his authority; and
where it is said if the corporation permits this means the same as if the thing is
permitted by the directing power of the corporation.
In this light, the bank is estopped from questioning the authority of the bank
manager to enter into the contract of sale. If a corporation knowingly permits one
of its officers or any other agent to act within the scope of an apparent authority,
it holds the agent out to the public as possessing the power to do those acts;
thus, the corporation will, as against anyone who has in good faith dealt with it
through such agent, be estopped from denying the agents authority.
There is absolutely no approval whatsoever by any responsible bank officer of
the petitioner. True it is that the signature of branch manager Lagrito appears
below the typewritten word NOTED at the bottom of respondents offer to
purchase dated May 25, 1988.14. However, the mere NOTING of such an offer
cannot be taken to mean an approval of the supposed sale..
The representation of Roy Palasan, a mere clerk at petitioners Cagayan de Oro
City branch, that the manager had already approved the sale, even if true, cannot
bind the petitioner bank to a contract of sale with respondents, it being obvious to
us that such a clerk is not among the bank officers upon whom such putative
authority may be reposed by a third party. There is, thus, no legal basis to bind
petitioner into any valid contract of sale with the respondents, given the absolute
absence of any approval or consent by any responsible officer of petitioner bank.
DISPOSITION
WHEREFORE, the instant petition is GRANTED and the assailed decision and
resolution of the Court of Appeals REVERSED and SET ASIDE. The complaint
filed in this case is accordingly DISMISSED.
No pronouncement as to costs.
DOCTRINE
Corporation Law; Actions; It is familiar doctrine that if a corporation
knowingly permits one of its officers, or any other agent, to do acts within
the scope of an apparent authority, and thus holds him out to the public as
possessing power to do those acts, the corporation will, as against any one
who has in good faith dealt with the corporation through such agent, be
estopped from denying his authority; and where it is said: if a corporation
permits this means the same as if the thing is permitted by the directing
power of the corporation. This Court has observed in Ramirez vs. Orientalist
Co., 38 Phil. 634, 654-655, thatIn passing upon the liability of a corporation in
cases of this kind it is always well to keep in mind the situation as it presents
itself to the third party with whom the contract is made. Naturally he can have
little or no information as to what occurs in corporate meetings; and he must
necessarily rely upon the external manifestation of corporate consent. The
237

integrity of commercial transactions can only be maintained by holding the


corporation strictly to the liability fixed upon it by its agents in accordance with
law; and we would be sorry to announce a doctrine which would permit the
property of man in the city of Paris to be whisked out of his hands and carried
into a remote quarter of the earth without recourse against the corporation whose
name and authority had been used in the manner disclosed in this case. As
already observed, it is familiar doctrine that if a corporation knowingly permits
one of its officers, or any other agent, to do acts within the scope of an apparent
authority, and thus holds him out to the public as possessing power to do those
acts, the corporation will, as against any one who has in good faith dealt with the
corporation through such agent, be estopped from denying his authority; and
where it is said if the corporation permits this means the same as if the thing is
permitted by the directing power of the corporation.
Same; Same; The very circumstance that the offer to purchase was merely
NOTED, by the branch manager and not approved, is a clear indication
that there is no perfected contract of sale to speak of.The bank is
estopped from questioning the authority of the bank manager to enter into the
contract of sale. If a corporation knowingly permits one of its officers or any other
agent to act within the scope of an apparent authority, it holds the agent out to
the public as possessing the power to do those acts; thus, the corporation will, as
against anyone who has in good faith dealt with it through such agent, be
estopped from denying the agents authority. Unquestionably, petitioner has
authorized Tena to enter into the Deed of Sale. Accordingly, it has a clear legal
duty to issue the board resolution sought by respondents. Having authorized her
to sell the property, it behooves the bank to confirm the Deed of Sale so that the
buyers may enjoy its full use. There is, however, a striking and very material
difference between the aforecited case and the one at bar. For, unlike in Milaor
where it was the branch manager who approved the sale for and in behalf of the
bank, here, there is absolutely no approval whatsoever by any responsible bank
officer of the petitioner. True it is that the signature of branch manager Lagrito
appears below the typewritten word NOTED at the bottom of respondents offer
to purchase dated May 25, 1988. By no stretch of imagination, however, can the
mere NOTING of such an offer be taken to mean an approval of the supposed
sale. Quite the contrary, the very circumstance that the offer to purchase was
merely NOTED by the branch manager and not approved, is a clear indication
that there is no perfected contract of sale to speak of.
Same; Same; A clerk is not among the bank officers upon whom such
putative authority may be reposed by a third party. There is, thus, no legal
basis to bind petitioner into any valid contract of sale with the respondents,
given the absolute absence of any approval or consent by any responsible
officer of petitioner bank. And because there is no perfected contract of
sale between the parties, respondents action for breach of contract and/or
specific performance is simply without any leg to stand on and must
therefore fall.The representation of Roy Palasan, a mere clerk at petitioners
Cagayan de Oro City branch, that the manager had already approved the sale,
even if true, cannot bind the petitioner bank to a contract of sale with
respondents, it being obvious to us that such a clerk is not among the bank
officers upon whom such putative authority may be reposed by a third party.
There is, thus, no legal basis to bind petitioner into any valid contract of sale with
the respondents, given the absolute absence of any approval or consent by any
responsible officer of petitioner bank. And because there is here no perfected
contract of sale between the parties, respondents action for breach of contract
and/or specific performance is simply without any leg to stand on and must
therefore fall.
238

Same; Same; Since there was never any approval or acceptance by the
higher authorities of petitioner of respondents offer to purchase, the
encashment of the check can not in any way represent partial payment of
any purchase price.We also disagree with the Court of Appeals that the
encashment of the check representing the P14,000.00 deposit in relation to
respondents offer to purchase is an indication or proof of perfection of a contract
of sale. It must be noted that the very documents signed by the respondents as
their offer to purchase unmistakably state that the deposit shall only form part of
the purchase price if the offer to purchase is approved, it being expressly
understood x x x that the same (i.e., the deposit) does not bind DBP to the offer
until my/our receipt of its approval by higher authorities of the bank. It may be so
that the official receipt issued therefor by the petitioner termed such deposit as a
downpayment. But the very written offers of the respondents unequivocably and
invariably speak of such amount as deposit, above deposit, we are depositing
the amount of P14,000.00. Since there never was any approval or acceptance
by the higher authorities of petitioner of respondents offer to purchase, the
encashment of the check can not in any way represent partial payment of any
purchase price.

239

LITONJUA JR
Vs.
ETERNIT CORPORATION
G.R. No. 144805
FACTS OF THE CASE
Eternit Corporation (EC) is a corporation duly organized and registered
under the Philippine laws, and is engaged in the manufacture of roofing materials
and pipe products. 90% of the shares of stocks of EC were owned by
Eteroutremer S.A. Corporation (ESAC), a corporation organized and registered
under the laws of Belgium. In 1986, the management of ESAC grew concerned
about the political situation on the Philippines and wanted to stop its operations in
the country. A member of the ECs Board of Directors, Adams, was instructed to
dispose of the eight parcels of land owned by EC. Adams engaged the services
of realtor Marquez, who coordinated with Litonjua as one of the prospective
buyers who responded to the offer. Then, President Glanville of EC
communicated with Marquez and Litonjua that he is prepared to press for a
satisfactory conclusion to the sale.
However, upon the improvement of the political situation in the Philippines,
Glanville informed Marquez that the decision has been taken at a Board Meeting
not to sell the properties on which EC is situated. Litonjua filed a complaint for
specific performance against EC. In their answer, EC alleged that the Board and
stockholders of EC never approved any resolution to sell subject properties nor
authorized Marquez to sell the same. Further, EC added that the communication
of Glanville on the satisfactory conclusion of the sale was his own personal
making and did not bind the EC. The trial court ruled in favor of EC, stating that
Litonjua could not assume that EC had agreed to sell the property without a clear
authorization from the corporation concerned, through the resolutions of the
Board of Directors and stockholders. CA affirmed trial court decision. G.R. NO.
151413.
ISSUE
Whether or not the CA erred in not holding that Glanville has the
necessary authority to sell the subject properties
RULING
The CA correctly ruled that Glanville does not have the necessary authority
to sell the subject properties.
Section 23 of the Corporation Code provides that unless otherwise provided in
this Code, the corporate powers of all corporations formed under this Code shall
be exercised, all business conducted and all property of such corporations
controlled and held by the board of directors or trustees to be elected from
among the holders of stocks, or where there is not stock, from among the
members of the corporation, who shall hold office for one year and until their
successors are elected and qualified.
A corporation is a juridical person separate and distinct from its members
or stockholders and is not affected by the personal rights, obligations and
transactions of the latter. It may act only through its board of directors, or when
authorized either by its by-laws or by its board resolution, through its officers or
agents in the normal course of business.
240

The property of a corporation is not the property of the stockholders or


members, and as such, may not be sold without express authority from the board
of directors. Absent a valid delegation/authorization, the rule is that the
declarations of an individual director relating to the affairs of the corporation, but
not in the course of, or connected with, the performance of authorized duties of
such director, are not binding on the corporation. Absent a board resolution, the
EC is not bound by the acts of Glanville.

241

EASYCALL COMMUNICATIONS PHILS INC.


VS.
EDWARD KING
478 SCRA 102 (2005)
FACTS OF THE CASE
Easycall Communications Phils., Inc. was a domestic corporation primarily
engaged in the business of message handling and through its general manager,
Roberto B. Malonzo, hired the services of respondent as assistant to the general
manager with the responsibility of ensuring that the expansion plans outside
Metro Manila and Metro Cebu were achieved at the soonest possible time.
Based on his performance, the respondent was promoted then appointed on
August 14 and December 22, 1992 to assistant vice president and vice president
position for nationwide expansion, respectively. He became responsible for the
sales and rentals of pager units in petitioners expansion areas and also in
charge of coordinating with the dealers in these areas.
The management then confronted respondent regarding his sales
performance and provincial sales development visits after Malonzo discovered
that the respondent spent around 40% of the total number of working days for the
period October 1992March 1993 in the field thereby making 78% of his sales
commitment and 70% of his sales target. On April 16, 1993, Rockwell Gohu,
petitioners deputy general manager, talked to respondent to discuss his sales
performance and informed him that Malonzo wanted his resignation. Hence, the
respondent wrote a memorandum to Malonzo to inquire whether Malonzo really
wanted him to resign, to emphasize that his work performance had yet to be
evaluated stating that based on the approved budget for fiscal year ending in
June 1993, he was within the budget and targets set forth by petitioner and
declared that he had no intention of resigning from his position.
On April 19, 1993, respondent received a notice of termination signed by
Malonzo. The notice informed him of the termination of his employment with
petitioner effective April 30, 1993 which prompted him to file filed a complaint for
illegal dismissal with the NLRC. The labor arbiter dismissed the complaint for
lack of merit. On appeal, the NLRC affirmed the decision of the labor arbiter with
the modification that petitioner was ordered to indemnify respondent in the
amount of P10,000 for violating respondents right to due process. Respondent
filed a partial motion for reconsideration to NLRC on its ruling insofar as it
declared that he was validly dismissed for cause which was also denied. He
elevated the matter to the CA granted the petition and ruled that the NLRC erred
in holding that it lacked jurisdiction over the case. The CA granted the petition
and ruled that the NLRC erred in holding that it lacked jurisdiction over the case.
The CA also ruled that the dismissal of respondent was illegal for having been
done without cause and in violation of his right to due process. Petitioner moved
for a reconsideration of the CA decision but the motion was denied in the CAs
November 8, 2000 resolution. Hence, this petition.
ISSUE
Whether CA gravely erred when it failed to consider that being a corporate
officer, the NLRC has no jurisdiction over the subject under PD 902-A.
242

RULING
No. The Supreme Court held that an office is created by the charter of the
corporation and the officer is elected by the directors or stockholders. On the
other hand, an employee occupies no office and generally is employed not by the
action of the directors or stockholders but by the managing officer of the
corporation who also determines the compensation to be paid to such employee.
In this case, respondent was appointed vice president for nationwide expansion
by Malonzo, petitioners general manager, not by the board of directors of
petitioner. It was also Malonzo who determined the compensation package of
respondent. Thus, respondent was an employee, not a corporate officer. The
CA was therefore correct in ruling that jurisdiction over the case was properly
with the NLRC, not the SEC.
Corporate officers in the context of PD 902-A are those officers of a
corporation who are given that character either by the Corporation Code or by
the corporations by-laws. Under Section 25 of the Corporation Code, the
corporate officers are the president, secretary, treasurer and such other officers
as may be provided for in the by-laws.

243

ELCEE FARMS INC.


VS. NLRC
G.R. No.150286
FACTS OF THE CASE
On December 26, 1990, a complaint for illegal dismissal was filed by 144
employees before the NLRC AGAINST Petitioners Elcee Farms and
Saguemuller; and Hilla Corporation, Rey Hilado and Roberto Montao. Of the
144 named complainants, only 28 submitted their affidavits and evidence of
employment.
The Labor Arbiter ordered Hilla Corporation to pay each of the 28
complainants the sum of P2, 235.62 as separation pay but dismissed all claims
against Elcee Farms, Saguemuller, Hilado and Montao, for lack of merit.
Not satisfied with the decision, private respondents and Hilla Corporation
appealed to the NLRC.
The NLRC modified the decision of the Labor Arbiter by holding all
defendants liable for the payment of separation pay and adding the payment
of P5, 000.00 as moral damages to each complainant for all the troubles and
sufferings from the disturbance of their rights to labor. All the parties moved for
reconsideration. NLRC issued a Resolution modifying its decision by: (a)
absolving Hilla Corporation from liability and held only petitioners ElceElcee
Farms and Saguemuller liable for the separation pay, moral and exemplary
damages; and (b) increasing the number of awardees from 28 to 131 based on
the list of remitted SSS contributions as of 1990.
ISSUE
Whether the petitioners must be held liable for the error of the private
respondents counsel.
RULING
The Rules of Court was conceived and promulgated to set forth guidelines
in the dispensation of justice but not to bind and chain the hand that dispenses it,
for otherwise, courts will be mere slaves to or robots of technical rules, shorn of
judicial discretion. That is precisely why courts in rendering real justice have
always been, as they in fact ought to be, conscientiously guided by the norm that
when on the balance, technicalities take a backseat against substantive rights,
and not the other way around
It is worthwhile to add that our courts are not only courts of justice but also
of equity. Equity as the complement of legal jurisdiction seeks to reach and do
complete justice where courts of law, through the inflexibility of their rules and
want of power to adapt their judgments to the special circumstances of cases,
are incompetent so to do. Equity regards the spirit and not the letter, the intent
and not the form, the substance rather than the circumstance, as it is variously
expressed by different courts.
In this case, the erroneous move taken by petitioners counsel in instituting the
petition for annulment of judgment before the Court of Appeals before the First
Division of this Court should not bind petitioners because they appear to have
legitimate grievances.
244

ARATEA
Vs.
SUICO
G.R. No. 170284
FACTS OF THE CASE
Petitioners Aratea and Canonigo are the controlling stockholders of Samar
Mining Development Corporation (SAMDECO), a domestic corporation engaged
in mining operations. On the other hand, private respondent Suico is a
businessman engaged in export and general merchandise. Sometime in 1989,
Suico entered into a Memorandum of Agreement (MOA) with SAMDECO. Armed
with the proper board resolution, Aratea and Canonigo signed the MOA as the
duly authorized representatives of the corporation. Under the MOA, Suico would
extend loans and cash advances to SAMDECO in exchange for the grant of the
exclusive right to market fifty percent (50%) of the total coal extracted by
SAMDECO from its mining sites in San Isidro, Wright, and Western Samar. Suico
was enticed into the aforementioned financing scheme because of the
assurances of profit promised by Aratea and Canonigo.
Pursuant to the same MOA, Suico started releasing loans and cash
advances to SAMDECO, still through Aratea and Suico. SAMDECO started
operations in its mining sites to gather the coal. As agreed in the MOA, fifty
percent (50%) of the coals produced were offered by Suico to different buyers.
However, SAMDECO, again through Aratea and Canonigo, prevented the full
implementation of the marketing arrangement by not accepting the prices offered
by Suicos coal buyers even though such prices were competitive and fair
enough, giving no other explanation for such refusal other than saying that the
price was too low. Because he failed to close any sale of his 50% share of the
coal-produce and gain profits therefrom, Suico could not realize payment of the
loans and advances he extended to SAMDECO.
SAMDECO, on the other hand, successfully disposed of its 50% share of
the coal-produce. Even with said coal sales, however, it absolutely made no
payment of its loan obligations to Suico, despite demands. Petitioners eventually
sold the mining rights and passed on the operations of SAMDECO to Southeast
Pacific Marketing, Inc. (SPMI). They also sold their shares in SAMDECO to
SPMIs President, Arturo E. Dy without notice to, or consent of Suico, in violation
of the MOA.
Hence, Suico filed a complaint for a Sum of Money and Damages against
SAMDECO, Aratea, Canonigo, and Seiko Philippines, Inc. (SEIKO, which was
later substituted by SPMI and Arturo E. Dy). The trial court ruled in his favor to
which defendants filed their appeal. The CA dismissed the appeal and affirmed
the appealed decision of the trial court. Petitioners filed their common motion for
reconsideration but the same was denied. Hence, this recourse by petition for
review on certiorari under Rule 45.
ISSUE
Whether or not Aratea and Canonigo, as SAMDECOs controlling
stockholders and/or representatives, be nonetheless held personally and
solidarily liable with SAMDECO and its successors-in-interest for obligations the
corporation incurred.

245

RULING
We rule in the affirmative.
A corporation is a juridical entity with legal personality separate and distinct
from those acting for and in its behalf and, in general, from the people comprising
it. The general rule is that obligations incurred by the corporation, acting through
its directors, officers and employees, are its sole liabilities. There are times,
however, when solidary liabilities may be incurred but only when exceptional
circumstances warrant such as in the following cases:
1. When directors and trustees or, in appropriate cases, the officers of a
corporation:
(a) Vote for or assent to patently unlawful acts of the corporation;
(b) Act in bad faith or with gross negligence in directing the corporate affairs;
(c) Are guilty of conflict of interest to the prejudice of the corporation, its
stockholders or members,
And other persons;
2. When a director or officer has consented to the issuance of watered stocks or
who, having knowledge thereof, did not forthwith file with the corporate secretary
his written objection thereto;
3. When a director, trustee or officer has contractually agreed or stipulated to
hold himself personally and solidarily liable with the corporation; or
4. When a director, trustee or officer is made, by specific provision of law,
personally liable for his corporate action.
Petitioners Aratea and Canonigo, despite having separate and distinct
personalities from SAMDECO may be held personally liable for the loans and
advances made by Suico to SAMDECO which they represent on account of their
bad faith in carrying out the business of the corporation.
Petitioners Aratea and Canonigo acted in bad faith when they, as officers
of SAMDECO, unreasonably prevented Suico from selling his part of the coalproduce of the mining site, in gross violation of their MOA. This resulted in Suico
not being unable to realize profits from his 50% share of the coal-produce, from
which Suico could obtain part of the payment for the loans and advances he
made in favor of SAMDECO. Moreover, petitioners also acted in bad faith when
they sold, transferred and assigned their proprietary rights over the mining area
in favor of SPMI and Dy, thereby causing SAMDECO to grossly violate its MOA
with Suico. Suico suffered grave injustice because he was prevented from
acquiring the opportunity to obtain payment of his loans and cash advances,
while petitioners Aratea and Canonigo profited from the sale of their
shareholdings in SAMDECO in favor of SPMI and Dy. These facts duly
established Aratea and Canonigos personal liability as officers/stockholders of
SAMDECO and their solidary liability with SAMDECO for its obligations in favor
of Suico for the loans and cash advances received by the corporation.
Wherefore, the instant petition is denied and the assailed CA decision and
resolution are affirmed in toto.

246

PAMPLONA PLANTATION COMPANY


Vs.
RAMON ACOSTA ET.AL.
510 SCRA 249 (2006)
FACTS OF THE CASE
This stems from a case before the Labor Arbiter for underpayment,
overtime pay, premium pay for rest day and holiday, service incentive leave pay,
damages, attorneys fees, and 13th month pay. The complainants claimed that
they were regular rank and file employees of petitioner Pamplona Plantation Co.,
Inc. with different hiring periods, work designations, and salary rates. Petitioner,
however, denied this, alleging that some of the complainants are seasonal
employees, some are contractors, and others were hired under the pakyaw
system, while the rest were hired by the Pamplona Plantation Leisure
Corporation, which has a separate and distinct entity from it. The Labor Arbiter
(LA) held petitioner and Pamplona Plantations manager, Jose Luis Bondoc,
liable for underpayment as complainants were regular employees of petitioner.
They were also held guilty of illegal dismissal with regard to two complainants.
The NLRC reversed the LAs decision, dismissing all the complaints, finding that
the complaint should have been directed against the Pamplona Plantation
Leisure Corporation since complainants individual affidavits contained the
allegations that their tasks pertained to their work in the golf course. The Court
of Appeals (CA) set aside the NLRCs dismissal and reinstated the LAs Decision
with modification.
ISSUE
the Pamplona Plantations manager is personally liable for the money
claims awarded to the workers.
RULING
The rule is that officers of a corporation are not personally liable for their
official acts unless it is shown that they have exceeded their authority. However,
the legal fiction that a corporation has a personality separate and distinct from
stockholders and members may be disregarded if it is used as a means to
perpetuate fraud or an illegal act or as a vehicle for the evasion of an existing
obligation, the circumvention of statutes, or to confuse legitimate issues.
Moreover, assuming Bondoc is a corporate officer, a corporate officer is not
personally liable for the money claims of discharged corporate employees unless
he acted with evident malice and bad faith in terminating their employment.

247

PEOPLE
Vs.
HERMENEGILDO DUMLAO AND EMILIO LAO
G.R. No. 168918
FACTS OF THE CASE
Hermenegildo C. Dumlao, Aber Canlas, Jacobo C. Clave, Roman A. Cruz,
Jr., and Fabian C. Ver were members of the Board of Trustees of the GSIS.
While in the performance of their official functions, they enter into contract
of lease-purchase with Emilio G. Lao, a private person whereby the GSIS
agreed to sell to said Emilio G. Lao, a GSIS acquired property consisting of three
parcels of land with an area of 821 square meters together with a 5-storey
building situated at 1203 A. Mabini St., Ermita, Manila, known as the Government
Counsel Centre for the sum of P2,000,000.00 with a down payment
of P200,000.00 with the balance payable in fifteen years at 12% interest per
annum compounded yearly, with a yearly amortization of P264,278.37 including
principal and interest granting Emilio G. Lao the right to sub-lease the ground
floor for his own account during the period of lease, from which he collected
yearly rentals in excess of the yearly amortization.
The respondent argued that the allegedly approved Board Resolution was
not in fact approved by the GSIS Board of Trustees, contrary to the allegations in
the information. Since the signatures of Fabian Ver, Roman Cruz, Aber Canlas
and Jacobo Clave did not appear in the minutes of the meeting held on 23 April
1982, he said it was safe to conclude that these people did not participate in the
alleged approval of the Lease-Purchase Agreement. This being the case, he
maintained that there was no quorum of the board to approve the supposed
resolution authorizing the sale of the GSIS property. There being no approval by
the majority of the Board of Trustees, there can be no resolution approving the
Lease-Purchase Agreement. The unapproved resolution, he added, proved his
innocence.
The Sandiganbayan ruled that the Board Resolution was not validly
passed by the Board of Trustees of GSIS since it was only signed by 3 members
of the Board. Thus, it never had the force and effect of a valid resolution and did
not in effect approve the Lease and Purchase Agreement subject matter
hereof. Therefore, the prosecution has no cause of action against herein
movant-accused Hermenegildo C. Dumlao.

ISSUE
Whether the signature of the majority of the GSIS Board of Trustees are
necessary on the minutes of the meeting to give force and effect in approving the
Lease-Purchase Agreement by and among the GSIS, OGCC and respondent
Lao.
RULING
No.
A resolution is distinct and different from the minutes of the meeting. A
board resolution is a formal action by a corporate board of directors or other
corporate body authorizing a particular act, transaction, or appointment. It is
ordinarily special and limited in its operation, applying usually to some single
248

specific act or affair of the corporation; or to some specific person, situation or


occasion. On the other hand, minutes are a brief statement not only of what
transpired at a meeting, usually of stockholders/members or directors/trustees,
but also at a meeting of an executive committee. The minutes are usually kept in
a book specially designed for that purpose, but they may also be kept in the form
of memoranda or in any other manner in which they can be identified as minutes
of a meeting.
The non-signing by the majority of the members of the GSIS Board of
Trustees of the said minutes does not necessarily mean that the supposed
resolution was not approved by the board. The signing of the minutes by all the
members of the board is not required. There is no provision in the Corporation
Code of the Philippines that requires that the minutes of the meeting should be
signed by all the members of the board.

249

GOSIACO
VS.
CHING
G.R. No. 173807
FACTS OF THE CASE
On 16 February 2000, petitioner Jaime Gosiaco (petitioner) invested P8,
000,000.00 with ASB Holdings, Inc. (ASB) by way of loan. The money was
loaned to ASB for a period of 48 days with interest at 10.5% which is equivalent
to P112, 000.00. In exchange, ASB through its Business Development Operation
Group manager Ching, issued DBS checks no. 0009980577 and 0009980578
for P8, 000,000.00 and P112, 000.00 respectively. The checks, both signed by
Ching, were drawn against DBS Bank Makati Head Office branch. ASB, through
a letter dated 31 March 2000, acknowledged that it owed petitioner the
abovementioned amounts.
Upon maturity of the ASB checks, petitioner went to the DBS Bank San
Juan Branch to deposit the two (2) checks. However, upon presentment, the
checks were dishonored and payments were refused because of a stop payment
order and for insufficiency of funds. Petitioner informed respondents, through
letters dated 6 and 10 April 2000,[about the dishonor of the checks and
demanded replacement checks or the return of the money placement but to no
avail. Thus, petitioner filed a criminal complaint for violation of B.P. Blg. 22 before
the Metropolitan Trial Court of San Juan against the private respondents.
Ching was arraigned and tried while Casta remained at large. Ching
denied liability and claimed that she was a mere employee of ASB. She asserted
that she did not have knowledge as to how much money ASB had in the banks.
Such responsibility, she claimed belonged to another department.
On 15 December 2000, petitioner moved that ASB and its president, Luke
Roxas, be impleaded as party defendants. Petitioner, then, paid the
corresponding docket fees. However, the MTC denied the motion as the case
had already been submitted for final decision.
On 8 February 2001, the MTC acquitted Ching of criminal liability but it did
not absolve her from civil liability. The MTC ruled that Ching, as a corporate
officer of ASB, was civilly liable since she was a signatory to the checks.
Both petitioner and Ching appealed the ruling to the RTC. Petitioner
appealed to the RTC on the ground that the MTC failed to hold ASB and Roxas
either jointly or severally liable with Ching. On the other hand, Ching moved for a
reconsideration which was subsequently denied. Thereafter, she filed her notice
of appeal on the ground that she should not be held civilly liable for the bouncing
checks because they were contractual obligations of ASB.
On 12 July 2005, the RTC rendered its decision sustaining Ching's appeal.
The RTC affirmed the MTCs ruling which denied the motion to implead ASB and
Roxas for lack of jurisdiction over their persons. The RTC also exonerated Ching
from civil liability and ruled that the subject obligation fell squarely on ASB. Thus,
Ching should not be held civilly liable.
Petitioner filed a petition for review with the Court of Appeals on the
grounds that the RTC erred in absolving Ching from civil liability; in upholding the
250

refusal of the MTC to implead ASB and Roxas; and in refusing to pierce the
corporate veil of ASB and hold Roxas liable.
On 19 July 2006, the Court of Appeals affirmed the decision of the RTC
and stated that the amount petitioner sought to recover was a loan made to ASB
and not to Ching. Roxas testimony further bolstered the fact that the checks
issued by Ching were for and in behalf of ASB. The Court of Appeals ruled that
ASB cannot be impleaded in a B.P. Blg. 22 case since it is not a natural person
and in the case of Roxas, he was not the subject of a preliminary investigation.
Lastly, the Court of Appeals ruled that there was no need to pierce the corporate
veil of ASB since none of the requisites were present.
Hence this petition.
ISSUE
Petitioner raised the following issues: (1) is a corporate officer who signed a
bouncing check civilly liable under B.P. Blg. 22; (2) can a corporation be
impleaded in a B.P. Blg. 22 case; and (3) is there a basis to pierce the corporate
veil of ASB?
RULING
The right to recover due and demandable pecuniary obligations incurred
by juridical persons such as corporations cannot be impaired by procedural rules.
Our rules of procedure governing the litigation of criminal actions for violation of
Batas Pambansa Blg. 22 (B.P. 22) have given the appearance of impairing such
substantive rights, and we take the opportunity herein to assert the necessary
clarifications.
When a corporate officer issues a worthless check in the corporate name
he may be held personally liable for violating a penal statute The statute imposes
criminal penalties on anyone who with intent to defraud another of money or
property, draws or issues a check on any bank with knowledge that he has no
sufficient funds in such bank to meet the check on presentment. Moreover, the
personal liability of the corporate officer is predicated on the principle that he
cannot shield himself from liability from his own acts on the ground that it was a
corporate act and not his personal act.
The general rule is that a corporate officer who issues a bouncing
corporate check can only be held civilly liable when he is convicted. In the recent
case of Bautista v. Auto Plus Traders Inc., the Court ruled decisively that the civil
liability of a corporate officer in a B.P. Blg. 22 case is extinguished with the
criminal liability. We are not inclined through this case to revisit so recent a
precedent, and the rule of stare decisis precludes us to discharge Ching of any
civil liability arising from the B.P. Blg. 22 case against her, on account of her
acquittal in the criminal charge.
There are two prevailing concerns should civil recovery against the
corporation be pursued even as the B.P. Blg. 22 case against the signatory
remains extant. First, the possibility that the plaintiff might be awarded the
amount of the check in both the B.P. Bldg. 22 case and in the civil action against
the corporation. For obvious reasons, that should not be permitted. Considering
that petitioner herein has no chance to recover the amount of the check through
251

the B.P. Blg. 22 case, we need not contend with that possibility through this case.
Nonetheless, as a matter of prudence, it is best we refer the matter to the
Committee on Rules for the formulation of proper guidelines to prevent that
possibility.
The other concern is over the payment of filing fees in both the B.P. Blg.
22 case and the civil action against the corporation. Generally, we see no evil or
cause for distress if the plaintiff were made to pay filing fees based on the
amount of the check in both the B.P. Blg. 22 case and the civil action. After all,
the plaintiff therein made the deliberate option to file two separate cases, even if
the recovery of the amounts of the check against the corporation could evidently
be pursued through the civil action alone.
Nonetheless, in petitioners particular case, considering the previous legal
confusion on whether he is authorized to file the civil case against ASB, he
should, as a matter of equity, be exempted from paying the filing fees based on
the amount of the checks should he pursue the civil action against ASB. In a
similar vein and for a similar reason, we likewise find that petitioner should not be
barred by prescription should he file the civil action as the period should not run
from the date the checks were issued but from the date this decision attains
finality. The courts should not be bound strictly by the statute of limitations or the
doctrine of laches when to do so, manifest wrong or injustice would results.
Wherefore, the petition is denied, without prejudice to the right of
petitioner Jaime U. Gosiaco to pursue an independent civil action against ASB
Holdings Inc. for the amount of the subject checks, in accordance with the terms
of this decision. No pronouncements as to costs.
Let a copy of this Decision be referred to the Committee on Revision of the
Rules for the formulation of the formal rules of procedure to govern the civil
action for the recovery of the amount covered by the check against the juridical
person which issued it.

252

PROFESSIONAL SERVICES INC.


VS.
COURT OF APPEALS
G.R. No. 126297 February 11, 2008
FACTS
Natividad Agana was admitted at the Medical City General Hospital
(Medical City) because of difficulty of bowel movement and bloody anal
discharge. Dr. Ampil diagnosed her to be suffering from cancer of the sigmoid.
Thus, Dr. Ampil performed an anterior resection surgery upon her. During the
surgery, he found that the malignancy had spread to her left ovary, necessitating
the removal thus; Dr. Ampil obtained the consent of Atty. Enrique Agana,
Natividads husband, to permit Dr. Juan Fuentes to perform hysterectomy upon
Natividad.
After a couple of days, Natividad complained of excruciating pain in her
anal region. Dr. Ampil and Dr. Fuentes told her that the pain was the natural
consequence of the surgical operation. Natividad, accompanied by her husband,
went to the United States to seek further treatment. After four (4) months of
consultations and laboratory examinations, Natividad was told that she was free
of cancer. Natividad flew back to the Philippines, still suffering from pains. Two
(2) weeks thereafter, her daughter found a piece of gauze protruding from her
vagina. Dr. Ampil managed to extract by hand a piece of gauze then assured
Natividad that the pains would soon vanish but it intensified which prompting
Natividad to seek treatment at the Polymedic General Hospital. Dr. Gutierrez
found that the gauze had badly infected her vaginal vault. Another surgical
operation was needed to remedy the situation.
On November 12, 1984, Natividad and her husband filed with the RTC a
complaint for damages against PSI (owner of Medical City), Dr. Ampil and Dr.
Fuentes. On February 16, 1986, pending the outcome of the above case,
Natividad died.
The trial court rendered judgment in favor of spouses Agana finding PSI,
Dr. Ampil and Dr. Fuentes jointly and severally liable. On appeal, the Court of
Appeals, in its Decision dated September 6, 1996, affirmed the assailed
judgment with modification in the sense that the complaint against Dr. Fuentes
was dismissed.

ISSUE
Whether or not there exist an employer-employee relationship as a basis for the
application of the Doctrine of Corporate Negligene.
RULING
The First Division ruled that an employer-employee relationship in effect
exists between the Medical City and Dr. Ampil. Consequently, both are jointly
and severally liable to the Aganas. This ruling proceeds from the following
ratiocination in Ramos:
253

In the first place, hospitals exercise significant control in the hiring and
firing of consultants and in the conduct of their work within the hospital
premises. Doctors who apply for consultant slots, visiting or attending, are
required to submit proof of completion of residency, their educational
qualifications; generally, evidence of accreditation by the appropriate board
(diplomate), evidence of fellowship in most cases, and references. These
requirements are carefully scrutinized by members of the hospital administration
or by a review committee set up by the hospital who either accept or reject the
application. This is particularly true with respondent hospital.
After a physician is accepted, either as a visiting or attending consultant,
he is normally required to attend clinico-pathological conferences, conduct
bedside rounds for clerks, interns and residents, moderate grand rounds and
patient audits and perform other tasks and responsibilities, for the privilege of
being able to maintain a clinic in the hospital, and/or for the privilege of admitting
patients into the hospital. In addition to these, the physicians performance as a
specialist is generally evaluated by a peer review committee on the basis of
mortality and morbidity statistics, and feedback from patients, nurses, interns and
residents. A consultant remiss in his duties, or a consultant who regularly falls
short of the minimum standards acceptable to the hospital or its peer review
committee, is normally politely terminated.
In other words, private hospitals hire, fire and exercise real control over
their attending and visiting consultant staff. While consultants are not,
technically employees, a point which respondent hospital asserts in denying all
responsibility for the patients condition, the control exercised, the hiring, and the
right to terminate consultants all fulfill the important hallmarks of an employeremployee relationship, with the exception of the payment of wages. In assessing
whether such a relationship in fact exists, the control test is determining.
Accordingly, on the basis of the foregoing, we rule that for the purpose of
allocating responsibility in medical negligence cases, an employer-employee
relationship in effect exists between hospitals and their attending and visiting
physicians. This being the case, the question now arises as to whether or not
respondent hospital is solidarily liable with respondent doctors for petitioners
condition.
The basis for holding an employer solidarily responsible for the negligence
of its employee is found in Article 2180 of the Civil Code which considers a
person accountable not only for his own acts but also
for those of others based on the formers responsibility under a relationship of
partia ptetas.
Clearly, in Ramos, the Court considered the peculiar relationship between
a hospital and its consultants on the bases of certain factors. One such factor is
the control test wherein the hospital exercises control in the hiring and firing of
consultants, like Dr. Ampil, and in the conduct of their work.
Actually, contrary to PSIs contention, the Court did not reverse its ruling in
Ramos. What it clarified was that the De Los Santos Medical Clinic did not
exercise control over its consultant, hence, there is no employer-employee
254

relationship between them. Thus, despite the granting of the said hospitals
motion for reconsideration, the doctrine in Ramos stays, i.e., for the purpose of
allocating responsibility in medical negligence cases, an employer-employee
relationship exists between hospitals and their consultants.
In the instant cases, PSI merely offered a general denial of responsibility,
maintaining that consultants, like Dr. Ampil, are independent contractors, not
employees of the hospital. Even assuming that Dr. Ampil is not an employee of
Medical City, but an independent contractor, still the said hospital is liable to the
Aganas.

255

LANUZA vs. CA
GR No. 131394 March 28, 2005
FACTS OF THE CASE
The Philippine Merchant Marine School (PMMI) was incorporated in 1952 with
700 founders, Shares and 76 common shares as its initial stock subscription
reflected in the articles of incorporation.it was only in 1978 when the companys
stock and transfer book was registered, recording 33 common shares as the only
issued and outstanding shares of PMMI. In a dispute over the basis of a quorum
in a stockholders meeting, private respondents contend that the same should be
based on the initial subscribed capital stock as reflected in the 1052 articles of
incorporation, and not on the number of issued and outstanding shares as recorded in
1978 in the companys stock and transfer book. Petitioners contend otherwise.
Both the SEC en banc and the Court of Appeals ruled in favor of private
respondents. Hence, this petition seeking to nullify the assailed decision.
ISSUE
What should be the basis in determining the quorum in the stockholders
meeting?
RULING
The initial subscribed capital stock as reflected in the articles of incorporation
should be made the basis in the determination of a quorum. The article of
incorporation defines the charter of the corporation and its contractual relations with the
state and the stockholders. The contents thereof are binding not only on the corporation
but also on its shareholders. In the instant case, the articles of incorporation
indicate that the company had 776 issued and outstanding shares. On the other
hand, the stock and transfer book is not in any sense a public record and only
constitutes prima facie evidence. Hence, it may be impeached by other competent
evidence. Therefore, the same cannot be used as the sole basis for determining the quorum
as it does not reflect the totality of shares which have been subscribed, more so when
the articles of incorporation show a significantly larger amount of shares issued
and outstanding.

256

DIGITAL MIOCROWAVE CORP.


Vs.
COURT OF APPEALS
G.R. No. 128550
FACTS OF THE CASE
On December 14, 1994, private respondent Asian High Technology Corp.
filed a complaint against petitioner Digital Microwave Corp. for a sum of money
and damages before the Regional Trial Court of Pasig city. Petitioner moved for
the dismissal of the complaint. The trial court denied the motion, as well as
petitioners subsequent motion for reconsideration.
Petitioner then initiated a special civil action for certiorari before the Court
of Appeals, alleging grave abuse of discretion on the part of the trial court.
However, the Court of Appeals dismissed the petition for failure to comply with
Revised Circular No. 28-91, as amended by Administrative Circular No. 04-94.
Said circular requires the petition filed before the Court of Appeals to be
accompanied by a sworn certification against forum shopping, signed by
petitioner himself. Petitioners certification was signed by counsel; the petition
was, thus, dismissed. Petitioner moved for a reconsideration of the dismissal and
submitted a sworn certification against forum shopping duly signed by one of its
senior officers. The motion was, however, denied.
ISSUE
Whether a counsel for the corporation in a case may sign the certification
against non-forum shopping.
RULING
No. The reason the certification against forum shopping is required to be
accomplished by petitioner himself is because only the petitioner himself has
actual knowledge of whether or not he has initiated similar actions or
proceedings in different courts or agencies. Even his counsel may be unaware of
such fact. For sure, his counsel is aware of the action for which he has been
retained. But what of other possible actions?
We disagree with petitioner that a corporation cannot possibly hope to
comply with the requirement laid down by Revised Circular No. 28-91 because it
is a juridical entity and not a natural person. If this were so, then it would have
been impossible for a corporation to do anything at all. Needless to say, this is
the reason why corporations have directors and officers, to represent it in its
transactions with others. The same is true for the certification against forum
shopping. It could easily have been made by a duly authorized director or officer
of the corporation. That petitioner did not in the first instance comply with the
requirement of revised Circular No. 28-91 by having the certification against
forum shopping signed by one of its officers, as it did after its petition before the
Court of Appeals had been dismissed, is beyond our comprehension.

257

BA SAVINGS BANK
Vs.
ROGER T. SIA
G.R. No. 131214
FACTS OF THE CASE
In 1997 the Court of Appeals issued a Resolution denying due course to a
Petition for Certiorari filed by BA Savings Bank, on the ground that the
Certification on anti-forum shopping incorporated in the petition was signed not
by the duly authorized representative of the petitioner, as required under
Supreme Court Circular No. 28-91, but by its counsel, in contravention of said
circular.
A Motion for Reconsideration was subsequently filed by the petitioner, attached
to which was a BA Savings Bank Corporate Secretarys Certificate dated August
14, 1997. The Certificate showed that the petitioners Board of Directors
approved a Resolution on May 21, 1996, authorizing the petitioners lawyers to
represent it in any action or proceeding before any court, tribunal or agency; and
to sign, execute and deliver the Certificate of Non-forum Shopping, among
others.
ISSUE
Whether a corporation is allowed to authorize its counsel to execute a certificate
of non-forum shopping for and on its behalf

RULING
Yes. A corporation, such as the petitioner, has no powers except those expressly
conferred on it by the Corporation Code and those that are implied by or are
incidental to its existence. In turn, a corporation exercises said powers through
its board of directors and/or its duly authorized officers and agents. Physical
acts, like the signing of documents, can be performed only by natural persons
duly authorized for the purpose by corporate bylaws or by a specific act of the
board of directors. All acts within the powers of a corporation may be performed
by agents of its selection; and, except so far as limitations or restrictions which
may be imposed by special charter, by-law, or statutory provisions, the same
general principles of law which govern the relation of agency for a natural person
govern the officer or agent of a corporation, of whatever status or rank, in respect
to his power to act for the corporation; and agents once appointed, or members
acting in their stead, are subject to the same rules, liabilities and incapacities as
are agents of individuals and private persons. In the present case, the
corporations board of directors issued a Resolution specifically authorizing its
lawyers to act as their agents in any action or proceeding before the Supreme
Court, the Court of Appeals, or any other tribunal or agency and to sign, execute
and deliver in connection therewith the necessary pleadings, motions,
verification, affidavit of merit, certificate of non-forum shopping and other
instruments necessary for such action and proceeding. The Resolution was
sufficient to vest such persons with the authority to bind the corporation and was
specific enough as to the acts they were empowered to do. Thus certificate of
non-forum shopping may be signed, for and on behalf of a corporation, by a
specifically authorized lawyer who has personal knowledge of the facts required
to be disclosed in such document.
258

SHIPSIDE INC.
Vs.
COURT OF APPEALS
GR 143377 February 20, 2001
FACTS OF THE CASE
In 1958, certificates of title were issued for 4 lots in San Fernando, La
Union in favor of Rafael Galvez. By 1960, lots 1 and 4 were sold to Filipina
Mamaril which she then sold to Lepanto Consolidated Mining that same year. A
transfer certificated was then issued in favor of Lepanto for the said lots.
Unknown to Lepanto, in 1963, the land registration case for the subject lots that
granted Galvez the land title was declared null and void and was cancelled. By
October of 63, Lepanto sold the lots to petitioner Shipside Inc to which the latter
exercised proprietary rights. Galvez appealed the cancellation of the title that was
granted to him but to no avail, both in RTC and Court of Appeals.
After 20 years, in 99, the Office of the Solicitor General received a letter
notifying the Government that the order of the cancellation of the title for the
subject lots was not executed. That same year, SolGen filed a complaint for the
revival of judgment. Shipside, in its defense, filed a motion to dismiss based on
the main argument that the action for revival had already prescribed.RTC denied
the petitioners motion to which the Shipside brought under certiorari with the
Court of Appeals. CA denied the motion as a complaint filed in behalf of a
corporation can be made only if authorized by its Board of Directors, and in the
absence thereof, the petition cannot prosper. According to the Solicitor General,
Lorenzo Balbin, who signed for and in behalf of petitioner in the verification and
certification of non-forum shopping portion of the petition, failed to show proof of
his authorization to institute the petition for certiorari and prohibition with the
Court of Appeals, thus the latter court acted correctly in dismissing the same
Hence, this petition.
ISSUE
WON an authorization from petitioner's Board of Directors is still required
in order for its resident manager to institute or commence a legal action for and in
behalf of the corporation
RULING
The instant case is a special circumstance to which the Court may lax its
technical requirements.
It is undisputed that on October 21, 1999, the time petitioner's Resident
Manager Balbin filed the petition, there was no proof attached thereto that Balbin
was authorized to sign the verification and non-forum shopping certification
therein, as a consequence of which the petition was dismissed by the Court of
Appeals. However, subsequent to such dismissal, petitioner filed a motion for
reconsideration, attaching to said motion a certificate issued by its "board
secretary stating that on October 11, 1999, or ten days prior to the filing of the
petition, Balbin had been authorized by petitioner's board of directors to file said
petition.
The Court has consistently held that the requirement regarding verification
of a pleading is formal, not. Such requirement is simply a condition affecting the
259

form of the pleading, non-compliance with which does not necessarily render the
pleading fatally defective. Verification is simply intended to secure an assurance
that the allegations in the pleading are true and correct and not the product of the
imagination or a matter of speculation, and that the pleading is filed in good faith.
The court may order the correction of the pleading if verification is lacking or act
on the pleading although it is not verified, if the attending circumstances are such
that strict compliance with the rules may be dispensed with in order that the ends
of justice may thereby be served.
On the other hand, the lack of certification, against forum shopping is
generally not curable by the submission thereof after the filing of the petition.
Section 5, Rule 45 of the 1997 Rules of civil Procedure provides that the failure
of the petitioner to submit the required documents that should accompany the
petition, including the certification against forum shopping, shall be sufficient
ground for the dismissal thereof. The same rule applies to certifications against
forum shopping signed by a person on behalf of a corporation which are
unaccompanied by proof that said signatory is authorized to file a petition on
behalf of the corporation.
In certain exceptional circumstances, however, the Court has allowed the
belated filing of the certification. In Loyola v. Court of Appeals, et. al. (245 SCRA
477 [1995]), the Court considered the filing of the certification one day after the
filing of an election protest as substantial compliance with the requirement. In
Roadway Express, Inc. v. Court of Appeals, et. al. (264 SCRA 696 [1996]), the
Court allowed the filing of the certification 14 days before the dismissal of the
petition. In "Uy v. LandBank, supra, the Court had dismissed Uy's petition for lack
of verification and certification against non-forum shopping. However, it
subsequently reinstated the petition after Uy submitted a motion to admit
certification and non-forum shopping certification. In all these cases, there were
special circumstances or compelling "reasons that justified the relaxation of the
rule requiring verification and certification on non-forum shopping.
In the instant case, the merits of petitioner' case should be considered
special circumstances or compelling reasons that justify tempering the
requirement in regard to the certificate of non-forum shopping. Moreover, in
Loyola, Roadway, and Uy, the Court excused non-compliance with the
requirement as to the certificate of non-forum shopping. With more reason should
we allow the instant petition since petitioner herein did submit a certification on
non-forum shopping, failing only to show proof that the signatory was authorized
to do so. That petitioner subsequently submitted a secretary's certificate attesting
that Balbin was authorized to file an action on behalf of petitioner likewise,
mitigates this oversight.
It must also be kept in mind that while the requirement of the certificate of
non-forum shopping is mandatory, nonetheless the requirements must not be
interpreted too literally and thus defeat the objective of preventing the
undesirable practice of forum-shopping (Bernardo v. NLRC, .255 SCRA 108
[1996]). Lastly, technical rules of procedure should be used to promote, not
frustrate justice. While the swift unclogging of court dockets is a laudable
objective, the granting of substantial justice is an even more urgent ideal.

260

SAN PABLO MANUFACTURING CORPORATION


VS
COMMISSIONER OF INTERNAL REVENUE
GR. NO.14779, JUNE 22, 2006
FACTS OF THE CASE
This is petition for review under Rule 45 of the Rules of Court brought by
petitioner San Pablo Manufacturing assailing the resolution of the Court of
Appeals.
SMPC is Domestic Corporation engaged in the business of milling,
Manufacturing and Exporting of coconut oil and other allied product. It was
assessed and ordered by the CIR to pay deficiency millers tax and
manufactures sales tax for the taxable year 1997. SMPC opposed the
assessments but the Commissioner denied its protest. SMPC appealed the
denial to the CTA. The CTA cancelled SMPC liability for deficiency on
Manufacturers sales tax, but upheld Commissioners assessment of deficiency
millers tax.
SMPC elevated the case to the Court of Appeals in so far as the CTA
upheld its deficiency millers tax. The appellate court dismissed the petition on
the principal ground that the verification attached to the petition was signed
merely by SMPCs Chief Financial officer without the corporate secretarys
certificate, board resolution or power of attorney authorizing such officer to sign
the verification and certification against forum shopping.
SMPC admitted that no power of attorney, secretarys certificate or board
resolution to prove the affiants authority to sign the verification was attached to
the petition. It merely relied on the alleged inherent power of its Chief Financial
officer to represent SMPC in all matters regarding the finances of the corporation,
including filling of suits for recovery of erroneously paid taxes.
ISSUE
Whether or not the Chief Finance Officer who signs the verification and
certification of non-forum shopping without the proper authority from the board
has the capacity to institute the suit in behalf of the corporation.
RULING
Under Rule 43, Sec. 5 of the Rules of Court, appeals from the CTA and
quasi-judicial bodies to the Court of Appeals should be verified. A pleading
required to be verified which lack proper verification shall be treated as unsigned
pleading. A petition for review under Rule 43 requires a sworn certification
against forum shopping. Failure to comply with the requirement is a sufficient
ground for the dismissal of the petition.
A corporation may exercise those powers expressly conferred upon it by
the Corporation Code and those that are implied or are incidental to its existence
though its board of directors and/or duly authorized officers and agents. In the
absence of authority from the board of directors no person, not even the officers
of the corporation, can bind the Corporation.
261

PHILIPPINE RABBIT BUS LINES, INC.,


vs.
ALADDIN TRANSIT CORP., ANACLETO VILLARICO and ESTEBAN
ZIPAGAN, respondents
GR, NO. 166279 (June 30, 2006)
FACTS OF THE CASE
Nature of the Case: Petition for Review on Certiorari on the Decision of the
Court of Appeals reversing and setting aside the decision of the Branch 221 of
the RTC Quezon City.
Petitioner Philippine Rabbit and Respondent Aladdin Transport figured in a traffic
accident involving their buses. A complaint for damages was filed by Philippine
Rabbit against Aladdin. The lower court ruled in favor of Petitioner. Aladdin
transport appealed citing among others a defective certification of non-forum
shopping in the Complaint for damages filed by Petitioner. The certification was
only signed by the counsel of Philippine Rabbit. The petitioner contends that
since it is represented by its counsel, the counsel serves as an agent of the
corporation, the verification and certification signed by him binds the corporation.
The Court of Appeals reversed the decision citing the defective certification
against forum shopping and that the documentary evidence contradicted the
complaint and testimonies of the witnesses. Philippine Rabbit filed a Motion for
Reconsideration which was denied. It then filed this Petition for Review on
Certiorari but without verification and certificate of non-forum shopping. Later on
Petitioners counsel filed a motion to admit an attached verification and certificate
of non-forum shopping signed by Ramon Nisce, Chairman of the Board of
Directors, Treasurer of Philippine Rabbit and its Chief Legal Counsel. The
certification states that he is the petitioner in the pleading.
ISSUE
Whether Mr. Nisce (Chairman of the Board of Directors, Treasurer of Philippine
Rabbit and its Chief Legal Counsel) is authorized to sign the petition for nonforum shopping?
RULING
The Court denied the petition and affirmed the decision of the Court of Appeals
favoring Aladdin Transport.
The complaint filed in the lower court as well as the Petition for Certiorari clearly
states that it was filed by Philippine Rabbit. Contrary to the claim of Mr. Nisce,
he is clearly not the petitioner. Philippine Rabbit is, however, a juridical entity,
hence, it may only exercise its right to file a suit by specific act of its board of
directors or any duly authorized officer or agent. Physical acts, like the signing of
documents, can be performed only by natural persons duly authorized for the
purpose by its by-laws of by a specific act of its board of directors. All acts within
the powers of a corporation may be performed by agents of its selection; and
except so far as limitations or restrictions which may imposed by special charter,
by-law, or statutory provisions, the same general principles of law which govern
262

the relation of agency for a natural person govern the officer or agent of a
corporation, of whatever status or rank, in respect to act for the corporation; and
agents once appointed, or members acting in their stead, are subject to the same
rules, liabilities, and incapacities as are agents of individuals and private persons.
The requirement that the petitioner sign the certificate is mandatory and the
Rules of Court makes no distinction between natural and juridical persons. Mr.
Nisce did not personally attest to the facts in the petition and did not adduce
proof that he is indeed the corporate officer authorized by the corporation.

263

CAGAYAN VALLEY DRUG CORPORATION


VS.
COMMISSIONER OF INTERNAL REVENUE
G.R.NO. 151413
FACTS OF THE CASE
Cagayan Valley Drug Corporation (CVDC) is a duly organized corporation
under Philippine laws, and a duly licensed retailer of medicine and other
pharmaceutical products. It operates two drugstores, under the name and style of
Mercury Drug. In 1998, CVDC filed a petition for review against BIR before G.R.
NO. 151413
Court of Tax Appeals due to BIRs inaction on petitioners claim for
refund/tax credit pursuant to RA 7432 on the 20% sales discount it gave to senior
citizens in 1995. CTA denied the petition. Hence CVDC elevated the case to the
Court of Appeals.
The Court of Appeals dismissed the petition. According to the court, the
person who signed the verification and certification of absence of forum
shopping, a certain Jacinto J. Concepcion, president of CVDC, failed to adduce
proof that he was duly authorized by the board of directors to do so.
ISSUE
Whether CVDCs president can sign the subject verification and
certification sans the approval of its Board of Directors.
RULING
With respect to a juridical person, Sec. 4, Rule 7 on verification and Sec. 5,
Rule 7 on certification against forum shopping are silent as to who the authorized
signatory should be. Said rules do not indicate if the submission of a board
resolution authorizing the officer or representative is necessary.
However, Sec. 23, in relation to Sec. 25 of the Corporation Code, clearly
enunciates that all corporate powers are exercised, all business conducted, and
all properties controlled by the board of directors. A corporation has a separate
and distinct personality from its directors and officers and can only exercise its
corporate powers through the board of directors. Thus, it is clear that an
individual corporate officer cannot solely exercise any corporate power pertaining
to the corporation without authority from the board of directors. But in previous
cases, the Court has recognized the authority of some corporate officers to sign
the verification and certification against forum shopping. The Court has held the
following officials or employees of the company can sign the verification and
certification without need of a board resolution: the Chairperson of the Board of
Directors; the President of a corporation; the General Manager or Acting General
Manager; Personnel Officer; and an Employment Specialist in a labor case. The
determination of the sufficiency of authority of corporate officers or
representatives of the corporation to sign the verification or certificate against
forum shopping, being in a position to verify the truthfulness and correctness of
the allegations in the petition. The Court however believes that appending the
board resolution to the complaint or petition is the better procedure to obviate any
question on the authority of the signatory to the verification and certification.

264

In the case at bar, CVDC has substantially complied with the Rules on Civil
Procedure. The President of CVDC is in a position to verify the truthfulness and
correctness of the allegations in the petition. Also, the requisite board resolution
has been submitted albeit belatedly by CVDC.

265

MARANAW HOTEL & RESORT CORPORATION


VS.
COURT OF APPEALS
576 SCRA 463 (2009)
FACTS OF THE CASE
This case emanated from a complaint for regularization, subsequently
converted into one for illegal dismissal, filed before Labor Arbiter Madjayran H.
Ajan by private respondent Sheryl Oabel. It appears that Oabel was initially hired
by petitioner as an extra beverage attendant on April 24, 1995 which lasted until
February 7, 1997. Respondent worked in Century Park Hotel, an establishment
owned by the petitioner. However, on September 16, 1996, petitioner contracted
with Manila Resource Development Corporation. Subsequently, Oabel was
transferred to MANRED, with the latter deporting itself as her employer.
MANRED has intervened at all stages of these proceedings and has consistently
claimed to be the employer of private respondent Oabel.
On July 20, 1998, private respondent filed before the Labor Arbiter a
petition for regularization of employment against the petitioner. On August 1,
1998, however, she was dismissed from employment. Respondent converted her
petition for regularization into a complaint for illegal dismissal. Labor Arbiter
Madjayran H. Ajan rendered a decision on July 13, 1999, dismissing the
complaint against the petitioner. Private respondent appealed before the National
Labor Relations Commission (NLRC). The NLRC reversed the ruling of the Labor
Arbiter and held that: (1) MANRED is a labor-only contractor, and (2) private
respondent was illegally dismissed. Petitioner subsequently appealed before the
Court of Appeals but dismissed the petition on the ground of non-compliance with
the rule on certification against forum shopping taking into account that the
aforesaid certification was subscribed and verified by the Personnel Director of
petitioner corporation without attaching thereto his authority to do so for and in
behalf of petitioner corporation per board resolution or special power of attorney
executed by the latter. The petitioner filed its motion for reconsideration which
was denied. Hence, this petition.
ISSUE
Whether the filing of a motion for reconsideration with an appended
certificate of non forum-shopping suffices to cure the defect in the pleading.
RULING
No. Petitioners contention that the filing of a motion for reconsideration
with an appended certificate of non forum-shopping suffices to cure the defect in
the pleading is absolutely specious. It negates the very purpose for which the
certification against forum shopping is required: to inform the Court of the
pendency of any other case which may present similar issues and involve similar
parties as the one before it. The requirement applies to both natural and juridical
persons. Well-settled is the rule that the certificate of non-forum shopping is a
mandatory requirement. Substantial compliance applies only with respect to the
contents of the certificate but not as to its presence in the pleading wherein it is
required
266

EXPERT TRAVEL &TOURS INC.


Vs.
COURT of APPEALS
G.R.NO. 152392
FACTS OF THE CASE
Korean Air Lines (KAL) filed a complaint against Expert Travel & Tours Inc
(ETI) with the RTC of Manila for collection of sum of money plus attorneys fees
and damages. The verification and certification against non-forum shopping was
signed by Atty. Mario Aguinaldo, who indicated therein that he was the resident
agent and legal counsel of KAL and had caused the preparation of the complaint.
ETI moved to dismiss the complaint on the ground that said lawyer was not
authorized to execute the verification and certification against non-forum
shopping as required by Section 5 Rule 7 of the Rules of Court. KAL opposed the
motion, contending that Atty. Aguinaldo was its resident agent and was reported
as such with the SEC as required by the Corporation Code of the Philippines.
Also, it further alleged that Atty. Aguinaldo was the Corporate Secretary of KAL.
At the hearing, Atty. Aguinaldo claimed that thru a resolution of KAL Board
of Directors approved during a special meeting, he was authorized to file the
complaint. Thru an affidavit submitted by its general manager, it was alleged that
a special teleconference was held and in that same teleconference the Board
approved a resolution authorizing him to execute the certification against nonforum shopping and to file the complaint. However, the general manager
provided no written copy of the said resolution.
The trial court gave due credence to the claim of Atty. Aguinaldo and the
general manager. ETI filed a motion for reconsideration, contending that the
court cannot take judicial notice of the said teleconference without any hearing,
which was denied by the RTC. CA also denied the appeal.
ISSUE
Whether or not the court can take judicial notice of the said teleconference.
RULING
Things of common knowledge of which courts take judicial matters
coming to the knowledge of men generally in the course of the ordinary
experiences of life, or they may be matters which are generally accepted by
mankind as true and are capable of ready and unquestionable determination. As
the common knowledge of man ranges far and wide, a wide variety of particular
facts have been judicially noticed as being matters of common knowledge. But a
court cannot take judicial notice of any fact which, in part, is dependent on the
existence or non-existence of a fact of which the court has no constructive
knowledge.
In this age of modern technology, the courts may take judicial notice that
business transactions may be made by individuals through teleconferencing.
Teleconferencing is interactive group communication through an electronic
medium, bringing people together under one roof even though they are
separated by hundreds of miles.
In the Philippines, teleconferencing and videoconferencing of members of
the board of directors of Private Corporation is a reality, in light of RA 8792. The
267

SEC issued SEC memorandum Circular No. 15, on November 30, 2001,
providing the guidelines to be complied with related to such conferences.
The Court is not convinced that one was conducted; even if there had
been one, the Court is not inclined to believe that a board resolution was duly
passed specifically authorizing Atty. Aguinaldo to file the complaint and execute
the required certification against non-forum shopping. Petition granted.

268

GOVERNMENT SERVICE INSURANCE SYSTEM


Vs.
COURT OF APPEALS
G.R. No. 183905
FACTS OF THE CASE
The annual stockholders meeting (annual meeting) of the Manila Electric
Company (Meralco) was scheduled and in connection therein, proxies were
required to be submitted and the proxy validation was slated for five days later. In
view of the resignation of Camilo Quiason, the position of corporate secretary of
Meralco became vacant. The board of directors of Meralco designated Jose
Vitug to act as corporate secretary for the annual meeting. However, when the
proxy validation began, the proceedings were presided over by respondent
Anthony Rosete (Rosete), assistant corporate secretary and in-house chief legal
counsel of Meralco. Private respondents nonetheless argue that Rosete was the
acting corporate secretary of Meralco. Petitioner Government Service Insurance
System (GSIS), a major shareholder in Meralco, was distressed over the proxy
validation proceedings, and the resulting certification of proxies in favor of the
Meralco management.
GSIS filed a complaint seeking the declaration of certain proxies as invalid.
Three days later, it filed a Notice with the RTC manifesting the dismissal of the
complaint. On the same day, GSIS filed an Urgent Petition with the Securities
and Exchange Commission (SEC) seeking to restrain Rosete from "recognizing,
counting and tabulating, directly or indirectly, notionally or actually or in whatever
way, form, manner or means, or otherwise honoring the shares covered by" the
proxies in favor of respondents and to annul and declare invalid said proxies.
GSIS also prayed for the issuance of a Cease and Desist Order (CDO) to
restrain the use of said proxies during the annual meeting scheduled for the
following day. During the annual meeting held on the following day, Rosete
announced that the meeting would push through, expressing the opinion that the
CDO is null and void. Respondents filed a petition for certiorari with prohibition
with the Court of Appeals, praying that the CDO and the SCO be annulled. The
Court of Appeals promulgated a decision dismissing the complaint filed by GSIS
in SEC.
The two remaining cases are docketed as G.R. No. 183905 and 184275. It
pertains to a petition for certiorari and prohibition filed by GSIS, against the Court
of Appeals and seeks of the Court to declare the decision of the Court of Appeals
null and void, affirm the SECs jurisdiction over the petition filed before it by
GSIS, and pronounce that the CDO and the SCO orders are valid.
ISSUE
Whether or not SEC has jurisdiction over the petition filed by GSIS.
RULING
The correct answer is not clear-cut, but there is one. In private
respondents favor, the provisions of law they cite pertain directly and exclusively
to the statutory jurisdiction of trial courts acquired by virtue of the transfer of
jurisdiction following the passage of the SRC. In contrast, the SRC provisions
relied upon by GSIS do not immediately or directly establish that bodys
jurisdiction over the petition, since it necessitates the linkage of Section 20 to
Section 53.1 of the SRC before the point can bear on us.
269

On the other hand, the distinction between "proxy solicitation" and "proxy
validation" cannot be dismissed offhand. The right of a stockholder to vote by
proxy is generally established by the Corporation Code, but it is the SRC which
specifically regulates the form and use of proxies, more particularly the
procedure of proxy solicitation, primarily through Section 20.
It is plain that proxy solicitation is a procedure that antecedes proxy
validation. The former involves the securing and submission of proxies, while the
latter concerns the validation of such secured and submitted proxies. GSIS
raises the sensible point that there was no election yet at the time it filed its
petition with the SEC, hence no proper election contest or controversy yet over
which the regular courts may have jurisdiction. And the point ties its cause of
action to alleged irregularities in the proxy solicitation procedure, a process that
precedes either the validation of proxies or the annual meeting itself.
Under Section 20.1, the solicitation of proxies must be in accordance with
rules and regulations issued by the SEC, such as AIRR-SRC Rule 4. And by
virtue of Section 53.1, the SEC has the discretion "to make such investigations
as it deems necessary to determine whether any person has violated" any rule
issued by it, such as AIRR-SRC Rule 4. The investigatory power of the SEC
established by Section 53.1 is central to its regulatory authority, most crucial to
the public interest especially as it may pertain to corporations with publicly traded
shares. For that reason, we are not keen on pursuing private respondents
insistence that the GSIS complaint be viewed as rooted in an intracorporate
controversy solely within the jurisdiction of the trial courts to decide. It is possible
that an intra-corporate controversy may animate a disgruntled shareholder to
complain to the SEC a corporations violations of SEC rules and regulations, but
that motive alone should not be sufficient to deprive the SEC of its investigatory
and regulatory powers, especially so since such powers are exercisable on a
motu proprio basis.
In the end, even assuming that the events narrated in our Resolution in
A.M. No. 08-8-11-CA constitute sufficient basis to nullify the assailed decision of
the Court of Appeals, still it remains clear that the reliefs GSIS seeks of this Court
have no basis in law.
Wherefore, the petition in G.R. No. 184275 is expunged for lack of capacity of the
petitioner to bring forth the suit. The petition in G.R. No. 183905 is dismissed for
lack of merit except that the second and third paragraphs of the fallow of the
assailed decision of the Court of Appeals, including subparagraphs (1), (2), 2(a),
2(b), 2(c) and 2(d) under the second paragraph, are deleted.

270

REPUBLIC OF THE PHILIPPINES


Vs.
ACOJE MINING COMPANY, INC.
G.R.No. L-18062
February 28, 1963
FACTS OF THE CASE
On May 17, 1948, the Acoje Mining Company, Inc. wrote the Director of
Posts requesting the opening of a post, telegraph and money order offices at its
mining camp at Sta. Cruz, Zambales, to service its employees and their families
that were living in said camp. Acting on the request, the Director of Posts wrote in
reply stating that if aside from free quarters the company would provide for all
essential equipment and assign a responsible employee to perform the duties of
a postmaster without compensation from his office until such time as funds
therefor may be available he would agree to put up the offices requested. The
company in turn replied signifying its willingness to comply with all the
requirements.
On April 11, 1949, the Director of Posts again wrote a letter to the
company stating among other things that "In cases where a post office will be
opened under circumstances similar to the present, it is the policy of this office to
have the company assume direct responsibility for whatever pecuniary loss may
be suffered by the Bureau of Posts by reason of any act of dishonesty,
carelessness or negligence on the part of the employee of the company who is
assigned to take charge of the post office," thereby suggesting that a resolution
be adopted by the board of directors of the company expressing conformity to the
above condition relative to the responsibility to be assumed buy it in the event a
post office branch is opened as requested.
On September 2, 1949, the company informed the Director of Posts of the
passage by its board of directors of a resolution The letter further states that the
company feels that that resolution fulfills the last condition imposed by the
Director of Posts and that, therefore, it would request that an inspector be sent to
the camp for the purpose of acquainting the postmaster with the details of the
operation of the branch office.

ISSUE
Whether the act of the Board in issuing the said resolution of conformity
was ultra vires.
RULING
The corporate act was a necessary corollary to promote the interest and
welfare of the corporation. This is further bolstered by the fact that the opening of
the post was upon the request of the company for the convenience and benefit of
its employees, and not an idea of the Director of Posts. Thus, having benefited
from the agreement, the corporation is estopped from raising the defense that the
said corporate act by its board in conforming to the condition imposed by the
Director of Posts is ultra vires.
Neither can the corporation interpose the defense that its liability is only
that of a guarantor. A mere reading of the resolution of the Board of Directors
dated August 31, 1949, upon which the plaintiff based its claim, would show that
the responsibility of the defendant company is not just that of a guarantor. The
271

phraseology and the terms employed are so clear and sweeping and that the
defendant assumed 'full responsibility for all cash received by the Postmaster.'
Here the responsibility of the defendant is not just that of a guarantor. It is clearly
that of a principal."

272

NATIONAL POWER CORPORATION


Vs.
VERA
G.R. No. 83558
FACTS OF THE CASE
Private respondent Sea Lion International Port Services (Sea Lion) filed a
complaint for prohibition and mandamus against petitioner National Power
Corporation (NPC) alleging that it has acted in bad faith and with grave abuse of
discretion in not renewing its Contract for Stevedoring Services for Coal-Handling
Operations at NPC's plant, and in taking over its stevedoring services.
respondent judge issued a restraining order against NPC enjoining the latter from
undertaking stevedoring services at its pier. Consequently, NPC filed an "Urgent
Motion" to dissolve the restraining order, asserting: (1) that by virtue of
Presidential Decree No. 1818, respondent judge had no jurisdiction to issue the
order; and (2) that private respondent, whose contract with NPC had expired
prior to the commencement of the suit, failed to establish a cause of action for a
writ of preliminary injunction. Respondent judge issued the assailed Order
denying NPC's motion and issuing a writ of preliminary injunction, after finding
that NPC was not empowered by its Charter, to engage in stevedoring and
arrastre services.
ISSUE
Whether the undertaking of stevedoring services is empowered by the
NPCs charter powers.
RULING
Yes.
To carry out the national policy of total electrification of the country,
specifically the development of hydroelectric generation of power and the
production of electricity from nuclear, geothermal and other sources to meet the
needs of industrial development and dispersal and the needs of rural
electrification [Secs. 1 and 2, Rep. Act No. 6395, as amended], the NPC was
created and empowered not only to construct, operate and maintain power
plants, reservoirs, transmission lines, and other works, but also to exercise such
powers and do such things as may be reasonably necessary to carry out the
business and purposes for which it was organized, or which, from time to time,
may be declared by the Board to be necessary, useful, incidental or auxiliary to
accomplish said purpose.
In determining whether or not an NPC act falls within the purview of the
above provision, the Court must decide whether or not a logical and necessary
relation exists between the act questioned and the corporate purpose expressed
in the NPC charter. For if that act is one which is lawful in itself and not otherwise
prohibited, and is done for the purpose of serving corporate ends, and
reasonably contributes to the promotion of those ends in a substantial and not in
a remote and fanciful sense, it may be fairly considered within the corporation's
charter powers.

273

AURBACH
VS.
SANITARY WARES MANUFACTURING CORP
G.R.NO. 75875
FACTS OF THE CASE
Saniwares (Domestic Corporation) and ASI (foreign corporation) entered
into an agreement to engage primarily in the business of manufacturing in the
Philippines and selling here and abroad vitreous china and sanitary wares.
They also agreed that the business operations in the Philippines shall be
carried on by an incorporated enterprise and that the name of the corporation
shall initially be Sanitary Wares Manufacturing Corp.
Unfortunately, with the business successes came the deterioration of the
initially harmonious relationship between the two. The disagreement was
allegedly due to Saniwares desire to expand the export operations which was
objected by ASI as it apparently had other subsidiaries of joint venture groups in
countries contemplated by Saniwares.
Several incidents in the annual stockholders meeting triggered the filing of
separate petitions by the parties, both parties claiming to be the legitimate
directors of the corporation.
According to ASI, the actual intention of the parties should be viewed from
the agreement wherein it is clearly stated that the parties intention was to form a
corporation and not a joint venture. No other evidence should be admitted on the
ground that it contravenes the parol evidence rule under sec. 7, Rule 130,
Revised Rules of Court.
Lagdameo and Young (Saniware): the agreement failed to express the true
intent of the parties.
ISSUE
Whether or not the business established by the parties was a joint venture
or a corporation.

RULING
It was a joint venture.
The rule is that whether the parties to a particular contract have thereby
established among themselves a joint venture or some other relation depends
upon their actual intention which is determined in accordance with the rules
governing the interpretation and construction of contracts.
In the instant cases, our examination of important provisions of the
Agreement as well as the testimonial evidence presented by the Lagdameo and
Young Group shows that the parties agreed to establish a joint venture and not a
corporation. The history of the organization of Saniwares and the unusual
arrangements which govern its policy making body are all consistent with a joint
274

venture and not with an ordinary corporation. According to the unrebutted


testimony of Mr. Baldwin Young, he negotiated the Agreement with ASI in behalf
of the Philippine nationals. He testified that ASI agreed to accept the role of
minority vis-a-vis the Philippine National group of investors, on the condition that
the Agreement should contain provisions to protect ASI as the minority.
The legal concept of a joint venture is of common law origin. It has no
precise legal definition but it has been generally understood to mean an
organization formed for some temporary purpose It is in fact hardly
distinguishable from the partnership, since their elements are similar community
of interest in the business, sharing of profits and losses, and a mutual right of
control. The main distinction cited by most opinions in common law jurisdictions
is that the partnership contemplates a general business with some degree of
continuity, while the joint venture is formed for the execution of a single
transaction, and is thus of a temporary nature. This observation is not entirely
accurate in this jurisdiction, since under the Civil Code, a partnership may be
particular or universal, and a particular partnership may have for its object a
specific undertaking. (Art. 1783, Civil Code). It would seem therefore that under
Philippine law, a joint venture is a form of partnership and should thus be
governed by the law of partnerships. The Supreme Court has however
recognized a distinction between these two business forms, and has held that
although a corporation cannot enter into a partnership contract, it may however
engage in a joint venture with others. (At p. 12, Tuazon v. Bolanos, 95 Phil. 906
[1954])

275

PENA
VS.
COURT OF APPEALS
193 SCRA 716
GR.NO.91478. FEBRUARY 7, 1991
FACTS OF THE CASE
Pampanga Bus Company (PAMBUSCO) mortgaged a lot covered by TCT
certificates to Development Bank of the Philippines. Said lot was foreclosed. On
the foreclosure sale, Rosita Pena was the highest bidder. The sheriff then issued
a certificate of sale in her favor.
A month thereafter, 3 out of 5 of the board of directors of PAMBUSCO
passed a resolution assigning its right of redemption to any person. A deed of
assignment was executed in favor of Marcelo Enriquez, who later on redeemed
the property by virtue of the deed of assignment. A certificate of redemption was
later issued by the deputy sheriff in favor of Enriquez. A day after, Enriquez sold
the sand lot to spouses yap.
The deputy sheriff wrote Pena that the said properties had been redeemed
and that she may get her money back. However, Pena wrote back advising the
sheriff that the redemption was not valid because it was made by virtue of a void
deed of assignment. A case was filed against Pena for the recovery of the
property. Pena countered that she is the legitimate owner of the property by
virtue of the foreclosure proceeding instituted by DBP, and that no valid
redemption had been affected within the allowed period. She further contended
that the deed of assignment executed in favor of Enriquez was void for being an
ultra-vires act of the BOD of PAMBUSCO.
The trial court decided in favor of Pena declaring the deed of assignment
null and void, and the subsequent sale of the property void as initio. Upon
appeal, the CA reversed the decision of the trial court on the ground that the
provision on the by-laws of PAMBUSCO requiring a quorum of four is only
applicable when there is a failure of notice, thus the resolution of the board then
is valid.
ISSUE
Whether the board resolution regarding the deed of assignment or the
disposition of the corporations property was valid.
RULING
The board resolution was invalid. The by-laws of PAMBUSCO that in
special meetings of the BOD a quorum shall be composed of four members. The
resolution assigning the right of redemption was passed with the presence of
only three out of its five boards of directors.
Under Section 25 of the Corporation Code, the articled of incorporation or
the by-laws may fix a greater number than the majority of the number of the
board to constitute a quorum necessary for the validity of the transaction or
business. Any number less than the number provided in the articles or by-laws
276

therein cannot constitute a quorum, and any act therein cannot bind the
corporation.
Further, the three alleged directors who attended the meeting and passed
the resolution were not listed directors of PAMBUSCO in the latest GIS filed by
PAMBUSCO at the SEC. Also, the three alleged directors were not listed
stockholders on the list of stockholders filed by PAMBUSCO at the SEC. Under
Section 30, of thee Corporation Code, only persons who own at least one share
in their own right may qualify as director of a corporation.
Moreover, under Section 40 of the Corporation Code, sale or disposition of
all or substantially all of the properties of the corporation requires, in addition to a
resolution passed by the majority of the BOD, an affirmative vote of at least 2/3 of
the stockholders representing the voting power in a meeting duly called for the
purpose.

277

LOPEZ REALTY INC. AND ASUNCION LOPEZ GONZALES,


Vs.
FONTECHA ET AL. and NLRC
G.R. No. 76801
August 11, 1995
FACTS OF THE CASE
Lopez Realty, Inc., is a corporation engaged in real estate business, while
petitioner Asuncion Lopez Gonzales is one of its majority shareholders.
Sometime in 1978, Arturo Lopez, one of the shareholders, submitted a
proposal relative to the distribution of certain assets of the corporation including
the reduction of employees with provision for their gratuity pay. It was approved
in a special meeting of the board of directors. Lopez Realty approved two
resolutions, both passed in 1980, providing for the gratuity pay of its employees.
In 1981, except for Asuncion Lopez Gonzales who was then abroad, the
remaining members of the Board of Directors passed another resolution on how
the gratuity of the employees will be given.
Private respondents were the retained employees of petitioner corporation.
They requested for the full payment of their gratuity pay. This was granted in a
special meeting but petitioner Asuncion was still abroad at that time. She sent a
cablegram to the corporation objecting to certain matters taken up by the board
in her absence. Upon her return, she filed a derivative suit with the SEC against
Arturo Lopez.
Notwithstanding the "corporate squabble" between petitioner Asuncion
Lopez Gonzales and Arturo Lopez, the first two (2) installments of the gratuity
pay were paid by petitioner corporation but the rest of the cash vouchers and
checks were cancelled by petitioner Asuncion. Despite private respondents'
repeated demands for their gratuity pay, corporation refused to pay the same.
ISSUE
Whether or not the corporation is bound to give the full gratuity pay
considering the lack of notice to one of the board directors during the resolution
that granted it.
RULING
Yes. The general rule is that a corporation, through its board of directors,
should act in the manner and within the formalities, if any, prescribed by its
charter or by the general law. Thus, directors must act as a body in a meeting
called pursuant to the law or the corporations by-laws, otherwise, any action
taken therein may be questioned by any objecting director or shareholder.
Be that as it may, jurisprudence tells us that an action of the board of
directors during a meeting, which was illegal for lack of notice, may be ratified
either expressly, by the action of the directors in subsequent legal meeting, or
impliedly, by the corporations subsequent course of conduct. Thus, despite lack
of notice at that time the assailed resolutions were passed, Asuncion is now
precluded from questioning the validity since she acquiesced thereto by signing
the vouchers of the gratuity pay.
Assuming, arguendo, that there was no notice given to Asuncion Lopez
Gonzales during the special meetings held on August 17, 1981 and September
278

1, 1981, it is erroneous to state that the resolutions passed by the board during
the said meetings were ultra vires. In legal parlance, ultra vires act refers to one
which is not within the corporate powers conferred by the Corporation Code or
articles of incorporation or not necessary or incidental in the exercise of the
powers so conferred.
The assailed resolutions before us cover a subject which concerns the
benefit and welfare of the companys employees. To stress, providing gratuity
pay for its employees is one of the express powers of the corporation under the
Corporation Code, hence, petitioners cannot invoke the doctrine of ultra vires to
avoid any liability arising from the issuance of the subject resolutions.
Petitioners try to convince us that the subject resolutions had no force and
effect in view of the non-approval thereof during the Annual Stockholders
Meeting held on March 1, 1982. To strengthen their position, petitioners cite
section 28 1/2 of the Corporation Law (Section 40 of the Corporation Code). We
are not persuaded. The cited provision is not applicable to the case at bench as it
refers to the sale, lease, exchange or disposition of all or substantially all of the
corporations assets, including its goodwill. In such a case, the action taken by
the board of directors requires the authorization of the stockholders on record.
It will be observed that, except for Arturo Lopez, the stockholders of
petitioner corporation also sit as members of the board of directors. Under the
circumstances in field, it will be illogical and superfluous to require the
stockholders approval of the subject resolutions. Thus, even without the
stockholders approval of the subject resolutions, petitioners are still liable to pay
private respondents gratuity pay.

279

LAUREANO INVESTMENT AND DEVELOPMENT CORPORATION


VS
COURT OF APPEALS
G.R. No. 100468
FACTS OF THE CASE
Spouses Reynaldo Laureano and Florence Laureano are majority
stockholders of petitioner Corporation entered into a series of loan and credit
transactions with Philippine National Cooperative Bank (PNCB for short). To
secure payment of the loans, they executed Deeds of Real Estate Mortgage. In
view of their failure to pay their indebtedness, PNCB applied for extrajudicial
foreclosure of the real estate mortgages. The bank was the purchaser of the
properties in question in the foreclosure sale and titles thereof were consolidated
in PNCBs name.
Private respondent Bormaheco, Inc. became the successor of the
obligations and liabilities of PNCB over subject lots by virtue of a Deed of
Sale/Assignment including the two parcels of land in question, formerly
registered in the name of the Laureano spouses.
Five (5) days after securing titles over the said properties, Bormaheco filed
an Ex-Parte Petition for the Issuance of Writ of Possession of the two lots
formerly owned by the Laureanos. LIDECO filed a motion to intervene. However,
BOMAHECO filed a motion to strike out LIDECOs complaint in intervention and
all other pleadings submitted by LIDECO on the ground that the latter has no
juridical personality of its own, and therefore, no capacity to sue.
ISSUE
Whether LIDECO, not registered with the SEC, has the capacity to sue.
RULING
No. Section 1, Rule 3 of the Rules of Court provides that only natural or
juridical persons or entities authorized by law may be parties to a civil
action. Under the Civil Code, a corporation has a legal personality of its own
(Article 44), and may sue or be sued in its name, in conformity with the laws and
regulations of its organization (Article 46). Additionally, Article 36 of the
Corporation Code similarly provides:
Article 36. Corporate powers and capacity. -- Every
incorporated under this Code has the power and capacity:
1. To sue and be sued in its corporate name;
x x x (underscoring supplied)

corporation

As the trial and appellate courts have held, Lideco Corporation had no
personality to intervene since it had not been duly registered as a corporation. If
petitioner legally and truly wanted to intervene, it should have used its corporate
name as the law requires and not another name which it had not
registered. Indeed, as the Respondent Court found, nowhere in the motion for
intervention and complaint in intervention does it appear that Lideco
Corporation
stands
for
Laureano
Investment
and
Development
Corporation. Bormaheco, Inc., thus, was not estopped from questioning the
juridical personality of Lideco Corporation, even after the trial court had allowed
it to intervene in the case.
280

ISLAMIC DIRECTORATE OF THE PHILIPPINES


Vs.
COURT OF APPEALS
G.R. No. 117897
FACTS OF THE CASE
Sometime in 1971, the ISLAMIC DIRECTORATE OF THE PHILIPPINES (IDP)
was organized and incorporated for the primary purpose of establishing an
Islamic Center in Quezon City. IDP purchased the subject lot in Tandang Sora,
Quezon City. When former President Marcos declared Martial Law in 1972, most
of the members of the IDPs Board of Trustees flew to the Middle East to escape
political persecution. Thereafter two groups emerged, the Carpizo Group and
Abbas Group claiming to be members of the IDPs board. In a suit between these
two groups, SEC ruled that the election of both groups is null and void. SEC
recommended that new by-laws be adopted and upon its approval an election be
conducted. In 1989, the Carpizo group without having been elected as new
members of the board caused to be signed a board resolution authorizing the
sale of the subject lot to private respondents Iglesias ni Cristo (INC) as
evidenced by a Deed of Absolute Sale. In 1991 the petitioner IDP Board of
Trustees headed by former Senator Mamintal Tamano, or the Tamano Group,
filed a petition before the SEC, seeking to declare null and void the said Deed of
Absolute Sale signed by the Carpizo Group and the INC since the group of
Carpizo was not the legitimate Board of Trustees of the IDP. The SEC declared
the sale null and void, however this was reversed by the CA.
ISSUE
Whether the contract of sale between INC and IDP signed by the Carpizo Group
is valid
RULING
No. Under Sec. 40 of the Corporation Code, a corporation may, by a majority
vote of its board of directors or trustees, sell, lease, exchange, mortgage, pledge
or otherwise dispose of all or substantially all of its property and assets, including
its goodwill, upon terms and conditions and for such consideration, which may be
money, stocks, bonds or other instruments for the payment of money or other
property or consideration, as its board of directors or trustees may deem
expedient, when authorized by the vote of the stockholders representing at least
two-thirds (2/3) of the outstanding capital stock; or in case of non-stock
corporation, by the vote of at least two-thirds (2/3) of the members, in a
stockholders or members meeting duly called for the purpose. Here, the
Tandang Sora property, it appears from the records, constitutes the only property
of the IDP. Hence, its sale to a third-party is a sale or disposition of all the
corporate property and assets of IDP falling squarely within the contemplation of
the foregoing section. For the sale to be valid, the majority vote of the legitimate
Board of Trustees, concurred in by the vote of at least 2/3 of the bona fide
members of the corporation should have been obtained. These twin
requirements were not met as the Carpizo Group which voted to sell the
Tandang Sora property was a fake Board of Trustees, and those whose names
and signatures were affixed by the Carpizo Group together with the sham Board
Resolution authorizing the negotiation for the sale were, from all indications, not
bona fide members of the IDP as they were made to appear.

281

HEIRS OF TAN ENG KEE


Vs
COURT OF APPEALS
GR 126881, October 3, 2000
FACTS OF THE CASE
Tan Eng Kee died September 13, 1984, his heirs- his common law wife
and his children, filed a complaint against the deceased brother, Tan Eng Lay.
According to the petitioners, the brothers pooled their resources together and
entered into a partnership engaged in the business of selling lumber and
hardware and construction supplies. They named their enterprise "Benguet
Lumber" which they jointly managed until Tan Eng Kee's death. Petitioners
herein averred that the business prospered due to the hard work and thrift of the
alleged partners. However, they claimed that in 1981, Tan Eng Lay and his
children caused the conversion of the partnership "Benguet Lumber" into a
corporation called "Benguet Lumber Company." The incorporation was
purportedly a ruse to deprive Tan Eng Kee and his heirs of their rightful
participation in the profits of the business. Petitioners prayed for accounting of
the partnership assets, and the dissolution, winding up and liquidation thereof,
and the equal division of the net assets of Benguet Lumber.
RTC ruled in favor of the petitioners, ruling that the business was a joint
venture, akin to a partnership, between the brothers. CA reversed the court a
quos decision; hence, this petition.
ISSUE
WON Benguet Lumber Company was a product of brothers Tan Eng Kee and
Tan Eng Lays partnership.
RULING
NO. The company was not a partnership venture between the brothers;
therefore, the petitioners had no right to Benguet Lumber Companys earnings.
The petitioners failed to prove, through the required quantum of evidence to
prove that partnership existed between Tan Eng Kee and Tan Eng Lay.
Unfortunately for petitioners, Tan Eng Kee has passed away. Only he,
aside from Tan Eng Lay, could have expounded on the precise nature of the
business relationship between them. In the absence of evidence, we cannot
accept as an established fact that Tan Eng Kee allegedly contributed his
resources to a common fund for the purpose of establishing a partnership. The
testimonies to that effect of petitioners' witnesses are directly controverted by
Tan Eng Lay. Tan Eng. Lay consistently testified that he had his business and his
brother had his, that it was only later on that his said brother, Tan Eng Kee, came
to work for him. Be that as it may, co-ownership or co-possession is not an
indicium of the existence of a partnership.
Besides, it is indeed odd, if not unnatural, that despite the forty years the
partnership was allegedly in existence, Tan Eng Kee never asked for an
accounting. The essence of a partnership is that the partners share in the profits
and losses. Each has the right to demand an accounting as long as the
282

partnership exists. The Court have allowed a scenario wherein "[i]f excellent
relations exist among the partners at the start of the business and all the partners
are more interested in seeing the firm grow rather than get immediate returns, a
deferment of sharing in the profits is perfectly plausible." But in the situation in
the case at bar, the deferment, if any, had gone on too long to be plausible. A
person is presumed to take ordinary care of his concerns.
A demand for periodic accounting is evidence of a partnership. During his
lifetime, Tan Eng. Kee appeared never to have made any such demand for
accounting from his brother, Tang Eng Lay.
The evidence of payroll purporting Tan Eng Kee to be just an employee of
his brother sums up the conclusion that there was no partnership between the
brothers on the business.

283

PILIPINAS LOAN COMPANY, INC.


VS
SECURITIES AND EXCHANCE COMMISSION
GR.NO.104720, APRIL 4, 2001
FACTS OF THE CASE
Petitioner Pilipinas Loan Company Inc. is a lending Corporation registered
with the SEC, and its primary purpose base on its Articles of Incorporation is To
act as a lending investor or to engage in the practice of lending money or
extending loans on the security of real or personal, tangible or intangible
properties whether as pledge, real or chattel mortgage, xxx without however
engaging in pawn broking as defined under P.D.114.
Private respondent Filipinas Pawnshop, Inc. is also a corporation duly
registered with the SEC, and its primary purpose under its Articles of
Incorporation, is to extend loans on the security of either personal or real
properties, or in other words, to operate as a pawn broker, engage in pawnshop
business as defined under P.D 114.
Private respondent Filipinas Pawnshop, Inc. filed a complaint against
petitioner, Pilipinas Loan Company with the Prosecution Enforcement
Department of the SEC alleging that the petitioner contrary to its prime purpose,
has been operating and doing business as a pawn broker, pawnshop or
sanglaan in the same area where private respondent has had its own
pawnshop for 30 years, without permission from the Central Bank to engage in
pawnshop business, thereby causing unjust and unfair competition with private
respondent.
Petitioner denied that it is engage in the pawnshop business alleging that it
is a lending investor duly registered with the Central Bank. Petitioner also
questioned the power of the SEC to take cognizance of the complaint involving
supposed violation of the Pawnshop Regulation Act (P.D.114) which is more
properly within the jurisdiction of the Central Bank.
The SEC en banc rendered a decision ordering the petitioner to cease and
desist from further engaging in business as a pawnshop or pawnbroker as
defined in P.D.114, until proper license shall have been secured from the Central
Bank.
On appeal, the Court of Appeals affirmed the decision of the SEC en banc
hence, this petition for certiorari under Rule 45 of the Rules of Court.
ISSUE
1) Whether or not petitioner had violated its Articles of Incorporation by engaging
in pawnshop business.
2) Whether or not the SEC has jurisdiction over complaints involving violation of
Pawnshop Regulation Act (P.D.114.)

284

RULING
On the first issue, a corporation has only such power as are expressly
granted to it by the Code and by its Articles of Incorporation, those that are
incidental to such conferred powers and to its existence. In the case at bar, the
limit of the powers of petitioner as a corporation is that it is prohibited from
engaging in pawn broking as defined in P.D.114. Petitioner engaged in the
pawnshop business when it is not authorized to do so by its Articles of
Incorporation is an ultra vires act, which amounts to fraud, detrimental not only to
the corporation but also to the public.
On the issue of the jurisdiction, the SC ruled that the SEC has jurisdiction
to entertain complaints involving ultra vires acts of a corporation. Thus, the
complaint treats of a violation of petitioners primary purpose, an ultra vires act.
Moreover, by law the SEC has absolute jurisdiction, supervision and control over
all corporations that are enfranchised to act as corporate entities and registered
under it. Furthermore, petitioner cannot invoke the jurisdiction of the Central
Bank in view of its own avowal that it is not a pawnshop and neither is it engaged
in the business as a pawnshop. P.D. 114 provides that the supervisory power of
the Central Bank extends merely to pawnshop registered with it hence, the
Central Bank does not acquire jurisdiction over the petitioner.

285

G.R. No. 133547. December 7, 2001.*


HEIRS OF ANTONIO PAEL and ANDREA ALCANTARA and CRISANTO
PAEL, petitioners, vs. COURT OF APPEALS, JORGE H. CHIN and RENATO
B. MALLARI, respondents.
G.R. No. 133843. December 7, 2001.*
MARIA DESTURA, petitioner, vs. COURT OF APPEALS, JORGE H. CHIN
and RENATO MALLARI, respondents.
FACTS OF THE CASE
Nature of the Case: A Motion for Reconsideration of the Decision of the
Supreme Court affirming the findings of the Court of Appeals.
The Motion for Reconsideration was filed by the Heirs of Antonio Pael, Andrea
Alacantara, and Crisante Pael. They alleged that the transfer of ownership in
1983 was not only dubious and fabricated, that it also produced no legal effect.
The transfer of title from the Heir of Pael in favor of PFINA PROPERTIES, INC
was replete with badges of fraud and irregularities which rendered nugatory and
imperative the existing doctrines on land registration and land titles.
The Heirs of Pael earlier disposed of their rights. There was nothing to transfer to
PFINA PROPERTIES, INC
PFINA PROPERTIES, INC. claims that it acquired the properties from the Heirs
of Pael by virtue of a deed of assignment. It filed for a motion to intervene.
It took them 15 years to act on the alleged deed of assignment and no steps
were taken by either of them to register the deed or secure the transfer certificate
of title evidencing the change of ownership.
At the time when PFINA PROPERTIES, INC acquired the disputed properties in
1983, its corporate name was PFINA Mining and Exploration , Inc. a mining
company which had no valid grounds to engage in the highly speculative
business of urban real estate development.
The Paels are no longer the owners of the land that they allegedly assigned.
During the pendency of the motion for reconsideration, the University of the
Philippines (UP) filed a motion for intervention alleging the properties covered by
TCT No. 52828 and TCT No. 52929 in the name of respondents Chin and Mallari
from part of the vast tract of land that is the UP Campus. It is registered in the
name of UP under TCT No 9462. Therefore any pronouncement by this Court
affecting the properties could create a cloud over UPs Title for which reason it
had a right to intervene in the proceedings.
The denial of the motion for intervention arising from the strict application of the
rule due lack of notice to, or the alleged failure of, movants to act seasonably will
lead the court to act of injustice to the movants, to their successors-in-interest
and to all purchasers for value and in good faith and thereby open the door to
fraud, falsehood and misrepresentation.
286

If the petition for reconstitution is finally granted, the chaos and confusion arising
from a situation where the certificate of title of the movant covering large areas of
land overlap or overreach on properties the title to which is being sought to be
reconstituted by private respondent.
Petitioners are indispensable parties with substantial interest in the controversy
that final adjudication cannot be made in their absence without affecting, nay
injuring such interest.
UPs motion for intervention was granted. The adjudication was limited to a
determination of the alleged overlapping or encroachment between UPs title and
respondents titles under the names of Chin and Mallari.
The respondents derive their titles from a defective title, and their titles should
also be null and void.
UPs title emanated from TCT 9462 from a sale by the Commonwealth of the
Philippines to the University in 1949. It came from OCT # 730 was under the
name of Mariano Tuason as early as 1914.
ISSUE
Whether the petition for intervention of UP should be granted given it has
intervened late in the case.

RULING
The motion for intervention of UP is granted. The case is remanded to the lower
Court for reception of evidence on conflicting claims over the property TCT No
52928 and 52929 between UP and Jorge H Chin and Renato B. Mallari.
Motion of reconsideration is denied for lack of merit.
DOCTRINE
Corporation Law; A mining company has no valid grounds to engage in the
highly speculative business of urban real estate development.At the time
PFINA acquired the disputed properties in 1983, its corporate name was PFINA
Mining and Exploration, Inc., a mining company which had no valid grounds to
engage in the highly speculative business of urban real estate development.

287

BENITO
Vs.
SECURITIES AND EXCHANGE COMMISSION,
G.R.NO. L-56655
FACTS OF THE CASE
In 1959, THE Articles of Incorporation of respondent Jamiatul Philippine-Al
Islamia, Inc. were filed with the SEC and were approved on 1962. Petitioner
Benito subscribed to 460 shares in the corporation. Then, the corporation filed a
certificate of increase of its capital stock from the then P200, 000 to P1, 000,000.
The increase was approved. Further, P110, 980 worth of shares were
subsequently issued by the corporation from the unissued portion of the
authorized capital stock of P200, 000.00.
Benito filed with respondent SEC a petition alleging that the additional
issue of previously subscribed shares of the corporation was made in violation of
his pre-emptive right to said additional issue and that the increase in the
authorized capital stock of the corporation was illegal considering that the
stockholders of record were not notified of the meeting wherein the proposed
increase was in the agenda. The SEC held that the issuance by the corporation
of its unissued shares was validly made and was not subject to the pre-emptive
rights of stockholders, including Benito.
ISSUE
Whether the issuance of the additional shares without the consent of the
stockholders or of the Board of Directors, and in the absence of consideration, is
null and void.
RULING
The issuance of additional shares is valid. SEC ruling is upheld.
The power to issue shares of stocks in a corporation is lodged in the board of
directors and no stockholders meeting is necessary to consider it because
issuance of the P110,980 worth of shares of stocks does not need approval of its
stockholders. The by-laws of the corporation itself states that the Board of
Trustees shall, in accordance with the law, provide for the issue and transfer of
shares of stock of the Institute and shall prescribe the form of the certificate of
stock of the Institute.
The general rule is that pre-emptive right is recognized only with respect to new
issue of shares, and not with respect to additional issues of originally authorized
shares. This is on the theory that when a corporation at its inception offers its first
shares, it is presumed to have offered all of those which it is authorized to issue.
An original subscriber is deemed to have taken his shares knowing that they form
a definite proportionate part of the whole number of authorized shares. When the
shares left unsubscribed are later re-offered, he cannot therefore claim a dilution
of interest.

288

DEE
VS.
SEC,
G.R.NO. L-60502, JULY 16, 1991
FACTS OF THE CASE
Naga Telephone Company, Inc. (Natelco) was organized in 1954; the
authorized capital was P100, 000.00. In 1974, it decided to increase its
authorized capital to P3, 000,000.00. As required by the Public Service Act,
Natelco filed an application for the approval of the increased authorized capital
with the Board of Communications (BOC) which was duly approved subject to
certain conditions imposed by the decision in BOC Case NO. 74-84 that the
issuance of the shares of stocks will be for a period of one year from January 8,
1975, after which no further issues will be made without previous authority from
BOC. Pursuant to the approval given by the then BOC, Natelco filed its Amended
Articles of Incorporation with the SEC. When the amended articles were filed with
the SEC, the original authorized capital of P100, 000.00 was already paid. Of the
increased capital of P2, 900,000.00 the subscribers subscribed to P580,000.00
of which P145,000 was fully paid.
On April 12, 1977, Natelco entered into a contract with Communication
Services, Inc. (CSI) for the manufacture, supply, delivery and installation of
telephone equipment. In accordance with this contract, Natelco issued 24,000
shares of common stocks to CSI on the same date as part of the down payment.
On May 5, 1979, another 12,000 shares of common stocks were issued to CSI.
In both instances, no prior authorization from the Board of Communications, now
the National Telecommunications Commission, was secured pursuant to the
conditions imposed by the decision in BOC Case NO. 74 - 84.
On May 19, 1979, the stockholders of the Natelco held their annual
stockholders meeting to elect their seven directors to their Board of Directors, for
the year 1979-1980, wherein Pedro Lopez Dee (Dee) was unseated as Chairman
of the Board and President of the Corporation, but was elected as one of the
directors, together with his wife, Amelia Lopez Dee. In the election CSI was able
to gain control of Natelco when the latters legal counsel; Atty. Maggay won a
seat in the Board with the help of CSI. In the reorganization Atty. Maggay
became president.
Petitioner Dee having been unseated in the election, filed a petition in the
SEC questioning the validity of the elections of May 19, 1979 upon the main
ground that there was no valid list of stockholders through which the right to vote
could be determined. SEC issued restraining order which was elevated to the
Supreme Court where the enforcement of the SEC restraining order was
restrained upon the ground that the same was premature and the Commission
should be allowed to conduct its hearing on the controversy. Subsequently, the
Supreme Court dismissed the petition which resulted in the unseating of the
Maggay group from the board of directors of Natelco in a hold-over capacity.
In the course of the proceedings in SEC Case No. 1748, respondent
hearing officer issued an order on June 23, 1981, declaring: (1) that CSI is a
stockholder of Natelco and, therefore, entitled to vote; (2) that unexplained
16,858 shares of Natelco appear to have been issued in excess to CSI which
should not be allowed to vote; (3) that 82 shareholders with their corresponding
number of shares shall be allowed to vote; and (4) consequently, ordering the
holding of special stock-holder meeting to elect the new members of the Board
289

of Directors for Natelco based on the findings made in the order as to who are
entitled to vote. Due to this order, petitioner Dee filed a petition for
certiorari/appeal with the SEC which was sustained by the Commission en banc.
A motion for reconsideration was filed but was denied.
On May 20, 1982, Antonio Villasenor filed with the CFI claiming that he
was an assignee of an option to repurchase 36,000 shares of CS of Natelco
under a Deed of Assignment executed in his favor. On May 21, 1982, restraining
order was issued by the lower court commanding desistance from the scheduled
election until further orders. Then on May 22, 1982, controlling majority of the
stockholders proceeded with the elections under the supervision of the SEC
representatives. SEC recognized the election and the duly elected directors of
Lopez Dee group headed by Messrs. Justino De Jesus and Julio Lopez Dee kept
insisting no elections were held and refused to vacate their positions. On May 28,
1982, SEC issued another order directing the hold-over directors and officers to
turn over their respective posts and directing the Sheriff of Naga City and other
enforcement agencies to enforce its order. Hold-over officers peacefully vacated.
On June 2, 1982, Villasenor filed a charge for contempt which on September 7,
1982, lower court rendered CSI Nilda Ramos, Luciano Maggay, Desiderio
Saavedra, Augusto Federis and Ernesto Miguel, guilty of contempt of court. On
September 17, 1982, CSI group filed a petition for certiorari and prohibition with
preliminary injunction or restraining order against the CFI. Then on April 14,
1983, Intermediate Appellate Court annulled the contempt charge. Hence, this
petition.
ISSUE
Whether the preemptive right of Natelco stockholders was violated by the
issuance of the 113,800 shares to CSI.
RULING
No.
While the group of Luciano Maggay was in control of Natelco by virtue of the
restraining order issued in G.R. No. 50885, the Maggay Board issued 113,800
shares of stock to CSI. Petitioner said that the Maggay Board, in issuing said
shares without notifying Natelco stockholders, violated their right of pre-emption
to the unissued shares. The Court reiterated its decision in Benito vs. SEC, et al.,
which ruled that the Petitioner bewails the fact that in view of the lack of notice to
him of such subsequent issuance, he was not able to exercise his right of
preemption over the unissued shares. However, the general rule is that preemptive right is recognized only with respect to new issues of shares, and not
with respect to additional issues of originally authorized shares. This is on the
theory that when a corporation at its inception offers its first shares, it is
presumed to have offered all of those which it is authorized to issue. An original
subscriber is deemed to have taken his shares knowing that they form a definite
proportionate part of the whole number of authorized shares. When the shares
left unsubscribed are later reoffered, he cannot therefore claim a dilution of
interest the questioned issuance of the 113,800 stocks is not invalid even
assuming that it was made without notice to the stockholders as claimed by the
petitioner. The power to issue shares of stocks in a corporation is lodged in the
board of directors and no stockholders meeting is required to consider it because
additional issuance of shares of stocks does not need approval of the
stockholders. Consequently, no preemptive right of Natelco stockholders was
violated by the issuance of the 113,800 shares to CSI.
290

NIELSEN AND COMPANY INC.


Vs.
LEPANTO CONSOLIDATED MINING CO. INC
G.R.No. L-21601
FACTS OF THE CASE
On January 30, 1937, Nielson & Co. executed an agreement with Lepanto
Consolidated Mining Co. Lepanto owned the mining properties. Nielson operated
and maintained the said properties for Php 2,500.00 / month as management fee
plus 10% participation in the net profits for 5 years.
In 1940, the 10% share was disputed. Lepantos Board of Directors authorized
C.A. De Witt, president to enter with an agreement with Nielson modifying same
provisions effective January 1, 1940 such that Nielson shall receive :
10% of the dividends paid during the contract period and every end of the year;
10% of any depletion reserve that may be set up;
10% of any amount expended during the year out of surplus earnings for capital
account.
In 1941, the parties renewed their contract for another 5 years but the Pacific
War broke out in December 1941. In January 1942, the operation was disrupted.
The U.S. Army ordered that the mill, power plant, supplies, equipment,
concentrates on hand and mines be destroyed to prevent the Japanese from
using. Thereafter, the Japanese army occupied the mining properties and was
ousted only in August 1945.
Lepanto then rebuilt the mines and mills including setting up new organizations,
repairs, clearings, salvages, etc. The reconstruction was completed until 1948.
On June 26, 1948 the mines resumed the operation under the exclusive
management of Lepanto. However, after the mines were liberated in 1945, a
disagreement arose between Nielson and Lepanto over the status of the
operating contract which expired in 1947. Under the terms thereof
the management contract shall remain in suspension in case of fortuitous event
or force majeure such as war, which adversely affects the work of the mining and
milling.
On February 6, 1958, Nielson brought an action against Lepanto before the
Court of First Instance (CFI) of Manila to recover damages suffered in view of the
refusal of Lepanto to comply with the terms of a management contract entered
into between them on January 30, 1937.
In its answer, Lepanto denied the allegations and set up certain defenses,
prescription and laches as bars against the institution of the action.
After trial, the court a quo rendered a decision dismissing the complaint with
costs. The court stated that it did not find sufficient evidence to establish the
counterclaim of Lepanto therefore, dismissed the same. Nielson appealed. The
Supreme Court reversed the decision of the trial court and ordered Lepanto to
pay:
291

10% Share of cash dividends of December 1941 in the amount of Php 17,500.00
with legal interest thereon from the date of the filling of the complaint;
Management fee for January 1942 in the amount of Php 2,500.00 with legal
interest thereon from the date of the filing of the complaint;
Management fees for the 60-month period of extension amounting to Php
150,000.00 with legal interest;
10% Share in the cash dividends during the period of extension;10% of the
depletion reserve amounting to Php 53,928.88 with legal interest;10% of the
expenses of the capital account amounting to Php 694,364.76 with legal interest;
To issue and deliver to Nielson shares of stock at a par value equivalent to the
total of Nielsons 10% share in the stock dividends declared on November 28,
1948 and August 22, 1950; and
The sum of Php 50,000.00 as attorneys fee and the cost that Lepanto seeks for
reconsideration.
ISSUE
Whether or not the management contract is a contract of agency?
RULING
NO. The Supreme Court ruled that the management contract is not a contract of
agency as defined in Article 1709 of the Old Civil Code, but as a contract of lease
of services as defined in Article 1544 of the same Code. Article 1709 defines the
contract of agency as one person binds himself to render some service or to do
something for the account or at the request of another. While Article 1544
defines contract of lease of service as in a lease of work or services, one of the
parties binds himself to make or construct something or to render a service to the
other for a price certain. The court determined the nature of the management
contract in question wherein there was agreement for Nielson for 5 years had the
right to renew, to explore, to develop, and to operate the mining claims of
Lepanto. In the performance of this principal undertaking Nielson was not acting
as an agent but one as performing material acts for an employer, for
compensation.

292

MADRIGAL & COMPANY, INC.


Vs.
ZAMORA
G.R.No. L-48237
FACTS OF THE CASE
The petitioner was engaged, among several other corporate objectives, in
the management of Rizal Cement Co., Inc. Admittedly, the petitioner and Rizal
Cement Co., Inc. are sister companies, both owned by the same or practically
the same stockholders.
By an alleged resolution of its stockholders, the petitioner reduced its
capital stock through the distribution of the marketable securities owned by the
petitioner to its stockholders in exchange for their shares in an equivalent amount
in the corporation. Then petitioner reduced again for the second time through the
same scheme.
The respondent, the Madrigal Central Office Employees Union, sought for
the renewal of its collective bargaining agreement with the petitioner, and
proposed a wage increase, an allowance, and other economic benefits. The
petitioner, however, requested for a deferment in the negotiations. The
respondent then commenced an action with the National Labor Relations
Commission on a complaint for unfair labor practice. Petitioner filed its position
paper, alleging operational losses. The labor arbiter then rendered a decision in
favor of respondents finding that petitioner had been making substantial profits in
its operation. The petitioner appealed but was denied. Following the successive
reversals, the petitioner came anew to the Court. (G.R. No. 49023.)
While pending the resolution of the case, petitioner in a letter informed the
Secretary of Labor that Rizal Cement Co., Inc., from which it derives income as
the General Manager or Agent had ceased operating temporarily, and requested
that it be allowed to effect retrenchment. The letter, not having been verified and
accompanied by the proper supporting papers, the Department of Labor took no
action on the petitioner's request. The petitioner then applied for clearance to
terminate the services of a number of employees pursuant supposedly to its
retrenchment program. Respondent union then went to the Department of Labor
to complain of illegal lockout against the petitioner. Acting on this complaint, the
Secretary of Labor, in a decision found the dismissals to be contrary to law and
ordered the petitioner to reinstate some employees with backwages. The
petitioner then moved for reconsideration but was denied. The succeeding
dismissals, petitioner came to the Court. (G.R. No. 48237.)
By resolution of the Court, G.R. No. 48237 is consolidated with G.R. No. 49023.
ISSUE
Whether or not the dividends earned by petitioners can be used to satisfy
the employees claims.
RULING
There is no merit in these two (2) petitions.
What clearly emerges from the recorded facts is that the petitioner, awash
with profits from its business operations but confronted with the demand of the
union for wage increases, decided to evade its responsibility towards the
293

employees by a devised capital reduction. While the reduction in capital stock


created an apparent need for retrenchment, it was, by all indications, just a mask
for the purge of union members, who, by then, had agitated for wage increases.
In the face of the petitioner company's piling profits, the unionists had the right to
demand for such salary adjustments.
The petitioner would, however, have us believe that it in fact sustained
losses. Whatever profits it earned, so it claims were in the nature of dividends
"declared on its shareholdings in other companies in the earning of which the
employees had no participation whatsoever. Cash dividends," according to it,
"are the absolute property of the stockholders and cannot be made available for
disposition if only to meet the employees' economic demands."
There is no merit in this contention. We agree with the National Labor
Relations Commission that "[t]he dividends received by the company are
corporate earnings arising from corporate investment." Indeed, as found by the
Commission, the petitioner had entered such earnings in its financial statements
as profits, which it would not have done if they were not in fact profits.
Moreover, it is incorrect to say that such profits in the form of dividends
are beyond the reach of the petitioner's creditors since the petitioner had
received them as compensation for its management services in favor of the
companies it managed as a shareholder thereof. As such shareholder, the
dividends paid to it were its own money, which may then be available for wage
increments. It is not a case of a corporation distributing dividends in favor of its
stockholders, in which case, such dividends would be the absolute property of
the stockholders and hence, out of reach by creditors of the corporation. Here,
the petitioner was acting as stockholder itself, and in that case, the right to a
share in such dividends, by way of salary increases, may not be denied its
employees.
Accordingly, this court is convinced that the petitioner's capital reduction
efforts were, to begin with, a subterfuge, a deception as it were, to camouflage
the fact that it had been making profits, and consequently, to justify the mass
layoff in its employee ranks, especially of union members. They were nothing but
a premature and plain distribution of corporate assets to obviate a just sharing to
labor of the vast profits obtained by its joint efforts with capital through the years.
Surely, we can neither countenance nor condone this. It is an unfair labor
practice.
Wherefore, the petitions are dismissed. Subject to the modification as to
the amount of backwages hereby awarded, the challenged decisions are
affirmed. The temporary restraining orders are lifted. This decision is immediately
executory.

294

REPUBLIC PLANTERS BANK


VS.
AGANA
GR 51765, MARCH 3, 1997
FACTS OF THE CASE
On 18 September 1961, the Robes-Francisco Realty & Development
Corporation (RFRDC) secured a loan from the Republic Planters Bank in the
amount of P120, 000.00. As part of the proceeds of the loan, preferred shares of
stocks were issued to RFRDC through its officers then, Adalia F. Robes and one
Carlos F. Robes. In other words, instead of giving the legal tender totaling to the
full amount of the loan, which is P120, 000.00, the Bank lent such amount
partially in the form of money and partially in the form of stock certificates
numbered 3204 and 3205, each for 400 shares with a par value of P10.00 per
share, or for P4, 000.00 each, for a total of P8, 000.00. Said stock certificates
were in the name of Adalia F. Robes and Carlos F. Robes, who subsequently,
however, endorsed his shares in favor of Adalia F. Robes. Said certificates of
stock bear the following terms and conditions: "The Preferred Stock shall have
the following rights, preferences, qualifications and limitations, to wit: 1. Of the
right to receive a quarterly dividend of 1%, cumulative and participating. xxx 2.
That such preferred shares may be redeemed, by the system of drawing lots, at
any time after 2 years from the date of issue at the option of the Corporation." On
31 January 1979, RFRDC and Robes proceeded against the Bank and filed a
complaint anchored on their alleged rights to collect dividends under the
preferred shares in question and to have the bank redeem the same under the
terms and conditions of the stock certificates. The bank filed a Motion to Dismiss
3 private respondents' Complaint on the following grounds: (1) that the trial court
had no jurisdiction over the subject-matter of the action; (2) that the action was
unenforceable under substantive law; and (3) that the action was barred by the
statute of limitations and/or laches. The bank's Motion to Dismiss was denied by
the trial court in an order dated 16 March 1979. The bank then filed its Answer on
2 May 1979. Thereafter, the trial court gave the parties 10 days from 30 July
1979 to submit their respective memoranda after the submission of which the
case would be deemed submitted for resolution. On 7 September 1979, the trial
court rendered the decision in favor of RFRDC and Robes; ordering the bank to
pay RFRDC and Robes the face value of the stock certificates as redemption
price, plus 1% quarterly interest thereon until full payment. The bank filed the
petition for certiorari with the Supreme Court, essentially on pure questions of
law.
ISSUE
1. Whether the bank can be compelled to redeem the preferred shares issued to
RFRDC and Robes.
2. Whether RFRDC and Robes are entitled to the payment of certain rate of
interest on the stocks as a matter of right without necessity of a prior
declaration of dividend.
295

RULING
First Issue: While the stock certificate does allow redemption, the option to do
so was clearly vested in bank. The redemption therefore is clearly the type
known as "optional". Thus, except as otherwise provided in the stock certificate,
the redemption rests entirely with the corporation and the stockholder is without
right to either compel or refuse the redemption of its stock. Furthermore, the
terms and conditions set forth therein use the word "may". It is a settled doctrine
in statutory construction that the word "may" denotes discretion, and cannot be
construed as having a mandatory effect. The redemption of said shares cannot
be allowed. The Central Bank made a finding that the Bank has been suffering
from chronic reserve deficiency, and that such finding resulted in a directive,
issued on 31 January 1973 by then Gov. G. S. Licaros of the Central Bank, to the
President and Acting Chairman of the Board of the bank prohibiting the latter
from redeeming any preferred share, on the ground that said redemption would
reduce the assets of the Bank to the prejudice of its depositors and creditors.
Redemption of preferred shares was prohibited for a just and valid reason. The
directive issued by the Central Bank Governor was obviously meant to preserve
the status quo, and to prevent the financial ruin of a banking institution that would
have resulted in adverse repercussions, not only to its depositors and creditors,
but also to the banking industry as a whole. The directive, in limiting the exercise
of a right granted by law to a corporate entity, may thus be considered as an
exercise of police power.
Second Issue: Both Section 16 of the Corporation Law and Section 43 of the
present Corporation Code prohibit the issuance of any stock dividend without the
approval of stockholders, representing not less than two-thirds (2/3) of the
outstanding capital stock at a regular or special meeting duly called for the
purpose. These provisions underscore the fact that payment of dividends to a
stockholder is not a matter of right but a matter of consensus. Furthermore,
"interest bearing stocks", on which the corporation agrees absolutely to pay
interest before dividends are paid to common stockholders, is legal only when
construed as requiring payment of interest as dividends from net earnings or
surplus only. In compelling the bank to redeem the shares and to pay the
corresponding dividends, the Trial committed grave abuse of discretion
amounting to lack or excess of jurisdiction in ignoring both the terms and
conditions specified in the stock certificate, as well as the clear mandate of the
law.

296

BITONG
Vs.
COURT OF APPEALS
G.R. No. 123553
FACTS OF THE CASE
Petitioner Nora Bitong alleged that she had been the treasurer and
member of the Board of Directors of Mr. and Mrs. Corporation. She filed a
complaint with the SEC to hold respondent spouses Apostol liable for fraud,
misrepresentation, disloyalty, evident bad faith, conflict of interest and
mismanagement in directing the affairs of the corporation to the prejudice of the
stockholders. She alleges that certain transactions entered into by the
corporation were not supported by any stockholders resolution. The complaint
sought to enjoin Apostol from further acting as president-director of the
corporation and from disbursing any money or funds.
Apostol contends that Bitong was merely a holder in trust of the JAKA
shares of the corporation, hence, not entitled to the relief she prays for. SEC
Hearing Panel issued a writ enjoining Apostol. After hearing the evidence, SEC
Hearing Panel dissolved the writ and dismissed the complaint filed by Bitong.
Bitong appealed to the SEC en banc which reversed SEC Hearing Panel
decision. Apostol filed petition for review with the CA. CA reversed SEC en banc
ruling holding that Bitong was not the owner of any shares of stock in the
corporation and therefore, not a real party in interest to prosecute the complaint.
ISSUE
Whether Bitong was the real party in interest.
RULING
No.
Bitong was not a bona fide stockholder of the corporation. Several
corporate documents disclose that the true party in interest was JAKA. Although
her buying of the shares were recorded in the Stock and Transfer Book of
corporation, and as provided by Sec. 63 of the Corporation Code that no transfer
shall be valid except as between the parties until the transfer is recorded in the
books of the corporation, and upon its recording the corporation is bound by it
and is estopped to deny the fact of transfer of said shares, this provision is not
conclusive even against the corporation but are prima facie evidence only.
Parol evidence may be admitted to supply the omissions in the records,
explain ambiguities, or show what transpired where no records were kept, or in
some cases where such records were contradicted.
The certificate of stock itself once issued is a continuing affirmation or
representation that the stock described therein is valid and genuine and is at
least prima facie evidence that it was legally issued in the absence of evidence to
the contrary. However, this presumption may be rebutted. However, the books
and records of a corporation are not conclusive even against the corporation but
are prima facie evidence only. Parol evidence may be admitted to supply
omissions in the records, explain ambiguities, or show what transpired where no
records were kept, or in some cases where such records were contradicted. The
effect of entries in the books of the corporation which purport to be regular
records of the proceedings of its board of directors or stockholders can be
destroyed by testimony of a more conclusive character than mere suspicion that
there was an irregularity in the manner in which the books were kept.
297

Citibank, N.A. vs. Chua, 220 SCRA 75 (1993)


Ponente : Campos, Jr., Jp:

FACTS:
Citibank entered into an arrangement with Cresencio and Zenaida Velez
whereby the former extended a line of credit to the latter sufficiently secured with
real estate and chattel mortgage on equipment. Citibank, later on initiated a
restructuring agreement with Velez and allegedly the former did not comply
hence a complaint lodged by the latter against Citibank. During the pre-trial,
Citibank were represented by attorney-in-fact which were not duly authorized by
the banks Board of Directors and so, counsel for the private respondent sought
that Citibank, be declared in default. Petitioner contended that its by-laws are not
binding because it lacks the approval of SEC and being a foreign corporation
engaged in trade or business in the Philippines, Citibank is bound by the law of
the land.

The court of Appeals were convinced and favored private respondent.


Hence, this petition.

ISSUE:

Whether Citibanks by-laws were effective.

RULING:
The Supreme Court (SC) held that the adoption of by-laws requiring prior
approval of SEC only applies to Domestic corporations. In the case at bar, the
issuance of license to Citibank by SEC was sufficient proof that the bank
complied with all the legal requirements necessary to do business in the
Philippines which necessarily includes the banks by-laws and for this reason, the
appointment of an attorney-in-fact by Citibank was valid.

Petition was granted and the case remanded for further proceedings.

298

China Banking Corporation vs. Court appeals, 270 SCRA 503 (1997)

FACTS:
On 21 August 1974, Galicano Calapatia, Jr., a stockholder of Valley Golf &
Country Club, Inc. (VGCCI), pledged his Stock Certificate 1219 to China Banking
Corporation (CBC). On 16 September 1974, CBC wrote VGCCI requesting that
the pledge agreement be recorded in its books. In a letter dated 27 September
1974, VGCCI replied that the deed of pledge executed by Calapatia in CBC's
favor was duly noted in its corporate books. On 3 August 1983, Calapatia
obtained a loan of P20,000.00 from CBC, payment of which was secured by the
pledge agreement still existing between Calapatia and CBC. Due to Calapatia's
failure to pay his obligation, CBC, on 12 April 1985, filed a petition for
extrajudicial foreclosure before Notary Public Antonio T. de Vera of Manila,
requesting the latter to conduct a public auction sale of the pledged stock. On 14
May 1985, CBC informed VGCCI of the foreclosure proceedings and requested
that the pledged stock be transferred to its name and the same be recorded in
the corporate books. However, on 15 July 1985, VGCCI wrote CBC expressing
its inability to accede to CBC's request in view of Calapatia's unsettled accounts
with the club. Despite the foregoing, Notary Public de Vera held a public auction
on 17 September 1985 and CBC emerged as the highest bidder at P20,000.00
for the pledged stock. Consequently, CBC was issued the corresponding
certificate of sale. On 21 November 1985, VGCCI sent Calapatia a notice
demanding full payment of his overdue account in the amount of P18,783.24.
Said notice was followed by a demand letter dated 12 December 1985 for the
same amount and another notice dated 22 November 1986 for P23,483.24. On 4
December 1986, VGCCI caused to be published in the newspaper Daily Express
a notice of auction sale of a number of its stock certificates, to be held on 10
December 1986 at 10:00 a.m. Included therein was Calapatia's own share of
stock (Stock Certificate 1219). Through a letter dated 15 December 1986, VGCCI
informed Calapatia of the termination of his membership due to the sale of his
share of stock in the 10 December 1986 auction. On 5 May 1989, CBC advised
VGCCI that it is the new owner of Calapatia's Stock Certificate 1219 by virtue of
being the highest bidder in the 17 September 1985 auction and requested that a
new certificate of stock be issued in its name. On 2 March 1990, VGCCI replied
that "for reason of delinquency" Calapatia's stock was sold at the public auction
held on 10 December 1986 for P25,000.00. On 9 March 1990, CBC protested the
sale by VGCCI of the subject share of stock and thereafter filed a case with the
299

Regional Trial Court of Makati for the nullification of the 10 December 1986
auction and for the issuance of a new stock certificate in its name. On 18 June
1990, the Regional Trial Court of Makati dismissed the complaint for lack of
jurisdiction over the subject matter on the theory that it involves an intracorporate dispute and on 27 August 1990 denied CBC's motion for
reconsideration. On 20 September 1990, CBC filed a complaint with the
Securities and Exchange Commission (SEC) for the nullification of the sale of
Calapatia's stock by VGCCI; the cancellation of any new stock certificate issued
pursuant thereto; for the issuance of a new certificate in petitioner's name; and
for damages, attorney's fees and costs of litigation. On 3 January 1992, SEC
Hearing Officer Manuel P. Perea rendered a decision in favor of VGCCI, stating
in the main that considering that the said share is delinquent, VGCCI had valid
reason not to transfer the share in the name of CBC in the books of VGCCI until
liquidation of delinquency. Consequently, the case was dismissed. On 14 April
1992, Hearing Officer Perea denied CBC's motion for reconsideration. CBC
appealed to the SEC en banc and on 4 June 1993, the Commission issued an
order reversing the decision of its hearing officer; holding that CBC has a prior
right over the pledged share and because of pledgor's failure to pay the principal
debt upon maturity, CBC can proceed with the foreclosure of the pledged share;
declaring that the auction sale conducted by VGCCI on 10 December 1986 is
declared NULL and VOID; and ordering VGCCI to issue another membership
certificate in the name of CBC. VGCCI sought reconsideration of the order.
However, the SEC denied the same in its resolution dated 7 December 1993.
The sudden turn of events sent VGCCI to seek redress from the Court of
Appeals. On 15 August 1994, the Court of Appeals rendered its decision
nullifying and setting aside the orders of the SEC and its hearing officer on
ground of lack of jurisdiction over the subject matter and, consequently,
dismissed CBC's original complaint. The Court of Appeals declared that the
controversy between CBC and VGCCI is not intra-corporate; nullifying the SEC
orders and dismissing CBCs complaint. CBC moved for reconsideration but the
same was denied by the Court of Appeals in its resolution dated 5 October 1994.
CBC filed the petition for review on certiorari.

Issue:

Whether CBC is bound by VGCCI's by-laws.

Ruling:
300

In order to be bound, the third party must have acquired knowledge of the
pertinent by-laws at the time the transaction or agreement between said third
party and the shareholder was entered into. Herein, at the time the pledge
agreement was executed. VGCCI could have easily informed CBC of its by-laws
when it sent notice formally recognizing CBC as pledgee of one of its shares
registered in Calapatia's name. CBC's belated notice of said by-laws at the time
of foreclosure will not suffice. By-laws signifies the rules and regulations or
private laws enacted by the corporation to regulate, govern and control its own
actions, affairs and concerns and its stockholders or members and directors and
officers with relation thereto and among themselves in their relation to it. In other
words, by-laws are the relatively permanent and continuing rules of action
adopted by the corporation for its own government and that of the individuals
composing it and having the direction, management and control of its affairs, in
whole or in part, in the management and control of its affairs and activities. The
purpose of a by-law is to regulate the conduct and define the duties of the
members towards the corporation and among themselves. They are self-imposed
and, although adopted pursuant to statutory authority, have no status as public
law. Therefore, it is the generally accepted rule that third persons are not bound
by by-laws, except when they have knowledge of the provisions either actually or
constructively. For the exception to the general accepted rule that third persons
are not bound by by-laws to be applicable and binding upon the pledgee,
knowledge of the provisions of the VGCCI By-laws must be acquired at the time
the pledge agreement was contracted. Knowledge of said provisions, either
actual or constructive, at the time of foreclosure will not affect pledgee's right
over the pledged share. Article 2087 of the Civil Code provides that it is also of
the essence of these contracts that when the principal obligation becomes due,
the things in which the pledge or mortgage consists maybe alienated for the
payment to the creditor. Further, VGCCI's contention that CBC is duty-bound to
know its by-laws because of Article 2099 of the Civil Code which stipulates that
the creditor must take care of the thing pledged with the diligence of a good
father of a family, fails to convince. CBC was never informed of Calapatia's
unpaid accounts and the restrictive provisions in VGCCI's by-laws. Furthermore,
Section 63 of the Corporation Code which provides that "no shares of stock
against which the corporation holds any unpaid claim shall be transferable in the
books of the corporation" cannot be utilized by VGCCI. The term "unpaid claim"
refers to "any unpaid claim arising from unpaid subscription, and not to any
indebtedness which a subscriber or stockholder may owe the corporation arising
301

from any other transaction." Herein, the subscription for the share in question has
been fully paid as evidenced by the issuance of Membership Certificate 1219.
What Calapatia owed the corporation were merely the monthly dues. Hence,
Section 63 does not apply.

302

Loyola Grand Villas Homeowners Association (South) Association,


Inc. vs. Court of Appeals, 276 SCRA 681 (1997)
FACTS
Loyola Grand Villas Homeowners Association (LGVHA) was organized on
February 8, 1983 as the association of homeowners and residents of the Loyola
Grand Villas. It was registered with the Home Financing Corporation, the
predecessor of herein respondent Home Insurance and Guaranty Corporation
(HIGC), as the sole homeowners organization in the said subdivision under
Certificate of Registration No. 04-197. It was organized by the developer of the
subdivision and its first president was Victorio V. Soliven, himself the owner of
the developer. For unknown reasons, however, LGVHAI did not file its corporate
by-laws.

Sometime in 1988, the officers of the LGVHAI tried to register its by-laws.
They failed to do so. To the officers consternation, they discovered that there
were two other organizations within the subdivision the North Association and
the South Association.

In July, 1989, when Soliven inquired about the status of LGVHAI, Atty.
Joaquin A. Bautista, the head of the legal department of the HIGC, informed him
that LGVHAI had been automatically dissolved for two reasons. First, it did not
submit its by-laws within the period required by the Corporation Code and,
second, there was non-user of corporate charter because HIGC had not received
any report on the associations activities. These developments prompted the
officers of the LGVHAI to lodge a complaint with the HIGC. They questioned the
revocation of LGVHAIs certificate of registration without due notice and hearing
and concomitantly prayed for the cancellation of the certificates of registration of
the North and South Associations by reason of the earlier issuance of a
certificate of registration in favor of LGVHAI.

The HIGC ruled in favor of of LGVHAI. The South Association appealed to


the Appeals Board of the HIGC but it was dismissed for lack of merit. The Court
of Appeals affirmed the Resolution of the HIGC Appeals Board. Hence, this
petition.

ISSUE

Whether or not the failure of a corporation to file its by-laws within one month
from the date of its incorporation, as mandated by Section 46 of the Corporation
Code, result in its automatic dissolution

303

RULING
No. As the rules and regulations or private laws enacted by the
corporation to regulate, govern and control its own actions, affairs and concerns
and its stockholders or members and directors and officers with relation thereto
and among themselves in their relation to it, by-laws are indispensable to
corporations in this jurisdiction. These may not be essential to corporate birth but
certainly, these are required by law for an orderly governance and management
of corporations. Nonetheless, failure to file them within the period required by law
by no means tolls the automatic dissolution of a corporation.

Although the Corporation Code requires the filing of by-laws, it does not
expressly provide for the consequences of the non-filing of the same within the
period provided for in Section 46. However, such omission has been rectified by
Presidential Decree No. 902-A, the pertinent provisions on the jurisdiction of the
Securities and Exchange Commission of which state: * * * That the failure to file
by-laws is not provided for by the Corporation Code but in another law is of no
moment. P.D. No. 902-A, which took effect immediately after its promulgation on
March 11, 1976, is very much apposite to the Code. Accordingly, the provisions
abovequoted supply the law governing the situation in the case at bar, inasmuch
as the Corporation Code and P.D. No. 902-A are statutes in parimateria.
Interpretareetconcordarelegibusestoptimusinterpretandi. Every statute must be
so construed and harmonized with other statutes as to form a uniform system of
jurisprudence.

Taken as a whole and under the principle that the best interpreter of a
statute is the statute itself (optima statutiinterpretatixestipsumstatutum), Section
46 aforequoted reveals the legislative intent to attach a directory, and not
mandatory, meaning for the word must in the first sentence thereof. Note
should be taken of the second paragraph of the law which allows the filing of the
by-laws even prior to incorporation. This provision in the same section of the
Code rules out mandatory compliance with the requirement of filing the by-laws
within one (1) month after receipt of official notice of the issuance of its
certificate of incorporation by the Securities and Exchange Commission. It
necessarily follows that failure to file the by-laws within that period does not imply
the demise of the corporation.

The incorporators must be given the chance to explain their neglect or


omission and to remedy the same.Even under the foregoing express grant of
power and authority, there can be no automatic corporate dissolution simply
because the incorporators failed to abide by the required filing of by-laws
embodied in Section 46 of the Corporation Code. There is no outright demise of
corporate existence. Proper notice and hearing are cardinal components of due
process in any democratic institution, agency or society. In other words, the
304

incorporators must be given the chance to explain their neglect or omission and
remedy the same.

305

PMI College vs. National labor Relations Commission, 277 SCRA 462
(1997)
ROMERO, J.:

FACTS:

On July 7, 1991, petitioner, an educational institution offering courses on basic


seaman's training and other marine-related courses, hired private respondent as
contractual instructor with an agreement that the latter shall be paid at an hourly
rate of P30.00 to P50.00, depending on the description of load subjects and on
the schedule for teaching the same. Pursuant to this engagement, private
respondent then organized classes in marine engineering.
Initially, private respondent and other instructors were compensated for services
rendered during the first three periods of the abovementioned contract. However,
for reasons unknown to private respondent, he stopped receiving payment for the
succeeding rendition of services. This claim of non-payment was embodied in a
letter dated March 3, 1992, written by petitioner's Acting Director, Casimiro A.
Aguinaldo, addressed to its President, Atty. Santiago Pastor, calling attention to
and appealing for the early approval and release of the salaries of its instructors
including that of private respondent.
Private respondent's claims, were resisted by petitioner. Later in the proceedings,
PMI Colleges manifested that Mr. Tomas Cloma Jr., a member of the board of
trustees write a letter to the Chairman of the Board, clarifying the case of Galvan
and stating therein, inter alia, that under PMIs by-laws only the Chairman is
authorized to sign any contract and that Galvan, in any event, failed to submit
documents on the alleged shipyard and plant visits in Cavite Naval Base.

ISSUE:

Whether or not the contract of employment of Galvan valid even if the signatory
therein was not the Chairman of the Board.

RULING:

YES.

306

The contract of employment is valid. The contract remained valid even if the
signatory thereon was not the chairman of the board which allegedly violated
petitioners by-laws. Since by-laws operate merely as internal rules among the
stockholders, they cannot affect or prejudice third persons who deal with the
corporation, unless they have knowledge of the same. No proof appears on
record that private respondent ever knew anything about the provisions of the
said by-laws. In fact, petitioner itself merely asserts the same without even
bothering to attach a copy or excerpt thereof to show that there is such provision.
That this allegation has never been denied to private respondent nor necessarily
signify admission of its existence because technicalities of law and procedure
and the rules obtaining in the courts of law do not strictly apply to proceeding of
this nature.

307

Nakpil vs. Intercontinental Broadcasting Corporation , 379 SCRA 653


(2002)
FACTS
Petitioner Dily Dany Nacpil states that he was Asst. General Manager for
Finance/Administration and comptroller for International Broadcasting
Corporation from 1996 until 1997. When Emiliano Templo replaced the current
president of the corporation in March of 97, he told the Board of Directors that he
will terminate the services of the petitioner as he blames the latter for
mismanagement. Templo allegedly harassed the petitioner until the latter retired.
Despite the termination, the petitioners benefits was put on hold as he has not
yet secured clearance from Presidential Commission on Good Governance and
the Commission on Audit. In addition, Templo did not recognize the employment
of the petitioner as the latter only usurped the position. This led the petitioner to
file a complaint for illegal dismissal and non-payment of benefits with the NLRC.

IBC, in its position paper, contended that the case is outside the
jurisdiction of NLRC as the petitioner is a corporate officer duly elected by the
Board of Directors, therefore qualifies the case as intra-corporate dispute falling
within the jurisdiction of the Securities and Exchange Commission. NLRC denied
the position and the Labor Arbiter rendered a decision stating that the petitioner
has been illegally dismissed. CA reversed the said decision, hence, this petition.

ISSUE

WON the petitioner is a corporate officer

RULING

Yes. The petitioner is a corporate officer; therefore, the issue is within the
jurisdiction of SEC and not of NLRC.
Under Presidential Decree No. 902-A (the Revised Securities Act), the law in
force when the complaint for illegal dismissal was instituted by petitioner in 1997,
the following cases fall under the exclusive of the SEC:
c) Controversies in the election or appointment of directors, trustees, officers, or
managers of such corporations, partnerships or associations
Petitioner argues that he is not a corporate officer of the IBC but an employee
thereof since he had not been elected nor appointed as Comptroller and
Assistant Manager by the IBC's Board of Directors. He points out that he had
actually been appointed as such on January 11, 1995 by the IBC's General
Manager, Ceferino Basilio. In support of his argument, petitioner underscores the
308

fact that the IBC's By-Laws does not even include the position of comptroller in
its roster of corporate officers. He therefore contends that his dismissal is a
controversy falling within the jurisdiction of the labor courts.
Petitioners argument is untenable. Even assuming that he was in fact appointed
by the General Manager, such appointment was subsequently approved by the
Board of Directors of the IBC. That the position of Comptroller is not expressly
mentioned among the officers of the IBC in the By-Laws is of no moment,
because the IBC's Board of Directors is empowered under Section 25 of the
Corporation Code and under the corporation's By-Laws to appoint such other
officers as it may deem necessary.

An "office" has been defined as a creation of the charter of a corporation,


while an "officer" as a person elected by the directors or stockholders. On the
other hand, an "employee" occupies no office and is generally employed not by
action of the directors and stockholders but by the managing officer of the
corporation who also determines the compensation to be paid to such employee.
As petitioner's appointment as comptroller required the approval and formal
action of the IBC's Board of Directors to become valid, it is clear therefore holds
that petitioner is a corporate officer whose dismissal may be the subject of a
controversy cognizable by the SEC under Section 5(c) of P.D. 902-A which
includes controversies involving both election and appointment of corporate
directors, trustees, officers, and managers. Had petitioner been an ordinary
employee, such board action would not have been required.

309

Ong vs. Tiu, 401 SCRA 1 (2003)

Facts:
In 1994, the construction of the Masagana Citimall in Pasay City was threatened
with stoppage and incompletion when its owner, the First Landlink Asia
Development Corporation (FLADC), which was owned by David S. Tiu, Cely Y.
Tiu, Moly Yu Gow, Belen See Yu, D. Terence Y. Tiu, John Yu and Lourdes C. Tiu
(the Tius), encountered dire financial difficulties. It was heavily indebted to the
Philippine National Bank (PNB) for P190 million. To stave off foreclosure of the
mortgage on the two lots where the mall was being built, the Tius invited Ong
Yong, Juanita Tan Ong, Wilson T. Ong, Anna L. Ong, William T. Ong and Julia
Ong Alonzo (the Ongs), to invest in FLADC. Under the Pre-Subscription
Agreement they entered into, the Ongs and the Tius agreed to maintain equal
shareholdings in FLADC: the Ongs were to subscribe to 1,000,000 shares at a
par value of P100.00 each while the Tius were to subscribe to an additional
549,800 shares at P100.00 each in addition to their already existing subscription
of 450,200 shares. Furthermore, they agreed that the Tius were entitled to
nominate the Vice-President and the Treasurer plus 5 directors while the Ongs
were entitled to nominate the President, the Secretary and 6 directors (including
the chairman) to the board of directors of FLADC. Moreover, the Ongs were
given the right to manage and operate the mall. Accordingly, the Ongs paid P100
million in cash for their subscription to 1,000,000 shares of stock while the Tius
committed to contribute to FLADC a four-storey building and two parcels of land
respectively valued at P20 million (for 200,000 shares), P30 million (for 300,000
shares) and P49.8 million (for 49,800 shares) to cover their additional 549,800
stock subscription therein. The Ongs paid in another P70 million 3 to FLADC and
P20 million to the Tius over and above their P100 million investment, the total
sum of which (P190 million) was used to settle the P190 million mortgage
indebtedness of FLADC to PNB. The business harmony between the Ongs and
the Tius in FLADC, however, was shortlived because the Tius, on 23 February
1996, rescinded the Pre-Subscription Agreement. The Tius accused the Ongs of
(1) refusing to credit to them the FLADC shares covering their real property
contributions; (2) preventing David S. Tiu and Cely Y. Tiu from assuming the
positions of and performing their duties as Vice-President and Treasurer,
respectively, and (3) refusing to give them the office spaces agreed upon. The
controversy finally came to a head when the case was commenced by the Tius
on 27 February 1996 at the Securities and Exchange Commission (SEC),
seeking confirmation of their rescission of the Pre-Subscription Agreement. After
hearing, the SEC, through then Hearing Officer Rolando G. Andaya, Jr., issued a
decision on 19 May 1997 confirming the rescission sought by the Tius. On
motion of both parties, the above decision was partially reconsidered but only
insofar as the Ongs' P70 million was declared not as a premium on capital stock
but an advance (loan) by the Ongs to FLADC and that the imposition of interest
on it was correct. Both parties appealed to the SEC en banc which rendered a
decision on 11 September 1998, affirming the 19 May 1997 decision of the
Hearing Officer. The SEC en banc confirmed the rescission of the PreSubscription Agreement but reverted to classifying the P70 million paid by the
310

Ongs as premium on capital and not as a loan or advance to FLADC, hence, not
entitled to earn interest. On appeal, the Court of Appeals (CA) rendered a
decision on 5 October 1999, modifying the SEC order of 11 September 1998.
Their motions for reconsideration having been denied, both parties filed separate
petitions for review before the Supreme Court. On 1 February 2002, the Supreme
Court promulgated its Decision, affirming the assailed decision of the Court of
Appeals but with the modifications that the P20 million loan extended by the
Ongs to the Tius shall earn interest at 12% per annum to be computed from the
time of judicial demand which is from 23 April 1996; that the P70 million
advanced by the Ongs to the FLADC shall earn interest at 10% per annum to be
computed from the date of the FLADC Board Resolution which is 19 June 1996;
and that the Tius shall be credited with 49,800 shares in FLADC for their property
contribution, specifically, the 151 sq. m. parcel of land. The Court affirmed the
fact that both the Ongs and the Tius violated their respective obligations under
the Pre-Subscription Agreement.
On 15 March 2002, the Tius filed before the Court a Motion for Issuance of a Writ
of Execution. Aside from their opposition to the Tius' Motion for Issuance of Writ
of Execution, the Ongs filed their own "Motion for Reconsideration; Alternatively,
Motion for Modification (of the February 1, 2002 Decision)" on 15 March 2002.
Willie Ong filed a separate "Motion for Partial Reconsideration" dated 8 March
2002, pointing out that there was no violation of the Pre-Subscription Agreement
on the part of the Ongs, among others. On 29 January 2003, the Special Second
Division of this Court held oral arguments on the respective positions of the
parties. On 27 February 2003, Dr. Willie Ong and the rest of the movants Ong
filed their respective memoranda. On 28 February 2003, the Tius submitted their
memorandum.
ISSUE: Whether the pre-Subscription Agreement executed by the Ongs is
actually a subscription contract.
RULING: FLADC was originally incorporated with an authorized capital stock of
500,000 shares with the Tius owning 450,200 shares representing the paid-up
capital. When the Tius invited the Ongs to invest in FLADC as stockholders, an
increase of the authorized capital stock became necessary to give each group
equal (50-50) shareholdings as agreed upon in the Pre-Subscription Agreement.
The authorized capital stock was thus increased from 500,000 shares to
2,000,000 shares with a par value of P100 each, with the Ongs subscribing to
1,000,000 shares and the Tius to 549,800 more shares in addition to their
450,200 shares to complete 1,000,000 shares. Thus, the subject matter of the
contract was the 1,000,000 unissued shares of FLADC stock allocated to the
Ongs. Since these were unissued shares, the parties' Pre-Subscription
Agreement was in fact a subscription contract as defined under Section 60, Title
VII of the Corporation Code. A subscription contract necessarily involves the
corporation as one of the contracting parties since the subject matter of the
transaction is property owned by the corporation its shares of stock. Thus, the
subscription contract (denominated by the parties as a Pre-Subscription
Agreement) whereby the Ongs invested P100 million for 1,000,000 shares of
stock was, from the viewpoint of the law, one between the Ongs and FLADC, not
between the Ongs and the Tius. Otherwise stated, the Tius did not contract in
311

their personal capacities with the Ongs since they were not selling any of their
own shares to them. It was FLADC that did. Considering therefore that the real
contracting parties to the subscription agreement were FLADC and the Ongs
alone, a civil case for rescission on the ground of breach of contract filed by the
Tius in their personal capacities will not prosper. Assuming it had valid reasons to
do so, only FLADC (and certainly not the Tius) had the legal personality to file
suit rescinding the subscription agreement with the Ongs inasmuch as it was the
real party in interest therein. Article 1311 of the Civil Code provides that
"contracts take effect only between the parties, their assigns and heirs. . ."
Therefore, a party who has not taken part in the transaction cannot sue or be
sued for performance or for cancellation thereof, unless he shows that he has a
real interest affected thereby.

312

Ong vs. Tiu, ibid


Ponente: CORONA, J.:

Facts:
In 1994, the construction of Masagana City Mall in Pasay was threatened
to be stopped in incompletion when its owner, First Land Asia Development
Corporation (FLADC), which was owned by the Tius-herein respondents, was
indebted to Philippine National Bank for P190M. To save the business, the Tius
invited the Ongs to invest in FLADC. Under the Pre-Subscription Agreement they
entered into, the Ongs and the Tius agreed to maintain equal shareholdings in
FLADC: the Ongs were to subscribe to 1,000,000 shares at a par value of
P100.00 each while the Tius were to subscribe to an additional 549,800 shares
at P100.00 each in addition to their already existing subscription of 450,200
shares. Furthermore, they agreed that the Tius were entitled to nominate the
Vice-President and the Treasurer plus five directors while the Ongs were entitled
to nominate the President, the Secretary and six directors (including the
chairman) to the board of directors of FLADC. Moreover, the Ongs were given
the right to manage and operate the mall.

The Ongs paid P100M as their share while Tius contributed to FLADC a
four-storey building and two parcels of land. Ongs added P70M to their
investment in FLADC and paid the Tius another P20M. The cash contributions of
the Ongs, totaled at P190M, were used to pay off FLADCs outstanding debt.

By 1996, Tius moved to rescind the Pre-subsciption Agreement with the


Ongs as the latter would not comply with the agreement. According to them, the
Ongs refused to credit to them the real property contributions to FLADC, the
Ongs prevented the Tius from assuming positions and performing their duties as
VP and Treasurer, and refusing them to give office spaces agreed upon. The
Ongs, in their defense, clarified that the Tius assumed positions in FLADC as VP
and Treasurers but it was them that refused to perform the corporate duties.
Also, according to the Ongs, the Tius owned offices in the building even before
the Ongs came into the corporation making it illogical for the Tius to be left out of
office spaces. And most important, the real properties that were contributed by
the Tius were already the property of FLADC and the other, the Tius refused to
pay the capital tax gains and documentary tax stamps which would prevent SEC
from considering such properties as contributions.

SEC, through its hearing officer, confirmed the rescission sought after by
the Tius, as both are in mutual guilt for failure to perform their end of the
agreement. Both parties appealed on SEC en banc which affirmed the decision.
The Court of Appeals, all the same, affirmed the rescission of the agreement.
The Supreme Court initially affirmed the decision but upon reconsideration and
perusal of circumstances, the Court reviewed the decision.
313

Issue:
Whether the Tius could legally rescind the Pre-Subscription Agreement
Ruling:
No. The Tius cannot rescind the Pre-Subscription Agreement. A
subscription contract necessarily involves the corporation as one of the
contracting parties since the subject matter of the transaction is property owned
by the corporation its shares of stock. Thus, the subscription contract
(denominated by the parties as a Pre-Subscription Agreement) whereby the
Ongs invested P100 million for 1,000,000 shares of stock was, from the
viewpoint of the law, one between the Ongs and FLADC, not between the Ongs
and the Tius. Otherwise stated, the Tius did not contract in their personal
capacities with the Ongs since they were not selling any of their own shares to
them. It was FLADC that did. Considering therefore that the real contracting
parties to the subscription agreement were FLADC and the Ongs alone, a civil
case for rescission on the ground of breach of contract filed by the Tius in their
personal capacities will not prosper. Assuming it had valid reasons to do so, only
FLADC (and certainly not the Tius) had the legal personality to file suit rescinding
the subscription agreement with the Ongs inasmuch as it was the real party in
interest therein.

Although it has been proven that the Ongs prevented the Tius form holding
position, rescission is not the proper remedy as the Corporation Code, SEC rules
and even the Rules of Court provide for adequate and appropriate intra-corporate
remedies for the situation. A contrary doctrine will tread on extremely dangerous
ground because it will allow just any stockholder, for just about any real or
imagined offense, to demand rescission of his subscription and call for the
distribution of some part of the corporate assets to him without complying with
the requirements of the Corporation Code. Also, the rescission of the PreSubscription Agreement will effectively result in the unauthorized distribution of
the capital assets and property of the corporation, thereby violating the Trust
Fund Doctrine and the Corporation Code, since rescission of a subscription
agreement is not one of the instances when distribution of capital assets and
property of the corporation is allowed. The Tius' case for rescission cannot
validly be deemed a petition to decrease capital stock because such action never
complied with the formal requirements for decrease of capital stock under
Section 33 of the Corporation Code. No majority vote of the board of directors
was ever taken. Neither was there any stockholders meeting at which the
approval of stockholders owning at least two-thirds of the outstanding capital
stock was secured. There was no revised treasurer's affidavit and no proof that
the said decrease will not prejudice the creditors' rights. On the contrary, all their
314

pleadings contained were alleged acts of violations by the Ongs to justify an


order of rescission.
Stripped to its barest essentials, the issue of rescission in this case is not
difficult to understand. If rescission is denied, will injustice be inflicted on any of
the parties? The answer is no because the financial interests of both the Tius and
the Ongs will remain intact and safe within FLADC. On the other hand, if
rescission is granted, will any of the parties suffer an injustice? Definitely yes
because the Ongs will find themselves out in the streets with nothing but the
money they had in 1994 while the Tius will not only enjoy a windfall estimated to
be anywhere from P450 million to P900 million but will also take over an
extremely profitable business without much effort at all.
The Ongs are also correct in not accepting the real property contributions of the
Tius as titles cannot be transferred to FLADCs name, the other lot is already a
property of FLADC and was factored before the Ong came into the corporation.

Without the Ongs, the Tius would have lost everything they originally
invested in said mall. If only for this and the fact that this Resolution can truly
pave the way for both groups to enjoy the fruits of their investments assuming
good faith and honest intentions the Court cannot allow the rescission of the
subject subscription agreement. The Ongs' shortcomings were far from serious
and certainly less than substantial; they were in fact remediable and correctable
under the law. It would be totally against all rules of justice, fairness and equity to
deprive the Ongs of their interests on petty and tenuous grounds.

315

Baltazar vs. Lingayen Gulf Electric Power Company, Inc., 14 SCRA 522
(1965)
Ponente: Justice Paredes
Facts:

Ireneo Baltazar subscribed 600 and Marvin Rose, 400 shares, to the Lingayen
Gulf Electric Power Co., Inc. who was doing business in the Philippines with their
principal office at Lingayen, Pangasinan. The said corporation has a total of
P300,000.00 authorized capital stock divided into 3,00 shares of voting stock at
P100.00 par value, per share. It is alleged that it has always been the practice
and procedure of the Corporation to issue certificates of stock to its individual
subscribers for unpaid shares of stock.
Of the 600, only 535 shares of stock of Baltazar were fully paid and only 341
shares remained during the trial. He had also 65 shares with par value of
P6,500.00, for which no certificate was issued to him. As for Rose, out of 400
shares, only 345 fully paid stock duly covered by certificates of stock issued to
him.
Ungson, Estrada, Fernandez and Yuson were also stockholders of hodling not
more than 100 fully paid shares of stock. Acena, on the other hand, has 600 fully
paid shares of stocks. During the stockholders meeting, the Ungson group
(including Acena) and the Baltazar group became members of the Board.
In the year of 1955, there was a fight for the control of the management of the
corporation and the annual stockholders meeting which was supposed to be
held every first Tuesday of February was moved to May 1, 1955. To retain the
control, the Ungson group passed three (3) resolutions on January 30, 1955. The
said resolutions became a threat to Baltazar group and deprived them of their
right to vote for the May 1, 1955 Board Meeting due to their unpaid shares of
stocks.

Issues:

Whether a stockholder is entitled to vote, notwithstanding the fact that he has not
paid the balance of his subscription, which has been called for payment or
declared delinquent.

Ruling:

Yes. It was the practice and procedure of the corporation to issue certificates of
stock to its individual subscribers for unpaid shares of stock and gave voting
powers to shares of stock fully paid.
316

Gonzales vs. Philippine National Bank, 122 SCRA 489 (1983)


Ponente: Justice Vasquez
Facts:
Ramon A. Gonzales initially instituted several cases in the Supreme Court
questioning different transactions entered into by the Bank with other parties.
First among them is Civil Case 69345 filed on 27 April 1967, by Gonzales as a
taxpayer versus Sec. Antonio Raquiza of Public Works and Communications, the
Commissioner of Public Highways, the Bank, Continental Ore Phil., Inc.,
Continental Ore, Huber Corporation, Allis Chalmers and General Motors
Corporation. In the course of the hearing of said case on 3 August 1967, the
personality of Gonzales to sue the bank and question the letters of credit it has
extended for the importation by the Republic of the Philippines of public works
equipment intended for the massive development program of the President was
raised. In view thereof, he expressed and made known his intention to acquire
one share of stock from Congressman Justiniano Montano which, on the
following day, 30 August 1967, was transferred in his name in the books of the
Bank. Subsequent to his aforementioned acquisition of one share of stock of the
Bank, Gonzales, in his dual capacity as a taxpayer and stockholder, filed the
following cases involving the bank or the members of its Board of Directors to wit:
(1) On 18 October 1967, Civil Case 71044 versus the Board of Directors of the
Bank; the National Investment and Development Corp., Marubeni Iida Co., Ltd.,
and Agro-Inc. Dev. Co. or Saravia; (2) On 11 May 1968, Civil Case 72936 versus
Roberto Benedicto and other Directors of the Bank, Passi (Iloilo) Sugar Central,
Inc., Calinog-Lambunao Sugar Mill Integrated Farming, Inc., Talog sugar Milling
Co., Inc., Safary Central, Inc., and Batangas Sugar Central Inc.; and (3) On 8
May 1969, Civil Case 76427 versus Alfredo Montelibano and the Directors of
both the PNB and DBP.
On 11 January 1969, however, Gonzales addressed a letter to the
President of the Bank, requesting submission to look into the records of its
transactions covering the purchase of a sugar central by the Southern Negros
Development Corp. to be financed by Japanese suppliers and financiers; its
financing of the Cebu-Mactan Bridge to be constructed by V.C. Ponce, Inc. and
the construction of the Passi Sugar Mills in Iloilo. On January 23, 1969, the Asst.
Vice President and Legal Counsel of the Bank answered petitioner's letter
denying his request for being not germane to his interest as a one share
stockholder and for the cloud of doubt as to his real intention and purpose in
acquiring said share. In view of the Bank's refusal, Gonzales instituted the
317

petition for mandamus. The Court of First Instance of Manila denied the prayer of
Gonzales that he be allowed to examine and inspect the books and records of
PNB regarding the transactions mentioned on the grounds that the right of a
stockholder to inspect the record of the business transactions of a corporation
granted under Section 51 of the former Corporation Law (Act No. 1459, as
amended) is not absolute, but is limited to purposes reasonably related to the
interest of the stockholder, must be asked for in good faith for a specific and
honest purpose and not gratify curiosity or for speculative or vicious purposes;
that such examination would violate the confidentiality of the records of the bank
as provided in Section 16 of its charter, RA 1300, as amended; and that
Gonzales has not exhausted his administrative remedies. Gonzales filed the
petition for review.

Issues:
1. Whether Gonzales' can ask for an examination of the books and records of
PNB, in light of his ownership of one share in the bank.
2. Whether the inspection sought to be exercised by Gonzales would be
violative of the provisions of PNB's charter.
Ruling:
1. The unqualified provision on the right of inspection previously contained in
Section 51, Act No. 1459, as amended, no longer holds true under the provisions
of the present law. The argument of Gonzales that the right granted to him under
Section 51 of the former Corporation Law should not be dependent on the
propriety of his motive or purpose in asking for the inspection of the books of
PNB loses whatever validity it might have had before the amendment of the law.
If there is any doubt in the correctness of the ruling of the trial court that the right
of inspection granted under Section 51 of the old Corporation Law must be
dependent on a showing of proper motive on the part of the stockholder
demanding the same, it is now dissipated by the clear language of the pertinent
provision contained in Section 74 of Batas Pambansa Bilang 68. Although
Gonzales has claimed that he has justifiable motives in seeking the inspection of
the books of the PNB, he has not set forth the reasons and the purposes for
which he desires such inspection, except to satisfy himself as to the truth of
published reports regarding certain transactions entered into by the respondent
bank and to inquire into their validity. The circumstances under which he
318

acquired one share of stock in the PNB purposely to exercise the right of
inspection do not argue in favor of his good faith and proper motivation.
Admittedly he sought to be a stockholder in order to pry into transactions entered
into by the PNB even before he became a stockholder. His obvious purpose was
to arm himself with materials which he can use against the PNB for acts done by
the latter when Gonzales was a total stranger to the same. He could have been
impelled by a laudable sense of civic consciousness, but it could not be said that
his purpose is germane to his interest as a stockholder.

3. Section 15 of the PNB's Charter (RA 1300, as amended) provides that


"Inspection by Department of Supervision and Examination of the Central
Bank. The National Bank shall be subject to inspection by the
Department of Supervision and Examination of the Central Bank." Section
16 thereof providest that "Confidential information. The Superintendent
of Banks and the Auditor General, or other officers designated by law to
inspect or investigate the condition of the National Bank, shall not reveal to
any person other than the President of the Philippines, the Secretary of
Finance, and the Board of Directors the details of the inspection or
investigation, nor shall they give any information relative to the funds in its
custody, its current accounts or deposits belonging to private individuals,
corporations, or any other entity, except by order of a Court of competent
jurisdiction." On the other hand, Section 30 of the same provides that
"Penalties for violation of the provisions of this Act. Any director, officer,
employee, or agent of the Bank, who violates or permits the violation of
any of the provisions of this Act, or any person aiding or abetting the
violations of any of the provisions of this Act, shall be punished by a fine
not to exceed ten thousand pesos or by imprisonment of not more than five
years, or both such fine and imprisonment." The Philippine National Bank
is not an ordinary corporation. Having a charter of its own, it is not
governed, as a rule, by the Corporation Code of the Philippines. The
provision of Section 74 of Batas Pambansa Blg. 68 of the new Corporation
Code with respect to the right of a stockholder to demand an inspection or
examination of the books of the corporation may not be reconciled with the
above quoted provisions of the charter of the PNB. It is not correct to
claim, therefore, that the right of inspection under Section 74 of the new
Corporation Code may apply in a supplementary capacity to the charter of
the PNB

319

Datu Tagoranao Benito vs. Securities & Exchange Commission 123 SCRA
722 (1983)
Relova, J.;

FACTS:

On February 6, 1959, the Articles of Incorporation of respondent Jamiatul


Philippine-Al Islamia, Inc. (originally Kamilol Islam Institute, Inc.) were filed with
the Securities and Exchange Commission (SEC) and were approved on
December 14, 1962. The corporation had an authorized capital stock of
P200,000.00 divided into 20,000 shares at a par value of P10.00 each. Of the
authorized capital stock, 8,058 shares worth P80,580.00 were subscribed and
fully paid for. Petitioner Datu Tagoranao Benito subscribed to 460 shares worth
P4,600.00.
On October 28, 1975, the respondent corporation filed a certificate of increase of
its capital stock from P200,000.00 to P1,000,000.00. Thus, P110,980.00 worth of
shares were subsequently issued by the corporation from the unissued portion of
the authorized capital stock of P200,000.00. Of the increased capital stock of
P1,000,000.00, P160,000.00 worth of shares were subscribed by Mrs. Fatima A.
Ramos, Mrs. Tarhata A. Lucman and Mrs. Moki-in Alonto.
Petitioner Datu Tagoranao filed a petition alleging that the additional issue (worth
P110,980.00) of previously subscribed shares of the corporation was made in
violation of his pre-emptive right to said additional issue and that the increase in
the authorized capital stock of the

corporation from

P200,000.00 to

P1,000,000.00 was illegal considering that the stockholders of record were not
notified of the meeting wherein the proposed increase was in the agenda.
Respondents denied the material allegations of the petition and claimed that
petitioner has no cause of action and that the stock certificates covering the
shares alleged to have been sold to petitioner were only given to him as
collateral for the loan of Domocao Alonto and Moki-in Alonto. The SEC affirmed
the sale.

ISSUE:

Whether or not the issuance of the unissued shares was subject to the preemptive right of the stockholders.
320

RULING:

NO.

The Court held that the questioned issuance of the unsubscribed portion of the
capital stock worth P110,980.00 is not invalid even if assuming that it was made
without notice to the stockholders as claimed by petitioner. The power to issue
shares of stocks in a corporation is lodged in the board of directors and no
stockholders' meeting is necessary to consider it because additional issuance of
shares of stocks does not need approval of the stockholders.
Petitioner bewails the fact that in view of the lack of notice to him of such
subsequent issuance, he was not able to exercise his right of pre-emption over
the unissued shares. However, the general rule is that pre-emptive right is
recognized only with respect to new issue of shares, and not with respect to
additional issues of originally authorized shares. This is on the theory that when a
corporation at its inception offers its first shares, it is presumed to have offered all
of those which it is authorized to issue. An original subscriber is deemed to have
taken his shares knowing that they form a definite proportionate part of the whole
number of authorized shares. When the shares left unsubscribed are later reoffered, he cannot therefore claim a dilution of interest.

321

Edward A. Keller 7 Co., Ltd. vs. COB Group Marketing, Inc., 141 SCRA
86 (1986)

AQUINO, C.J.:

FACTS:

Edward A. Keller & Co., Ltd. appointed COB Group Marketing, Inc. as exclusive
distributor of its household products, Brite and Nuvan in Panay and Negros, as
shown in the sales agreement dated March 14, 1970 . Under that agreement
Keller sold on credit its products to COB Group Marketing.

As security for COB Group Marketing's credit purchases up to the amount of


P35,000, one Asuncion Manahan mortgaged her land to Keller. Manahan
assumed solidarily with COB Group Marketing the faithful performance of all the
terms and conditions of the sales agreement.

In July, 1970 the parties executed a second sales agreement whereby COB
Group Marketing's territory was extended to Northern and Southern Luzon. As
security for the credit purchases up to P25,000 of COB Group Marketing for that
area, Tomas C. Lorenzo, Jr. and his father Tomas, Sr. (now deceased) executed
a mortgage on their land in Nueva Ecija. Like Manahan, the Lorenzos were
solidarily liable with COB Group Marketing for its obligations under the sales
agreement.

The credit purchases of COB Group Marketing, which started on October 15,
1969, limited up to January 22, 1971. On May 8, the board of directors of COB
Group Marketing were apprised by Jose E. Bax the firm's president and general
manager, that the firm owed Keller about P179,000. Bax was authorized to
negotiate with Keller for the settlement of his firm's liability. On the same day,
May 8, Bax and R. Oefeli of Keller signed the conditions for the settlement of
COB Group Marketing's liability. Twelve days later, or on May 20, COB Group
Marketing, through Bax executed two second chattel mortgages over its 12
trucks (already mortgaged to Northern Motors, Inc.) as security for its obligation
to Keller amounting to P179,185.16 as of April 30, 1971. The stockholders of
COB Group Marketing, Moises P. Adao and Tomas C. Lorenzo, Jr., in a letter
dated July 24, 1971 to Keller's counsel, proposed to pay Keller P5,000 on
November 30, 1971 and thereafter every thirtieth day of the month for three
322

years until COB Group Marketing's mortgage obligation had been fully satisfied.
They also proposed to substitute the Manahan mortgage with a mortgage on
Adao's lot at 72 7th Avenue, Cubao, Quezon City

ISSUE:

Whether the lower courts erred in nullifying the admissions of liability made in
1971 by Bax as president and general manager of COB Group Marketing and in
giving credence to the alleged overpayment computed by Bax. As to the
stockholders, the extent of financial liability they m ay shoulder.

HELD:

YES.

The lower courts not only allowed Bax to nullify his admissions as to the liability
of COB Group Marketing but they also erroneously rendered judgment in its favor
in the amount of its supposed overpayment in the sum of P100,596.72, in spite of
the fact that COB Group Marketing was declared in default and did not file any
counterclaim for the supposed overpayment. The lower courts harped on Keller's
alleged failure to thresh out with representatives of COB Group Marketing their
"diverse statements of credits and payments". This contention has no factual
basis. That means that there was a conference on the COB Group Marketing's
liability. Bax in that discussion did not present his reconciliation statements to
show overpayment.

Bax admitted that Keller sent his company monthly statements of accounts but
he could not produce any formal protest against the supposed inaccuracy of the
said statements. He lamely explained that he would have to dig up his company's
records for the formal protest. He did not make any written demand for
reconciliation of accounts.

As to the liability of the stockholders, it is settled that a stockholder is personally


liable for the financial obligations of a corporation to the extent of his unpaid
subscription.

323

Remo vs. Intermediate Appellate Court, 172 SCRA 405 (1989)


FACTS:

The board of directors of Akron Customs Brokerage Corporation adopted a


resolution authorizing the purchase of thirteen (13) trucks for use in its business
to be paid out of a loan the corporation may secure from any lending institution.

Feliciano Coprada, as President and Chairman of Akron, purchased thirteen


trucks from private respondent on January 25, 1978 for and in consideration of
P525,000.00 as evidenced by a deed of absolute sale. In a side agreement of the
same date, the parties agreed on a downpayment in the amount of P50,000.00
and that the balance of P475,000.00 shall be paid within sixty (60) days from the
date of the execution of the agreement. The parties also agreed that until said
balance is fully paid, the down payment of P50,000.00 shall accrue as rentals of
the 13 trucks; and that if Akron fails to pay the balance within the period of 60
days, then the balance shall constitute as a chattel mortgage lien covering said
cargo trucks and the parties may allow an extension of 30 days and thereafter
private respondent may ask for a revocation of the contract and the
reconveyance of all said trucks.

The obligation is further secured by a promissory note executed by Coprada in


favor of Akron. It is stated in the promissory note that the balance shall be paid
from the proceeds of a loan obtained from the Development Bank of the
Philippines (DBP) within sixty (60) days. After the lapse of 90 days, private
respondent tried to collect from Coprada but the latter promised to pay only upon
the release of the DBP loan.

Private respondent sent Coprada a letter of demand dated May 10, 1978.9 In his
reply to the said letter, Coprada reiterated that he was applying for a loan from
the DBP from the proceeds of which payment of the obligation shall be made.
Meanwhile, two of the trucks were sold under a pacto de retro sale to a certain
Mr. Bais of the Perpetual Loans and Savings Bank at Baclaran. The sale was
authorized by a board resolution made in a meeting held on March 15, 1978.
Upon inquiry, private respondent found that no loan application was ever filed by
Akron with DBP. In the meantime, Akron paid rentals of P500.00 a day pursuant
to a subsequent agreement, from April 27, 1978 (the end of the 90-day period to
pay the balance) to May 31, 1978. Thereafter, no more rental payments were
made.

On June 17, 1978, Coprada wrote private respondent begging for a grace period
of until the end of the month to pay the balance of the purchase price; that he will
324

update the rentals within the week; and in case he fails, then he will return the 13
units should private respondent elect to get back the same. Private respondent,
through counsel, wrote Akron on August 1, 1978 demanding the return of the 13
trucks and the payment of P25,000.00 back rentals covering the period from
June 1 to August 1, 1978.14

Again, Coprada wrote private respondent on August 8, 1978 asking for another
grace period of up to August 31, 1978 to pay the balance, stating as well that he
is expecting the approval of his loan application from a certain financing
company, and that ten (10) trucks have been returned to Bagbag, Novaliches.15
On December 9, 1978, Coprada informed private respondent anew that he had
returned ten (10) trucks to Bagbag and that a resolution was passed by the board
of directors confirming the deed of assignment to private respondent of P475,000
from the proceeds of a loan obtained by Akron from the State Investment House,
Inc.16

In due time, private respondent filed a compliant for the recovery of P525,000.00
or the return of the 13 trucks with damages against Akron and its officers and
directors, with the then Court of First Instance of Rizal. Only petitioner answered
the complaint denying any participation in the transaction and alleging that Akron
has a distinct corporate personality.

In the meanwhile, petitioner sold all his shares in Akron to Coprada. It also
appears that Akron amended its articles of incorporation thereby changing its
name to Akron Transport International, Inc. which assumed the liability of Akron
to private respondent.
Court of First Instance favored the complaint of plaintiff thus it went ot IAC. A
motion for new trial filed by petitioner was denied so he appealed to the then
Intermediate Appellate Court (IAC). However, upon a motion for reconsideration
filed by private respondent, the IAC, in a resolution dated February 8, 1984, set
aside the decision dated June 30, 1983. The appellate court entered another
decision affirming the appealed decision of the trial court, with costs against
petitioner.

Issues:

I. The Intermediate Appellate Court (IAC) erred in disregarding the corporate


fiction and in holding the petitioner personally liable for the obligation of the
Corporation which decision is patently contrary to law and the applicable decision
thereon.

325

II. The Intermediate Appellate Court (IAC) committed grave error of law in its
decision by sanctioning the merger of the personality of the corporation with that
of the petitioner when the latter was held liable for the corporate debts.

Ruling:
The Court reverse.

# A corporation is an entity separate and distinct from its stockholders; Corporate


fiction.A corporation is an entity separate and distinct from its stockholders.
While not in fact and in reality a person, the law treats a corporation as though it
were a person by process of fiction or by regarding it as an artificial person
distinct and separate from its individual stockholders. However, the corporate
fiction or the notion of legal entity may be disregarded when it is used to defeat
public convenience, justify wrong, protect fraud, or defend crime in which
instances the law will regard the corporation as an association of persons, or in
case of two corporations, will merge them into one. The corporate fiction may
also be disregarded when it is the mere alter ego or business conduit of a
person. There are many occasions when this Court pierced the corporate veil
because of its use to protect fraud and to justify wrong.

The environmental facts of this case show that there is no cogent basis to pierce
the corporate veil of Akron and hold petitioner personally liable for its obligation
to private respondent. While it is true that in December, 1977 petitioner was still a
member of the board of directors of Akron and that he participated in the
adoption of a resolution authorizing the purchase of 13 trucks for the use in the
brokerage business of Akron to be paid out of a loan to be secured from a
lending institution, it does not appear that said resolution was intended to defraud
anyone and more particularly private respondent. It was Coprada, President and
Chairman of Akron, who negotiated with said respondent for the purchase of 13
cargo trucks on January 25, 1978. It was Coprada who signed a promissory note
to guarantee the payment of the unpaid balance of the purchase price out of the
proceeds of a loan he supposedly sought from the DBP. The word WE in the
said promissory note must refer to the corporation which Coprada represented in
the execution of the note and not its stockholders or directors. Petitioner did not
sign the said promissory note so he cannot be personally bound thereby.

As to the sale through pacto de retro of two units to a third person by the
corporation by virtue of a board resolution, petitioner asserts that he never signed
said resolution. Be that as it may, the sale is not inherently fraudulent as the 13
units were sold through a deed of absolute sale to Akron so that the corporation
is free to dispose of the same. Of course, it was stipulated that in case of default
326

in payment to private respondent of the balance of the consideration, a chattel


mortgage lien shall be constituted on the 13 units. Nevertheless, said mortgage
is a prior lien as against the pacto de retro sale of the 2 units. Amendment of
articles of incorporation thereby changing the name of the corporation is not an
indication to evade payment by the corporation of its obligations to another.
As to the amendment of the articles of incorporation of Akron thereby changing
its name to Akron Transport International, Inc., petitioner alleges that the change
of corporate name was in order to include trucking and container yard operations
in its customs brokerage of which private respondent was duly informed in a
letter. Indeed, the new corporation confirmed and assumed the obligation of the
old corporation. There is no indication of an attempt on the part of Akron to evade
payment of its obligation to private respondent. A stockholder has an inherent
right to dispose of his shares of stock anytime he so desires.There is the fact
that petitioner sold his shares in Akron to Coprada during the pendency of the
case. Since petitioner has no personal obligation to private respondent, it is his
inherent right as a stockholder to dispose of his shares of stock anytime he so
desires. If private respondent is the victim of fraud, there was no showing that the
corporation had any participation in the perpetration of the fraud; Fraud must be
established by clear and convincing evidence.Mention is also made of the
alleged dumping of 10 units in the premises of private respondent at Bagbag,
Novaliches which to the mind of the Court does not prove fraud and instead
appears to be an attempt on the part of Akron to attend to its obligations as
regards the said trucks. Again petitioner has no part in this. If the private
respondent is the victim of fraud in this transaction, it has not been clearly shown
that petitioner had any part or participation in the perpetration of the same. Fraud
must be established by clear and convincing evidence. If at all, the principal
character on whom fault should be attributed is Feliciano Coprada, the President
of Akron, whom private respondent dealt with personally all through out.
Fortunately, private respondent obtained a judgment against him from the trial
court and the said judgment has long been final and executory.

327

Saw vs. Court of Appeals, 195 SCRA 797. (1991)


CRUZ, J.:
Facts: A collection suit with preliminary attachment was filed by Equitable
Banking Corporation against Freeman, Inc. and Saw Chiao Lian.
The petitioners, as stockholders of Freeman moved to intervene.
The motion to intervene was denied, and the petitioners appealed to the Court of
Appeals.
Meanwhile, Equitable and Saw Chiao Lian entered into a compromise agreement
which was approved by the lower court. But because it was not complied with,
Equitable secured a writ of execution, and two lots owned by Freeman, Inc. were
levied upon and sold at public auction.
The Court of Appeals sustained the denial of the petitioners' motion for
intervention holding that "the compromise agreement will not necessarily
prejudice petitioners whose rights to corporate assets are at most inchoate and
the motion is not proper in intervention under Sec. 2, Rule 12 of the Revised
Rules of Court.
The petitioners are now before this Court.
Issues:

a) Do petitioners

as stockholders of Freeman have the right to

intervene?
b) Is the Court of Appeals correct in ruling that the notice of appeal
concerns only the denial of the motion for intervention and not the whole
case?
Ruling: a) After examining the issues and arguments of the parties, the Court
finds that the respondent court committed no reversible error in sustaining the
denial by the trial court of the petitioners' motion for intervention.
In the case of Magsaysay-Labrador v. Court of Appeals, we ruled as follows:
Viewed in the light of Section 2, Rule 12 of the Revised Rules of Court, this
Court affirms the respondent court's holding that petitioners herein have no
legal interest in the subject matter in litigation so as to entitle them to
intervene in the proceedings below. In the case of Batama Farmers'
328

Cooperative Marketing Association, Inc. v. Rosal, we held: "As clearly


stated in Section 2 of Rule 12 of the Rules of Court, to be permitted to
intervene in a pending action, the party must have a legal interest in the
matter in litigation, or in the success of either of the parties or an interest
against both, or he must be so situated as to be adversely affected by a
distribution or other disposition of the property in the custody of the court or
an officer thereof."
To allow intervention, [a] it must be shown that the movant has legal
interest in the matter in litigation, or otherwise qualified; and [b]
consideration must be given as to whether the adjudication of the rights of
the original parties may be delayed or prejudiced, or whether the
intervenor's rights may be protected in a separate proceeding or not. Both
requirements must concur as the first is not more important than the
second.
The interest which entitles a person to intervene in a suit between other
parties must be in the matter in litigation and of such direct and immediate
character that the intervenor will either gain or lose by the direct legal
operation and effect of the judgment. Otherwise, if persons not parties of
the action could be allowed to intervene, proceedings will become
unnecessarily complicated, expensive and interminable. And this is not the
policy of the law.
The words "an interest in the subject" mean a direct interest in the cause of
action as pleaded, and which would put the intervenor in a legal position to
litigate a fact alleged in the complaint, without the establishment of which
plaintiff could not recover.
Here, the interest, if it exists at all, of petitioners-movants is indirect,
contingent, remote, conjectural, consequential and collateral. At the very
least, their interest is purely inchoate, or in sheer expectancy of a right in
the management of the corporation and to share in the profits thereof and
in the properties and assets thereof on dissolution, after payment of the
corporate debts and obligations.
While a share of stock represents a proportionate or aliquot interest in the
property of the corporation, it does not vest the owner thereof with any
legal right or title to any of the property, his interest in the corporate
property being equitable or beneficial in nature. Shareholders are in no
329

legal sense the owners of corporate property, which is owned by the


corporation as a distinct legal person.
b) On the second assignment of error, the respondent court correctly noted that
the notice of appeal was filed by the petitioners on October 24, 1988, upon the
denial of their motion to intervene, and the writ of execution was issued by the
lower court on January 30, 1989. The petitioners' appeal could not have
concerned the "whole" case (referring to the decision) because the petitioners
"did not appeal the decision as indeed they cannot because they are not parties
to the case despite their being stockholders of respondent Freeman, Inc." They
could only appeal the denial of their motion for intervention as they were never
recognized by the trial court as party litigants in the main case.
Intervention is "an act or proceeding by which a third person is permitted to
become a party to an action or proceeding between other persons, and which
results merely in the addition of a new party or parties to an original action, for
the purpose of hearing and determining at the same time all conflicting claims
which may be made to the subject matter in litigation.
It is not an independent proceeding, but an ancillary and supplemental one
which, in the nature of things, unless otherwise provided for by the statute or
Rules of Court, must be in subordination to the main proceeding. It may be laid
down as a general rule that an intervenor is limited to the field of litigation open to
the original parties.
In the case at bar, there is no more principal action to be resolved as a writ
of execution had already been issued by the lower court and the claim of
Equitable had already been satisfied. The decision of the lower court had
already become final and in fact had already been enforced. There is
therefore no more principal proceeding in which the petitioners may
intervene.

330

Africa vs. Presidential Commission on Good Government


205 SCRA 39 (1992)
Facts: On July 22, 1987, the petitioner Republic of the Philippines, through the
Presidential Commission on Good Government (PCGG), filed a complaint
against Jose L. Africa, Manuel H. Nieto, Jr., Ferdinand E. Marcos, Imelda R.
Marcos, Ferdinand R. Marcos, Jr., Juan Ponce Enrile, and Potenciano Ilusorio
(collectively, the

respondents)

for reconveyance,

reversion,

accounting,

restitution,and damages before the Sandiganbayan. The petitioner alleged that


the respondents illegally manipulated the purchase of the major shareholdings of
Cable and Wireless Limited in Eastern Telecommunications Philippines, Inc.
(ETPI), which shareholdings respondents Jose Africa and Manuel Nieto, Jr. held
for themselves and, through their holdings and the corporations they organized,
beneficially for respondents Ferdinand E. Marcos and Imelda R. Marcos. Africa,
as an ETPI stockholder, filed a petition for certiorari, with prayer for a temporary
restraining order/preliminary injunction with the Sandiganbayan seeking to
nullify the order of the Sandiganbayan for the former to account for his
sequestered shares in ETPI and to cease and desist from exercising voting
rights. During the pendency of Africas petition, Africa filed a motion with the
Sandiganbayan, alleging that the PCGG had been illegally exercising the rights
of stockholders of ETPI, especially in the election of the members of the board of
directors and prayed for the issuance of an order for the calling and holding of
[ETPI] annual stockholders meeting under the courts control and supervision.
Sandiganbayan favored Africas motion.The PCGG assailed this resolution
before this Court via a petition for certiorari docketed imputing grave abuse of
discretion on the Sandiganbayan for holding, inter alia, that the registered
stockholders of ETPI had the right to vote.To resolve the Urgent Petition, the
testimony of Mr. Maurice V. Bane (former director and treasurer-in-trust of ETPI)
was taken at the petitioners instance for the purpose for Bane to identify and
testify on the facts in his affidavit so as to prove the ownership issue in favor of
the petitioner and/or establish the prima facie factual foundation for sequestration
of ETPIs Class A stock in support of the Urgent Petition The Sandiganbayan
granted the Urgent Petition hence, Africa went to this Court via a petition for
certiorari docketed as G.R. No. 147214 (Africas petition). The court resolved the
PCGG and Africas petition and were REFERRED to the Sandiganbayan for
reception of evidence and to decide it within 60 days from receipt of this
resolution.The petitioners did not question the 1998 resolution, and instead they
made its Formal Offer of Evidence. Significantly, the Bane deposition was not
331

included as part of its offered exhibits. In order to correct this, they filed a second
motion with prayer for re-opening of the case for the purpose of introducing
additional evidence and requested the court to take judicial notice of the facts
established by the Bane deposition. This was denied by the Sandiganbayan in
its November 6, 2000 resolution. A third motion was filed by the petitioners on
November 16, 2001 seeking once more to admit the Bane deposition which the
Sandiganbayan

for

the

reason

that without

plaintiff

having

moved

for

reconsideration within the reglementary period, the resolution has attained


finality and its effect cannot be undone by the simple expedient of filing a motion.

Issue: Whether the Sandiganbayan committed grave abuse of jurisdiction on


holding that the 1998 resolution already attained finality and that the 3rd motion
for reconsideration of the petitioner is prohibited.

Held: Judgment or order is considered final if the order disposes of the action or
proceeding completely, or terminates a particular stage of the same action; in
such case, the remedy available to an aggrieved party is appeal. If the order or
resolution, however, merely resolves incidental matters and leaves something
more to be done to resolve the merits of the case, the order is interlocutory and
the aggrieved partys remedy is a petition for certiorari under Rule 65. Therefore,
the 1998 resolution is interlocutory.

The Sandiganbayans denial of the

petitioners 1st motion through the 1998 Resolution came at a time when the
petitioner had not even concluded the presentation of its evidence. Plainly, the
denial of the motion did not resolve the merits of the case, as something still had
to

be

done

to

achieve

this

end.

the Sandiganbayans 1998 resolution which merely denied the adoption of the
Bane deposition as part of the evidence in Civil Case No. 0009 could not have
attained finality.The Sandiganbayan undoubtedly erred on a question of law in its
ruling, but this legal error did not necessarily amount to a grave abuse of
discretion in the absence of a clear showing that its action was a capricious and
whimsical exercise of judgment affecting its exercise of jurisdiction. Without this
showing, the Sandiganbayans erroneous legal conclusion was only an error of
judgment, or, at best, an abuse of discretion but not a grave one.The
3rd motion could not also be considered as a prohibited motion because Section
5, Rule 37 of the Rules of Court clearly provides, the proscription against a
second motion for reconsideration is directed against a judgment or final order.
But a second motion for reconsideration of an interlocutory order can be denied
332

on the ground that it is discusses again the arguments already passed upon and
resolved by the court. In this case, the latter is the reason cited by the
respondents for the denial of the motion

333

Lee vs. Court of appeals


205 SCRA 752 (1992)

A trust receipt is considered as a security transaction intended to aid in financing


importers and retail dealers who do not have sufficient funds or resources to
finance the importation or purchase of merchandise, and who may not be able to
acquire credit except through utilization, as collateral of the merchandise
imported or purchased.
Facts:
Charles Lee, as President of MICO wrote private respondent Philippine Bank of
Communications (PBCom) requesting for a grant of a discounting loan/credit line
in the sum of Three Million Pesos (P3,000,000.00) for the purpose of carrying out
MICOs line of business as well as to maintain its volume of business. On the
same day, Charles Lee requested for another discounting loan/credit line of
Three Million Pesos (P3,000,000.00) from PBCom for the purpose of opening
letters of credit and trust receipts. nother loan of One Million Pesos
(P1,000,000.00) was availed of by MICO from PBCom which was likewise later
on renewed.

Charles Lee, Chua Siok Suy, Mariano Sio, Alfonso Yap and

Richard Velasco, in their personal capacities executed a Surety Agreement in


favor of PBComwhereby the petitioners jointly and severally, guaranteed the
prompt payment on due dates or at maturity of overdrafts, promissory notes,
discounts, drafts, letters of credit, bills of exchange, trust receipts, and other
obligations of every kind and nature, for which MICO may be held accountable by
PBCom. Charles Lee, in his capacity as president of MICO, wrote PBCom and
applied for an additional loan in the sum of Four Million Pesos (P4,000,000.00).
The loan was intended for the expansion and modernization of the companys
machineries. Upon approval of the said application for loan, MICO availed of the
additional loan of Four Million Pesos (P4,000,000.00).
To secure the trust receipts transactions, MICO and Lee executed a real estate
mortgage in favor of PBCOM over several properties it owns. Upon maturity of
all credit availments obtained by MICO from PBCom, the latter made a demand
for payment.[For failure of petitioner MICO to pay the obligations incurred despite
repeated demands, PBCom extrajudicially foreclosed MICOs real estate
mortgage and sold the said mortgaged properties in a public auction sale. Lee
334

contends that the letters of credit, surety agreements and loan transactions did
not ripen into valid and binding contracts since no part of the proceeds of the
loan transactions were delivered to MICO or to any of the petitioners-sureties.
Petitioners-sureties allege that Chua Siok Suy was the beneficiary of the
proceeds of the loans and that the latter made them sign the surety agreements
in blank. Thus, they maintain that they should not be held accountable for any
liability that might arise therefrom.
Issue:
1) whether or not the proceeds of the loans and letters of credit transactions were
ever delivered to MICO
2) whether or not the individual petitioners, as sureties, may be held liable under
the two (2) Surety Agreements
Held:
1) whether or not the proceeds of the loans and letters of credit transactions were
ever delivered to MICO
The letter of credita, as well as the security agreements, have not merely created
a prima facie case but have actually proved the solidary obligation of MICO and
the petitioners, as sureties of MICO, in favor of respondent PBCom.
While the presumption found under the Negotiable Instruments Law may not
necessarily be applicable to trust receipts and letters of credit, the presumption
that the drafts drawn in connection with the letters of credit have sufficient
consideration. Under Section 3(r), Rule 131 of the Rules of Court there is also a
presumption that sufficient consideration was given in a contract.
Hence, petitioners should have presented credible evidence to rebut that
presumption as well as the evidence presented by private respondent PBCom.
The letters of credit show that the pertinent materials/merchandise have been
received by MICO. The drafts signed by the beneficiary/suppliers in connection
with the corresponding letters of credit proved that said suppliers were paid by
PBCom for the account of MICO. On the other hand, aside from their bare
denials petitioners did not present sufficient and competent evidence to rebut the
evidence of private respondent PBCom.
335

2) whether or not the individual petitioners, as sureties, may be held liable under
the two (2) Surety Agreements
A perusal of the By-Laws of MICO, however, shows that the power to borrow
money for the company and issue mortgages, bonds, deeds of trust and
negotiable instruments or securities, secured by mortgages or pledges of
property belonging to the company is not confined solely to the president of the
corporation. The Board of Directors of MICO can also borrow money, arrange
letters of credit, execute trust receipts and promissory notes on behalf of the
corporation.[35] Significantly, this power of the Board of Directors according to
the by-laws of MICO, may be delegated to any of its standing committee, officer
or agent.[36] Hence, PBCom had every right to rely on the Certification issued by
MICOs corporate secretary, P.B. Barrera, that Chua Siok Suy was duly
authorized by its Board of Directors to borrow money and obtain credit facilities in
behalf of MICO from PBCom.

336

Lopez Realty, Inc. vs. Fontecha


247 SCRA 183 (1995)

Facts:
Lopez Realty, Inc., is a corporation engaged in real estate
business, petitioner Gonzales is one of its majority shareholders. Sometime in
1978, Lopez submitted a proposal relative to the the reduction of employees with
provision for their gratuity pay. The proposal was deliberated upon and
approved in a special meeting of the board of directors. It appears
that petitioner corporation approved two (2) resolutions providing for
the gratuity pay of its employees. Private respondents were the retained
employees of the Corporation. In a letter, the private respondents requested
for the full payment of their gratuity pay. Their request was granted in a special
meeting held. At that, time, however, Gonzales was still abroad. Allegedly, while
she was still out of the country, she sent a cablegram to the corporation,
objecting to certain matters taken up by the board in her absence, such as the
sale of some of the assets of the corporation. Upon her return, she filed a
derivative suit with the SEC against majority shareholder Lopez. Notwithstanding
the "corporate squabble" between Gonzales and Lopez, the
first
two (2)
installments of the gratuity pay of the private respondents were paid by
the corporation. Also, the corporation had prepared the cash vouchers and
checks for the third installments of gratuity pay of said private respondents. For
some reason, said vouchers were cancelled by Gonzales. Likewise, the first,
second and third installments of gratuity pay ofthe rest of private respondents
were prepared but cancelled by Gonzales. Despite
private respondents'
repeated demands fortheir gratuity pay, corporation refused to pay the same.

Issue:
Whether the corporation is bound to grant its employees gratuitypay
despite the lack of notice to a board director during themeeting
wherein the said resolution was passed

Held:
YES. As a general rule, a corporation through its board of
directors should act in the manner and within the formalities
prescribed by its charter or by the general law. Thus, directors must act as a
body in a meeting called pursuant to the law or corporations by- laws, otherwise
any action may be questioned by any objecting stockholder. However, an action
of the board of directors during a meeting, which was illegal for lack of notice
may be ratified either expressly, by the action of the directors in subsequent
legal meeting or impliedly by the corporations subsequent course of
conduct. Thus, a director who was not notified of a board meeting is
precluded from questioning the validity of the resolution granting gratuity
337

pay to employee approved at that meeting if she later on acquiesced


to it by signing the vouchers for the payment of the gratuity pay

338

Republic Planters Bank vs. Agana, Sr., 296 SCRA 1 (1998)


Ponente: Justice Hermosisima, Jr.

FACTS:

On 18 September 1961, the Robes-Francisco Realty & Development Corporation


(RFRDC) secured a loan from the Republic Planters Bank in the amount of
P120,000.00. As part of the proceeds of the loan, preferred shares of stocks
were issued to RFRDC through its officers then, Adalia F. Robes and one Carlos
F. Robes. In other words, instead of giving the legal tender totaling to the full
amount of the loan, which is P120,000.00, the Bank lent such amount partially in
the form of money and partially in the form of stock certificates numbered 3204
and 3205, each for 400 shares with a par value of P10.00 per share, or for
P4,000.00 each, for a total of P8,000.00. Said stock certificates were in the name
of Adalia F. Robes and Carlos F. Robes, who subsequently, however, endorsed
his shares in favor of Adalia F. Robes.

Said certificates of stock bear the following terms and conditions: "The Preferred
Stock shall have the following rights, preferences, qualifications and limitations,
to wit: 1. Of the right to receive a quarterly dividend of 1%, cumulative and
participating. xxx 2. That such preferred shares may be redeemed, by the system
of drawing lots, at any time after 2 years from the date of issue at the option of
the Corporation." On 31 January 1979, RFRDC and Robes proceeded against
the Bank and filed a complaint anchored on their alleged rights to collect
dividends under the preferred shares in question and to have the bank redeem
the same under the terms and conditions of the stock certificates. The bank filed
a Motion to Dismiss 3 private respondents' Complaint on the following grounds:
(1) that the trial court had no jurisdiction over the subject-matter of the action; (2)
that the action was unenforceable under substantive law; and (3) that the action
was barred by the statute of limitations and/or laches. The bank's Motion to
Dismiss was denied by the trial court in an order dated 16 March 1979. The bank
then filed its Answer on 2 May 1979. Thereafter, the trial court gave the parties
10 days from 30 July 1979 to submit their respective memoranda after the
submission of which the case would be deemed submitted for resolution. On 7
September 1979, the trial court rendered the decision in favor of RFRDC and
Robes; ordering the bank to pay RFRDC and Robes the face value of the stock
certificates as redemption price, plus 1% quarterly interest thereon until full
339

payment. The bank filed the petition for certiorari with the Supreme Court,
essentially on pure questions of law.

ISSUE:

1. Whether the bank can be compelled to redeem the preferred shares


issued to RFRDC and Robes.
2. Whether RFRDC and Robes are entitled to the payment of certain rate of
interest on the stocks as a matter of right without necessity of a prior
declaration of dividend.

HELD:

1. While the stock certificate does allow redemption, the option to do so was
clearly vested in the bank. The redemption therefore is clearly the type known as
"optional". Thus, except as otherwise provided in the stock certificate, the
redemption rests entirely with the corporation and the stockholder is without right
to either compel or refuse the redemption of its stock. Furthermore, the terms
and conditions set forth therein use the word "may". It is a settled doctrine in
statutory construction that the word "may" denotes discretion, and cannot be
construed as having a mandatory effect. The redemption of said shares cannot
be allowed. The Central Bank made a finding that the Bank has been suffering
from chronic reserve deficiency, and that such finding resulted in a directive,
issued on 31 January 1973 by then Gov. G. S. Licaros of the Central Bank, to the
President and Acting Chairman of the Board of the bank prohibiting the latter
from redeeming any preferred share, on the ground that said redemption would
reduce the assets of the Bank to the prejudice of its depositors and creditors.
Redemption of preferred shares was prohibited for a just and valid reason. The
directive issued by the Central Bank Governor was obviously meant to preserve
the status quo, and to prevent the financial ruin of a banking institution that would
have resulted in adverse repercussions, not only to its depositors and creditors,
but also to the banking industry as a whole. The directive, in limiting the exercise
of a right granted by law to a corporate entity, may thus be considered as an
exercise of police power.

2. Both Section 16 of the Corporation Law and Section 43 of the present


Corporation Code prohibit the issuance of any stock dividend without the
approval of stockholders, representing not less than two-thirds (2/3) of the
340

outstanding capital stock at a regular or special meeting duly called for the
purpose. These provisions underscore the fact that payment of dividends to a
stockholder is not a matter of right but a matter of consensus. Furthermore,
"interest bearing stocks", on which the corporation agrees absolutely to pay
interest before dividends are paid to common stockholders, is legal only when
construed as requiring payment of interest as dividends from net earnings or
surplus only. In compelling the bank to redeem the shares and to pay the
corresponding dividends, the Trial committed grave abuse of discretion
amounting to lack or excess of jurisdiction in ignoring both the terms and
conditions specified in the stock certificate, as well as the clear mandate of the
law.

341

Neugene Marketing Inc. vs. Court of Appeals, 303 SCRA 295 (1999)
Purisima, J.
Facts:
On January 27, 1978, NEUGENE was duly registered with this Commission to
engage in trading business for a term of fifty (50) year. The authorized capital
stock of NEUGENE is THREE MILLION PESOS (P3,000,000.00) divided into
THIRTY THOUSAND (30,000) shares with a par value of ONE HUNDRED
PESOS (P100.00) each. Out of this authorized capital stock, SIX HUNDRED
THOUSAND PESOS (P600,000.00) had been subscribed by the following
subscribers, namely:
NAME

NO.

SHARES SUBSCRIBED AMOUNT PAID

OF AMOUNT

UP

Johnson Lee

600

P 60,000.00

P15,000.00

Lok Chun Suen

1,200

120,000.00

30,000.00

Charles O. Sy

1,800

180,000.00

45,000.00

Flores, 2,100

210,000.00

52,500.00

Eugenio
Jr.

Arsenio Yang, Jr.

300

30,000.00

7,500.00

TOTAL

6,000

P600,000.00

P15,000.00

The original shareholdings of the incorporators/stockholders of NEUGENE were


increased by ten percent (10%) each by virtue of stock dividend declaration in
the amount of SIXTY THOUSAND PESOS (P60,000.00) made by its board of
directors in a special meeting and subsequently an additional dividend in the
amount of FORTY THOUSAND PESOS (P40,000.00) in proportion to the
shareholdings of the stockholders of record. The outstanding capital stock of
342

NEUGENE became SEVEN HUNDRED THOUSAND PESOS (P700,000.00)


represented by SEVEN THOUSAND (7,000) shares.
On May 15, 1986, Eugenio Flores, Jr. assigned, transferred and conveyed his
entire shareholdings of TWO THOUSAND FOUR HUNDRED FIFTY (2,450)
shares in NEUGENE to the following, to wit:
Pet. Sonny Moreno 1,050 shares (Exh. B)
Resp. Arsenio Yang, Jr.. 700 shares (Exh. C)
Resp. Charles O. Sy 700 shares (Exh. D)
TOTAL 2,450
On November 30, 1987, the private respondents, Charles O. Sy, Arsenio Yang,
Jr. and Lok Chun Suen, the directors and stockholders then present, voted for
and approved a resolution dissolving NEUGENE.
On March 1, 1988, acting upon private respondentss Petition for Dissolution,
SEC issued a Certificate of Dissolution of NEUGENE.
On March 22, 1988, the petitioners brought an action to annul or set aside the
said SEC Certification on the Dissolution of Neugene.

Issue:
Whether the private respondents lacked the requisite number of shares of stock
or had divested themselves of their stockholdings as of November 30, 1987
when they voted for the resolution dissolving NEUGENE.

Ruling:
The entries on the right hand portion of NEUGENES Stock and Transfer Book,
under the column Certificates Issued, indubitably record the private respondents
as the holders of 5,250 shares, constituting at least two-thirds (2/3) of
NEUGENEs outstanding capital stock of 7,000 shares.

343

Republic vs. Sandiganbayan, 246 SCRA 760 (2000)


Ponente: Ynares, Santiago, J:

Facts: On July 22, 1987, the petitioner Republic of the Philippines, through the
Presidential Commission on Good Government (PCGG), filed a complaint
against Jose L. Africa, Manuel H. Nieto, Jr., Ferdinand E. Marcos, Imelda R.
Marcos, Ferdinand R. Marcos, Jr., Juan Ponce Enrile, and Potenciano Ilusorio
(collectively, the

respondents)

for reconveyance,

reversion,

accounting,

restitution,and damages before the Sandiganbayan. The petitioner alleged that


the respondents illegally manipulated the purchase of the major shareholdings of
Cable and Wireless Limited in Eastern Telecommunications Philippines, Inc.
(ETPI), which shareholdings respondents Jose Africa and Manuel Nieto, Jr. held
for themselves and, through their holdings and the corporations they organized,
beneficially for respondents Ferdinand E. Marcos and Imelda R. Marcos. Africa,
as an ETPI stockholder, filed a petition for certiorari, with prayer for a temporary
restraining order/preliminary injunction with the Sandiganbayan seeking to
nullify the order of the Sandiganbayan for the former to account for his
sequestered shares in ETPI and to cease and desist from exercising voting
rights. During the pendency of Africas petition, Africa filed a motion with the
Sandiganbayan, alleging that the PCGG had been illegally exercising the rights
of stockholders of ETPI, especially in the election of the members of the board of
directors and prayed for the issuance of an order for the calling and holding of
[ETPI] annual stockholders meeting under the courts control and supervision.
Sandiganbayan favored Africas motion.The PCGG assailed this resolution
before this Court via a petition for certiorari docketed imputing grave abuse of
discretion on the Sandiganbayan for holding, inter alia, that the registered
stockholders of ETPI had the right to vote.To resolve the Urgent Petition, the
testimony of Mr. Maurice V. Bane (former director and treasurer-in-trust of ETPI)
was taken at the petitioners instance for the purpose for Bane to identify and
testify on the facts in his affidavit so as to prove the ownership issue in favor of
the petitioner and/or establish the prima facie factual foundation for sequestration
of ETPIs Class A stock in support of the Urgent Petition The Sandiganbayan
granted the Urgent Petition hence, Africa went to this Court via a petition for
certiorari docketed as G.R. No. 147214 (Africas petition). The court resolved the
PCGG and Africas petition and were REFERRED to the Sandiganbayan for
reception of evidence and to decide it within 60 days from receipt of this
resolution.The petitioners did not question the 1998 resolution, and instead they
344

made its Formal Offer of Evidence. Significantly, the Bane deposition was not
included as part of its offered exhibits. In order to correct this, they filed a second
motion with prayer for re-opening of the case for the purpose of introducing
additional evidence and requested the court to take judicial notice of the facts
established by the Bane deposition. This was denied by the Sandiganbayan in
its November 6, 2000 resolution. A third motion was filed by the petitioners on
November 16, 2001 seeking once more to admit the Bane deposition which the
Sandiganbayan

for

the

reason

that without

plaintiff

having

moved

for

reconsideration within the reglementary period, the resolution has attained


finality and its effect cannot be undone by the simple expedient of filing a motion.

Issue: Whether the Sandiganbayan committed grave abuse of jurisdiction on


holding that the 1998 resolution already attained finality and that the 3rd motion
for reconsideration of the petitioner is prohibited.

Held: Judgment or order is considered final if the order disposes of the action or
proceeding completely, or terminates a particular stage of the same action; in
such case, the remedy available to an aggrieved party is appeal. If the order or
resolution, however, merely resolves incidental matters and leaves something
more to be done to resolve the merits of the case, the order is interlocutory and
the aggrieved partys remedy is a petition for certiorari under Rule 65. Therefore,
the 1998 resolution is interlocutory.

The Sandiganbayans denial of the

petitioners 1st motion through the 1998 Resolution came at a time when the
petitioner had not even concluded the presentation of its evidence. Plainly, the
denial of the motion did not resolve the merits of the case, as something still had
to

be

done

to

achieve

this

end.

the Sandiganbayans 1998 resolution which merely denied the adoption of the
Bane deposition as part of the evidence in Civil Case No. 0009 could not have
attained finality.The Sandiganbayan undoubtedly erred on a question of law in its
ruling, but this legal error did not necessarily amount to a grave abuse of
discretion in the absence of a clear showing that its action was a capricious and
whimsical exercise of judgment affecting its exercise of jurisdiction. Without this
showing, the Sandiganbayans erroneous legal conclusion was only an error of
judgment, or, at best, an abuse of discretion but not a grave one.The
3rd motion could not also be considered as a prohibited motion because Section
5, Rule 37 of the Rules of Court clearly provides, the proscription against a
second motion for reconsideration is directed against a judgment or final order.
345

But a second motion for reconsideration of an interlocutory order can be denied


on the ground that it is discusses again the arguments already passed upon and
resolved by the court. In this case, the latter is the reason cited by the
respondents for the denial of the motion

346

Reyes vs. Tan


3 SCRA 198 (1961)
FACTS:
The board of directors of Akron Customs Brokerage Corporation adopted a
resolution authorizing the purchase of thirteen (13) trucks for use in its business
to be paid out of a loan the corporation may secure from any lending institution.
Feliciano Coprada, as President and Chairman of Akron, purchased thirteen
trucks from private respondent on January 25, 1978 for and in consideration of
P525,000.00 as evidenced by a deed of absolute sale. In a side agreement of the
same date, the parties agreed on a downpayment in the amount of P50,000.00
and that the balance of P475,000.00 shall be paid within sixty (60) days from the
date of the execution of the agreement. The parties also agreed that until said
balance is fully paid, the down payment of P50,000.00 shall accrue as rentals of
the 13 trucks; and that if Akron fails to pay the balance within the period of 60
days, then the balance shall constitute as a chattel mortgage lien covering said
cargo trucks and the parties may allow an extension of 30 days and thereafter
private respondent may ask for a revocation of the contract and the
reconveyance of all said trucks.
The obligation is further secured by a promissory note executed by Coprada
in favor of Akron. It is stated in the promissory note that the balance shall be paid
from the proceeds of a loan obtained from the Development Bank of the
Philippines (DBP) within sixty (60) days. After the lapse of 90 days, private
respondent tried to collect from Coprada but the latter promised to pay only upon
the release of the DBP loan.
Private respondent sent Coprada a letter of demand dated May 10,
1978.9 In his reply to the said letter, Coprada reiterated that he was applying for
a loan from the DBP from the proceeds of which payment of the obligation shall
be made. Meanwhile, two of the trucks were sold under a pacto de retro sale to a
certain Mr. Bais of the Perpetual Loans and Savings Bank at Baclaran. The sale
was authorized by a board resolution made in a meeting held on March 15, 1978.
Upon inquiry, private respondent found that no loan application was ever
filed by Akron with DBP. In the meantime, Akron paid rentals of P500.00 a day
pursuant to a subsequent agreement, from April 27, 1978 (the end of the 90-day
period to pay the balance) to May 31, 1978. Thereafter, no more rental payments
were made.

On June 17, 1978, Coprada wrote private respondent begging for a grace
period of until the end of the month to pay the balance of the purchase price; that
he will update the rentals within the week; and in case he fails, then he will return
the 13 units should private respondent elect to get back the same. Private
respondent, through counsel, wrote Akron on August 1, 1978 demanding the
347

return of the 13 trucks and the payment of P25,000.00 back rentals covering the
period from June 1 to August 1, 1978.14
Again, Coprada wrote private respondent on August 8, 1978 asking for
another grace period of up to August 31, 1978 to pay the balance, stating as well
that he is expecting the approval of his loan application from a certain financing
company, and that ten (10) trucks have been returned to Bagbag, Novaliches.15
On December 9, 1978, Coprada informed private respondent anew that he had
returned ten (10) trucks to Bagbag and that a resolution was passed by the board
of directors confirming the deed of assignment to private respondent of P475,000
from the proceeds of a loan obtained by Akron from the State Investment House,
Inc
In due time, private respondent filed a compliant for the recovery of
P525,000.00 or the return of the 13 trucks with damages against Akron and its
officers and directors, with the then Court of First Instance of Rizal. Only
petitioner answered the complaint denying any participation in the transaction
and alleging that Akron has a distinct corporate personality.
In the meanwhile, petitioner sold all his shares in Akron to Coprada. It also
appears that Akron amended its articles of incorporation thereby changing its
name to Akron Transport International, Inc. which assumed the liability of Akron
to private respondent.
Court of First Instance favored the complaint of plaintiff thus it went ot IAC.
A motion for new trial filed by petitioner was denied so he appealed to the then
Intermediate Appellate Court (IAC). However, upon a motion for reconsideration
filed by private respondent, the IAC, in a resolution dated February 8, 1984, set
aside the decision dated June 30, 1983. The appellate court entered another
decision affirming the appealed decision of the trial court, with costs against
petitioner

ISSUE:
The Intermediate Appellate Court (IAC) erred in disregarding the corporate
fiction and in holding the petitioner personally liable for the obligation of the
Corporation which decision is patently contrary to law and the applicable decision
thereon.

RULING:
A corporation is an entity separate and distinct from its stockholders;
Corporate fiction.A corporation is an entity separate and distinct from its
stockholders. While not in fact and in reality a person, the law treats a
corporation as though it were a person by process of fiction or by regarding it as
an artificial person distinct and separate from its individual stockholders.
However, the corporate fiction or the notion of legal entity may be disregarded
when it is used to defeat public convenience, justify wrong, protect fraud, or
defend crime in which instances the law will regard the corporation as an
348

association of persons, or in case of two corporations, will merge them into one.
The corporate fiction may also be disregarded when it is the mere alter ego or
business conduit of a person. There are many occasions when this Court
pierced the corporate veil because of its use to protect fraud and to justify wrong.
The environmental facts of this case show that there is no cogent basis to
pierce the corporate veil of Akron and hold petitioner personally liable for its
obligation to private respondent. While it is true that in December, 1977 petitioner
was still a member of the board of directors of Akron and that he participated in
the adoption of a resolution authorizing the purchase of 13 trucks for the use in
the brokerage business of Akron to be paid out of a loan to be secured from a
lending institution, it does not appear that said resolution was intended to defraud
anyone and more particularly private respondent. It was Coprada, President and
Chairman of Akron, who negotiated with said respondent for the purchase of 13
cargo trucks on January 25, 1978. It was Coprada who signed a promissory note
to guarantee the payment of the unpaid balance of the purchase price out of the
proceeds of a loan he supposedly sought from the DBP. The word WE in the
said promissory note must refer to the corporation which Coprada represented in
the execution of the note and not its stockholders or directors. Petitioner did not
sign the said promissory note so he cannot be personally bound thereby.
As to the sale through pacto de retro of two units to a third person by the
corporation by virtue of a board resolution, petitioner asserts that he never signed
said resolution. Be that as it may, the sale is not inherently fraudulent as the 13
units were sold through a deed of absolute sale to Akron so that the corporation
is free to dispose of the same. Of course, it was stipulated that in case of default
in payment to private respondent of the balance of the consideration, a chattel
mortgage lien shall be constituted on the 13 units. Nevertheless, said mortgage
is a prior lien as against the pacto de retro sale of the 2 units. Amendment of
articles of incorporation thereby changing the name of the corporation is not an
indication to evade payment by the corporation of its obligations to another.
As to the amendment of the articles of incorporation of Akron thereby changing
its name to Akron Transport International, Inc., petitioner alleges that the change
of corporate name was in order to include trucking and container yard operations
in its customs brokerage of which private respondent was duly informed in a
letter. Indeed, the new corporation confirmed and assumed the obligation of the
old corporation. There is no indication of an attempt on the part of Akron to evade
payment of its obligation to private respondent. A stockholder has an inherent
right to dispose of his shares of stock anytime he so desires.There is the fact
that petitioner sold his shares in Akron to Coprada during the pendency of the
case. Since petitioner has no personal obligation to private respondent, it is his
inherent right as a stockholder to dispose of his shares of stock anytime he so
desires. If private respondent is the victim of fraud, there was no showing that the
corporation had any participation in the perpetration of the fraud; Fraud must be
established by clear and convincing evidence.Mention is also made of the
alleged dumping of 10 units in the premises of private respondent at Bagbag,
Novaliches which to the mind of the Court does not prove fraud and instead
appears to be an attempt on the part of Akron to attend to its obligations as
regards the said trucks. Again petitioner has no part in this. If the private
349

respondent is the victim of fraud in this transaction, it has not been clearly shown
that petitioner had any part or participation in the perpetration of the same. Fraud
must be established by clear and convincing evidence. If at all, the principal
character on whom fault should be attributed is Feliciano Coprada, the President
of Akron, whom private respondent dealt with personally all through out.
Fortunately, private respondent obtained a judgment against him from the trial
court and the said judgment has long been final and executory

350

Republic Bank vs. Cuaderno, 19 SCRA 671. (1976)


Ponente: Justice Jose B.L. Reyes

Facts:

Damaso Perez, a stockholder of Republic Bank instituted a derivative suit


for and in behalf of said Bank, against Miguel Cuaderno and several others
including the Board of Directors (BOD) of the Republic Bank, and the Monetary
Board of the Central Bank of the Philippines. According to Perez, he had
complained to the Monetary Board against certain frauds committed by Pablo
Roman as the chairman of the BOD by granting loans to fictitious and nonexisting persons and to their close friends, relatives and/or employees. Acting
upon the complaint, Miguel Cuaderno (then Governor of the Central Bank) and
the Monetary Board ordered an investigation which found that certain mortgage
loans were indeed granted in violation of the General Banking Act. However, to
neutralize the impending action against him, Roman engaged Cuaderno as
technical consultant and one Bienvenido Dizon as chairman of BOD. For Perez,
the selection of these individuals by Roman was made in order to shield the latter
from the alleged wrongdoing and from any prosecution that may be instituted
against him. The case, however, was dismissed.

Issue:

Whether the dismissal of the case was proper

Ruling:

No. Philippine jurisprudence is settled that an individual stockholder is


permitted to institute a derivative or representative suit on behalf of the
corporation wherein he holds stock in order to protect or vindicate corporate
rights, whenever the officials of the corporation refuse to sue, or are the ones to
be sued or hold the control of the corporation. In such actions, the suing
stockholder is regarded as a nominal party, with the corporation as the real party
in interest. Damasos action is precisely in conformity with these principles. He is
neither alleging nor vindicating his own individual interest or prejudice, but the
interest of the Republic Bank and the damage caused to it. The action he has
brought is a derivative one, expressly manifested to be for and in behalf of the
Republic Bank, because it was futile to demand action by the corporation, since
351

its Directors were nominees and creatures of defendant Pablo Roman. The
frauds charged by plaintiff are frauds against the Bank that redounded to its
prejudice.

Defendants urge that the action is improper because the plaintiff was not
authorized by the corporation to bring suit in its behalf. Any such authority could
not be expected as the suit is aimed to nullify the action taken by the manager
and the board of directors of the Republic Bank; and any demand for intracorporate remedy would be futile. These circumstances permit a stockholder to
bring a derivative suit

352

San Miguel Corporation vs. Kahn


176 SCRA 73 (1991)
FACTS: San Miguel Corporation were acquired by fourteen (14)corporationsand
were placed under a Voting Trust Agreement in favor of the late Andres Soriano,
Jr. When the latter died, Eduardo M. Cojuangco, Jr. was elected Substitute
Trustee with power to delegate the trusteeship in writing to Andres Soriano III.
Shortly after the Revolution, he left the country amid "persistent reports" that
"huge and unusual cash disbursements from the funds of SMC. An"Agreement"
was executed between Andres Soriano III, as "Buyer," and the 14 corporations,
as "Sellers," for the purchase by Soriano, "for himself and as agent of several
persons. The Agreement revoked the voting trust above mentioned, and
expressed the desire of the 14 corporations to sell the shares of stock "to pay
certain outstanding and unpaid debts,". Actually the buyer of the shares was
Neptunia Corporation, a foreign corporation and wholly-owned subsidiary of
another subsidiary wholly owned by San Miguel Corporation. Neptunia paid the
downpayment from the proceeds of certain loans. PCGG then sequestered the
shares subject of the sale so San Miguel suspended all the other installments of
the price to the sellers. The 14 corporations then sued for recission and
damages. Meanwhile, PCGG directed San Miguel to issue qualifying shares to
seven (7) individuals including Eduardo de los Angeles from the sequestered
shares for them to hold in trust.Then, the San Miguel board of directors passed a
resolution assuming the loans incurred by Neptunia for the downpayment. De los
Angeles assailed the resolution alleging that it was not passed by the board
aside from its delitorious effects on the corporations interest. When his efforts to
obtain relief within the corporation proved futile, he filed this action with the SEC.
ISSUE:
Whether de los Angeles has legal standing in the instant case.
HELD:
Yes. The requisites of a derivative suit are:1. the party bringing the suit should
be a stockholder as of the time of the act or transactions complained of, the
number of shares not being material;2. exhaustion of intra-corporate remedies
(has made a demand on the board of directors for the appropriate relief but the
latter has failed or refused to heed his plea); and 3. the cause of action actually
devolves on the corporation and not to the particular stockholder bringing the
suit.The bona fide ownership by a stockholder in his own right suffices to invest
him with the standing to bring a derivative suit for the benefit of the corporation.
The number of his shares is immaterial since he is not suing in his own behalf,
or for the protection or vindication of his own particular right, or the redress of a
wrong committed against him individually but in behalf and for the benefit of the
corporation. It is undisputed that apart from the qualifying shares given to him by
the PCGG, he owns 20 shares in his own right, as regards which he cannot from
any aspect be deemed to be beholden to the PCGG, his ownership of his shares
being precisely what he invokes as the source of his authority to bring the
derivative suit. Furthermore, it was not necessary for de los Angeles to be a
director in order to bring a derivative suit.
De los Angeles complaint is confined to the issue of the validity of the
assumption by the corporation of the indebtedness of Neptunia, allegedly for the
benefit of certain of its officers and stockholders and is distinct from the
ownership of the sequestered shares. The dispute concerns the acts of the
board of directors claimed to amount to fraud and misrepresentation which may
be detrimental to the interest of the stockholders, or is one arising out of intracorporate relations between and among stockholders, or between any or all of
353

them and the corporation of which they are stockholders (meaning that the
cause of action still belongs to the corporation).

354

Western Institute of Technology, Inc. vs. Salas


278 SCRA 216 (1997)

FACTS: Private respondents Ricardo T. Salas, Salvador T. Salas, Soledad


Salas-Tubilleja, Antonio S. Salas, and Richard S. Salas, belonging to the same
family, are the majority and controlling members of the Board of Trustees of
Western Institute of Technology, Inc., a stock corporation engaged in the
operation, among others, of an educational institution. According to petitioners,
the minority stockholders of WIT, a Special Board Meeting was held. In
attendance were other members of the Board including one of the petitioners
Reginald Villasis. In said meeting, the Board of Trustees passed Resolution No.
48, s. 1986, granting monthly compensation to the private respondents as
corporate officers retroactive June 1, 1985. A few years later, petitioners Homero
Villasis, Prestod Villasis, Reginald Villasis and Dimas Enriquez filed an affidavitcomplaint against private respondents before the Office of the City Prosecutor,
as a result of which two (2) separate criminal informations, one for falsification of
a public document and the other for estafa, were filed before the Regional Trial
Court. The charge for falsification of public document was anchored on the
private respondents' submission of WIT's income statement for the fiscal year
1985-1986 with the Securities and Exchange Commission reflecting therein the
disbursement of corporate funds for the compensation of private respondents
based on Resolution No. 4, series of 1986, making it appear that the same was
passed by the board on March 30, 1986, when in truth, the same was actually
passed on June 1, 1986, a date not covered by the corporation's fiscal year
1985-1986. Thereafter, trial for the two criminal cases was consolidated. After a
full-blown hearing, Judge Porfirio Parian handed down a verdict of acquittal on
both counts without imposing any civil liability against the accused therein.
Petitioners filed a Motion for Reconsideration of the civil aspect of the RTC
Decision which was, however, denied in an Order.
ISSUE: Whether or not the case is derivative suit correctly filed in the Regional
Trial Court.
RULING: No. Granting, for purposes of discussion, that this is a derivative suit as
insisted by petitioners, which it is not, the same is outrightly dismissible for
having been wrongfully filed in the regular court devoid of any jurisdiction to
entertain the complaint. The ease should have been filed with the Securities and
Exchange Commission (SEC) which exercises original and exclusive jurisdiction
over derivative suits, they being intra-corporate disputes, per Section 5 (b) of
P.D. No. 902-A: In addition to the regulatory and adjudicative functions of the
Securities and Exchange Commission over corporations, partnerships and other
forms of associations registered with it as expressly granted under existing laws
and decrees, it shall have original and exclusive jurisdiction to hear and decide
cases involving: Controversies arising out of intra-corporate or partnership
relations, between and among stockholders, members, or associates; between
any or all of them and the corporation, partnership or association of which they
are stockholders, members or associates, respectively; and between such
corporation, partnership or association and the State insofar as it concerns their
individual franchise or right to exist as such entity.

355

Bitong vs. Court of Appeals, 292 SCRA 503 (1998)


BELLOSILLO, J.:

FACTS:

Petitioner Alleged before the SEC that she had been the Treasurer and a
Member of the Board of Directors of Mr. & Ms. from the time it was incorporated
on 29 October 1976 to 11 April 1989, and was the registered owner of 1,000
shares of stock out of the 4,088 total outstanding shares, petitioner complained
of irregularities committed from 1983 to 1987 by Eugenia D. Apostol, President
and Chairperson of the Board of Directors. Petitioner claimed that except for the
sale of the name Philippine Inquirer to Philippine Daily Inquirer (PDI hereafter) all
other transactions and agreements entered into by Mr. & Ms. with PDI were not
supported by any bond and/or stockholders' resolution. And, upon instructions of
Eugenia D. Apostol, Mr. & Ms. made several cash advances to PDI on various
occasions amounting to P3.276 million. On some of these borrowings PDI paid
no interest whatsoever. Despite the fact that the advances made by Mr. & Ms. to
PDI were booked as advances to an affiliate, there existed no board or
stockholders' resolution, contract nor any other document which could legally
authorize the creation of and support to an affiliate.

Petitioner further alleged that respondents Eugenia and Jose Apostol were
stockholders, directors and officers in both Mr. & Ms. and PDI. In fact on 2 May
1986 respondents Eugenia D. Apostol, Leticia J. Magsanoc and Adoracion G.
Nuyda subscribed to PDI shares of stock at P50,000.00 each or a total of
P150,000.00.

ISSUE:

Whether or not the petitioner is the holder of the proper certificates of share of
stock.
HELD:

NO. The certificate of stock itself once issued is a continuing affirmation or


representation that the stock described therein is valid and genuine and is at
least prima facie evidence that it was legally issued in the absence of evidence to
the contrary. However, this presumption may be rebutted. 13 Similarly, books and
records of a corporation which include even the stock and transfer book are
356

generally admissible in evidence in favor of or against the corporation and its


members to prove the corporate acts, its financial status and other matters
including one's status as a stockholder. They are ordinarily the best evidence of
corporate acts and proceedings.

However, the books and records of a corporation are not conclusive even against
the corporation but are prima facie evidence only. Parol evidence may be
admitted to supply omissions in the records, explain ambiguities, or show what
transpired where no records were kept, or in some cases where such records
were contradicted. The effect of entries in the books of the corporation which
purport to be regular records of the proceedings of its board of directors or
stockholders can be destroyed by testimony of a more conclusive character than
mere suspicion that there was an irregularity in the manner in which the books
were kept.

Thus, while petitioner asserts in her petition that Certificate of Stock No. 008
dated 25 July 1983 was issued in her name, private respondents argue that this
certificate was signed by respondent Eugenia D. Apostol as President only in
1989 and was fraudulently antedated by petitioner who had possession of the
Certificate Book and the Stock and Transfer Book.

357

Lim vs. Lim Yu, 352 SCRA 216 (2001)


PANGANIBAN, J.:
Facts: At a special meeting on 07 October 1994, the Board of Directors of
LIMPAN resolved that the corporation shall make a partial payment for the legal
services of Gilda C. Lim in the handling of various cases on behalf of, or involving
the corporation in the amount of P1,551,500.00 to be paid in equivalent value in
shares of stock of the corporation totaling 15,515 shares, the same being found
to be reasonable, and there being no available funds to pay the same. In July
1996, the private respondent Patricia Lim Yu, a sister of the petitioner, Lim, filed
a complaint against the members of the Board of Directors of LIMPAN who
approved the aforesaid resolution. Bundang, Velez, and Manzano filed an
Answer, asserting as affirmative defenses that the complaint failed to state a
cause of action against them; that Yu had no legal capacity to sue; and that the
issuance of the shares in Lims favor was bona fide and valid pursuant to law and
LIMPANs By-Laws. In turn, the herein petitioners Lim, Joven and Lerios filed a
Motion to Dismiss on the following grounds: that Yu had no legal capacity to sue;
that the complaint failed to state a cause of action against Joven and Lerios, and
that no earnest efforts were exerted towards a compromise, Yu and Lim being
siblings.
Issue: That Yus suit to enforce her pre-emptive rights in a corporation is in the
nature of a derivative suit, hence he has the legal capacity to file her Complaint
before the SEC.

Ruling: A derivative suit has been defined as an action brought by minority


shareholders in the name of the corporation to redress wrongs committed against
it, for which the directors refuse to sue. It is a remedy designed by equity and has
been the principal defense of the minority shareholders against abuses by the
majority. In a derivative action, the real party in interest is the corporation itself,
not the shareholder(s) who actually instituted it. If the suit filed by Yu was indeed
derivative in character, then Yu may not have the capacity to sue. The reason is
that she would be acting in representation of the corporation, an act which the
TRO enjoins her from doing. Hence, Yu's suit cannot be characterized as
derivative, because she was complaining only of the violation of her pre-emptive
right under Section 39 of the Corporation Code. She was merely praying that she
be allowed to subscribe to the additional issuances of stocks in proportion to her
shareholdings to enable her to preserve her percentage of ownership in the
358

corporation. She was therefore not acting for the benefit of the corporation. Quite
the contrary, she was suing on her own behalf, out of a desire to protect and
preserve her pre-emptive rights. Unquestionably, the TRO did not prevent her
from pursuing that action.

359

Goachan vs. Young, 354 SCRA 207 (2001)


Ponente : Justice Panganiban
Facts :
Felix Gochan and Sons Realty Corporation (Gochan Realty) was
registered with the SEC on June 1951, with Felix Gochan, Sr., Maria Pan Nuy Go
Tiong, Pedro Gochan, Tomasa Gochan, Esteban Gochan and Crispo Gochan as
its incorporators. Felix Gochan Sr.'s daughter, Alice inherited 50 shares of stock
in Gochan Realty from the former. Alice died in 1955, leaving the 50 shares to
her husband, John Young, Sr. In 1962, the Regional Trial Court of Cebu
adjudicated 6/14 of these shares to her children, Richard Young, David Young,
Jane Young Llaban, John Young Jr., Mary Young Hsu and Alexander Thomas
Young (the Youngs). Having earned dividends, these stocks numbered 179 by
20 September 1979. 5 days later (25 September), at which time all the children
had reached the age of majority, their father John Sr., requested Gochan Realty
to partition the shares of his late wife by cancelling the stock certificates in his
name and issuing in lieu thereof, new stock certificates in the names of the
Youngs. On 17 October 1979, Gochan Realty refused, citing as reason, the right
of first refusal granted to the remaining stockholders by the Articles of
Incorporation. In 1990, John, Sr. died, leaving the shares to the Youngs. On 8
February 1994, Cecilia Gochan Uy and Miguel Uy filed a complaint with the SEC
for issuance of shares of stock to the rightful owners, nullification of shares of
stock, reconveyance of property impressed with trust, accounting, removal of
officers and directors and damages against Virginia Gochan, et. al. (Gochans) A
Notice of Lis Pendens was annotated to the real properties of the corporation.

On 16 March 1994, the Gochans moved to dismiss the complaint alleging that:
(1) the SEC had no jurisdiction over the nature of the action; (2) the the Youngs
were not the real parties-in-interest and had no capacity to sue; and (3) the
Youngs' causes of action were barred by the Statute of Limitations. The motion
was opposed by the Youngs. On 29 March 1994, the Gochans filed a Motion for
cancellation of Notice of Lis Pendens. The Youngs opposed the said motion. On
9 December 1994, the SEC, through its Hearing Officer, granted the motion to
dismiss and ordered the cancellation of the notice of lis pendens annotated upon
the titles of the corporate lands; holding that the Youngs never been stockholders
of record of FGSRC to confer them with the legal capacity to bring and maintain
their action, and thus, the case cannot be considered as an intra-corporate
360

controversy within the jurisdiction of the SEC; and that on the allegation that the
Youngs brought the action as a derivative suit on their own behalf and on behalf
of Gochan Realty, rhe failure to comply with the jurisdictional requirement on
derivative action necessarily result in the dismissal of the complaint. The Youngs
filed a Petition for Review with the Court of Appeals. On 28 February 1996, the
Court of Appeals ruled that the SEC had no jurisdiction over the case as far as
the heirs of Alice Gochan were concerned, because they were not yet
stockholders of the corporation. On the other hand, it upheld the capacity of
Cecilia Gochan Uy and her spouse Miguel Uy. It also held that the Intestate
Estate of John Young Sr. was an indispensable party. The appellate court further
ruled that the cancellation of the notice of lis pendens on the titles of the
corporate real estate was not justified. Moreover, it declared that the Youngs'
Motion for Reconsideration before the SEC was not pro forma; thus, its filing
tolled the appeal period. The Gochans moved for reconsideration but were
denied in a Resolution dated 18 December 1997. The Gochans filed the Petition
for Review on Certiorari.

Issue:
Whether the action filed by the Spouses Uy was not a derivative suit,
because the spouses and not the corporation were the injured parties.

Ruling :
The following portions of the Complaint shows allegations of injury to the
corporation itself, to wit: "That on information and belief, in further pursuance of
the said conspiracy and for the fraudulent purpose of depressing the value of the
stock of the Corporation and to induce the minority stockholders to sell their
shares of stock for an inadequate consideration as aforesaid, respondent
Esteban T. Gochan . . ., in violation of their duties as directors and officers of the
Corporation . . ., unlawfully and fraudulently appropriated [for] themselves the
funds of the Corporation by drawing excessive amounts in the form of salaries
and cash advances . . . and by otherwise charging their purely personal
expenses to the Corporation"; and "That the payment of P1,200,000.00 by the
Corporation to complainant Cecilia Gochan Uy for her shares of stock constituted
an unlawful, premature and partial liquidation and distribution of assets to a
stockholder, resulting in the impairment of the capital of the Corporation and
361

prevented it from otherwise utilizing said amount for its regular and lawful
business, to the damage and prejudice of the Corporation, its creditors, and of
complainants as minority stockholders." As early as 1911, the Court has
recognized the right of a single stockholder to file derivative suits. "Where
corporate directors have committed a breach of trust either by their frauds, ultra
vires acts, or negligence, and the corporation is unable or unwilling to institute
suit to remedy the wrong, a single stockholder may institute that suit, suing on
behalf of himself and other stockholders and for the benefit of the corporation, to
bring about a redress of the wrong done directly to the corporation and indirectly
to the stockholders." Herein, the Complaint alleges all the components of a
derivative suit. The allegations of injury to the Spouses Uy can coexist with those
pertaining to the corporation. The personal injury suffered by the spouses cannot
disqualify them from filing a derivative suit on behalf of the corporation. It merely
gives rise to an additional cause of action for damages against the erring
directors. This cause of action is also included in the Complaint filed before the
SEC. The Spouses Uy have the capacity to file a derivative suit in behalf of and
for the benefit of the corporation. The reason is that the allegations of the
Complaint make them out as stockholders at the time the questioned transaction
occurred, as well as at the time the action was filed and during the pendency of
the action.

362

Reyes vs. Hon. RTC Makati Br. 142


561 SCRA 593 (2008)
Facts:
Oscar and private respondent Rodrigo C. Reyes (Rodrigo) are two of the four
children of the spouses Pedro and Anastacia Reyes. Pedro, Anastacia, Oscar,
and Rodrigo each owned shares of stock of Zenith Insurance Corporation
(Zenith), a domestic corporation established by their family. Pedro died in 1964,
while Anastacia died in 1993. Although Pedros estate was judicially partitioned
among his heirs sometime in the 1970s, no similar settlement and partition
appear to have been made with Anastacias estate, which included her
shareholdings in Zenith. As of June 30, 1990, Anastacia owned 136,598 shares
of Zenith; Oscar and Rodrigo owned 8,715,637 and 4,250 shares, respectively. ,
Zenith and Rodrigo filed a complaint4 with the Securities and Exchange
Commission (SEC) against Oscar, The complaint stated that it is "a derivative
suit initiated and filed by the complainant Rodrigo C. Reyes to obtain an
accounting of the funds and assets of ZENITH INSURANCE
CORPORATION which are now or formerly in the control, custody, and/or
possession of respondent and to determine the shares of stock of deceased
spouses Pedro and Anastacia Reyes that were arbitrarily and fraudulently
appropriated [by Oscar] for himself which were not collated and taken into
account in the partition, distribution, and/or settlement of the estate of the
deceased spouses, for which he should be ordered to account for all the income
from the time he took these shares of stock, and should now deliver to his
brothers and sisters their just and respective shares. Oscar denied the charge
that he illegally acquired the shares of Anastacia Reyes. He asserted, as a
defense, that he purchased the subject shares with his own funds from the
unissued stocks of Zenith, and that the suit is not a bona fide derivative suit
because the requisites therefor have not been complied with. He thus questioned
the SECs jurisdiction to entertain the complaint because it pertains to the
settlement of the estate of Anastacia Reyes.
Oscar filed a Motion to Declare Complaint as Nuisance or Harassment Suit. He
claimed that the complaint is a mere nuisance or harassment suit and should,
according to the Interim Rules of Procedure for Intra-Corporate Controversies, be
dismissed; and that it is not a bona fide derivative suit as it partakes of the nature
of a petition for the settlement of estate of the deceased Anastacia that is outside
the jurisdiction of a special commercial court settlement of estate of the
deceased Anastacia that is outside the jurisdiction of a special commercial court.
The RTC denied the motion. Oscar thereupon went to the CA on a petition
for certiorari, prohibition, and mandamus and prayed that the RTC Order be
annulled and set aside and that the trial court be prohibited from continuing with
the proceedings.

Issue:
Whether the complaint is a mere harassment and should be dismissed.
363

Ruling:
A review of relevant jurisprudence shows a development in the Courts approach
in classifying what constitutes an intra-corporate controversy. Initially, the main
consideration in determining whether a dispute constitutes an intra-corporate
controversy was limited to a consideration of the intra-corporate relationship
existing between or among the parties.19 The types of relationships embraced
under Section 5(b), as declared in the case of Union Glass & Container Corp. v.
SEC,20 were as follows:
a) between the corporation, partnership, or association and the public;
b) between the corporation, partnership,
stockholders, partners, members, or officers;

or

association

and

its

c) between the corporation, partnership, or association and the State as far


as its franchise, permit or license to operate is concerned; and
d) among the stockholders, partners, or associates themselves. [Emphasis
supplied.]
The existence of any of the above intra-corporate relations was sufficient to
confer jurisdiction to the SEC, regardless of the subject matter of the dispute.
This came to be known as the relationship test.
However, in the 1984 case of DMRC Enterprises v. Esta del Sol Mountain
Reserve, Inc.,21 the Court introduced the nature of the controversy test. We
declared in this case that it is not the mere existence of an intra-corporate
relationship that gives rise to an intra-corporate controversy; to rely on the
relationship test alone will divest the regular courts of their jurisdiction for the sole
reason that the dispute involves a corporation, its directors, officers, or
stockholders. We saw that there is no legal sense in disregarding or minimizing
the value of the nature of the transactions which gives rise to the dispute.
Under the nature of the controversy test, the incidents of that relationship must
also be considered for the purpose of ascertaining whether the controversy itself
is intra-corporate.22 The controversy must not only be rooted in the existence of
an intra-corporate relationship, but must as well pertain to the enforcement of the
parties correlative rights and obligations under the Corporation Code and the
internal and intra-corporate regulatory rules of the corporation. If the relationship
and its incidents are merely incidental to the controversy or if there will still be
conflict even if the relationship does not exist, then no intra-corporate controversy
exists. Rhe Supreme Court grant the petition and reverse the decision of Court
of Appeals.

364

Hi-Yield, Inc. vs. Court of Appeals


GR 168863 590 SCRA 548 (2009)

FACTS:
Leonora H. Torres was a major stockholder of Honorio Torres & Sons, Inc.
(HTSI) owning 55% of the outstanding shares who allegedly, together with Glenn
and Stephanie Torres, entered into a loan obligation on behalf of HTSI and
without authority of the Board of Directors. However, the mortgage was
foreclosed and sold to Hi-Yield Realty. For this, Roberto Torres, a stockholder of
HTSI filed a petition for annulment of the Real Estate Mortgage and Foreclosure
Sale. Hi-Yield Realty moved for its dismissal but was denied by the RTC and
later by CA ruling that the case was a real action in a form of a derivative suit and
that the prayer for annulment of mortgage and foreclosure proceedings was
merely incidental to the main action. Hence, this petition for certiorari against CA.

ISSUE:

Whether the suit filed by Roberto Torres was a derivative suit.

RULING:
The Supreme Court (SC) ruled that the case was indeed a derivative suit which
does not require to be filed were the subject properties were located rather, may
be instituted were the corporations principal office is located. Moreover, SC
explained that Robert had fulfilled the requisites before a stockholder could file a
derivative suit: a.) He was a stockholder at the time of the transaction; b.) He has
tried to exhaust intra-corporate remedies (Found to be inapplicable in Roberts
case because the earnest effort of Robert to arrive at a compromise was
rendered inutile considering that those impleaded control the corporation); c.)
The cause of action devolves on the corporation.
In conclusion, SC settled that CA did not commit grave abuse of discretion hence
the petition was dismissed.

365

Stockholders of F. Guanzon vs. Register of Deeds of Manila


6 SCRA 373 (1962)
FACTS:
The 5 stockholders of F. Guanzon and Sons, Inc. executed a certificate of
liquidation of the assets of the said corporation, reciting that by virtue of a
resolution passed by them dissolving the corporation they have distributed the
among themselves, in proportion to their stockholdings, the assets of the
corporation as liquidating dividends.

When the certificate of liquidation was presented to the register of deeds of


manila so that the real properties of the corporation would be registered in their
names, the former denied said registration on the ground that said certificate of
liquidation is tantamount to transfer or conveyance of the corporations properties
hence the need to conform with the requirements for transfer such as, among
others payment of registration fees, and the number of parcels involved in the
distribution. In his view, thought the certificate involves a distribution of assets of
the corporation, in the last analysis it actually represents a transfer of the said
assets to the stockholders. Hence in substance it is transfer or conveyance. The
appellants argued that the certificate is not a conveyance much less a transfer
but mere distribution of the assets of the corporation which ceased to exist.

ISSUE
Whether the certificate of liquidation merely involves a distribution of the
corporations assets or a transfer of conveyance.

RULING
The certificate of liquidation executed by the stockholders is not and
cannot be considered as a mere distribution or partition of community property,
but rather a transfer or conveyance of the title of its assets to the stockholder.
A corporation is a juridical person separate and distinct from the person
composing it. Propertied registered in the name of the corporation are owned by
it as a separate entity. While shares of stocks constitute personal property, they
do not represent property of the corporation. The corporation has properties of its
own which consist mainly of real state. A share of stock only typifies n aliquot
part of the corporations property, or the right to share in the proceeds to that
extent when distributed according to law and equity, but its holder is not the
owner of ay part of the capital of the corporation nor is he entitled to any definite
portion of its property. The stockholder is not a co-owner or tenant in common of
the corporate property.
Since the purpose of the liquidation is to transfer their title from the
corporation to the stockholders in proportion their shareholdings, that transfer
366

cannot be effected without the corresponding deed of conveyance from the


corporation to the stockholders. The certificate of conveyance therefore is in the
nature of a transfer or conveyance.

367

Philippine National Bank vs. Bitulok Sawmill, Inc.


23 SCRA 1366 (1968)

FACTS:
The Philippine Lumber Distributing Agency, Inc., according to the lower court,
"was organized sometime in the early part of 1947 upon the initiative and
insistence of the late President Manuel Roxas of the Republic of the Philippines
who for the purpose, had called several conferences between him and the
subscribers and organizers of the Philippine Lumber Distributing Agency, Inc."
The purpose was praiseworthy, to insure a steady supply of lumber, which could
be sold at reasonable prices to enable the war sufferers to rehabilitate their
devastated homes. At the beginning, the lumber producers were reluctant to
organize the cooperative agency as they believed that it would not be easy to
eliminate from the retail trade the alien middlemen who had been in this
business from time immemorial, but because the late President Roxas made it
clear that such a cooperative agency would not be successful without a
substantial working capital which the lumber producers could not entirely
shoulder, and as an inducement he promised and agreed to finance the agency
by making the Government invest P9.00 by way of counterpart for every peso
that the members would invest therein." Accordingly, "the late President Roxas
instructed the Hon. Emilio Abello, then Executive Secretary and Chairman of the
Board of Directors of the Philippine National Bank, for the latter to grant said
agency an overdraft in the original sum of P250,000.00 which was later
increased to P350,000.00, which was approved by said Board of Directors of the
Philippine National Bank on July 28, 1947, payable on or before April 30, 1958,
with interest at the rate of 6% per annum, and secured by the chattel mortgages
on the stock of lumber of said agency." The Philippine Government did not
invest the P9.00 for every peso coming from defendant lumber producers. The
loan extended to the Philippine Lumber Distributing Agency by the Philippine
National Bank was not paid.

ISSUE:

Whether or not the non-compliance with a plain statutory command,


considering the persuasiveness of the plea that defendants-appellees would "not
have subscribed to the capital stock" of the Philippine Lumber Distributing
Agency "were it not for the assurance of the then President of the Republic that
the Government would back it up by investing P9.00 for every peso" subscribed,
a condition which was not fulfilled, such commitment not having been complied
with, be justified.
RULING:
NO.

It would be unwarranted to ascribe to the late President Roxas the view that the
payment of the stock subscriptions, as thus required by law, could be condoned
368

in the event that the counterpart fund to be invested by the Government would
not be available. Even if such were the case, however, and such a promise were
in fact made, to further the laudable purpose to which the proposed corporation
would be devoted and the possibility that the lumber producers would lose
money in the process, still the plain and specific wording of the applicable legal
provision as interpreted by this Court must be controlling. It is a well-settled
principle that with all the vast powers lodged in the Executive, he is still devoid of
the prerogative of suspending the operation of any statute or any of its terms.

369

Rivera vs. Florendo, 144 SCRA 643 (1986)


PARAS, J.:

FACTS:
Rivera, a registered stockholder of Fujuyama Hotel and Restaurant, Inc., is
allegedly just a front of a Japanese investor named Akasako. The latter sold the
shares registered under Rivera to the respondents. Initially, everybody agreed
to effect the sale including Rivera. However, upon the consummation of such,
Rivera refused to make the endorsement unless he is also paid.
Respondents attempted several times to have the shares registered but
were refused compliance by the corporation.

The trial court granted to the

respondents by a writ of preliminary injunction the right to manage the company


upon the filing of a bond.

ISSUES:
Whether Rivera has the right to refuse indorsement of the shares of stock
in question.
Whether the Corporation has the right to refuse the registration of the
respondents shares.
Whether the SEC has jurisdiction over the case.

RULING:
The Supreme Court denied the writ of preliminary mandatory injunction
and remanded the case to the lower court for a trial on the merits. As found in
Sec. 63 of the Corporation Code, shares of stock may be transferred by delivery
of the certificate after endorsement by the owner or his attorney-in-fact or other
person legally authorized to make the transfer. By this provision it is evident that
Riveras endorsement must be obtained before any transfer of the questioned
shares is affected.

On the matter of jurisdiction, the SEC does not have

jurisdiction of the case since the dispute is not an intra-corporate controversy.


What it simply involves is a conflict on the ownership of a group of shares
between the registered owner and an outside party.

Hence, because of this

conflict in ownership rights, a mandatory injunction cannot lie.

370

Embassy Farms, Inc. vs. Court of Appeals


188 SCRA 492 (1990)
FACTS:
It appears on record that sometime on August 2, 1984, Alexander G. Asuncion
(AGA for short) and Eduardo B. Evangelists (EBE for short) entered into a
Memorandum of Agreement (Annex "A" of the petition). Under said agreement
EBE obligated himself to transfer to AGA 19 parcels of agricultural land
registered in his name with an aggregate area of 104,447 square meters located
in Loma de Gato, Marilao, Bulacan, together with the stocks, equipment and
facilities of a piggery farm owned by Embassy Farms, Inc., a registered
corporation wherein ninety (90) per cent of its shares of stock is owned by EBE.
EBE also obligated himself to cede, transfer and convey "in a manner absolute
and irrevocable any and all of his shares of stocks" in Embassy Farins Inc. to
AGA or his nominees "until the total of said shares of stock so transferred shall
constitute 90% of the paid-in-equity of said corporation" within a reasonable time
from signing of the document. Likewise, EBE obligated to turnover to AGA the
effective control and management of the piggery upon the signing of the
agreement.
On the other hand, AGA obligated himself, upon signing of the agreement to pay
to EBE the total sum of close to P8,630,000.00. Within reasonable time from
signing of the agreement AGA obligated himself to organize and register a new
corporation with an authorized capital stock of P10,000,000.00 which upon
registration will take over all the rights and liabilities of AGA.
ISSUE:
Whether or not there has been an effective transfer of shares of stock from AGA
to other persons.
RULING:
NO.
There being no delivery of the indorsed shares of stock AGA cannot therefore
effectively transfer to other person or his nominees the undelivered shares of
stock. For an effective transfer of shares of stock the mode and manner of
transfer as prescribed by law must be followed (Navea v. Peers Marketing Corp.,
74 SCRA 65). As provided under Section 3 of Batas Pambansa Bilang 68,
otherwise known as the Corporation Code of the Philippines, shares of stock may
be transferred by delivery to the transferree of the certificate properly indorsed.
Title may be vested in the transferree by the delivery of the duly indorsed
certificate of stock (18 C.J.S. 928, cited in Rivera v. Florendo, 144 SCRA 643).
However, no transfer shall be valid, except as between the parties until the
transfer is properly recorded in the books of the corporation.
In the case at bar the indorsed certificate of stock was not actually delivered to
AGA so that EBE is still the controlling stockholder of Embassy Farms despite
the execution of the memorandum of agreement and the turn over of control and
management of the Embassy Farms to AGA on August 2, 1984.
When AGA filed on April 10, 1986 an action for the rescission of contracts with
damages the Pasig Court merely restored and established the status quo prior to
the execution of the memorandum of agreement by the issuance of a restraining
order on July 10, 1987 and the writ of preliminary injunction on July 30, 1987. It
would be unjust and unfair to allow AGA and his nominees to control and
371

manage the Embassy Farms despite the fact that AGA who is the source of their
supposed shares of stock in the corporation is not asking for the delivery of the
indorsed certificate of stock but for the rescission of the memorandum of
agreement. Rescission would result in mutual restitution (Magdalena Estate v.
Myrick, 71 Phil. 344) so it is but proper to allow EBE to manage the farm.

372

Tan vs. Securities and Exchange Commission, 206 SCRA 740 (1992)
Ponente: PARAS, J.:

Facts:
Respondent Corporation, Visayan Educational Supply was a registered
corporation on October 1979. Petitioner Alfonso S. Tan is an incorporator of said
corporation holding 400 shares of capital stock with a par value of 100,
evidenced by certificate of stock No.2. He was elected as President and held
such position until 1982 but remained in the Board of Directors until 1983. In
1981 two other incorporators withdraw from the corporation and assigned their
shares represented by certificate of stock No. 4 and No. 5 to the corporation and
were paid 40% of the corporate stock in trade. Due to the withdrawal of the said
incorporators and to complete the membership of the five(5) directors of the
board, petitioner sold fifty(50) shares out of his 400 shares of capital stock to his
brother, Angel S. Tan. Another incorporator also sold fifty (50) of his share of
capital stock to Teodora S. Tan in order to complete the minimum required
number of Board of Directors. As a result of the sale by petitioner of his fifty (50)
shares, certificate of stock No.2 was cancelled by the corporate secretary and
respondent Patricia Aguillar, by virtue of a resolution which was passed and
approved while petitioner was still a member of the Board of Directors of the
respondent Corporation. But said cancelled certificate of stock was not
surrendered by the petitioner.

The cancelled certificate of stock No.2 was replaced by the issuance of


certificate of stock No.6 for the new owner and newly elected member of the
board and at the same time the new Vice-President, Angel S. Tan, certificate of
stock No.8 for the reduced share of respondent Alfonso Tan. A request was
made from petitioner Alfonso Tan to make proper endorsement of the cancelled
certificate of stock No.2 as well as the newly issued certificate of stock No.8 but
he did not indorsed, instead he kept the cancelled certificate of stock No.2 and
return only certificate of stock No.8. When petitioner dislodge from his position
as President, he withdrew from the corporation on the condition that he be paid
with stock in trade in lieu of the value of his shares. After he withdraw of the
stocks, the board of directors, held in a meeting cancelled certificate of stock
No.2 and No.8 of the withdrawing stockholder, here in petitioner, in the corporate
stock and transfer book, and submitted thereof the minutes of the meeting to the
Securities and Exchange commission. The petitioner question the cancellation of
his aforesaid stock certificate No.2 and No.8 countering that the deprivation of his
shares despite non- endorsement or surrender was without the process required
in Sec.63 of the Corporation code.

The Securities and Exchange Commission Extension Office hearing officer


in Cebu, Mr. Felix Chan ruled and held that the cancellation of the petitioners
stock certificate No.2 and the subsequent insurance of stock certificate No.8
373

were null and void. On appeal, the Securities and Exchange Commission en
banc unanimously overturned the decision of the hearing officer hence, a petition
for certiorari was filed by the petitioner before the Supreme Court.

Issue:

Whether or not the cancellation and transfer of petitioners shares and certificate
of stock No.2 as well as the issuance and cancellation of the certificate of Stock
No.8 was null or void and runs counter with the Provision of Section 63 of the
Corporation Code.

Ruling:
The court ruled that the requirement of Section 63 regarding the actual delivery
and endorsement of the certificate in question for a valid transfer of certificate of
stock is not mandatory but merely permissive become of the use of the term
may. From the given facts, there was already delivery of unendorsed stock
certificate No 2, which is essential to the insurance of stock certificate No 6. It
was only returned to the petitioner for his proper endorsement which he
deliberately failed to do so. Since Stock Certificate No.2 was already cancelled
and such cancellation was reported the respondent Commission, there was no
more necessity for the same certificate to be endorsed by the petitioner.
Moreover said transfer was earlier recorded or registered in the corporate stock
and transfer book. Furthermore, the certificate is not stock in the corporation but
is merely evidence of the holders interest and status in the corporation, his
ownership of the share represented thereby, but is not in law the equivalent of
ownership. The Supreme Court affirmed the decision of the Securities and
Exchange Commission declaring the cancellation and transfer of stock petitioner
valid.

374

Rural Bank of Salinas, Inc vs. Court of Appeals, 210 SCRA 510 (1992)
Ponente: Justice Paras
Facts:
The original owner of 473 shares was Clemente Guerrero. He executed a
Special Power of Attorney in favor of his Spouse Melania Guerrero. She then
executed 2 deeds of assignment: 1. In favor of Luz Andico (457 shares),
Wilhelmina Rosales (10 shares) and Francisco Guerrero Jr. (5 shares); 2.
Francisco Guerrero Sr. (1 share).
When Melania Guerrero requested for the transfer in the Banks stock so
assigned, the cancellation of stock certificates in the name of Clemente G.
Guerrero, and the issuance of new stock certificates covering the transferred
shares of stocks in the name of the new owners thereof, however, the Bank
denied the request.
On December 5, 1980, Melania Guerrero filed with SEC and action for
mandamus against Rural Bank of Salinas, its President and Corporate Secretary
which was granted by SEC even SEC en banc.
Issue:
Whether the Rural Bank of Salinas can refuse registration of the
transferred shares in its stock and transfer book.
Ruling:
No. The right of a transferee/assignee to have stocks transferred to his
name is an inherent right flowing from his ownership of the stocks. The refusal of
the petitioner Rural Bank of Salinas is in contravention of Sec. 63 of the
Corporation Code.

375

China Banking Corporation vs. Court of Appeals


270 SCRA 503 (1997)
Facts:

On 21 August 1974, Galicano Calapatia, Jr., a stockholder of Valley Golf &


Country Club, Inc. (VGCCI), pledged his Stock Certificate 1219 to China Banking
Corporation (CBC). On 16 September 1974, CBC wrote VGCCI requesting that
the pledge agreement be recorded in its books. In a letter dated 27 September
1974, VGCCI replied that the deed of pledge executed by Calapatia in CBC's
favor was duly noted in its corporate books. On 3 August 1983, Calapatia
obtained a loan of P20,000.00 from CBC, payment of which was secured by the
pledge agreement still existing between Calapatia and CBC. Due to Calapatia's
failure to pay his obligation, CBC, on 12 April 1985, filed a petition for
extrajudicial foreclosure before Notary Public Antonio T. de Vera of Manila,
requesting the latter to conduct a public auction sale of the pledged stock. On 14
May 1985, CBC informed VGCCI of the foreclosure proceedings and requested
that the pledged stock be transferred to its name and the same be recorded in
the corporate books. However, on 15 July 1985, VGCCI wrote CBC expressing
its inability to accede to CBC's request in view of Calapatia's unsettled accounts
with the club. Despite the foregoing, Notary Public de Vera held a public auction
on 17 September 1985 and CBC emerged as the highest bidder at P20,000.00
for the pledged stock. Consequently, CBC was issued the corresponding
certificate of sale. On 21 November 1985, VGCCI sent Calapatia a notice
demanding full payment of his overdue account in the amount of P18,783.24.
Said notice was followed by a demand letter dated 12 December 1985 for the
same amount and another notice dated 22 November 1986 for P23,483.24. On 4
December 1986, VGCCI caused to be published in the newspaper Daily Express
a notice of auction sale of a number of its stock certificates, to be held on 10
December 1986 at 10:00 a.m. Included therein was Calapatia's own share of
stock (Stock Certificate 1219). Through a letter dated 15 December 1986, VGCCI
informed Calapatia of the termination of his membership due to the sale of his
share of stock in the 10 December 1986 auction. On 5 May 1989, CBC advised
VGCCI that it is the new owner of Calapatia's Stock Certificate 1219 by virtue of
being the highest bidder in the 17 September 1985 auction and requested that a
new certificate of stock be issued in its name. On 2 March 1990, VGCCI replied
that "for reason of delinquency" Calapatia's stock was sold at the public auction
held on 10 December 1986 for P25,000.00. On 9 March 1990, CBC protested the
376

sale by VGCCI of the subject share of stock and thereafter filed a case with the
Regional Trial Court of Makati for the nullification of the 10 December 1986
auction and for the issuance of a new stock certificate in its name. On 18 June
1990, the Regional Trial Court of Makati dismissed the complaint for lack of
jurisdiction over the subject matter on the theory that it involves an intracorporate dispute and on 27 August 1990 denied CBC's motion for
reconsideration. On 20 September 1990, CBC filed a complaint with the
Securities and Exchange Commission (SEC) for the nullification of the sale of
Calapatia's stock by VGCCI; the cancellation of any new stock certificate issued
pursuant thereto; for the issuance of a new certificate in petitioner's name; and
for damages, attorney's fees and costs of litigation. On 3 January 1992, SEC
Hearing Officer Manuel P. Perea rendered a decision in favor of VGCCI, stating
in the main that considering that the said share is delinquent, VGCCI had valid
reason not to transfer the share in the name of CBC in the books of VGCCI until
liquidation of delinquency. Consequently, the case was dismissed. On 14 April
1992, Hearing Officer Perea denied CBC's motion for reconsideration. CBC
appealed to the SEC en banc and on 4 June 1993, the Commission issued an
order reversing the decision of its hearing officer; holding that CBC has a prior
right over the pledged share and because of pledgor's failure to pay the principal
debt upon maturity, CBC can proceed with the foreclosure of the pledged share;
declaring that the auction sale conducted by VGCCI on 10 December 1986 is
declared NULL and VOID; and ordering VGCCI to issue another membership
certificate in the name of CBC. VGCCI sought reconsideration of the order.
However, the SEC denied the same in its resolution dated 7 December 1993.
The sudden turn of events sent VGCCI to seek redress from the Court of
Appeals. On 15 August 1994, the Court of Appeals rendered its decision
nullifying and setting aside the orders of the SEC and its hearing officer on
ground of lack of jurisdiction over the subject matter and, consequently,
dismissed CBC's original complaint. The Court of Appeals declared that the
controversy between CBC and VGCCI is not intra-corporate; nullifying the SEC
orders and dismissing CBCs complaint. CBC moved for reconsideration but the
same was denied by the Court of Appeals in its resolution dated 5 October 1994.
CBC filed the petition for review on certiorari.

Issue:

Whether CBC is bound by VGCCI's by-laws.


377

Ruling:

In order to be bound, the third party must have acquired knowledge of the
pertinent by-laws at the time the transaction or agreement between said third
party and the shareholder was entered into. Herein, at the time the pledge
agreement was executed. VGCCI could have easily informed CBC of its by-laws
when it sent notice formally recognizing CBC as pledgee of one of its shares
registered in Calapatia's name. CBC's belated notice of said by-laws at the time
of foreclosure will not suffice. By-laws signifies the rules and regulations or
private laws enacted by the corporation to regulate, govern and control its own
actions, affairs and concerns and its stockholders or members and directors and
officers with relation thereto and among themselves in their relation to it. In other
words, by-laws are the relatively permanent and continuing rules of action
adopted by the corporation for its own government and that of the individuals
composing it and having the direction, management and control of its affairs, in
whole or in part, in the management and control of its affairs and activities. The
purpose of a by-law is to regulate the conduct and define the duties of the
members towards the corporation and among themselves. They are self-imposed
and, although adopted pursuant to statutory authority, have no status as public
law. Therefore, it is the generally accepted rule that third persons are not bound
by by-laws, except when they have knowledge of the provisions either actually or
constructively. For the exception to the general accepted rule that third persons
are not bound by by-laws to be applicable and binding upon the pledgee,
knowledge of the provisions of the VGCCI By-laws must be acquired at the time
the pledge agreement was contracted. Knowledge of said provisions, either
actual or constructive, at the time of foreclosure will not affect pledgee's right
over the pledged share. Article 2087 of the Civil Code provides that it is also of
the essence of these contracts that when the principal obligation becomes due,
the things in which the pledge or mortgage consists maybe alienated for the
payment to the creditor. Further, VGCCI's contention that CBC is duty-bound to
know its by-laws because of Article 2099 of the Civil Code which stipulates that
the creditor must take care of the thing pledged with the diligence of a good
father of a family, fails to convince. CBC was never informed of Calapatia's
unpaid accounts and the restrictive provisions in VGCCI's by-laws. Furthermore,
Section 63 of the Corporation Code which provides that "no shares of stock
against which the corporation holds any unpaid claim shall be transferable in the
books of the corporation" cannot be utilized by VGCCI. The term "unpaid claim"
refers to "any unpaid claim arising from unpaid subscription, and not to any
378

indebtedness which a subscriber or stockholder may owe the corporation arising


from any other transaction." Herein, the subscription for the share in question has
been fully paid as evidenced by the issuance of Membership Certificate 1219.
What Calapatia owed the corporation were merely the monthly dues. Hence,
Section 63 does not apply.

379

Bitong vs. Court of Appeals, 292 SCRA 503 (1998)


BELLOSILLO, J.:

FACTS:

Petitioner Alleged before the SEC that she had been the Treasurer and a
Member of the Board of Directors of Mr. & Ms. from the time it was incorporated
on 29 October 1976 to 11 April 1989, and was the registered owner of 1,000
shares of stock out of the 4,088 total outstanding shares, petitioner complained
of irregularities committed from 1983 to 1987 by Eugenia D. Apostol, President
and Chairperson of the Board of Directors. Petitioner claimed that except for the
sale of the name Philippine Inquirer to Philippine Daily Inquirer (PDI hereafter) all
other transactions and agreements entered into by Mr. & Ms. with PDI were not
supported by any bond and/or stockholders' resolution. And, upon instructions of
Eugenia D. Apostol, Mr. & Ms. made several cash advances to PDI on various
occasions amounting to P3.276 million. On some of these borrowings PDI paid
no interest whatsoever. Despite the fact that the advances made by Mr. & Ms. to
PDI were booked as advances to an affiliate, there existed no board or
stockholders' resolution, contract nor any other document which could legally
authorize the creation of and support to an affiliate.

Petitioner further alleged that respondents Eugenia and Jose Apostol were
stockholders, directors and officers in both Mr. & Ms. and PDI. In fact on 2 May
1986 respondents Eugenia D. Apostol, Leticia J. Magsanoc and Adoracion G.
Nuyda subscribed to PDI shares of stock at P50,000.00 each or a total of
P150,000.00.

ISSUE:

Whether or not the petitioner is the holder of the proper certificates of share of
stock.

HELD:

380

NO.The certificate of stock itself once issued is a continuing affirmation or


representation that the stock described therein is valid and genuine and is at
least prima facie evidence that it was legally issued in the absence of evidence to
the contrary. However, this presumption may be rebutted. 13 Similarly, books and
records of a corporation which include even the stock and transfer book are
generally admissible in evidence in favor of or against the corporation and its
members to prove the corporate acts, its financial status and other matters
including one's status as a stockholder. They are ordinarily the best evidence of
corporate acts and proceedings.

However, the books and records of a corporation are not conclusive even against
the corporation but are prima facie evidence only. Parol evidence may be
admitted to supply omissions in the records, explain ambiguities, or show what
transpired where no records were kept, or in some cases where such records
were contradicted. The effect of entries in the books of the corporation which
purport to be regular records of the proceedings of its board of directors or
stockholders can be destroyed by testimony of a more conclusive character than
mere suspicion that there was an irregularity in the manner in which the books
were kept.

Thus, while petitioner asserts in her petition that Certificate of Stock No. 008
dated 25 July 1983 was issued in her name, private respondents argue that this
certificate was signed by respondent Eugenia D. Apostol as President only in
1989 and was fraudulently antedated by petitioner who had possession of the
Certificate Book and the Stock and Transfer Book.

381

Thomson vs. Court of Appeals, 298 SCRA 280 (1998)


Ponente: Justice Quisumbing
Facts:
Petitioner Marsh Thomson (Thomson) was the Executive Vice-President
and, later on, the Management Consultant of private respondent, the American
Chamber of Commerce of the Philippines, Inc. (AmCham) for over ten years.
While petitioner was still working with private respondent, his superior, A. Lewis
Burridge, retired as AmCham's President. Before Burridge decided to return to
his home country, he wanted to transfer his proprietary share in the Manila Polo
Club (MPC) to petitioner. However, through the intercession of Burridge, private
respondent paid for the share but had it listed in petitioner's name. This was
made clear in an employment advice dated January 13, 1986, wherein petitioner
was informed by private respondent. On April 25, 1986, Burridge transferred said
proprietary share to petitioner, as confirmed in a letter of notification to the
Manila Polo Club. Upon his admission as a new member of the MPC, petitioner
paid the transfer fee of P40,000.00 from his own funds; but private respondent
subsequently reimbursed this amount. On November 19, 1986, MPC issued
Proprietary Membership Certificate Number 3398 in favor of petitioner. But
petitioner, however, failed to execute a document recognizing private
respondent's beneficial ownership over said share. Following AmCham's policy
and practice, there was a yearly renewal of employment contract between the
petitioner and private respondent. But petitioner never acknowledged that private
respondent is the beneficial owner of the share. When petitioner's contract of
employment was up for renewal in 1989, he notified private respondent that he
would no longer be available. The private respondent asked the petitioner to stay
on for another six (6) months. Pending the negotiation for the consultancy
arrangement, private respondent executed a Release and Quitclaim, stating that
AMCHAM, its directors, officers and assigns, employees and/or representatives
do hereby release, waive, abandon and discharge J. MARSH THOMSON from
any and all existing claims. The quitclaim, expressed in general terms, did not
mention specifically the MPC share. On April 5, 1990, private respondent,
through counsel sent a letter to the petitioner demanding the return and delivery
of the MPC share. Failing to get a favorable response, private respondent filed
on May 15, 1990, a complaint against petitioner. The trial court awarded the MPC
share to defendant (petitioner now) on the ground that the Articles of
Incorporation and By-laws of Manila Polo Club prohibit artificial persons, such as

382

corporations, to be club members. Private respondent appealed to the Court of


Appeals which in turn reversed the RTC decision.
Issue/s:
Whether the Court of Appeals in declaring AmCham to be the real owner of the
shares
Ruling:
The Supreme court ruled that the Court of Appeals did not err and affirmed the
appellate courts decision. It held that there is a trust relationship that existed
between the parties. Moreover, petitioner failed to present evidence to support
his allegation of being merely a debtor when the private respondent paid the
purchase price of the MPC share. Applicable here is the rule that a trust arises in
favor of one who pays the purchase money of property in the name of another,
because of the presumption that he who pays for a thing intends a beneficial
interest therein for himself. Although petitioner initiated the acquisition of the
share, evidence on record shows that private respondent acquired said share
with its funds. Petitioner did not pay for said share, although he later wanted to,
but according to his own terms, particularly the price. The quitclaim executed by
private respondent does not clearly show the intent to include therein the
ownership over the MPC share. Private respondent even asserts that at the time
the Release and Quitclaim was executed on September 29, 1989, the ownership
of the MPC share was not controversial nor contested. Settled is the rule that a
waiver to be valid and effective must, in the first place, be couched in clear and
unequivocal terms which leave no doubt as to the intention of a party to give up a
right or benefit which legally pertains to him. A waiver may not be attributed to a
person when the terms thereof do not explicitly and clearly evidence an intent to
abandon a right vested in such person. If we apply the standard rule that waiver
must be cast in clear and unequivocal terms, then clearly the general terms of
the cited release and quitclaim indicates merely a clearance from general
accountability, not specifically a waiver of AmCham's beneficial ownership of the
disputed shares.

383

Garcia vs. Jomouad, 323 SCRA 424 (2000)


KAPUNAN, J.:
FACTS:
Petitioner filed an action for injunction with prayer for preliminary injunction
against respondents spouses and ex-officio sheriff of Cebu. Said action stemmed
from an earlier case for collection of sum of money, filed by the spouses Atinon
against Jaime Dico. In that case (collection of sum of money), the trial court
rendered judgment ordering Dico to pay the spouses Atinon the sum of
P900,000.00 plus interests. After said judgment became final and executory,
respondent sheriff proceeded with its execution. In the course thereof, the
Proprietary Ownership Certificate (POC) No. 0668 in the Cebu Country Club,
which was in the name of Dico, was levied on and scheduled for public auction.
Claiming ownership over the subject certificate, petitioner filed the aforesaid
action for injunction with prayer for preliminary injunction to enjoin respondents
from proceeding with the auction.
After trial, the lower court rendered its Decision, dismissing petitioner's complaint
for injunction for lack of merit. On appeal, the CA affirmed in toto the decision of
the RTC upon finding that it committed no reversible error in rendering the same.
Hence, this petition.
ISSUE:
Whether a bona fide transfer of the shares of a corporation, not registered or
noted in the books of the corporation, is valid as against a subsequent lawful
attachment of said shares, regardless of whether the attaching creditor had
actual notice of said transfer or not?
RULING:
Section 63 of the Corporation Code reads:
"Sec. 63 Certificate of stock and transfer of shares. - The capital
stock of corporations shall be divided into shares for which
certificates signed by the president or vice- president, countersigned
by the secretary or assistant secretary, and sealed with the seal of
the corporation shall be issued in accordance with the by-laws.
Shares of stock so issued are personal property and may be
384

transferred by delivery of the certificate or certificates indorsed by


the owner or his attorney-in-fact or other person legally authorized to
make the transfer. No transfer, however, shall be valid, except as
between the parties, until the transfer is recorded in the books
of the corporation showing the names of the parties to the
transaction, the date of the transfer, the number of the
certificate or certificates and the number of shares transferred.
No shares of stock against which the corporation holds any unpaid
claim shall be transferable in the books of the corporation."
The Court hold that the transfer of the subject certificate made by Dico to
petitioner was not valid as to the spouses Atinon, the judgment creditors, as the
same still stood in the name of Dico, the judgment debtor, at the time of the levy
on execution. In addition, as correctly ruled by the CA, the entry in the minutes of
the meeting of the Club's board of directors noting the resignation of Dico as
proprietary member thereof does not constitute compliance with Section 63 of the
Corporation Code. Said provision of law strictly requires the recording of the
transfer in the books of the corporation, and not elsewhere, to be valid as against
third parties.

385

Rural Bank of Lipa City, Inc. vs. Court of appeals, 366 SCRA 188
(2001). See also Batangas Laguna Tayabas Bus company, Inc.,
et al., vs. Benjamin Bitanga, et al., 362 SCRA 635 (2001)
YNARES-SANTIAGO, J.:
Facts: Private respondent Reynaldo Villanueva, Sr., a stockholder of the Rural
Bank executed a Deed of Assignment, which he assigned his shares, as well as
those of eight (8) other shareholders under his control with a total of 10,467
shares, in favor of the stockholders of the Bank represented by its directors.
When the Spouses failed to settle their obligation amounting to P400000 covered
by an Agreement on the due date, the Board sent them a letter demanding: (1)
the surrender of all the stock certificates issued to them; and (2) the delivery of
sufficient collateral to secure the balance of their debt which the latter ignored.
On January 15, 1994, the stockholders of the Bank met to elect the new directors
and set of officers for the year 1994. The Villanuevas were not notified of said
meeting. The legality of the meeting was questioned by Atty. Amado Ignacio,
counsel for the Villanueva spouses.
In reply, the new set of officers informed Atty. Ignacio that the Villanuevas were
no longer entitled to notice of the said meeting since they had relinquished their
rights as stockholders in favor of the Bank.
The spouses filed with the SEC, a petition for annulment of the stockholders'
meeting and election of directors and officers, with damages and prayer for
preliminary injunction against newly-elected officers.
The SEC issued a temporary restraining order enjoining the petitioners herein,
from acting as directors and officers of the Bank, and from performing their duties
and functions as such.
Petitioners, moved for the lifting of the temporary restraining order and the
dismissal of the petition for lack of merit, and for the upholding of the validity of
the stockholders' meeting and election of directors and officers.
Villanuevas' application for the issuance of a writ of preliminary injunction was
denied.
Upon a motion for reconsideration, the writ was granted upon finding that since
the Villanuevas' have not disposed of their shares, whether voluntarily or
386

involuntarily, they were still stockholders entitled to notice of the annual


stockholders' meeting was sustained by the SEC.
A writ of preliminary injunction was issued enjoining the petitioners from acting as
directors and officers of the bank.
Petitioners filed an urgent motion to quash the writ of preliminary injunction.
SEC Hearing Officer granted Villanuevas filed an Omnibus Motion praying that
the meeting and election of officers scheduled on January 14, 1995 be
suspended or held in abeyance, and that the 1993 Board of Directors be allowed,
in the meantime, to act as such.
A petition for Certiorari and Annulment with Damages was filed by the Rural
Bank, its directors and officers before the SEC en banc, which the latter denied.
The decision states that petitioners could not show any proof of despotic or
arbitrary exercise of discretion committed by the hearing officer in issuing the
assailed and the non-delivery of the stock certificate does not make the transfer
of the shares of stock effective.
Motion for reconsideration was likewise denied by SEC en banc.
A Court of Appeals dismissed the petition for review upon finding that public
respondent is correct in holding that the Hearing Officer did not commit grave
abuse of discretion and the questioned Orders issued by the Hearing Officer
were based on pertinent law and the facts of the case.
A motion for reconsideration was likewise denied.
Hence, this present petition.
Issue: Did spouses Villanueva validly transfer their shares notwithstanding the
execution of the deed of assignment in favor of the petitioners?
Ruling: The Corporation Code specifically provides:
SECTION 63. Certificate of stock and transfer of shares. The capital
stock of stock corporations shall be divided into shares for which
certificates signed by the president or vice president, countersigned by the
secretary or assistant secretary, and sealed with the seal of the
corporation shall be issued in accordance with the by-laws. Shares of
stocks so issued are personal property and may be transferred by delivery
of the certificate or certificates indorsed by the owner or his attorney-in-fact
387

or other person legally authorized to make the transfer. No transfer,


however, shall be valid, except as between the parties, until the transfer is
recorded in the books of the corporation so as to show the names of the
parties to the transaction, the date of the transfer, the number of the
certificate or certificates and the number of shares transferred.
No shares of stock against which the corporation holds any unpaid claim
shall be transferable in the books of the corporation. (Emphasis ours)
xxxx.
We have uniformly held that for a valid transfer of stocks, there must be strict
compliance with the mode of transfer prescribed by law. The requirements are:
(a) There must be delivery of the stock certificate: (b) The certificate must be
endorsed by the owner or his attorney-in-fact or other persons legally authorized
to make the transfer; and (c) To be valid against third parties, the transfer must
be recorded in the books of the corporation. As it is, compliance with any of these
requisites has not been clearly and sufficiently shown.
It may be argued that despite non-compliance with the requisite endorsement
and delivery, the assignment was valid between the parties, meaning the private
respondents as assignors and the petitioners as assignees. While the
assignment may be valid and binding on the petitioners and private respondents,
it does not necessarily make the transfer effective. Consequently, the petitioners,
as mere assignees, cannot enjoy the status of a stockholder, cannot vote nor be
voted for, and will not be entitled to dividends, insofar as the assigned shares are
concerned Parenthetically, the private respondents cannot, as yet, be deprived of
their rights as stockholders, until and unless the issue of ownership and transfer
of the shares in question is resolved with finality.
There being no showing that any of the requisites mandated by law was complied
with, the SEC Hearing Officer did not abuse his discretion in granting the
issuance of the preliminary injunction prayed for by petitioners in SEC Case No.
02-94-4683 (herein private respondents). Accordingly, the order of the SEC en
banc affirming the ruling of the SEC Hearing Officer, and the Court of Appeals
decision upholding the SEC en banc order, are valid and in accordance with law
and jurisprudence, thus warranting the denial of the instant petition for review.

388

Ponce vs. Alsons Cement Corporation 393, SCRA 602 (2002)


QUISUMBING, J.:
FACTS:

Vicente C. Ponce and Fausto Gaid, incorporator of Victory Cement Corporation


(VCC), executed
a
Deed
of
Undertaking
and
Indorsement
whereby Gaid acknowledges that Ponce is the owner of the shares and he was
therefore assigning/endorsing it to Ponce.
VCC was renamed Floro Cement Corporation (FCC) and then to Alsons Cement
Corporation (ACC). However, conflict arose when no certificates of stock
corresponding to the 239,500 subscribed and fully paid shares of Gaid were
issued in the name of Gaid, the plaintiff.

Despite repeated demands, the ACC refused to issue the certificates of stocks.
During the hearing before Securities and Exchange Commission, a motion to
dismiss was granted while on appeal, the Commission En Banc reversed the
decision. Ponce filed a complaint with SEC for mandamus which the Court of
Appeals dismissed for failure to state cause of action.

ISSUE:

Whether the stock certificates of Gaid can be transferred to Ponce.

HELD:

NO. the Certificate of Stocks cannot be transferred.

The Corporation Code expressly provides:


SEC. 63. Certificate of stock and transfer of shares.The capital stock of stock
corporations shall be divided into shares for which certificates signed by the
president or vice-president, countersigned by the secretary or assistant
secretary, and sealed with the seal of the corporation shall be issued in
accordance with the by-laws. Shares of stock so issued are personal property
and may be transferred by delivery of the certificate or certificates indorsed by
the owner or his attorney-in-fact or other person legally authorized to make the
transfer. No transfer, however, shall be valid, except as between the parties,
until the transfer is recorded in the books of the corporation so as to show the
389

names of the parties to the transaction, the date of the transfer, the number of
the certificate or certificates and the number of shares transferred.

The stock and transfer book is the basis for ascertaining the persons entitled to
the rights and subject to the liabilities of a stockholder. Where a transferee is not
yet recognized as a stockholder, the corporation is under no specific legal duty to
issue stock certificates in the transferees name. In the present case,
a mandamus should not issue to compel the secretary of a corporation to make a
transfer of the stock on the books of the company unless it affirmatively appears
that he has failed or refused so to do, upon the demand either of the person in
whose name the stock is registered, or of some person holding a power of
attorney for that purpose from the registered owner of the stock.

Mere indorsement shall not suffice to be recognized as such by the corporation


and its officers, in the absence of express instructions of the registered owner to
make such transfer to the indorsee, or a power of attorney authorizing such
transfer.

390

Republic of the Phil. (PCGG) vs. Sandiganbayan and Victor Africa,


402 SCRA 84 (2003)

FACTS: A case was filed against the respondents for before the Sandiganbayan
(SB) for reconveyance, reversion, accounting, restitution, and damages in
relation to the allegation that respondents illegally manipulated the purchase of
the

major

shareholdings

of

Cable

and

Wireless

Limited

in

Eastern

Telecommunications Philippines, Inc. (ETPI). This case docketed as Civil Case


No. 0009 spawned numerous incidental cases, among them, Civil Case No.
0130, a petition instituted by Victor Africa (Son of the late Jose Africa) which
sought to nullify the orders of the PCGG directing him to account for the alleged
sequestered shares in ETPI and to cease and desist from exercising voting
rights. The present respondents were not made parties either in Civil Case No.
0130. In the former case, Victor Africa (Africa) was not impleaded in and so is
plainly not a party thereto.

In the proceedings for Civil Case No. 0130, testimony of Mr. Maurice V. Bane
(former director and treasurer-in-trust of ETPI) was taken by way of deposition
upon oral examination (Bane deposition) before Consul General Ernesto Castro
of the Philippine Embassy in London, England. The purpose was for Bane to
identify and testify on the facts set forth in his affidavit so as to prove the
ownership issue in favor of the petitioner and/or establish the prima facie factual
foundation for sequestration of ETPIs Class A stock.

As to Civil Case No. 009, the petitioner filed a motion (1st Motion) to adopt the
testimonies of the witnesses in Civil Case No. 0130, including the deposition of
Mr. Maurice Bane which was denied by SB in its April 1998 Resolution because
he was not available for cross-examination. The petitioners did not in any way
question the 1998 resolution, and instead made its Formal Offer of Evidence on
December 14, 1999. Significantly, the Bane deposition was not included as part
of its offered exhibits. In rectifying this, they filed a second motion with prayer for
re-opening of the case for the purpose of introducing additional evidence and
requested the court to take judicial notice of the facts established by the Bane
deposition. This was however denied by the SB in its November 6, 2000
resolution (2000 resolution). A third motion was filed by the petitioners on
November 16, 2001 seeking once more to admit the Bane deposition which the
391

SB denied for the reason that the 1998 resolution has become final in view of the
petitioners failure to file a motion for reconsideration or appeal within the 15-day
reglementary period.

ISSUE/S:

1. Whether the SB committed grave abuse of discretion in holding that the 1998
resolution has already attained finality and in refusing to re-open the case.

2. Whether the Bane deposition is admissible under the rules of court and under
the principle of judicial notice.

HELD: 1. The court ruled that the SBs ruling on the finality of its 1998 resolution
was legally erroneous but did not constitute grave abuse of discretion due to the
absence of a clear showing that its action was a capricious and whimsical
exercise of judgment affecting its exercise of jurisdiction. The SBs ruling,
although an erroneous legal conclusion was only an error of judgment, or, at
best, an abuse of discretion but not a grave one.

The 1998 resolution is an interlocutory decision, thus petition for certiorari is still
premature since the rules of court provides that certiorari should be availed in a
situation where neither an appeal nor any plain, speedy and adequate remedy in
the ordinary course of law is available to the aggrieved party except if such
remedy is inadequate or insufficient in relieving the aggrieved party of the
injurious effects of the order complained of. At the time of the 1st motion, the
presentation of evidence has not yet concluded. The remedy after the denial of
the 1st motion should have been for the petitioner to move for a reconsideration
to assert and even clarify its position on the admission of the Bane deposition.
But upon denial of the 2nd motion, petitioners should have already questioned it
by way of certiorari since it effectively foreclosed all avenues available to it for the
consideration of the Bane deposition. Instead of doing so, however, the petitioner
allowed the 60-day reglementary period, under Section 4, Rule 65 of the Rules of
Court, to lapse, and proceeded to file its 3rd motion.
392

However, the court ruled that the Sandiganbayan gravely abused its discretion in
ultimately refusing to reopen the case for the purpose of introducing and
admitting in evidence the Bane deposition. The Rules of Court does not prohibit a
party from requesting the court to allow it to present additional evidence even
after it has rested its case provided that the evidence is rebuttal in character,
whose necessity, for instance, arose from the shifting of the burden of evidence
from one party to the other; or where the evidence sought to be presented is in
the nature of newly discovered evidence. At the time the petitioner moved to reopen its case, the respondents had not yet even presented their evidence in
chief. The respondents, therefore, would not have been prejudiced by allowing
the petitioners introduction of the Bane deposition, which was concededly
omitted through oversight.

2. Despite the cases being closely related, admissibility of the Bane deposition
still needs to comply with the rules of court on the admissibility of testimonies or
deposition taken in a different proceeding. Depositions are not meant as
substitute for the actual testimony in open court of a party or witness. Generally,
the deponent must be presented for oral examination in open court at the trial or
hearing otherwise, the adverse party may oppose it as mere hearsay. Crossexamination will test the truthfulness of the statements of the witness; it is an
essential safeguard of the accuracy and completeness of a testimony.
Depositions from the former trial may be introduced as evidence provided that
the parties to the first proceeding must be the same as the parties to the later
proceeding. In the present case, the petitioner failed to establish the identity of
interest or privity between the opponents of the two cases. While Victor Africa is
the son of the late respondent Jose Africa, the deposition is admissible only
against him as an ETPI stockholder who filed Civil Case No. 0130.

Further, the rule of judicial notice is not applicable in this case as it would create
confusion between the two cases. It is the duty of the petitioner, as a partylitigant, to properly lay before the court the evidence it relies upon in support of
the relief it seeks, instead of imposing that same duty on the court.

The petition was DISMISSED for lack of merit


393

Republic of the Phils. vs. Sandiganbayan, ibid.

The Facts:
After the EDSA Revolution, Philippine Communication Satellite Corporation
(PHILCOMSAT) and Philippine Overseas Telecommunications Corporation
(POTC) were among those sequestered by the Philippine Commission on Good
Government. PHILCOMSAT, whose majority shareholders are the same families
who control and own POTC (Ilusorio, Nieto, Poblador, Africa, Benedicto, Ponce
Enrile and Elizalde), own 81% of the outstanding capital stock of Phicomsat
Holdings Corporation (PHC). During the Arroyo administration, respondents
Locsin, Andal and Jalandoni were appointed nominee directors representing the
Republic of the Philippines through the PCGG in the board of directors of
PHILCOMSAT. They sided with the Nieto Group.

In the July 28, 2004 meeting of the POTC and PHICOMSAT, Victor Africa was
elected as one of the directors in the Africa-Bildner Group; he was designated as
the POTC proxy to the PHILCOMSAT stockholders meeting. While Andal and
Nieto were elected Directors to both POTC and PHICOMSAT, they did not
accept their nominations. Instead, the Nieto-PCGG group on the other hand,
held their own election for the PHILCOMSAT on August 9, 2004, where Manuel
Nieto Jr. And Enrique Locsin were elected as Chairman and President of
PHILCOMSAT. They also issued a proxy in favour of Nieto Jr/Locsin authorising
them to represent PHILCOMSAT and vote the PHILCOMSAT shares in the PHC
stockholders meeting on August 28, 2004.

During the August 31, 2004 annual stockholders meeting of the PHC conducted
by the PCGG-Nieto group, the following were elected: Locsin (Director and
Acting Chairman); Oliverio Laperal (Director and Vice-Chairman); Manuel H.
Nieto, Jr. (Director, President and Chief Executive Officer); Philip G. Brodett
(Director and Vice-President); Andal (Director, Treasurer and Chief Financial
Officer); Roberto V. San Jose (Director and Corporate Secretary); Jalandoni,
Lokin, Jr., Prudencio Somera, Roberto Abad and Benito Araneta as Directors.

Thereafter, various suits were filed against each other by the two factions to gain
legitimacy of their election as respective officers of POTC and PHILCOMSAT.
The Africa group sought the invalidation of the proxy issued in favor of Nieto, Jr.
and/or Locsin and consequent nullification of the elections held during the annual
stockholders meeting of PHC on August 31, 2004 (Civil Case No. 04-1049 of
RTC, Makati City, Branch 138). Prior to this, there was the pending case
involving the compromise agreement dated June 28, 1996 entered into by Atty.
Potenciano Ilusorio with the Republic of the Philippines and the PCGG relative to
the Ilusorio familys shareholdings in POTC, including those shares forcibly taken
from him by former President Ferdinand Marcos which were placed in the name
of Independent Realty Corporation (IRC) and Mid-Pasig Land Development (Mid394

Pasig). By Decision dated June 15, 2005, thE Court affirmed the validity of the
said compromise agreement in G.R. Nos. 141796 and 141804. As a result of the
compromise agreement, the Ilusorio, Africa, Poblador, Benedicto and Ponce
Enrile families gained majority control (51.37%) and the Nieto family and PCGG
became the minority.

Victor Africa, the president and CEO of the PHILCOMSAT, then wrote a letter to
the PHC Board, informing it that PHILCOMSAT will exercise its right of inspection
over the books, records, papers, etc. pertinent to the business transactions of
PHC for the 3rd quarter of 2005, specifically the companys financial documents.
In reply, Nieto Jr. averred that he will refer the matter to the Executive Committee
or PHC Board in view of the several pending cases involving the two factions;
Africa wrote back, asserting that the PHICOMSAT Board was elected on
September 22, 2005, in accordance with the final decision of the Supreme Court
in G.R. Nos. G.R. Nos. 141796 and 141804, thus there is question anymore on
its legitimacy. On the day of the scheduled inspection, Africas group was
disallowed by Philip Brodett Jr., which prompted PHILCOMSAT to inquire if the
conduct of Brodett was with the knowledge and authority of the PHC Board of
Directors. No reply to this query was received, hence Victor Africa, in his
capacity as president/CEO of PHILCOMSAT and as a stockholder, filed with the
RTC a Complaint for Inspection of Books against the incumbent PHC Board to
enforce its rights under Sections 74 and 75 of the Corporation Code of the
Philippines. By order dated June 21, 2007, the RTC dismissed the case for lack
of jurisdiction; citing the cases of Del Moral v. Republic of the Philippines1 and
Olaguer v. RTC, National Capital Judicial Region, Br. 48, Manila2, the RTC
held that it is the Sandiganbayan, not the RTC, which has jurisdiction since
plaintiff is a sequestered corporation of the Republic through the PCGG alleging
a right of inspection over PHC but which right or authority was being raised as a
defense by the defendants.

PHILCOMSAT appealed the ruling to the Court of Appeals via a petition for
review under Rule 43. While respondents argue that it is the RTC which has
jurisdiction over the inspection of corporate books by a stockholder, the
petitioners in this case argue otherwise. They maintain that the main issue is
who are the rightful representatives of PHILCOMSAT, whose right of inspection
hinges on the ongoing power struggle within the PHILCOMSAT, especially on
who between the Africa-Bildner and the PCGG-Nieto group is the legitimate
board of directors. Since both POTC and PHICOMSAT were both under
sequestration by the PCGG, all issues and controversies arising therefrom or
related or incidental thereof fall under the exclusive and original jurisdiction of the
Sandiganbayan. They also argue that the case must be dismissed due to lit is
penedencia.

The CA ruled in favour of respondent, and reversed and set aside the RTC order.
Hence, the PCGG-Nieto group, composed of petitioners appealed to the
Supreme Court.
395

The Issue:

(1) Whether it is the Sandiganbayan or RTC which has jurisdiction over a


stockholders suit to enforce its right of inspection under Section 74 of the
Corporation Code; and

(2) Whether the complaint failed to state a cause of action considering that
PHILCOMSAT never authorized Africa or any other person to file the said
complaint.

The Ruling:

The petition has no merit.

Both issues presented in this case pertaining to the jurisdiction of the RTC in
intra-corporate disputes within the sequestered corporations of PCGG, and who
between the contending groups held the controlling interest in POTC, and
consequently in PHILCOMSAT and PHC, have already been resolved in the
consolidated petitions docketed as G.R. No. 184622 (Philippine Overseas
Telecommunications Corp. [POTC] and Philippine Communications Satellite
Corporation [PHILCOMSAT] v. Victor Africa, et al.), G.R. Nos. 184712-14 (POTC
and PHILCOMSAT v. Hon. Jenny Lin Aldecoa-Delorino, Pairing Judge of RTC
Makati City, Br. 138, et al.), G.R. No. 186066 (Philcomsat Holdings Corp.,
represented by Concepcion Poblador v. PHILCOMSAT, represented by Victor
Africa), and G.R. No. 186590 (Philcomsat Holdings Corp., represented by
Erlinda I. Bildner v. Philcomsat Holdings Corp., represented by Enrique L.
Locsin).3

On the first issue, we ruled that it is the RTC and not the Sandiganbayan which
has jurisdiction over cases which do not involve a sequestration-related incident
but an intra-corporate controversy.

Originally, Section 5 of Presidential Decree (P.D.) No. 902-A vested the original
and exclusive jurisdiction over cases involving the following in the SEC, to wit:

xxxx

396

(a) Devices or schemes employed by, or any acts of the board of directors,
business associates, its officers or partners, amounting to fraud and
misrepresentation which may be detrimental to the interest of the public and/or of
the stockholder, partners, members of associations or organization registered
with the Commission;

(b) Controversies arising out of intra-corporate or partnership relations, between


and among stockholders, members or associates; between any or all of them
and the corporation, partnership or association of which they are stockholders,
members or associates, respectively; and between such corporation, partnership
or association and the State insofar as it concerns their individual franchise or
right as such entity;

(c) Controversies in the election or appointment of directors, trustees, officers or


managers of such corporations, partnership or associations;

(d) Petitions of corporations, partnerships or associations to be declared in the


state of suspension of payment in cases where the corporation, partnership or
association possesses sufficient property to cover all its debts but foresees the
impossibility of meeting them when they respective fall due or in cases where the
corporation, partnership or association has no sufficient assets to cover its
liabilities but is under the management of a Rehabilitation Receiver or
Management Committee created pursuant to this Decree.

Upon the enactment of Republic Act No. 8799 (The Securities Regulation Code),
effective on August 8, 2000, the jurisdiction of the SEC over intra-corporate
controversies and the other cases enumerated in Section 5 of P.D. No. 902-A
was transferred to the Regional Trial Court pursuant to Section 5.2 of the law,
which provides:

5.2. The Commissions jurisdiction over all cases enumerated in Section 5 of


Presidential Decree No. 902-A is hereby transferred to the Courts of general
jurisdiction or the appropriate Regional Trial Court; Provided, That the Supreme
Court in the exercise of its authority may designate the Regional Trial Court
branches that shall exercise jurisdiction over these cases. The Commission shall
retain jurisdiction over pending cases involving intra-corporate disputes
submitted for final resolution which should be resolved within one (1) year from
the enactment of this Code. The Commission shall retain jurisdiction over
pending suspension of payments/rehabilitation cases filed as of 30 June 2000
until finally disposed.

397

To implement Republic Act No. 8799, the Court promulgated its resolution of
November 21, 2000 in A.M. No. 00-11-03-SC designating certain branches of the
RTC to try and decide the cases enumerated in Section 5 of P.D. No. 902-A.
Among the RTCs designated as special commercial courts was the RTC (Branch
138) in Makati City, the trial court for Civil Case No. 04-1049.

On March 13, 2001, the Court adopted and approved the Interim Rules of
Procedure for Intra-Corporate Controversies under Republic Act No. 8799 in
A.M. No. 01-2-04-SC, effective on April 1, 2001, whose Section 1 and Section 2,
Rule 6 state:

Section 1. Cases covered. The provisions of this rule shall apply to election
contests in stock and non-stock corporations.

Section 2. Definition. An election contest refers to any controversy or dispute


involving title or claim to any elective office in a stock or non-stock corporation,
the validation of proxies, the manner and validity of elections, and the
qualifications of candidates, including the proclamation of winners, to the office of
director, trustee or other officer directly elected by the stockholders in a close
corporation or by members of a non-stock corporation where the articles of
incorporation or by-laws so provide. (bold underscoring supplied)

Conformably with Republic Act No. 8799, and with the ensuing resolutions of the
Court on the implementation of the transfer of jurisdiction to the Regional Trial
Court, the RTC (Branch 138) in Makati had the authority to hear and decide the
election contest between the parties herein. There should be no disagreement
that jurisdiction over the subject matter of an action, being conferred by law,
could neither be altered nor conveniently set aside by the courts and the parties.

To buttress its position, however, the Nieto-Locsin Group relied on Section 2 of


Executive Order No. 14, which expressly mandated that the PCGG shall file all
such cases, whether civil or criminal, with the Sandiganbayan, which shall have
exclusive and original jurisdiction thereof.

The reliance was unwarranted.

Section 2 of Executive Order No. 14 had no application herein simply because


the subject matter involved was an intra-corporate controversy, not any incidents
arising from, incidental to, or related to any case involving assets whose nature
as ill-gotten wealth was yet to be determined. In San Miguel Corporation v. Kahn,
the Court held that:
398

The subject matter of his complaint in the SEC does not therefore fall within the
ambit of this Courts Resolution of August 10, 1988 on the cases just mentioned,
to the effect that, citing PCGG v. Pena, et al., all cases of the Commission
regarding the funds, moneys, assets, and properties illegally acquired or
misappropriated by former President Ferdinand Marcos, Mrs. Imelda Romualdez
Marcos, their close relatives, Subordinates, Business Associates, Dummies,
Agents, or Nominees, whether civil or criminal, are lodged within the exclusive
and original jurisdiction of the Sandiganbayan, and all incidents arising from,
incidental to, or related to, such cases necessarily fall likewise under the
Sandiganbayans exclusive and original jurisdiction, subject to review on
certiorari exclusively by the Supreme Court. His complaint does not involve any
property illegally acquired or misappropriated by Marcos, et al., or any incidents
arising from, incidental to, or related to any case involving such property, but
assets indisputably belonging to San Miguel Corporation which were, in his (de
los Angeles) view, being illicitly committed by a majority of its board of directors
to answer for loans assumed by a sister corporation, Neptunia Co., Ltd.

De los Angeles complaint, in fine, is confined to the issue of the validity of the
assumption by the corporation of the indebtedness of Neptunia Co., Ltd.,
allegedly for the benefit of certain of its officers and stockholders, an issue
evidently distinct from, and not even remotely requiring inquiry into the matter of
whether or not the 33,133,266 SMC shares sequestered by the PCGG belong to
Marcos and his cronies or dummies (on which, issue, as already pointed out, de
los Angeles, in common with the PCGG, had in fact espoused the affirmative).
De los Angeles dispute, as stockholder and director of SMC, with other SMC
directors, an intra-corporate one, to be sure, is of no concern to the
Sandiganbayan, having no relevance whatever to the ownership of the
sequestered stock. The contention, therefore, that in view of this Courts ruling as
regards the sequestered SMC stock above adverted to, the SEC has no
jurisdiction over the de los Angeles complaint, cannot be sustained and must be
rejected. The dispute concerns acts of the board of directors claimed to amount
to fraud and misrepresentation which may be detrimental to the interest of the
stockholders, or is one arising out of intra-corporate relations between and
among stockholders, or between any or all of them and the corporation of which
they are stockholders.

Moreover, the jurisdiction of the Sandiganbayan has been held not to extend
even to a case involving a sequestered company notwithstanding that the
majority of the members of the board of directors were PCGG nominees. The
Court marked this distinction clearly in Holiday Inn (Phils.), Inc. v.
Sandiganbayan, holding thusly:

The subject-matter of petitioners proposed complaint-in-intervention involves


basically, an interpretation of contract, i.e., whether or not the right of first refusal
399

could and/or should have been observed, based on the Addendum/Agreement of


July 14, 1988, which extended the terms and conditions of the original agreement
of January 1, 1976. The question of whether or not the sequestered property was
lawfully acquired by Roberto S. Benedicto has no bearing on the legality of the
termination of the management contract by NRHDCs Board of Directors. The
two are independent and unrelated issues and resolution of either may proceed
independently of each other. Upholding the legality of Benedictos acquisition of
the sequestered property is not a guarantee that HIPs management contract
would be upheld, for only the Board of Directors of NRHDC is qualified to make
such a determination.

Likewise, the Sandiganbayan correctly denied jurisdiction over the proposed


complaint-in-intervention. The original and exclusive jurisdiction given to the
Sandiganbayan over PCGG cases pertains to (a) cases filed by the PCGG,
pursuant to the exercise of its powers under Executive Order Nos. 1, 2 and 14,
as amended by the Office of the President, and Article XVIII, Section 26 of the
Constitution, i.e., where the principal cause of action is the recovery of ill-gotten
wealth, as well as all incidents arising from, incidental to, or related to such cases
and (b) cases filed by those who wish to question or challenge the commissions
acts or orders in such cases.

Evidently, petitioners proposed complaint-in-intervention is an ordinary civil case


that does not pertain to the Sandiganbayan. As the Solicitor General stated, the
complaint is not directed against PCGG as an entity, but against a private
corporation, in which case it is not per se, a PCGG case.

In the cases now before the Court, what are sought to be determined are the
propriety of the election of a party as a Director, and his authority to act in that
capacity. Such issues should be exclusively determined only by the RTC
pursuant to the pertinent law on jurisdiction because they did not concern the
recovery of ill-gotten wealth.4 (Emphasis supplied)

In the case at bar, the complaint concerns PHILCOMSATs demand to exercise


its right of inspection as stockholder of PHC but which petitioners refused on the
ground of the ongoing power struggle within POTC and PHILCOMSAT that
supposedly prevents PHC from recognizing PHILCOMSATs representative
(Africa) as possessing such right or authority from the legitimate directors and
officers. Clearly, the controversy is intra-corporate in nature as they arose out of
intra-corporate relations between and among stockholders, and between
stockholders and the corporation.

400

As to the issue of whether the complaint should be dismissed for failure to state a
cause of action since PHILCOMSAT never authorized Africa to file it, we rule in
the negative.

A complaint should not be dismissed for insufficiency of cause of action if it


appears clearly from the complaint and its attachments that the plaintiff is entitled
to relief. Conversely, a complaint may be dismissed for lack of cause of action if
it is obvious from the complaint and its annexes that the plaintiff is not entitled to
any relief.5 Here, attached to the complaint is the Board Secretarys
Certificate6 stating, among others, that PHILCOMSAT board of directors had
authorized its President to exercise the right of inspection in its subsidiary PHC,
and to file a case in court in case of refusal by PHC.

Petitioners insist that the board meeting held on September 22, 2005 where the
aforesaid resolution was approved, is void for want of a quorum as the majority
of the legitimate directors of PHILCOMSAT were not present at and notified of
the meeting. This clearly alludes to the Nieto-PCGG groups non-recognition of
the election of the board of directors of POTC and PHILCOMSAT conducted by
the Africa-Bildner group.

Issue:
Whether Africa-Bildner is a legitimate group as to the controlling interest in
PHILCOMSAT?

Ruling:
Correctly concluding that the Nieto-PCGG Group, it did not have the majority
control of POTC, could not have validly convened and held the stockholders
meeting and election of POTC officers on August 5, 2004 during which Nieto, Jr.
and PCGG representative Guy De Leon were respectively elected as President
and Chairman; and that there could not be a valid authority for Nieto, Jr. and/or
Locsin to vote the proxies of the group in the PHILCOMSAT meeting.

For the same reason, the POTC proxies used by Nieto, Jr. and Locsin to elect
themselves respectively as Chairman and President of PHILCOMSAT; and the
PHILCOMSAT proxies used by Nieto, Jr. and Locsin in the August 31, 2004 PHC
elections to elect themselves respectively as President and Acting Chairman of
PHC, were all invalid for not having the support of the majority shareholders of
said corporations.

401

Republic vs. Estate of Hans Menzi 476 SCRA (2005)


Ponente: Justice Tinga

FACTS:

In 1957, Menzi purchased the entire interest in Bulletin from its founder and
owner, Mr. Carson Taylor. In 1961, Yap, owner of US Automotive, purchased
Bulletin shares from Menzi and became one of the corporations major
stockholders.

Several years later, on June 5, 1984, Atty. Amorsolo V. Mendoza (Atty.


Mendoza), Vice President of US Automotive, executed a promissory note with his
personal guarantee in favor of Menzi, promising to pay the latter the sum of
P21,304,921.16 with interest at 18% per annum as consideration for Menzis sale
of his 154 block on or before December 31, 1984.

Accordingly, on May 15, 1985, Atty. Montecillo received from US Automotive two
(2) checks in the amounts of P21,304,778.24 andP3,664,421.85 in full payment
of the agreed purchase price and interest for the sale of the 154 block. On the
same day, Atty. Montecillo signed a company voucher acknowledging receipt of
the payment for the shares, indicating on the dorsal portion thereof the certificate
numbers of the 12 stock certificates covering the 154 block, the number of
shares covered by each certificate and the date of issuance thereof.

Atty. Montecillo also wrote on the lower portion of the promissory note executed
by Atty. Mendoza the words Paid May 15, 1985 (signed) M.G. Montecillo,
Executor of the Estate of Hans M. Menzi.

In 1957, Menzi purchased the entire interest in Bulletin from its founder and
owner, Mr. Carson Taylor. In 1961, Yap, owner of US Automotive, purchased
Bulletin shares from Menzi and became one of the corporations major
stockholders.

Several years later, on June 5, 1984, Atty. Amorsolo V. Mendoza (Atty.


Mendoza), Vice President of US Automotive, executed a promissory note with his
personal guarantee in favor of Menzi, promising to pay the latter the sum of
402

P21,304,921.16 with interest at 18% per annum as consideration for Menzis sale
of his 154 block on or before December 31, 1984.

Accordingly, on May 15, 1985, Atty. Montecillo received from US Automotive two
(2) checks in the amounts of P21,304,778.24 andP3,664,421.85 in full payment
of the agreed purchase price and interest for the sale of the 154 block. On the
same day, Atty. Montecillo signed a company voucher acknowledging receipt of
the payment for the shares, indicating on the dorsal portion thereof the certificate
numbers of the 12 stock certificates covering the 154 block, the number of
shares covered by each certificate and the date of issuance thereof.

Atty. Montecillo also wrote on the lower portion of the promissory note executed
by Atty. Mendoza the words Paid May 15, 1985 (signed) M.G. Montecillo,
Executor of the Estate of Hans M. Menzi.

ISSUE:

Whether the sale and transfer of Bulletin shares of stocks from Menzi to Yap is
valid and legal.

RULING:

Yes. The Corporation Code acknowledges that the delivery of a duly indorsed
stock certificate is sufficient to transfer ownership of shares of stock in stock
corporations. Such mode of transfer is valid between the parties. In order to
bind third persons, however, the transfer must be recorded in the books of the
corporation.
Clearly then, the absence of a deed of assignment is not a fatal flaw which
renders the transfer invalid as the Republic posits. In fact, as has been held
in Rural Bank of Lipa City, Inc. v. Court of Appeals, the execution of a deed of
sale does not necessarily make the transfer effective.

403

Nautica Canning Corporation v. Yumul 473 SCRA 415 (2005)


Ynares-Santiago, J.
Facts:
Nautica Canning Corporation (Nautica) was organized and incorporated on May
11, 1994 with an authorized capital stock of P40,000,000 divided into 400,000
shares with a par value of P100.00 per share. It had a subscribed capital stock of
P10,000,000 with paid-in subscriptions from its incorporators as follows:
Name

No. of Shares

Amount Subscribed

Amount Paid

ALVIN Y. DEE

89,991

P8,999,100

JONATHAN Y. DEE

200

200

JOANNA D. LAUREL

200

200

GONZALES

200

200

JENNIFER Y. DEE

200

200

ROBERTO C. YUMUL

100

100

JERRY ANGPING

10,000

1,000,000

P4,499,100

DARLENE EDSA MARIE

500,000
--------------

--------------------

----------

100,000

P10,000,000

---------

P5,000,000

Sometime in 1994, respondent Roberto C. Yumul was appointed Chief Operating


Officer/General Manager of Nautica with a monthly compensation of P85,000
and an additional compensation equal to 5% of the companys operating profit for
the calendar year. On the same date, First Dominion Prime Holdings, Inc.,
Nauticas parent company, through its Chairman Alvin Y. Dee, granted Yumul
an Option to Purchase up to 15% of the total stocks it subscribed from Nautica.
404

On June 22, 1995, a Deed of Trust and Assignment was executed


between First Dominion Prime Holdings, Inc. and Yumul whereby the former
assigned 14,999 of its subscribed shares in Nautica to the latter. The deed stated
that the 14,999 shares were acquired and paid for in the name of the ASSIGNOR
only for convenience, but actually executed in behalf of and in trust for the
ASSIGNEE.
In March 1996, Nautica declared a cash dividend, some of which was paid
to Yumul representing his 15% share.
After Yumuls resignation from Nautica on August 5, 1996, he wrote a letter
to Dee requesting the latter to formalize his offer to buy Yumuls 15% share in
Nautica on or before August 20, 1996; and demanding the issuance of the
corresponding certificate of shares in his name should Dee refuse to buy the
same. Dee, through Atty. Fernando R. Arguelles, Jr., Nauticas corporate
secretary, denied the request claiming that Yumul was not a stockholder of
Nautica.
September 6, 1996 and September 9, 1996, Yumul requested that
the Deed of Trust and Assignment be recorded in the Stock and Transfer Book of
Nautica, and that he, as a stockholder, be allowed to inspect its books and
records.
Yumuls requests were denied allegedly because he neither exercised the
option to purchase the shares nor paid for the acquisition price of the 14,999
shares. Atty. Arguelles maintained that the cash dividend received by Yumul is
held by him only in trust for First Dominion Prime Holdings, Inc.
The SEC en Banc found Yumul to be a stockholder of Nautica, of one
share of stock recorded in Yumuls name, although allegedly held in trust for Dee.
On appeal, the Court of Appeals affirmed the decision of the SEC en Banc.
Hence, this petition.

Issue:
Whether Yumul is a stockholder of Nautica Canning Corporation.

Ruling:

405

It is possible for a business to be wholly owned by one individual. The validity of


its incorporation is not affected when such individual gives nominal ownership of
only one share of stock to each of the other four incorporators. This is not
necessarily illegal. But, this is valid only between or among the incorporators
privy to the agreement. It does bind the corporation which, at the time the
agreement is made, was non-existent. Thus, incorporators continue to be
stockholders of a corporation unless, subsequent to the incorporation, they have
validly transferred their subscriptions to the real parties in interest. As between
the corporation on the one hand, and its shareholders and third persons on the
other, the corporation looks only to its books for the purpose of determining who
its shareholders are.
Nauticas Articles of Incorporation and By-laws, as well as the General
Information Sheet filed with the SEC indicated that Yumul was an incorporator
and subscriber of one share. Even granting that there was an agreement
between Yumul and Dee whereby the former is holding the share in trust for Dee,
the same is binding only as between them. From the corporations vantage point,
Yumul is its stockholder with one share, considering that there is no showing that
Yumul transferred his subscription to Dee, the alleged real owner of the share,
after Nauticas incorporation.

406

Pacific Basin Securities v. Oriental Petroleum 531 SCRA 667 (2007)


FACTS
On May 31, 1991, Pacific Basin Securities, Inc. (Pacific Basin), through the
stock brokerage firm First Resources Management and Securities Corporation
(FRMSC), purchased 308,300,000 Class A shares of Oriental Petroleum and
Minerals Corporation (OPMC). Pacific Basin fully paid for the OPMC shares in
the total amount of P17,727,000.00 or P.05750 per share.[5] The shares were
listed and traded in the Makati Stock Exchange.
The OPMC shares turned out to be owned by Piedras Petroleum Mining
Corporation (Piedras Petroleum), a sequestered company controlled by the
nominees of the Presidential Commission on Good Government (PCGG). PCGG
sent a letter dated June 10, 1991 to Equitable Banking Corporation (EBC),
OPMCs stock and transfer agent, confirming PiedrasPetroleums sale of the
OPMC shares in favor of Pacific Basin through FRMSC. In the same letter,
PCGG requested EBC to record the acquisition of said shares and to issue the
corresponding certificates of stock in favor of Pacific Basin
The requests were left unheeded. EBC informed FRMSC that it cannot
effect the transfer of the OPMC s hares to Pacific Basin on the following grounds:
first, that the endorser of the stock certificate, a certain Mr. Clemente Madarang,
was not among the authorized signatories of Piedras Petroleum; and second,
there was no board resolution from Piedras Petroleum which authorized the sale
of the OPMC shares
OPMC and EBC also argued that even on the assumption that the
government has a valid and effective title over the subject OPMC shares, the
sale by Piedras Petroleum to Pacific Basin was void as there was no showing
that Piedras Petroleum complied with the legal requirements for the disposition of
government owned assets as embodied in Proclamation No. 50, as amended,
and related rules and regulations on the matter. The non-holding of a public
bidding for the sale of the shares was allegedly a blatant violation of the said law
The Securities and Exchange Commission Hearing Officer ruled in favor of
Pacific Basin. In the Decision[13] dated December 28, 1995, the Hearing Officer
took judicial notice of the Courts January 10, 1993 and January 18, 1994 En
Banc Resolutions which dismissed the petition and denied the Motion for
Reconsideration filed by PCGG in G.R. No. 108368. Thus, the issue of the
Temporary Restraining Order on the Compromise Agreement executed between
PCGG and Mr. Benedicto was rendered moot. The Decision further held that
since the subject shares have been fully paid by Pacific Basin, it is the obligation
407

and a ministerial duty of OPMC and EBC to transfer the shares in the corporate
books and issue certificates of stock in favor of Pacific Basin under Section 63 of
the Corporation Code and Section I of Article I of the amended by-laws of OPMC.
The corporate officers of OPMC were also found to have acted in bad faith when
they refused to transfer the shares to Pacific Basin. Hence, they were ordered to
jointly and severally pay Pacific Basin the following amounts: P20,000,000.00
representing actual damages; P300,000.00 representing exemplary damages;
P300,000.00 representing attorneys fees; and P50,000.00 for the cost and
expenses of the suit

ISSUE:
THE COURT OF APPEALS COMMITTED GRAVE ERROR WHEN IT
RULED THAT RESPONDENTS DID NOT ACT IN BAD FAITH, NOR IN
WANTON,

FRAUDULENT,

RECKLESS

OR

OPPRESSIVE

MANNER.

MOREOVER, THIS CASE AFFECTS THE EXPECTATION OF THE INVESTING


PUBLIC ON THE MARKETABILITY OF THE SHARES LISTED AND TRADED
IN THE STOCK EXCHANGE. AS AN EXAMPLE TO THE PUBLIC GOOD,
RESPONDENTS SHOULD BE ORDERED TO PAY EXEMPLARY DAMAGES

RULING:
The subject OPMC shares do not fall within the ambit of assets, as the
term contemplates properties which are government-owned. To repeat, the
OPMC shares originally owned by Piedras Petroleum, a sequestered corporation
controlled by the nominees of PCGG, remain to be privately owned until such
time when the court declares that the subject shares were acquired through
government funds.

Even on the assumption that the OPMC shares are government assets, the
Court finds that the sale of the subject shares through the stock exchange is valid
and binding, as there is no law which mandates that listed shares which are
owned by the government be sold only through public bidding.

As conceded by both Pacific Basin and OPMC, the subject OPMC shares are
listed and traded in the stock exchange. OPMC is a listed corporation in the
Philippine Stock Exchange (PSE).[27] As a listed corporation, it shall be bound
by the provisions of the Revised Listing Rules of the PSE[28] the objective of
which is to provide a fair, orderly, efficient, and transparent market for the trading
of securities
408

The petition is not meritorious.

409

Valley Golf and Country Club, Inc. vs. Vda. De Caram


585 SCRA 218 (2009)
Ponente: Justice Dante O. Tinga

Facts:

Rep. Fermin Z. Caram, Jr. subscribed and paid in full 1 Golf Share of the
Valley Golf and Country Club, Inc. According to Valley Golf, Fermin stopped
paying his monthly dues and the 5 letters it sent to Caram concerning his
delinquent account were ignored. Hence, Fermins share was sold at public
auction.

Fermin thereafter died and his wife, Rosa, initiated intestate proceedings
before the RTC of Iloilo. Unaware of the pending controversy over the Golf
Share, the Caram family and the RTC included the same as part of the of
Fermins estate. The share was subsequently adjudicated to the wife. It was only
through a letter that the heirs of Fermin learned of the sale of the Golf Share
following their inquiry with Valley Golf about the same. After a series of
correspondence, the Caram heirs were subsequently informed in a letter that
they were entitled to the refund out of the proceeds of the sale of the Golf Share.

Rosa filed an action for reconveyance of the Golf Share before the SEC.
The SEC Hearing Officer rendered a decision in favor of the wife, ordering Valley
Golf to convey ownership of the Golf Share or in the alternative to issue one fully
paid share of stock of Valley Golf of the same class as the Golf Share to the wife.
The SEC en banc and the Court of Appeals affirmed the hearing officers
decision.

Issue:

May a non-stock corporation seize and dispose of the membership share


of a fully-paid member on account of its unpaid debts to the corporation when it is
authorized to do so under the corporate by-laws but not by the Articles of
Incorporation?

Ruling:

410

No. A share can only be deemed delinquent and sold at public auction only
upon the failure of the stockholder to pay the unpaid subscription. Delinquency in
monthly club dues is merely an ordinary debt enforceable by judicial action in a
civil case. A provision creating a lien upon shares of stock for unpaid debts,
liabilities, or assessments of stockholders to the corporation, should be embodied
in the Articles of Incorporation, and not merely in the by-laws. Moreover, the bylaws of Valley Golf should have provided formal notice and hearing procedure
before a members share may be seized and sold.

The Supreme Court also declared the sale as invalid. It found that Valley
Golf acted in bad faith when it sent the final notice to Fermin under the pretense
they believed him to be still alive, when in fact they had very well known that he
had already died. That reason alone, evocative as it is of the absence of
substantial justice in the sale of the Golf Share, is sufficient to nullify the sale and
sustain the rulings of SEC and CA.

411

Republic vs. Sandiganbayan, 199 SCRA 39 (1991)


FACTS: In a Joint Letter-Complaint to the Ombudsman dated January 8,1989,
Messrs. Arnel Blancaflor and Rodolfo Santos, residents of Kalookan City,
charged respondent Macario Asistio, Jr., who is the incumbent Mayor of
Kalookan City, with having violated the Anti-Graft and Corrupt Practices Act (R.A.
3019), specifically Section 8 thereof.
In said Joint Sworn Letter-Complaint, they alleged that during his incumbency as
Kalookan City Mayor in 1981, 1982 and 1983, respondent Asistio acquired
wealth in the amounts of P2,142,637.50, P11,463,734.40 and P3,658,351.00,
respectively, or a total of P17,264,722.90, which he deposited in his personal
account, CA-4670-00136-3, in the Republic Planters Bank, Sangandaan Branch,
Kalookan City. In support of their allegations, they attached the original copies of
the bank deposits and receipts which indicated the various sums deposited within
the three-year period and which had been machine validated from January 5,
1981 thru December 20, 1983. However, in his Sworn Statements of Assets and
Liabilities for the period ending December 31,1982 and December 31, 1984, said
respondent had a total income of only P234,128.68 and P255,324.02,
respectively; and as against its total assets (real and personal properties) of
P2,859,327.94 as of December 31, 1982, he had loans payable in the amount of
P2,425,575.60, and against total assets of P5,143,260.98 as of December 31,
1984, he had loans payable in the amount of P2,660,094.74. The Preliminary
Investigation was conducted by Special Prosecution Officer Margarito P.
Gervacio, Jr., before whom authenticated xerox copies of the original ledger
cards of CA-4670- 00136-3 in the name of respondent Macario Asistio, Jr. were
produced and presented by the officer-in-charge of the Republic Planters Bank,
Sangandaan Branch, Kalookan City.
ISSUE: Whether or not the Sandiganbayan should be the proper court to try the
subject of the case.
RULING: No. A perusal of the law originally creating the Office of the
Ombudsman then (to be known as the Tanodbayan), and the amendatory laws
issued subsequent thereto will show that, at its inception, the Office of the
Ombudsman was already vested with the power to investigate and prosecute
civil and criminal cases before the Sandiganbayan and even the regular courts.
Nonetheless, while we do not discount the authority of the Ombudsman, we
believe and so hold that the exercise of his correlative powers to both investigate
and initiate the proper action for the recovery of ill-gotten and/or unexplained
wealth is restricted only to cases for the recovery of ill-gotten and/or unexplained
wealth which were amassed after February 25, 1986. 18 Prior to said date, the
Ombudsman is without authority to initiate such forfeiture proceedings. We,
however, uphold his authority to investigate cases for the forfeiture or recovery of
such ill-gotten and/or unexplained wealth amassed even before the
aforementioned date, pursuant to his general investigatory power under Section
15(l) of Republic Act No. 6770.
In the case at bar, the alleged unexplained wealth of respondent Macario
Asistio, Jr. was supposed to have been acquired from 1981 to 1983. Verily, the
Ombudsman, like the Special Prosecutor, is without authority to initiate and file
the petition for forfeiture against respondent Asistio.

412

Bitong vs. Court of Appeals, 292 SCRA 503 (1998)


BELLOSILLO, J.:

FACTS
These twin cases originated from a derivative suit filed by petitioner Nora A.
Bitong before the Securities and Exchange Commission allegedly for the benefit
Mr. & Ms. Publishing Co., Inc. and to hold spouses Eugenia D. Apostol and Jose
A. Apostol liable for fraud, misrepresentation, disloyalty, evident bad faith,
conflict of interest and mismanagement in directing the affairs of Mr. & Ms. to the
damage and prejudice of Mr. & Ms. and its stockholders, including petitioner.
Alleging before the SEC that she had been the Treasurer and a Member of the
Board of Directors of Mr. & Ms. from the time it was incorporated 1976 to 1989,
and was the registered owner of shares of stock, as a petitioner complained of
irregularities committed by Eugenia D. Apostol, President and Chairperson of the
Board of Directors, that all other transactions and agreements entered into by Mr.
& Ms. with PDI were not supported by any bond and/or stockholders' resolution,
also there made several cash advances to PDI on various occasions amounting
to P3.276 million, whereby no interest was paid whatsoever.
Apostol spouses, Magsanoc, Nuyda, and Mr. & Ms., on the other hand, refuted
the allegations of petitioner Respondents averred that all the PDI shares owned
by Apostols were acquired through their own private funds and that the loan of
P750,000.00 by PDI from Mr. & Ms. had been fully paid with 20% interest per
annum. Private respondents further argued that petitioner was not the true party
to this case, the real party being JAKA which continued to be the true stockholder
of Mr. & Ms. Hence, petitioner did not have the personality to initiate and
prosecute the derivative suit an such, must be dismissed.
The SEC Hearing Panel rendered decision in favor of petitioner and issued a writ
of preliminary injunction enjoining respondents from disbursing any money
except for the payment of salaries and other similar expenses in the regular
course of business.
Respondents filed a Motion to Amend Pleadings to Conform to Evidence alleging
that the issue of whether petitioner is the real party-in-interest had been tried by
express or implied consent of the parties through the admission of documentary
exhibits presented by private respondents proving that the real party-in-interest
was JAKA, not petitioner Bitong.
Petitioner testified at the trial that she became the registered and beneficial
owner of 997 shares of stock of Mr. & Ms. out of the 4,088 total outstanding
shares after she acquired them from JAKA through a deed of sale executed on
25 July 1983 and recorded in the Stock and Transfer Book of Mr. & Ms. under
Certificate of Shares of Stock No. 008.

413

Private respondents refuted the statement of petitioner that Bitong was a


stockholder stating that Certificate of Stock No. 008 was fraudulent.
The SEC Hearing Panel dismissed the derivative suit filed by petitioner and
dissolved the writ of preliminary injunction barring private respondents from
disposing of their PDI shares and any of Mr. & Ms. Assets which they sold to Mr,
Edgardo Espiritu.
Petitioner Bitong appealed to the SEC En Banc. Which reversed the decision of
the Hearing Panel and, among others, ordered private respondents to account
for, return and deliver to Mr. & Ms. any and all funds and assets that they
disbursed from the coffers of the corporation including shares of stock, profits,
dividends and/or fruits and cease and desist from managing the affairs of Mr. &
Ms. for reasons of fraud, mismanagement, disloyalty and conflict of interest.
Consequently, respondent Apostol spouses, Magsanoc, Nuyda, and Mr. & Ms.
filed a petition for review before respondent Court of Appeals, while respondent
Edgardo B. Espiritu filed a petition for certiorari and prohibition also before
respondent Court of Appeals.
Appellate court rendered a decision reversing the SEC En Banc and held that
from the evidence on record petitioner was not the owner of any share of stock in
Mr. & Ms. and therefore not the real party-in-interest to prosecute the complaint
she had instituted. Petitioner as an agent could not file a derivative suit in behalf
of her principal. For not being the real party-in-interest, petitioner's complaint did
not state a cause of action, a defense which was never waived. Hence, her
petition should have been dismissed.
Petitioner file a motion for reconsideration but was then denied for lack of merit.
Hence this recourse.
ISSUE
Whether Nora Bitong has the legal personality to file the derivative suit against
sposes Apostol, Magsanoc and Nuyda.
HELD
The petition is DENIED.
For a valid transfer of stocks, the requirements are as follows: (a) There must be
delivery of the stock certificate; (b) The certificate must be endorsed by the
owner or his attorney-in-fact or other persons legally authorized to make the
transfer; and, (c) to be valid against third parties, the transfer must be recorded in
the books of the corporation. In the instant case, petitioner has satisfied only the
third requirement. Compliance with the first two requisites has not been clearly
and sufficiently shown.
Hence, JAKA retained its ownership of its Mr. & Ms. shares and not to petitioner.

414

There is no doubt that petitioner was an employee of JAKA as its managing


officer. However, in the absence of a special authority from the board of directors
of JAKA to institute a derivative suit for and in its behalf, petitioner is disqualified
by law to sue in her own name. The power to sue and be sued in any court by a
corporation even as a stockholder is lodged in the board of directors that
exercises its corporate powers and not in the president or officer thereof.
As found by the Hearing Panel and affirmed by respondent Court of Appeals,
there is overwhelming evidence that despite what appears on the certificate of
stock and stock and transfer book, petitioner was not a bona fidestockholder of
Mr. & Ms. at the time the complained acts were committed to qualify her to
institute a stockholder's derivative suit against private respondents.
It is well settled that stockholder's right to institute a derivative suit is not based
on any express provision of The Corporation Code but is impliedly recognized
when the law makes corporate directors or officers liable for damages suffered
by the corporation and its stockholders for violation of their fiduciary duties.
The basis of a stockholder's suit is always one in equity. However, it cannot
prosper without first complying with the legal requisites for its institution. The
most important of these is the bona fide ownership by a stockholder of a stock in
his own right at the time of the transaction complained of which invests him with
standing to institute a derivative action for the benefit of the corporation.

415

Boman Environment Development Corporation vs. Court of Appeals,


167 SCRA 540 (1988)
GRIO-AQUINO, J.:
Facts: Respondent Nilcar Y. Fajilan offered in writing to resign as President and
Member of the Board of Directors of petitioner, Boman Environmental
Development Corporation (BEDECO), and to sell to the company all his shares,
rights, and interests therein for P300,000 plus the transfer to him of the
companys Isuzu pick-up truck which he had been using. At a meeting of the
Board of Directors of BEDECO on June 14, 1984, Fajilans resignation as
president was accepted and new officers were elected, the Board promising to
pay for them on a staggered basis from July 15, 1984 to December 15, 1984.
Boman paid the first two P50,000.00 installments but defaulted in paying the
remaining P200,000.00. Fajilan then sued Boman in the RTC of Makati.

Issue: That a suit brought by a withdrawing stockholder against the corporation


to enforce payment of the balance due on the consideration for the surrender of
his shares of stock and interests in the corporation, involves an intra-corporate
dispute. The resolution of that issue will determine whether the Securities and
Exchange Commission (SEC) or a regular court has jurisdiction over the action

Ruling: This case involves an intra-corporate controversy because the parties


are a stockholder and the corporation. As correctly observed by the trial court,
the perfection of the agreement to sell Fajilans participation and interests in
BEDECO and the execution of the promissory note for payment of the price of
the sale did not remove the dispute from the coverage of Section 5(b) of P.D. No.
902, as amended, for both the said agreement (Annex C) and the promissory
note (Annex D) arose from intra-corporate relations. Indeed, all the signatories of
both documents were stockholders of the corporation at the time of signing the
same. It was an intra-corporate transaction; hence, this suit is an intra-corporate
controversy. Fajilans suit against the corporation to enforce the latters
promissory note or compel the corporation to pay for his shareholdings is
cognizable by the SEC alone which shall determine whether such payment will
not constitute a distribution of corporate assets to a stockholder in preference
over creditors of the corporation. The SEC has exclusive supervision, control and
regulatory jurisdiction to investigate whether the corporation has unrestricted
retained earnings to cover the payment for the shares, and whether the purchase
is for a legitimate corporate purpose as provided in Sections 41 and 122 of the
Corporation Code
416

NOTE: This is a 1988 case, now the RTC has expanded jurisdiction. Some RTCs
are granted special jurisdiction to hear and decide intra-corporate disputes.

417

Edward J. Nell Company vs. Pacific Farms, Inc., 15 SCRA 415 (1965)
Ponente : Justice Concepcion

Facts : March 21, 1958: Pacific Farms Inc. (Pacific) purchased as highest bidder
from a bank auction 1,000 shares of stock of Insular Farms for P285,126.99 and
BOD of Insular as reorganized, then caused its assets, including its leasehold
rights over a public land in Bolinao, Pangasinan, to be sold to Insular for
P10,000.00 and paid for the other assets of Insular Farms. October 9, 1958:
Edward J. Nell Co. (Edward) in Civil Case No. 58579 of the Municipal Court of
Manila against Insular Farms, Inc. (Insular) a judgment for the sum of P1,853.80
unpaid balance for a pump sold with interest plus P125 attorney's fees and
P84.00 as costs. August 14, 1959: A writ of execution, issued after the judgment
had become final returned unsatisfied, stating that Insular Farms had no leviable
property. November 13, 1959: Edward filed the present action against Pacific
upon the theory that Pacific is the alter ego of Insular Farms. CA affirmed
Municipal Court: dismissed the complaint.

Issue : Whether Pacific Farms is an alter ego of Insular Farms.

Ruling : NO. Appeal Affirmed


GR: where one corporation sells or otherwise transfers all of its assets to another
corporation, the latter is not liable for the debts and liabilities of the transferor
EX:
1.
2.

where the purchaser expressly or impliedly agrees to assume such debts.


where the transaction amounts to a consolidation or merger of the
corporations.
3. where the purchasing corporation is merely a continuation of the selling
corporation; - no proof.
4. where the transaction is entered into fraudulently in order to escape liability
for such debts.

418

Associated Bank vs. Court of Appeals,


291 SCRA 511
Facts:
About September 16, 1975 Associated Banking Corporation and Citizens
Bank and Trust Company merged to form just one banking corporation known as
Associated Citizens Bank, the surviving bank. On or about March 10, 1981, the
Associated Citizens Bank changed its corporate name to Associated Bank by
virtue of the Amended Articles of Incorporation. On September 7, 1977, the
defendant executed in favor of Associated Bank a promissory note whereby the
former undertook to pay the latter the sum of P2,50000000 payable on or before
March 6, 1978. As per said promissory note, the defendant agreed to pay interest
at 14% per annum, 3% per annum in the form of liquidated damages,
compounded interests, and attorney's fees, in case of litigation equivalent to 10%
of the amount due. The defendant, to date, still owes plaintiff bank the amount of
P2,250,000.00 exclusive of interest and other charges. Despite repeated
demands the defendant failed to pay the amount due. the defendant was
declared as if in default for failure to appear at the Pre-Trial Conference despite
due notice. A Motion to Lift Order of Default and/or Reconsideration of Order
dated May 22, 1986 was filed by defendant's counsel which was denied by the
Court in [an] order dated September 16, 1986 and the plaintiff was allowed to
present its evidence before the Court ex-parte on October 16, 1986. Based on
the evidence presented by petitioner, the trial court ordered Respondent
Sarmiento to pay the bank his remaining balance plus interests and attorney's
fees.
Issues:
Whether the Court of Appeals also erred in declaring that, since the
promissory note was executed in favor of Citizens Bank and Trust Company two
years after the merger between Associated Banking Corporation and Citizens
Bank and Trust Company, respondent is not liable to petitioner because there is
no privity of contract between respondent and Associated Bank.
Ruling:
The fact that the promissory note was executed after the effectivity date of the
merger does not militate against petitioner. The agreement itself clearly provides
that all contracts irrespective of the date of execution entered into in the name of
CBTC shall be understood as pertaining to the surviving bank, herein petitioner.
Since, in contrast to the earlier aforequoted provision, the latter clause no longer
specifically refers only to contracts existing at the time of the merger, no
distinction should be made. The clause must have been deliberately included in
the agreement in order to protect the interests of the combining banks;
specifically, to avoid giving the merger agreement a farcical interpretation aimed
at evading fulfillment of a due obligation. No other construction can be given to
the unequivocal stipulation. Being clear, plain and free of ambiguity, the provision
must be given its literal meaning and applied without a convoluted interpretation.
The Supreme Court granted the petition.
419

Philippine National Bank vs. Andrada Electric and Engineering Company,


381 SCRA 244 (2002)
FACTS:
PNB acquired the assets of Pampanga Sugar Mills (PASUMIL) which was
foreclosed by DBP. Subsequently, PNB organized the National Sugar
Development Corporation (NASUDECO) to consolidate PNBs controlled sugar
mills. Prior to PNBs acquisition of PASUMIL, the latter contracted the services of
Andrada Electric and Engineering Company (AEEC) for electrical repairs which
PASUMIL partially paid but several accounts were left unpaid. Since PASUMIL
was acquired by PNB, AEEC demanded that PNB pay the remaining balance
which the latter refused. AEEC, then, filed a complaint against PNB which the
trial court rendered judgement in favor of AEEC which was affirmed by CA.
Hence, the petition for review.

ISSUE:

Whether PNB is liable for PASUMILs debt.

RULING:
The Supreme Court (SC) held that while PNB acquired the assets of PASUMIL
there was no merger or consolidation with respect to PASUMIL and PNB
because the procedure prescribed under the Corporation Code was not followed
hence, PASUMILs corporate existence has not been legally extinguished or
terminated. SC expounded that prior to PNBs acquisition of the foreclosed
assets, PASUMIL to AEEC. Clearly, the corporate separateness of PASUMIL
and PNB remained.
Hence, PNBs petition was granted.

420

Babst vs. Court of Appeals, 350 SCRA 341 (2001)


Ynares-Santiago, J.

FACTS
Elizalde Steel Consolidated, Inc. (ELISCON) obtained a loan from
Commercial Bank and Trust Company (CBTC) evidenced by a promissory note
and 3 letters of credit. The promissory was obtained by ELICSON itself, and the
letters of credit were opened for ELISCON using the credit facilities of Pacific
Multi-Commercial Corporation (MULTI). In short, Multi served as guarantor for
Eliscon using the three letters of credit. Subsequently, Antontio Roxas-Chua and
Chester G. Babst executed a continuing suretyship, binding themselves solidarily
liable for any existing debt of Multi to CBTC to the extent of 8,000,000.00 each.
Moreover, CBTC opened another 3 domestic letters of credit for Elicson in favor
of National Steel Corporation, the proceeds of which to be used by Elicson for
purchase of tin black plates from NSC. Elicson defaulted on these three domestic
letters of credit.
The following year, the Bank of Philippine Islands entered into a merger
with CBTC, wherein BPI, being the surviving corporation acquired all the assets
and assumed all the liabilities of CBTC.
Meanwhile, Elicson suffered irreversible losses and became heavily
indebted to DBP. It proposed to transfer all its assets to DBP as payment of its
entire debt, DBP agreed and formally took over the properties of Eliscon
including its debt to BPI. DBP proposed formulas for the settlement of all the
debts of Eliscon but BPI expressly rejected the formula.
Consequently, BPI instituted a case for collection of sum of money against
Eliscon, Multi, and Babst. The trial court rendered a decision ordering all three
defendants to pay BPI the said amount solidarily. Babst and the other two
appealed the said decision, but the Court of Appeals only affirmed the decision of
the trial court. Hence they appealed to the Supreme Court. On their appeal they
argued that BPI has no right to recover from them the amount owed by Eliscon to
CBTC.

421

ISSUE

Whether BPI is entitled to collect the amount owed by ELICSON to CBTC


from Multi and Babst.

RULING

BPI is entitled to recover from the defendants the amount they owe from
CBTC. There is no question that there was a valid merger between BPI and
CBTC. It is settled that in the merger of two existing corporations, one of the
corporations survives and continues the business, while the other is dissolved
and all its rights, properties and liabilities are acquired by the surviving
corporation. Hence, BPI has the right to institute the case

422

Omictin vs. Court of Appeals


512 SCRA 70 (2007)
FACTS OF THE CASE
Petitioner Vincent E. Omictin, Operations Manager Ad Interim of
SaagPhils., Inc., filed a complaint for two counts of estafa with the Office of the
City Prosecutor of Makati against private respondent George I. Lagos. He
alleged that private respondent, despite repeated demands, refused to return the
two company vehicles entrusted to him when he was still the president of
SaagPhils., Inc.

On June 24, 1999, private respondent filed a motion to suspend


proceedings on the basis of a prejudicial question because of a pending petition
with the Securities and Exchange Commission (SEC) involving the same parties.
In the action before the SEC, private respondent averred that Saag (S) Pte. Ltd.
is a foreign corporation organized and existing under the laws of Singapore, and
is fully owned by Saag Corporation (Bhd). On July 1, 1994, he was appointed as
Area Sales Manager in the Philippines by ThiangShiangHiang, Manager of Saag
(S) Pte. Ltd. Pursuant to his appointment, respondent was authorized to organize
a local joint venture corporation to be known as Saag Philippines, Inc. for the
wholesale trade and service of industrial products for oil, gas and power
industries in the Philippines. On September 9, 1994, Saag Philippines, Inc. was
incorporated with Saag (S) Pte. Ltd. as the majority stockholder.
Later, due to intra-corporate disputes, the other board of directors resigned
and divested their shares in Saag Corporation (Bhd), thereby resulting in a
change in the controlling interest in Saag (S) Pte. Ltd. Barely three months after,
or on June 23, 1998, private respondent resigned his post as president of
SaagPhils., Inc. while still retaining his position as a director of the company.
Thus, since private respondent did not give his consent as regards the transfer of
shares made by Gan and Thiang, he made several requests to Nicholas Ng, who
replaced Gan as director, and Janifer Yeo, Executive Director of Saag (S) Pte.
Ltd., to call for a board meeting in order to discuss the following: a)
implementation of the board resolution declaring dividends; b) acquisition of
private respondents shares by Saag (S) Pte. Ltd.; c) dissolution of SaagPhils.,
Inc.; and d) the termination of the JVA.

Ng and Yeo failed to appear, however, in the scheduled board meetings.


Instead, on September 30, 1998 they issued a letter appointing Alex Y. Tan as
President Ad Interim of SaagPhils., Inc. Tan, in turn, appointed petitioner Omictin
as the companys Operations Manager Ad Interim. Citing as a reason the
absence of a board resolution authorizing the continued operations of
SaagPhils., Inc., private respondent retained his possession of the office
equipment of the company in a fiduciary capacity as director of the corporation
pending its dissolution and/or the resolution of the intra-corporate dispute. He
423

likewise changed the locks of the offices of the company allegedly to prevent Tan
and petitioner from seizing company property.

Private respondent stressed that Tans appointment was invalid because it


was in derogation of the company by-laws requiring that the president must be
chosen from among the directors, and elected by the affirmative vote of a
majority of all the members of the board of directors. As Tans appointment did
not have the acquiescence of the board of directors, petitioners appointment by
the former is likewise allegedly invalid. Thus, neither has the power or the
authority to represent or act for SaagPhils., Inc. in any transaction or action
before the SEC or any court of justice.

ISSUE

Whether or not a prejudicial question exists to warrant the suspension of the


criminal proceedings pending the resolution of the intra-corporate controversy
that was originally filed with the SEC.

RULING

Yes. The doctrine of primary jurisdiction may be applied in this case. The
issues raised by petitioner particularly the status of SaagPhils., Inc. vis--vis
Saag (S) Pte. Ltd., as well as the question regarding the supposed authority of
the latter to make a demand on behalf of the company, are proper subjects for
the determination of the tribunal hearing the intra-corporate case which in this
case is the RTC of Mandaluyong, Branch 214. These issues would have been
referred to the expertise of the SEC in accordance with the doctrine of primary
jurisdiction had the case not been transferred to the RTC of Mandaluyong.

While the above doctrine refers specifically to an administrative tribunal,


the Court believes that the circumstances in the instant case do not proscribe the
application of the doctrine, as the role of an administrative tribunal such as the
SEC in determining technical and intricate matters of special competence has
been taken on by specially designated RTCs by virtue of Republic Act No. 8799.
Hence, the RTC of Mandaluyong where the intra-corporate case is pending has
the primary jurisdiction to determine the issues under contention relating to the
status of the domestic corporation, SaagPhils., Inc., vis--vis Saag Pte. Ltd.; and
the authority of petitioner to act on behalf of the domestic corporation, the
determination of which will have a direct bearing on the criminal case. The law
recognizes that, in place of the SEC, the regular courts now have the legal
competence to decide intra-corporate dispute.
424

McLeod v. NLRC 512 SCRA 222 (2007)


CARPIO, J.:

FACTS:
Feb 1995 McLeod filed a complaint for retirement benefits, vacation and
sick leave benefits, non-payment of unused airline tickets, holiday pay,
underpayment of salary and 13th month pay, damages, attorneys fees against
Filsyn, Far Eastern Textile Mills, Inc., Sta. Rosa Textiles, Inc., Patricio Lim and
Eric Hu.
McLeod, an expert in textile manufacturing process, was hired as the
Assistant Spinning Manager of Universal Textiles, Inc. (UTEX), promoted to
Senior Manager. UTEX President Patricio Lim formed Peggy Mills, Inc. with
Filsyn having controlling interest. McLeod was absorbed by Peggy as its VP and
Laguna Plant Manager. McLeod claimed that respondents failed to pay him
vacation and leave credits since Peggy was short of funds; that he was entitled to
the monetary value of 4 round trip business class plane tickets on a ManilaLondon-Manila itinerary; that his monthly salary of 60k was reduced by 9.9k for
39 months.
Filsyn sold Peggy Mills, Inc. to Far Eastern Textile Mills, Inc. Peggy Mills
was renamed as Sta. Rosa Textile with Patricio Lim as Chairman and President.
When McLeod reached the retirement age, he was only given a reduced 13th
month pay of P44,183.63, leaving a balance of P15,816.87.
The owners of Far Eastern Textiles decided for cessation of operations of
Sta. Rosa Textiles. McLeod wrote to Lim requesting his retirement and other
benefits. Respondents offered compromise settlement of 300k which McLeod
rejected.
According to Respondents, Peggy Mills closed operations due to
irreversible losses but the corporation still exists at present. Peggys assets were
acquired by Sta. Rosa Textile Corporation which was established but still
remains non-operational. McLeod was hired as consultant by Sta. Rosa but
resigned. Respondents also allege that Filsyn and Far Eastern Textiles are
separate legal entities and have no employer relationship with McLeod; that Lim
is Sta Rosas President and Board Chairman; that respondent Eric Hu is Sta
Rosas Taiwanese Director; that complainant has no cause of action against
Filsyn, Far Eastern, Sta. Rosa Textile Corporation and Eric Hu; that Sta. Rosa
425

only acquired the assets and not the liabilities of Peggy Mills, Inc.; that Lim was
only impleaded as Board Chairman of Sta. Rosa Textile and not as private
individual; that while McLeod was Vice President and Plant Manager of Peggy
Mills, the union staged a strike up to July 1992 resulting in closure of operations
due to irreversible losses and it was due to McLeods lack of attention and
absence the strike continued. The attendance records of McLeod show that he
was either absent or worked at most two hours a day; the McLeods monthly
salary at Peggy Mills was P50,495.00 and not P60,000.00; that Peggy Mills, does
not have a retirement program; that whatever amount complainant is entitled
should be offset with the counterclaims. McLeod was only hired as a consultant
and not an employee by Sta. Rosa. The attendance records wipes out any
vacation/sick leave accumulated. There is no basis for the claim of business
class airline tickets.]
McLeod alleged that all respondents, one and the same entities, are
solidarily liable. They bear the same address at 12/F B.A. Lepanto Building,
Makati City; that their counsel holds office in the same address; same offices and
key personnel such as Patricio Lim and Eric Hu; [that the veil of corporate fiction
may be pierced if it is used as a shield to perpetuate fraud and confuse legitimate
issues; that he never accepted the change in his position from Vice-President
and Plant Manger to consultant and it is incumbent upon respondents to prove
that he was only a consultant; that he never resigned from his job but applied for
retirement; Eric Hu is a top official of Peggy Mills that the closure of Peggy Mills
cannot be the fault of McLeod also because that the strike was staged on the
issue of CBA negotiations which is not part of the usual duties and
responsibilities as Plant Manager; that complainant is a British national and is
prohibited by law in engaging in union activities; that the alleged attendance was
lifted from the logbook of a security agency and is hearsay evidence; his limited
hours was due to the strike but was on call 24 hours a day as plant manager; the
law itself provides for retirement benefits; that Lim by way of Memorandum
approved vacation and sick leave benefits; that complainant was not made to
sign an acknowledgement that their monthly compensation includes holiday pay
precisely because he is entitled to holiday pay over and above his monthly pay.
Respondents alleged that except for Peggy Mills, the other respondents
are not proper persons in interest due to the lack of employer-employee
relationship. Peggy Mills alleged that it offered complainant his retirement
benefits under RA 7641 but McLeod refused.
426

The Labor Arbiter decided to hold all respondents as jointly and solidarily
liable. On Filsyn, Far Eastern, Sta Rosa, Lim an Hus appeal, the NLRC
reversed. The NLRC held that only Peggy was to pay McLeod. McLeods MR
was dismissed. In resolving the certiorari petition the CA held that Lim is jointly
and solidarily liable with Peggy Mills.
The Court of Appeals ruled that the fact that (1) all respondent
corporations have the same address; (2) all were represented by the same
counsel, Atty. Isidro S. Escano; (3) Atty. Escano holds office at respondent
corporations address; and (4) all respondent corporations have common officers
and key personnel, would not justify the application of the doctrine of piercing the
veil of corporate fiction. Peggy and Filsyn have only two interlocking
incorporators and directors, namely, Patricio and Carlos Palanca, Jr. Patricio
deliberately and maliciously evaded PMIs financial obligation to McLeod. Despite
his approval, Patricio refused and ignored to pay McLeods retirement benefits.
Hence this petition, Mcleods argument pertinent to the topic of mergers is
that after Far Eastern purchased Peggy Mills in January 1993, McLeod
"continued to work at the same plant with the same responsibilities" until 30
November 1993. xxx Far Eastern merely renamed Peggy Mills as Sta Rosa. It
was for this reason that when he reached the retirement age in 1993, he asked
all the respondents for the payment of his benefits.
ISSUE:
WON there was a merger or consolidation of PMI and SRTI.
RULING:
NO THERE WAS NO MERGER OR CONSOLIDATION. There is no
employer-employee relationship between the other corporations except Peggy
Mills. The SC affirmed the CAs decision insofar as Peggys liability but absolved
Patricio Lim.
What took place between PMI and SRTI was dation in payment with lease.
Peggy is indebted to the DBP so the former executed REMs in favor of the latter.
By virtue of an inter-governmental agency arrangement, DBP transferred the
Obligations, including the Assets, to the Asset Privatization Trust ("APT") and the
latter has received payment for the Obligations from Peggy, under the Direct
Debt Buy-Out ("DDBO") program thereby causing APT to completely discharge
and cancel the mortgage in the Assets and to release the titles of the Assets
back to PMI. PMI obtained cash advances totaling to 210M from Sta Rosa to
427

enable Peggy to consummate the DDBO with APT, with SRTC subrogating APT
as PMIs creditor thereby. PMI agreed to transfer all its rights, title and interests
in the Assets by way of a dation in payment to SRTC, provided that simultaneous
with the dation in payment, SRTC shall grant unto PMI the right to lease the
Assets.
As a rule, a corporation that purchases the assets of another will not be
liable for the debts of the selling corporation, provided the former acted in good
faith and paid adequate consideration for such assets, except when any of the
following circumstances is present: (1) where the purchaser expressly or
impliedly agrees to assume the debts, (2) where the transaction amounts to a
consolidation or merger of the corporations, (3) where the purchasing corporation
is merely a continuation of the selling corporation, and (4) where the selling
corporation fraudulently enters into the transaction to escape liability for those
debts. None of the foregoing exceptions is present. The SC was not convinced
that PMI fraudulently transferred these assets to escape its liability for any of its
debts. PMI had already paid its employees, except McLeod, their money claims.
Consolidation is the union of two or more existing corporations to form a
new corporation called the consolidated corporation. It is a combination by
agreement between two or more corporations by which their rights, franchises,
and property are united and become those of a single, new corporation,
composed generally, although not necessarily, of the stockholders of the original
corporations.
, on the other hand, is a union whereby one corporation absorbs one or
more existing corporations, and the absorbing corporation survives and
continues the combined business.
The parties to a merger or consolidation are called constituent
corporations. In consolidation, all the constituents are dissolved and absorbed by
the new consolidated enterprise. In merger, all constituents, except the surviving
corporation, are dissolved. In both cases, however, there is no liquidation of the
assets of the dissolved corporations, and the surviving or consolidated
corporation acquires all their properties, rights and franchises and their
stockholders usually become its stockholders.
The surviving or consolidated corporation assumes automatically the
liabilities of the dissolved corporations, regardless of whether the creditors have
consented or not to such merger or consolidation.27
428

In this case, there is no showing that the subject dation in payment


involved any corporate merger or consolidation. Neither is there any showing of
those indicative factors that SRTI is a mere instrumentality of PMI. SRTI did not
expressly or impliedly agree to assume any of PMIs debts.
Also, McLeod did not present any evidence to show the alleged renaming
of "Peggy Mills, Inc." to "Sta. Rosa Textiles, Inc."
McLeod could have presented evidence to support his allegation of
employer-employee relationship between him and any of Filsyn, SRTI, and
FETMI, but he did not. McLeod claims that "for purposes of determining employer
liability, all private respondents are one and the same employer" because: (1)
they have the same address; (2) they are all engaged in the same business; and
(3) they have interlocking directors and officers.35
The fact that SRTI and PMI shared the same addres can be explained by
the two companies stipulation in their Deed of Dation in Payment with Lease that
"simultaneous with the dation in payment, SRTC shall grant unto PMI the right to
lease the Assets under terms and conditions stated hereunder." Filsyn and
FETMI did not have the same address as that of PMI. That respondent
corporations have interlocking incorporators, directors, and officers is of no
moment. The only interlocking incorporators of PMI and Filsyn were Patricio and
Carlos Palanca, Jr. Patricio was never an officer of FETMI. Respondent Eric Hu
was never an officer of PMI.
In light of the foregoing, and there being no proof of employer-employee
relationship between McLeod and respondent corporations and Eric Hu,
McLeods cause of action is only against his former employer, PMI.

429

Spouses Ramon Nisce vs. Equitable PCI Bank 516 SCRA 231
CALLEJO, SR., J.:

FACTS:

Equitable PCI Bank (Bank) as creditor-mortgagee filed a petition for extrajudicial


foreclosure before the Office of the Clerk of Court as Ex-Officio Sheriff of the
Regional Trial Court (RTC) of Makati City. It sought to foreclose the following real
estate mortgage contracts executed by the spouses Ramon and Natividad Nisce
over two parcels of land covered by Transfer Certificate of Title (TCT) Nos. S83466 and S-83467 of the Registry of Deeds of Rizal and an Amendment to Real
Estate Mortgage dated February 28, 2000. The mortgage contracts were
executed by the spouses Nisce to secure their obligation under Promissory Note
Nos. 1042793 and BD-150369, including a Suretyship Agreement executed by
Natividad. The obligation of the Nisce spouses totaled P34,087,725.76.

Nisce spouses filed before the RTC of Makati City a complaint for nullity of the
Suretyship Agreement, damages and legal compensation with prayer for
injunctive relief against the Bank and the Ex-Officio Sheriff. They alleged the
following: in a letter dated December 7, 2000 they had requested the bank
(through their lawyer-son Atty. Rosanno P. Nisce) to setoff the peso equivalent of
their obligation against their US dollar account with PCI Capital Asia Limited
(Hong Kong), a subsidiary of the Bank, under Certificate Deposit No. 01612 and
Account No. 090-0104 (Passbook No. 83-3041); the Bank accepted their offer
and requested for an estimate of the balance of their account; they complied with
the Banks request and in a letter dated February 11, 2002, informed it that the
estimated balance of their account as of December 1991 (including the 11.875%
per annum interest) was US$51,000.42, and that as of December 2002,
Natividads US dollar deposit with it amounted to at least P9,000,000.00; they
were surprised when they received a letter from the Bank demanding payment of
their loan account, and later a petition for extrajudicial foreclosure.

In its Answer to the complaint, the Bank alleged that the spouses had no
cause of action for legal compensation since PCI Capital was a different
430

corporation with a separate and distinct personality; if at all, offsetting may


occur only with respect to the spouses US$500.00 deposit account in its Paseo
de Roxas branch.

RTC declared that justice would be best served if a writ of preliminary injunction
would be issued to preserve the status quo. The Bank opted not to file a motion
for reconsideration of the order, and instead assailed the trial courts order before
the CA via petition for certiorari under Rule 65 of the Rules of Court. The Bank
alleged that the RTC had acted without or in excess of its jurisdiction. The CA
rendered judgment granting the petition and nullifying the assailed Order of the
RTC; that a petition for certiorari under Rule 65 of the Rules of Court may be filed
despite the failure to file a motion for reconsideration, particularly in instances
where the issue raised is one of law; where the error is patent; the assailed order
is void, or the questions raised are the same as those already ruled upon by the
lower court. The spouses Nisce moved to have the decision reconsidered, but
the appellate court denied the motion. They thus filed the instant petition for
review.

ISSUE:

Whether petitioner can set-off the peso equivalent of their obligation to Equitable
PCI Bank against their US dollar account with PCI Capital Asia Limited (Hong
Kong), a subsidiary of the Bank.

HELD:

No. The fact that a corporation owns all of the stocks of another corporation,
taken alone, is not sufficient to justify their being treated as one entity.
Admittedly, PCI Capital is a subsidiary of respondent Bank. Even then, PCI
Capital [PCI Express Padala (HK) Ltd.] has an independent and separate
juridical personality from that of the respondent Bank, its parent company; hence,
any claim against the subsidiary is not a claim against the parent company and
vice versa. The evidence on record shows that PCIB, which had been merged
431

with Equitable Bank, owns almost all of the stocks of PCI Capital. However, the
fact that a corporation owns all of the stocks of another corporation, taken alone,
is not sufficient to justify their being treated as one entity. If used to perform
legitimate functions, a subsidiarys separate existence shall be respected, and
the liability of the parent corporation, as well as the subsidiary shall be confined
to those arising in their respective business. A corporation has a separate
personality distinct from its stockholders and from other corporations to which it
may be conducted. This separate and distinct personality of a corporation is a
fiction created by law for convenience and to prevent injustice.

Test in determining the application of the instrumentality or alter ego


doctrine. The Court likewise declared in the same case that the test in
determining the application of the instrumentality or alter ego doctrine is as
follows: 1. Control, not mere majority or complete stock control, but complete
dominion, not only of finances but of policy and business practice in respect to
the transaction attacked so that the corporate entity as to this transaction had at
the time no separate mind, will or existence of its own; 2. Such control must have
been used by the defendant to commit fraud or wrong, to perpetuate the violation
of a statutory or other positive legal duty, or dishonest and unjust act in
contravention of plaintiffs legal rights; and 3. The aforesaid control and breach of
duty must proximately cause the injury or unjust loss complaint of.

Absence of any one of these elements prevents piercing the corporate


veil. The Court emphasized that the absence of any one of these elements
prevents piercing the corporate veil. In applying the instrumentality or alter
ego doctrine, the courts are concerned with reality and not form, with how the
corporation operated and the individual defendants relationship to that operation.

432

Litonjua vs. Court of Appeals, 286 SCRA 136 (1998)


Ponente: KAPUNAN, J.:

Facts:

Respondent Wack Wack Golf and Country Club is a non-profit corporation


which offers sports, recreational and social activities to its members. Petitioner
Antonio Litonjua is an Associate Member of said corporation and his son, copetitioner Arnold Litonjua, is a Junior Member thereof.

Wack Wack, pursuant to its bylaws, posted the monthly list of its delinquent
members on January 10, 1985. Antonio, discovering his name on the list, went to
the cashier and contested that his non-payment of dues for November was
because he only receive, according to him, the November bill on January 12,
1985 (November bill should have been settled by the end of December). To
prove his dilemma, he presented a sealed envelope alleging it was the
November bill; this was actually the bill for December. Upon inspection, the bill
for November was, as on accounting records, was received by Litonjuas
employee, Aquino. This was denied by Petitioner as he has no employee
named Aquino. Petitioner was able to convince the Cashier to delete his name
from the delinquent list. Petitioner was then allowed to use the clubs facilities
again.

On February on 1985, before he was able to play golf, Antonio was informed that
he was on Februarys delinquent list and will not be allowed to pay. This led the
General Manager of Wack Wack to suspend him, and his son, for 60 days from
using the clubs facilities. The suspension has been appealed but to no avail.
Petitioner filed a complaint to SEC to nullify his suspension on the respondent
club and for damages. SEC favored the petitioner as the suspension was in
accordance to its by-laws, and on appeal, denied the respondent club. The case
was brought up to SEC en banc but the decision was affirmed. A different tune,
however, was sung by the Court of Appeals when it reversed the decision of SEC
and favored the respondent club.

Hence, this petition.

Issue:
Whether Litonjua was illegally suspended.

433

Ruling:
The suspension was valid. According to Wack-Wacks by-laws, members who
despite being in the delinquent list continued using the facilities of the club shall
go through an automatic 60-day suspension. This suspension was that imposed
on the petitioner. Litonjua argued that his name had been already taken off the
delinquent list when he used the facility. However, as correctly determined by the
Court of Appeals, it was through misrepresentation that Antonio Litonjua was
able to have his name deleted from the list of delinquent members. He insisted
that he did not receive his statement of account for November 1984 on 12
December 1984. This claim turned out to be unsubstantiated.

An indication of Antonio Litonjuas duplicity is the sealed envelope he presented


at the Cashier's office on 13 January 1985, the same day he found out he was on
the January 1985 delinquent list. He told the auxiliary clerks that he received his
account (contained in the sealed envelope) only the day before or on 12 January
1985 and, therefore, since the bill was delivered late he should not be included in
the posted list of delinquent members. With this explanation coupled with the
sealed envelope Antonio Litonjua turned in, he managed to persuade the clerks
to cross out his name from the delinquent list. Later, however, when the sealed
envelope was opened, it was discovered that it contained not the statement of
account for November 1984 but the bill for December 1984.

The Court viewed with skepticism Antonio Litonjua's claim that he was
unaware that the sealed envelope contained his December 1984 bill and that he
simply presumed it to contain his account for November 1984. It must be recalled
that at the accounting office of Wack Wack, he was informed that his November
1984 bill was delivered to his office in Mandaluyong. If he thought that the sealed
envelope contained his November 1984 bill, he could have simply opened said
envelope at the presence of the auxiliary clerks to prove, right there and then, the
veracity of his claims. But, strangely, he did not.

Hence, since it was under false pretenses that Antonio Litonjua managed to have
his name deleted from the list of delinquent members the same has no force and
effect. Consequently, being a delinquent member in the posted list and having
used club facilities while posted as such, the imposition of suspension by
respondents on Antonio Litonjua (and his son Arnold Litonjua) pursuant to Sec.
34 (d) of the club by-laws, was valid and legal.

434

Manuel R. Dulay Enterprises, Inc. vs. Court of appeals,


225 SCRA 678 (1993)
Facts:
Manuel R. Dulay Enterprises, Inc, a domestic corporation obtained various loans
for the construction of its hotel project, Dulay Continental Hotel (now Frederick
Hotel).
Manuel Dulay by virtue of Board Resolution No 18 sold the subject property to
spouses Maria Theresa and Castrense Veloso.

Maria Veloso (buyer), without the knowledge of Manuel Dulay, mortgaged the
subject property to private respondent Manuel A. Torres. #fluffypeaches Upon
the failure of Maria Veloso to pay Torres, the property was sold to Torres in an
extrajudicial foreclosure sale.
Torres filed an action against the corporation, Virgilio Dulay and against the
tenants of the apartment.

RTC ordered the corporation and the tenants to vacate the building.
Petitioners: RTC had acted with GAD when it applied the doctrine of piercing
the veil of corporate entity considering that the sale has no binding effect on
corporation as Board Resolution No. 18 which authorized the sale of the subject
property was resolved without the approval of all the members of the board
of directors and said Board Resolution was prepared by a person not
designated by the corporation to be its secretary.

Issue:
WON the sale to Veloso is valid notwithstanding that it was resolved without
the approval of all the members of the board of directors. (YES)

Ruling:
Section 101 of the Corporation Code of the Philippines provides:
Sec. 101. When board meeting is unnecessary or improperly held. Unless the
by-laws provide otherwise, any action by the directors of a close corporation
without a meeting shall nevertheless be deemed valid if:

Before or after such action is taken, written consent thereto is signed by all
the directors, or
435

All the stockholders have actual or implied knowledge of the action and make
no prompt objection thereto in writing; or

The directors are accustomed to take informal action with the express or
implied acquiese of all the stockholders, or

All the directors have express or implied knowledge of the action in question
and none of them makes prompt objection thereto in writing.
If a directors' meeting is held without call or notice, an action taken therein
within the corporate powers is deemed ratified by a director who failed to
attend, unless he promptly files his written objection with the secretary of the
corporation after having knowledge thereof.

Dulay Inc. is classified as a close corporation and consequently a board


resolution authorizing the sale or mortgage is not necessary to bind the
corporation for the action of its president. At any rate, corporate action
taken at a board meeting without proper call or notice in a close corporation is
deemed ratified by the absent director unless the latter promptly files
his written objection with the secretary of the corporation after having
knowledge of the meeting which, in his case, Virgilio Dulay failed to do.

Although a corporation is an entity which has a personality distinct and


separate from its individual stockholders or members, the veil of corporate
fiction may be pierced when it is used to defeat public convenience
justify wrong, protect fraud or defend crime.

436

San Juan Structural and Steel Fabricators, Inc. vs. Court of Appeals,
296 SCRA 63 (1998)
PANGANIBAN, J.

FACTS:

Petitioner San Juan Structural and Steel Fabricators, Inc. alleges that on
February 14, 1989, it entered through its president, Andres Co, into the disputed
Agreement with Respondent Motorich Sales Corporation, which was in turn
allegedly represented by its treasurer, Nenita Lee Gruenberg. Petitioner insists
that when Gruenberg and Co affixed their signatures on the contract they both
consented to be bound by the terms thereof. Ergo, petitioner contends that the
contract is binding on the two corporations. Gruenberg and Co signed the
Agreement according to which a lot owned by Motorich Sales Corporation was
purportedly sold.

In its answer, defendants-appellees Motorich Sales Corporation and Nenita Lee


Gruenberg interposed as affirmative defense that the President and Chairman of
Motorich did not sign the agreement adverted to in par. 3 of the amended
complaint; that Mrs. Gruenbergs signature on the agreement (ref: par. 3 of
Amended Complaint) is inadequate to bind Motorich. The other signature, that of
Mr. Reynaldo Gruenberg, President and Chairman of Motorich, is required; that
plaintiff knew this from the very beginning as it was presented a copy of the
Transfer of Rights (Annex B of amended complaint) at the time the Agreement
(Annex B of amended complaint) was signed; that plaintiff-appellant itself drafted
the Agreement and insisted that Mrs. Gruenberg accept the P100,000.00 as
earnest money; that granting, without admitting, the enforceability of the
agreement, plaintiff-appellant nonetheless failed to pay in legal tender within the
stipulated period (up to March 2, 1989); that it was the understanding between
Mrs. Gruenberg and plaintiff-appellant that the Transfer of Rights/Deed of
Assignment will be signed only upon receipt of cash payment; thus they agreed
that if the payment be in check, they will meet at a bank designated by plaintiffappellant where they will encash the check and sign the Transfer of Rights/Deed.
However, plaintiff-appellant informed Mrs. Gruenberg of the alleged availability of
the check, by phone, only after banking hours.

437

ISSUE:

Was there a valid contract of sale between petitioner and Motorich?

HELD:

There was none. True, Gruenberg and Co signed on February 14, 1989, the
Agreement according to which a lot owned by Motorich Sales Corporation was
purportedly sold. Such contract, however, cannot bind Motorich, because it never
authorized or ratified such sale.

A corporation is a juridical person separate and distinct from its stockholders or


members. Accordingly, the property of the corporation is not the property of its
stockholders or members and may not be sold by the stockholders or members
without express authorization from the corporations board of directors.[10]
Section 23 of BP 68, otherwise known as the Corporation Code of the
Philippines, provides:
SEC. 23. The Board of Directors or Trustees. -- Unless otherwise provided in this
Code, the corporate powers of all corporations formed under this Code shall be
exercised, all business conducted and all property of such corporations
controlled and held by the board of directors or trustees to be elected from
among the holders of stocks, or where there is no stock, from among the
members of the corporation, who shall hold office for one (1) year and until their
successors are elected and qualified.

Indubitably, a corporation may act only through its board of directors, or, when
authorized either by its bylaws or by its board resolution, through its officers or
agents in the normal course of business. The general principles of agency
govern the relation between the corporation and its officers or agents, subject to
the articles of incorporation, bylaws, or relevant provisions of law. Thus, this
Court has held that a corporate officer or agent may represent and bind the
corporation in transactions with third persons to the extent that the authority to do
so has been conferred upon him, and this includes powers which have been
intentionally conferred, and also such powers as, in the usual course of the
particular business, are incidental to, or may be implied from, the powers
intentionally conferred, powers added by custom and usage, as usually
pertaining to the particular officer or agent, and such apparent powers as the
438

corporation has caused persons dealing with the officer or agent to believe that it
has conferred.

Furthermore, the Court has also recognized the rule that persons dealing with an
assumed agent, whether the assumed agency be a general or special one, are
bound at their peril, if they would hold the principal liable, to ascertain not only
the fact of agency but also the nature and extent of authority, and in case either
is controverted, the burden of proof is upon them to establish it (Harry Keeler v.
Rodriguez, 4 Phil. 19).[13] Unless duly authorized, a treasurer, whose powers
are limited, cannot bind the corporation in a sale of its assets.

In the case at bar, Respondent Motorich categorically denies that it ever


authorized Nenita Gruenberg, its treasurer, to sell the subject parcel of land.[15]
Consequently, petitioner had the burden of proving that Nenita Gruenberg was in
fact authorized to represent and bind Motorich in the transaction. Petitioner failed
to discharge this burden. Its offer of evidence before the trial court contained no
proof of such authority. It has not shown any provision of said respondents
articles of incorporation, bylaws or board resolution to prove that Nenita
Gruenberg possessed such power.

That Nenita Gruenberg is the treasurer of Motorich does not free petitioner from
the responsibility of ascertaining the extent of her authority to represent the
corporation. Petitioner cannot assume that she, by virtue of her position, was
authorized to sell the property of the corporation. Selling is obviously foreign to a
corporate treasurers function, which generally has been described as to receive
and keep the funds of the corporation, and to disburse them in accordance with
the authority given him by the board or the properly authorized officers.

Neither was such real estate sale shown to be a normal business activity of
Motorich. The primary purpose of Motorich is marketing, distribution, export and
import in relation to a general merchandising business.[18] Unmistakably, its
treasurer is not cloaked with actual or apparent authority to buy or sell real
property, an activity which falls way beyond the scope of her general authority.
Articles 1874 and 1878 of the Civil Code of the Philippines provides:

ART. 1874. When a sale of a piece of land or any interest therein is through an
agent, the authority of the latter shall be in writing; otherwise, the sale shall be
void.
439

ART. 1878 Special powers of attorney are necessary in the following case:

xxxxxxxxx

(5) To enter any contract by which the ownership of an immovable is transmitted


or acquired either gratuitously or for a valuable consideration;

x x x x x x x x x.

Petitioner further contends that Respondent Motorich has ratified said contract of
sale because of its acceptance of benefits, as evidenced by the receipt issued by
Respondent Gruenberg. Petitioner is clutching at straws.

As a general rule, the acts of corporate officers within the scope of their authority
are binding on the corporation. But when these officers exceed their authority,
their actions cannot bind the corporation, unless it has ratified such acts or is
estopped from disclaiming them.

In this case, there is a clear absence of proof that Motorich ever authorized
Nenita Gruenberg, or made it appear to any third person that she had the
authority, to sell its land or to receive the earnest money. Neither was there any
proof that Motorich ratified, expressly or impliedly, the contract. Petitioner rests
its argument on the receipt, which, however, does not prove the fact of
ratification. The document is a hand-written one, not a corporate receipt, and it
bears only Nenita Gruenbergs signature. Certainly, this document alone does not
prove that her acts were authorized or ratified by Motorich.

Article 1318 of the Civil Code lists the requisites of a valid and perfected contract:
(1) consent of the contracting parties; (2) object certain which is the subject
matter of the contract;(3) cause of the obligation which is established. As found
by the trial court[21] and affirmed by the Court of Appeals,[22] there is no
evidence that Gruenberg was authorized to enter into the contract of sale, or that
the said contract was ratified by Motorich. This factual finding of the two courts is
binding on this Court.[23] As the consent of the seller was not obtained, no
contract to bind the obligor was perfected. Therefore, there can be no valid
contract of sale between petitioner and Motorich.
440

Because Motorich had never given a written authorization to Respondent


Gruenberg to sell its parcel of land, we hold that the February 14, 1989
Agreement entered into by the latter with petitioner is void under Article 1874 of
the Civil Code. Being inexistent and void from the beginning, said contract cannot
be ratified.

441

Mambulao Lumber Company vs. Philippine National Bank, 22 SCRA


989 (1968) National Abaca & Other Fibers Corporation vs. Pore, 2
SCRA 989 (1961) (See however, Gelano and Reburiano cases, infra)
FACTS:

The plaintiff applied for an industrial loan with interest with the PNB. To secure
the payment of the loan, the plaintiff mortgaged to defendant PNB a parcel of
land, together with the buildings and improvements existing thereon as well as
various sawmill equipment, rolling unit and other fixed assets of the plaintiff.
However, the plaintiff failed to pay the amortizations on the amounts released to
and received by it.

Repeated demands were made upon the plaintiff to pay its obligation but it failed
or otherwise refused to do so.

Upon inspection and verification made by

employees of the PNB, it was found that the plaintiff had already stopped
operation about the end of 1957 or early part of 1958. Thus, PNB requested for
the foreclosure of the real estate mortgage as well as the chattel mortgage.

ISSUE:

Whether or not petitioner foreclosure of the mortgage is tenable.

HELD:

NO.

Due to the computation reflecting excess payment made to PNB, it is clear that
there was no further necessity to foreclose the mortgage of herein appellant's
chattels since the obligation has already been paid for. On this ground alone, it
may be declared that the sale of appellant's chattels, illegal and void. The Court
took into consideration the fact that the PNB must have been led to believe that
the stipulated 10% of the unpaid loan for attorney's fees in the real estate
mortgage was legally maintainable, and in accordance with such belief, herein
appellee bank insisted that the proceeds of the sale of appellant's real property
was deficient to liquidate the latter's total indebtedness. Be that as it may,
however, still the subsequent sale of herein appellant's chattels illegal and
objectionable on other grounds. The parties have agreed that in case of
442

foreclosure, the sale should be made elsewhere not necessarily where the
properties are located. This stipulation is allowed under the law which provides
for the general rule. However, the sale was made in the place where the
properties are situated. A clear violation of the agreement of the parties. Thus,
the foreclosure is not tenable.

443

Gelano vs. Court of appeals, 103 SCRA 90 (1981)


FACTS:

The plaintiff applied for an industrial loan with interest with the PNB. To secure
the payment of the loan, the plaintiff mortgaged to defendant PNB a parcel of
land, together with the buildings and improvements existing thereon as well as
various sawmill equipment, rolling unit and other fixed assets of the plaintiff.
However, the plaintiff failed to pay the amortizations on the amounts released to
and received by it.

Repeated demands were made upon the plaintiff to pay its obligation but it failed
or otherwise refused to do so.

Upon inspection and verification made by

employees of the PNB, it was found that the plaintiff had already stopped
operation about the end of 1957 or early part of 1958. Thus, PNB requested for
the foreclosure of the real estate mortgage as well as the chattel mortgage.

ISSUE:

Whether or not petitioner foreclosure of the mortgage is tenable.

HELD:

NO.

Due to the computation reflecting excess payment made to PNB, it is clear that
there was no further necessity to foreclose the mortgage of herein appellant's
chattels since the obligation has already been paid for. On this ground alone, it
may be declared that the sale of appellant's chattels, illegal and void. The Court
took into consideration the fact that the PNB must have been led to believe that
the stipulated 10% of the unpaid loan for attorney's fees in the real estate
mortgage was legally maintainable, and in accordance with such belief, herein
appellee bank insisted that the proceeds of the sale of appellant's real property
was deficient to liquidate the latter's total indebtedness. Be that as it may,
however, still the subsequent sale of herein appellant's chattels illegal and
objectionable on other grounds. The parties have agreed that in case of
foreclosure, the sale should be made elsewhere not necessarily where the
properties are located. This stipulation is allowed under the law which provides
444

for the general rule. However, the sale was made in the place where the
properties are situated. A clear violation of the agreement of the parties. Thus,
the foreclosure is not tenable.

445

Chung Ka Bio vs. Intermediate Appellate Court,


163 SCRA 534 (1988)
CRUZ, J.:
Facts: The Philippine Blooming Mills Company, Inc.s term of 25 years expired
on January 19, 1977. The members of its board of directors executed a deed of
assignment of all of the accounts receivables, properties, obligations and
liabilities of the old PBM in favor of Chung Siong Pek in his capacity as treasurer
of the new PBM, then in the process of reincorporation.
On May 5, 1981, Chung Ka Bio and the other petitioners herein, all stockholders
of the old PBM, filed with the SEC a petition for liquidation alleging that the old
PBM had become legally non-existent for failure to extend its corporate life and
that the new PBM had likewise been ipso facto dissolved for non-use of the
charter and continuous failure to operate within 2 years from incorporation.
After dismissal, the case was reinstated on appeal to SEC en banc and
remanded to a new panel of hearing officers for further proceedings, including
the proper accounting of the assets and liabilities of the old PBM.
In a related development, Alfredo Ching, one of the members of the board of
directors of the old PBM who executed the deed of assignment, filed with the
Intermediate Appellate Court a separate petition for certiorari alleging the action
amounted to a quo warranto proceeding which only the state could institute
through the Solicitor General.
This order was appealed to the Intermediate Appellate Court in a petition for
partial review questioning the authority of the SEC to adjudicate a matter not
properly raised on appeal or resolved in the order appealed from.
Earlier, on April 1, 1982, the new PBM and Alfredo Ching had filed with the SEC
a petition for suspension of payment, which was opposed by Chung Ka Bio, et
al., on the ground that the SEC had no jurisdiction over a petition for suspension
of payments initiated by a mere individual.
The opposition was rejected and the case was set for hearing.
Chung Ka Bio elevated the matter to the SEC en banc on certiorari with
preliminary injunction and receivership, praying for the annulment and setting
aside of the proceedings.
446

Chung Ka Bio came to this Court but we referred his case to the Intermediate
Appellate.
The three cases were then consolidated in the respondent court. The decision
affirmed the orders issued by the SEC in the said cases except the requirement
for the accounting of the assets of the old PBM, which was set aside.
Hence, this present petition.
Issues: a) Does the board of directors of an already dissolved corporation have
the inherent power, without the express consent of the stockholders, to convey
all its assets to a new corporation?
b)Did the new corporation substantially comply with the two-year
requirement of Section 22 of the new Corporation Code on non-user because its
stockholders never adopted a set of by-laws?
c) Does the Securities and Exchange Commission has jurisdiction over a
petition for suspension of payments filed by an individual only?
Ruling: a) On the first contention, the petitioners insist that they have never
given their consent to the creation of the new corporation nor have they indicated
their agreement to transfer their respective stocks in the old PBM to the new
PBM. The creation of the new corporation with the transfer thereto of the assets
of the old corporation was not within the powers of the board of directors of the
latter as it was authorized only to wind up the affairs of such company and not in
any case to continue its business. Moreover, no stockholders' meeting had been
convened to discuss the deed of assignment and the 2/3 vote required by the
Corporation Law to authorize such conveyance had not been obtained.
The pertinent provisions of the Corporation Law, which was the law then in force,
are the following:
SEC. 77. Every corporation whose charter expired by its own
limitation or is annulled by forfeiture or otherwise, or whose
corporate existence for other purposes is terminated in any other
manner, shall nevertheless be continued as a body corporate for
three years after the time when it would have been dissolved, for the
purpose of prosecuting and defending suits by or against it and of
enabling it gradually to settle and close its affairs, to dispose of and
447

convey its property and to divide its capital stock, but not for the
purpose of continuing the business for which it was established."
SEC. 28-1/2. A corporation may, by action taken at any meeting of
its board of directors, sell, lease, exchange, or otherwise dispose of
all or substantially all of its property and assets, including its
goodwill,

upon

such

terms

and

conditions

and

for

such

considerations, which may be money, stocks bonds, or other


instruments for the payment of money or other property or other
considerations, as its board of directors deem expedient, when and
as authorized by the affirmative vote of shareholders holding shares
in the corporation entitling them to exercise at least two-thirds of the
voting power on such a proposal at a shareholders' meeting called
for that purpose. Notice of such meeting shall be given to all of the
shareholders of record of the corporation whether or not they shall
be entitled to vote thereat: Provided, however, That any stockholder
who did not vote to authorize the action of the board of directors,
may, within forty days after the date upon which such action was
authorized, object thereto in writing and demand payment for his
shares. If, after such a demand by a stockholder, the corporation
and the stockholder can not agree upon the value of his share or
shares at the time such corporate action was authorized, such value
shall be ascertained by three disinterested persons, one of whom
shall be named by the stockholder, another by the corporation, and
the third by the two thus chosen. The finding of the appraisers shall
be final and if their award is not paid by the corporation within thirty
days after it is made, it may be recovered in an action by the
stockholder against the corporation. Upon payment by the
corporation to the stockholder of the agreed or awarded price of his
shares, the stockholder shall forthwith transfer and assign the share
or shares held by him as directed by the corporation.
Unless and until such sale, lease, or exchange shall be abandoned,
the stockholder making such demand in writing ceases to be a
stockholder and shall have no rights with respect to such shares
except the right to receive payment therefor as aforesaid.
A stockholder shall not be entitled to payment for his shares under
the provisions of this section unless the value of the corporate
448

assets which would remain after such payment would be at least


equal to the aggregate amount of its debts and liabilities exclusive of
capital stock.
Nothing in this section is intended to restrict the power of any
corporation, without the authorization thereof by the shareholders, to
sell, lease, exchange, or otherwise dispose of, any of its property if
thereby the corporate business be not substantially limited, or if the
proceeds of such property be appropriated to the conduct or
development of its remaining business.
As the first contention is based on the negative averment that no stockholders'
meeting was held and the 2/3 consent vote was not obtained, there is no need
for affirmative proof. Even so, there is the presumption of regularity which must
operate in favor of the private respondents, who insist that the proper
authorization as required by the Corporation Law was duly obtained at a meeting
called for the purpose. (That authorization was embodied in a unanimous
resolution dated March 19, 1977, which was reproduced verbatim in the deed of
assignment.) 11 Otherwise, the new PBM would not have been issued a
certificate of incorporation, which should also be presumed to have been done
regularly. It must also be noted that under Section 28-1/2, "any stockholder who
did not vote to authorize the action of the board of directors may, within forty
days after the date upon which such action was authorized, object thereto in
writing and demand payment for his shares." The record does not show, nor
have the petitioners alleged or proven, that they filed a written objection and
demanded payment of their shares during the reglementary forty-day period. This
circumstance should bolster the private respondents' claim that the authorization
was unanimous.
While we agree that the board of directors is not normally permitted to undertake
any activity outside of the usual liquidation of the business of the dissolved
corporation, there is nothing to prevent the stockholders from conveying their
respective shareholdings toward the creation of a new corporation to continue
the business of the old. Winding up is the sole activity of a dissolved corporation
that does not intend to incorporate anew. If it does, however, it is not unlawful for
the old board of directors to negotiate and transfer the assets of the dissolved
corporation to the new corporation intended to be created as long as the
stockholders have given their consent. This was not prohibited by the
Corporation Act. In fact, it was expressly allowed by Section 28-1/2.
449

What the Court finds especially intriguing in this case is the fact that although the
deed of assignment was executed in 1977, it was only in 1981 that it occurred to
the petitioners to question its validity.
xxxx.
The essential elements of laches are: (1) conduct on the part of the defendant, or
of one under whom he claims, giving rise to the sitution complained of; (2) delay
in asserting complainant's right after he had knowledge of the defendant's
conduct and after he has an opportunity to sue; (3) lack of knowledge or notice
on the part of the defendant that the complainant would assert the right on which
he bases his suit; (4) injury or prejudice to the defendant in the event relief is
accorded to the complainant.
All the requisites are present in the case at bar. Xxxx.
b) The third contention is likewise rejected for, as already shown, it is
undeniable that the new PBM has in fact been operating all these years. Xxxx.
Section 19 of the Corporation Law, part of which is now Section 22 of the
Corporation Code, provided that the powers of the corporation would cease if it
did not formally organize and commence the transaction of its business or the
continuation of its works within two years from date of its incorporation. Section
20, which has been reproduced with some modifications in Section 46 of the
Corporation Code, expressly declared that "every corporation formed under this
Act, must within one month after the filing of the articles of incorporation with the
Securities and Exchange Commission, adopt a code of by-laws." Whether this
provision should be given mandatory or only directory effect remained a
controversial question until it became academic with the adoption of PD 902-A.
Under this decree, it is now clear that the failure to file by-laws within the required
period is only a ground for suspension or revocation of the certificate of
registration of corporations.
xxxx.
It should be stressed in this connection that substantial compliance with
conditions subsequent will suffice to perfect corporate personality. Organization
and commencement of transaction of corporate business are but conditions
subsequent and not prerequisites for acquisition of corporate personality. The
adoption and filing of by-laws is also a condition subsequent. Under Section 19 of
450

the Corporation Code, a corporation commences its corporate existence and


juridical personality and is deemed incorporated from the date the Securities and
Exchange Commission issues certificate of incorporation under its official seal.
This may be done even before the filing of the by-laws, which under Section 46
of the Corporation Code, must be adopted "within one month after receipt of
official notice of the issuance of its certificate of incorporation."
xxxx.
In any case, the deficiency claimed by the petitioners was corrected when the
new PBM adopted and filed its by-laws on September 6, 1981, thus rendering the
third issue also moot and academic.
c) On the fifth and final issue, the respondent court justifies assumption by the
SEC of jurisdiction over the petition for suspension of payment filed by the
individual on the general principle against multiplicity of suits.
Under Section 5(d), PD 902-A, as amended by PD 1758, however, it is clearly
provided that such jurisdiction may be exercised only in:
d) Petitions of corporations, partnerships or associations to be
declared in the state of suspension of payments in cases where the
corporation, partnership or association possess sufficient property to
cover all its debts but foresees the impossibility of meeting them
when they respectively fall due or in cases where the corporation,
partnership or association has no sufficient assets to cover its
liabilities but is under the management of a Rehabilitation Receiver
or Management Committee created pursuant to this Decree.
This section clearly does not allow a mere individual to file the petition which is
limited to "corporations, partnerships or associations." Administrative agencies
like the SEC are tribunals of limited jurisdiction and, as such, can exercise only
those powers which are specifically granted to them by their enabling
statutes. Consequently, where no authority is granted to hear petitions
of individuals for suspension of payments, such petitions are beyond the
competence of the SEC. The analogy offered by the respondent court is clearly
inappropriate for while it is true that the Sandiganbayan may assume jurisdiction
over private individuals, it is because its charter expressly allows this in specified
cases. No similar permission is found in PD 902-A.

451

Clemente vs. Court of Appeals, 242 SCRA 717 (1995). See also Reburiano
vs. Court of Appeals, 301 SCRA 342 (1999)
Facts:
RTC rendered judgment in favor of Pepsi Cola Bottling Co. ordering Reburiano to
pay P55,000 with interest for the unpaid bottles of softdrinks it received from the
company. RTC issued a writ of execution. However, it appears that prior to the
promulgation of the decision of the trial court, private respondent amended its
articles of incorporation to shorten its term of existence to July 8, 1983.

The amended articles of incorporation was approved by the Securities and


Exchange Commission on March 2, 1984. The trial court was not notified of this
fact. Reburiano moved to quash the writ of execution on the grounds that when
the Court of Appeals rendered its decision, the private respondent was no longer
in existence and had no more juridical personality and so, as such, it no longer
had the capacity to sue and be sued; and that after Pepsi lost its existence and
juridical personality, Atty. Romualdo M. Jubay had no more client in this case and
so his appearance in this case was no longer possible and tenable;Private
respondent opposed petitioners' motion. It argued that the jurisdiction of the court
as well as the respective parties capacity to sue had already been established
during the initial stages of the case; and that when the complaint was filed
in1982, private respondent was still an existing corporation so that the mere fact
that it was dissolved at the time the case was yet to be resolved did not warrant
the dismissal of the case or oust the trial court of its jurisdiction.

Issue:
Whether or not Pepsi still had juridical personality to pursue its case against
Reburiano after a shortening of its corporate existence.

RULING:
Yes. Petitioners are in error in contending that "a dissolved and non-existing
corporation could no longer be represented by a lawyer and that a lawyer could
not appear as counsel for a non-existing judicial person.

452

The only reason for their refusal to execute the same is that there is no existing
corporation to which they are indebted. Such argument is untenable. The law
specifically allows a trustee to manage the affairs of the corporation in liquidation.
Consequently, any supervening fact, such as the dissolution of the corporation,
repeal of a law, or any other fact of similar nature would not serve as an effective
bar to the enforcement of such right.

As clearly stated in Section 122 of the Corporation Code:


Section122: Corporate Liquidation. Every Corporation whose charter
expires by its own limitation or is annulled by forfeiture or otherwise, or
whose corporate existence for other purposes is terminated in any other
manner, shall nevertheless be continued as a body corporate for three (3)
years after the time when it would have been so dissolved, for the purpose
of prosecuting and defending suits by or against it and enabling it to settle
and close its affairs, to dispose of and convey its property and to distribute
its assets, but not for the purpose of continuing the business for which it
was established. At any time during said three (3) years, said corporation
is authorized the empowered to convey all of its property to trustees for the
benefit of stockholders, members, creditors, and other persons in interest.
From and after any such conveyance by the corporation of its property in
trust for the benefit of its stockholders, members, creditors and others in
interests, all interests which the corporation had in the property in
terminates, the legal interest vests in the trustees, and the beneficial
interest in the stockholders, members, creditors or other persons in
interest.
Petitioners argue that while private respondent Pepsi Cola Bottling Company of
the Philippines, Inc. undertook a voluntary dissolution on July 3, 1983 and the
process of liquidation for three (3) years thereafter, there is no showing that a
trustee or receiver was ever appointed. They contend that Section 122 of the
Corporation Code does not authorize a corporation, after the three-year
liquidation period, to continue actions instituted by it within said period of three
years.

Here, the change in the status of private respondent took place in 1983, when it
was dissolved, during the pendecy of its case in the trial court. The change
453

occurred prior to the rendition of judgment by the trial court. Rules of fair play,
justice, and due process dictate that parties cannot raise for the first time on
appeal issues which they could have raised but never did during the trial and
even on appeal from the decision of the trial court

454

Philippine National Bank vs. Bitulok Sawmill Inc., SCRA 269


FACTS:
The Philippine Lumber Distributing Agency, Inc., according to the lower court,
"was organized sometime in the early part of 1947 upon the initiative and
insistence of the late President Manuel Roxas of the Republic of the Philippines
who for the purpose, had called several conferences between him and the
subscribers and organizers of the Philippine Lumber Distributing Agency, Inc."
The purpose waspraiseworthy, to insure a steady supply of lumber, which could
be sold at reasonable prices to enable the war sufferers to rehabilitate their
devastated homes. At the beginning, the lumber producers were reluctant to
organize the cooperative agency as they believed that it would not be easy to
eliminate from the retail trade the alien middlemen who had been in this business
from time immemorial, but because the late President Roxas made it clear that
such a cooperative agency would not be successful without a substantial working
capital which the lumber producers could not entirely shoulder, and as an
inducement he promised and agreed to finance the agency by making the
Government invest P9.00 by way of counterpart for every peso that the
members would invest therein."
Accordingly, "the late President Roxas instructed the Hon. Emilio Abello, then
Executive Secretary and Chairman of the Board of Directors of the Philippine
National Bank, for the latter to grant said agency an overdraft in the original sum
of P250,000.00 which was later increased to P350,000.00, which was approved
by said Board of Directors of the Philippine National Bank on July 28, 1947,
payable on or before April 30, 1958, with interest at the rate of 6% per annum,
and secured by the chattel mortgages on the stock of lumber of said agency."
The Philippine Government did not invest the P9.00 for every peso coming from
defendant lumber producers. The loan extended to the Philippine Lumber
Distributing Agency by the Philippine National Bank was not paid.
ISSUE:

Whether or not the non-compliance with a plain statutory command,


considering the persuasiveness of the plea that defendants-appellees would "not
have subscribed to the capital stock" of the Philippine Lumber Distributing
Agency "were it not for the assurance of the then President of the Republic that
the Government would back it up by investing P9.00 for every peso" subscribed,
a condition which was not fulfilled, such commitment not having been complied
with, be justified.
RULING:
NO.
It would be unwarranted to ascribe to the late President Roxas the view that the
payment of the stock subscriptions, as thus required by law, could be condonedin
the event that the counterpart fund to be invested by the Government would not
be available. Even if such were the case, however, and such a promise were in
455

fact made, to further the laudable purpose to which the proposed corporation
would be devoted and the possibility that the lumber producers would lose
money in the process, still the plain and specific wording of the applicable legal
provision as interpreted by this Court must be controlling. It is a well-settled
principle that with all the vast powers lodged in the Executive, he is still devoid of
the prerogative of suspending the operation of any statute or any of its terms.

456

Vesagas vs. Court of Appeals, 371 SCRA 506 (2001)


Puno, J.

Facts:
The respondent spouses Delfino and Helenda Raniel are members in good
standing of the Luz Village Tennis Club, Inc. (club). They alleged that petitioner
Teodoro B. Vesagas, who claims to be the clubs duly elected president, in
conspiracy with petitioner Wilfred D. Asis, who, in turn, claims to be its duly
elected vice-president and legal counsel, summarily stripped them of their lawful
membership, without due process of law.Thereafter, respondent spouses filed a
Complaint with the Securities and Exchange Commission (SEC) on March 26,
1997 against the petitioner to declare as illegal their expulsion from the club as it
was allegedly done in utter disregard of the provisions of its by-laws as well as
the requirements of due process. They likewise sought the annulment of the
amendments to the by-laws made on 1996, changing the annual meeting of the
club from the last Sunday of January to November and increasing the number of
trustees from nine to fifteen. Finally, they prayed for the issuance of a Temporary
Restraining Order and Writ of Preliminary Injunction. The application for TRO
was denied by SEC Hearing Officer Soller.
Before the hearing officer could start proceeding with the case, however,
petitioners filed a motion to dismiss on the ground that the SEC lacks jurisdiction
over the subject matter of the case. The motion was denied.

Issue:
Whether the dispute between the respondents and petitioners is a corporate
matter within the exclusive competence of the SEC to decide.

Ruling:
In order that the commission can take cognizance of a case, the controversy
must pertain to any of the following relationships: a) between the corporation,
457

partnership or association and the public; b) between the corporation, partnership


or association and its stockholders, partners, members, or officers; c) between
the corporation, partnership, or association and the state as far as its franchise,
permit or license to operate is concerned; and d) among the stockholders,
partners or associates themselves. The fact that the parties involved in the
controversy are all stockholders or that the parties involved are the stockholders
and the corporation, does not necessarily place the dispute within the loop of
jurisdiction of the SEC. Jurisdiction should be determined by considering not only
the status or relationship of the parties but also the nature of the question that is
the subject of their controversy.
We rule that the present dispute is intra-corporate in character. In the first
place, the parties here involved are officers and members of the club.
Respondents claim to be members of good standing of the club until they were
purportedly stripped of their membership in illegal fashion. Petitioners, on the
other hand, are its President and Vice-President, respectively. More significantly,
the present conflict relates to, and in fact arose from, this relation between the
parties. The subject of the complaint, namely, the legality of the expulsion from
membership of the respondents and the validity of the amendments in the clubs
by-laws are, furthermore, within the Commissions jurisdiction.

458

Paramount Insurance Corp. vs. A.C. Ordonez Corp.


561 SCRA 327 (2008)
Ponente: Ynares, Santiago, J.;

Facts: Petitioner Paramount Insurance Corp. is the subrogee of Maximo Mata,


the registered owner of a Honda City sedan involved in a vehicular accident with
a truck mixer owned by respondent corporation. On February 22, 2000, petitioner
filed before the Metropolitan Trial Court of Makati City, a complaint for damages
against respondents. Based on the Sheriffs Return of Service, summons
remained unserved on respondent Suspine,5 while it was served on respondent
corporation and received by Samuel D. Marcoleta of its Receiving Section on
April 3, 2000. On May 19, 2000, petitioner filed a Motion to Declare Defendants
in Default; however, on June 28, 2000, respondent corporation filed an Omnibus
Motion (And Opposition to Plaintiffs Motion to Declare Defendant in Default)
alleging that summons was improperly served upon it because it was made to a
secretarial staff who was unfamiliar with court processes; and that the summons
was received by Mr. Armando C. Ordoez, President and General Manager of
respondent corporation only on June 24, 2000. Respondent corporation asked
for an extension of 15 days within which to file an Answer.Pending resolution of
its first motion to declare respondents in default, petitioner filed on June 30, 2000
a Second Motion to Declare Defendants in Default.On July 26, 2000, respondent
corporation filed a Motion to Admit Answer alleging honest mistake and business
reverses that prevented them from hiring a lawyer until July 10, 2000, as well as
justice and equity. On August 25, 2000, the Metropolitan Trial Court of Makati
City, Branch 66, issued an Order admitting the answer and setting the case for
pre-trial. Petitioner moved for reconsideration but it was denied. Thus, it filed a
petition for certiorari and mandamus with prayer for preliminary injunction and
temporary restraining order before the Regional Trial Court of Makati City. On
October 16, 2000, the Regional Trial Court of Makati City, Branch 58 issued a
temporary restraining order, and on May 22, 2001, issued a writ of preliminary
injunction. On September 21, 2005, the Regional Trial Court rendered a
Decision7 granting the petition. Respondent corporation moved for
reconsideration but it was denied
Issue: Whether a party without corporate existence may file an appeal.
Held: There is likewise no merit in petitioners claim that respondent corporation
lacks legal personality to file an appeal. Although the cancellation of a
corporations certificate of registration puts an end to its juridical personality, Sec.
122 of the Corporation Code, however provides that a corporation whose
corporate existence is terminated in any manner continues to be a body
corporate for three years after its dissolution for purposes of prosecuting and
defending suits by and against it and to enable it to settle and close its
affairs.Moreover, the rights of a corporation, which is dissolved pending litigation,
are accorded protection by law pursuant to Sec. 145 of the Corporation Code, to
wit:
Section 145. Amendment or repeal. No right or remedy in favor of or
against any corporation, its stockholders, members, directors, trustees, or
officers, nor any liability incurred by any such corporation, stockholders,
members, directors, trustees, or officers, shall be removed or impaired
either by the subsequent dissolution of said corporation or by any
459

subsequent amendment or repeal of this Code or of any part thereof.


(Emphasis ours)
Dissolution or even the expiration of the three-year liquidation period should not
be a bar to a corporations enforcement of its rights as a corporation.

460

Top-Weld Manufacturing, Inc. vs. Eced, S.A. 138 SCRA 118. See also
Granger Associates vs. Microwave Systems, Inc., 189 SCRA 631
(1990)
Ponente: Justice Hugo E. Gutierrez, Jr.

Facts:

Top-weld Manufacturing, Inc. is a Philippine corporation engaged in the


business of manufacturing and selling welding supplies and equipment. It
entered into separate contracts with two different foreign entities IRTI, S.A., a
Swiss corporation, and ECED, S.A., a Panamanian company. Upon learning that
the two foreign entities were negotiating with another group to replace Top-weld
as their licensee and distributor, the latter instituted a civil case against IRTI,
ECED, an American corporation named EUTECTIC Corporation, and Victor
Gaerlan, a Filipino citizen alleged to be the representative and employee of these
three corporations. Top-weld sought the issuance of a writ of preliminary
injunction to restrain the corporations from negotiating with third persons or from
actually carrying out the transfer of its distributorship and franchising rights. It
also applied for a preliminary mandatory injunction to compel ECED to ship and
deliver various items covered by the distributorship contract, and to prohibit the
corporations from importing into the Philippines directly or indirectly any
EUTECTIC materials, supplies or equipment except to and/or through the
petitioner. Among others, the petitioner invoked the provisions of No. 9, Sec. 4 of
RA 5455 on alien firms doing business in the Philippines. The trial court granted
Top-welds applications for writ of preliminary injunction and writ mandatory
preliminary injunction. The Court of Appeals however set aside the order of the
trial court.

Issue:

Whether respondent corporations can be considered as "doing business"


in the Philippines and, therefore, subject to the provisions of RA 5455

Ruling:

Yes, foreign corporations ECED and IRTI were doing business in the
Philippines. There is no general rule or governing principle laid down as to what
461

constitutes "doing" or engaging in" or "transacting" business in the Philippines.


Each case must be judged in the light of its peculiar circumstances. The acts of
these corporations should be distinguished from a single or isolated business
transaction or occasional, incidental and casual transactions which do not come
within the meaning of the law. Where a single act or transaction, however, is not
merely incidental or casual but indicates the foreign corporation's intention to do
other business in the Philippines, said single act or transaction constitutes
"doing" or "engaging in" or "transacting" business in the Philippines.

The Supreme Court agreed with the Court of Appeals in considering the
respondents as "doing business" in the Philippines. When the respondents
entered into separate contracts with Top-weld, they were carrying out the
purposes for which they were created, i.e. to manufacture and market welding
products and equipment. The terms and conditions of the contracts as well as the
respondents' conduct indicate that they established within our country a
continuous business, and not merely one of a temporary character. This fact is
even more strengthened by the admission of the respondents that they are
negotiating with another group for the transfer of the distributorship and
franchising rights from the petitioner. Respondents' acts enabled them to enter
into the mainstream of our economic life in competition with our local business
interests. This necessarily brings them under the provisions of R.A. No. 5455.

462

Antam Consolidated, Inc. vs. Court of Appeals


143 SCRA 534 (1986)

FACTS: On April 9, 1981, respondent Stokely Van Camp. Inc. (Stokely) filed a
complaint against Banahaw Milling Corporation (Banahaw), Antam Consolidated,
Inc., Tambunting Trading Corporation (Tambunting), Aurora Consolidated
Securities and Investment Corporation, and United Coconut Oil Mills, Inc.
(Unicom) for collection of sum of money. In its complaint, Stokely alleged: 1) That
it is a corporation organized and existing under the laws of the state of Indiana,
U.S.A. and 2)It have undertook contract with Comphil to sell and deliver and
Capital City agreed to buy 500 long tons of crude coconut oil but Comphil failed
to deliver the coconut oil and so that Capital City covered its coconut oil needs in
the open market at a price substantially in excess of the contract and sustained a
loss of US$103,600. Subsequent contracts were undertaken by the parties but
still Comphil failed to deliver its obligation. Capital City sustained damages in the
amount of US$175,00. And that after repeated demands from Comphil to pay the
said amount, the latter still refuses to pay the same. Respondent Stokely further
prayed that a writ of attachment be issued against any and all the properties of
the petitioners in an amount sufficient to satisfy any lien of judgment that the
respondent may obtain in its action. The trial court ordered the issuance of a writ
of attachment in favor of the respondent upon the latter's deposit of a bond in the
amount of P l,285,000.00. Petitioners filed a petition for certiorari before the
Indianapolis intermediate Appellate Court but the same was dismissed.
ISSUE: Whether the respondent has the personality to sue since the respondent
should have secured first the requisite license to do business in the Philippines.
RULING: The Supreme Court held that the transactions entered into by the
respondent with the petitioners are not a series of commercial dealings which
signify an intent on the part of the respondent to do business in the Philippines
but constitute an isolated one which does not fall under the category of "doing
business." The records show that the only reason why the respondent entered
into the second and third transactions with the petitioners was because it wanted
to recover the loss it sustained from the failure of the petitioners to deliver the
crude coconut oil under the first transaction and in order to give the latter a
chance to make good on their obligation. Instead of making an outright demand
on the petitioners, the respondent opted to try to push through with the
transaction to recover the amount of US$103,600.00 it lost. From these facts
alone, it can be deduced that in reality, there was only one agreement between
the petitioners and the respondent and that was the delivery by the former of 500
long tons of crude coconut oil to the latter, who in turn, must pay the
corresponding price for the same. We rule that the defect in the original
verification was cured when Renato Calma subsequently executed an affidavit to
the effect that the allegations he made in support of the prayer for attachment
were verified by him from the records of Comphil and the Securities and
Exchange Commission. Moreover, petitioner had the opportunity to oppose the
issuance of the writ. As to the merit of the attachment order itself, we find that
the allegations in the respondent's complaint satisfactorily justify the issuance of
said order.

463

Converse Rubber Corporation vs. Universal Rubber Products Inc., 147


SCRA 154 (1987)
FERNAN, J.:

FACTS
Universal Rubber Products, Inc. filed an application with the Philippine Patent
office for registration of the trademark "UNIVERSAL CONVERSE AND DEVICE"
used on rubber shoes and rubber slippers.
Converse Rubber Corporation filed its opposition to the application for
registration on grounds that the trademark sought to be registered is confusingly
similar to the word "CONVERSE" which is part of petitioner's corporate name
"CONVERSE RUBBER CORPORATION" as to likely deceive purchasers and
such will cause great and irreparable injury to the business reputation and
goodwill
Respondent filed its answer that the petitioner's corporate name is "CONVERSE
RUBBER CORPORATION" and has been in existence since July 31, 1946, it is
duly organized under the laws of Massachusetts, USA and not licensed to do
business in the Philippines and it is not doing business on its own in the
Philippines; and,
Petitioner's lone witness, Mrs. Carmen B. Pacquing, a duly licensed private
merchant selling CONVERSE rubber shoes in the local market since 1956 with
an averaged of twelve to twenty pairs a month purchased mostly by basketball
players of local private educational institutions like Ateneo, La Salle and San
Beda.
Respondent presented as its lone witness the secretary of said corporation who
testified that respondent has been selling on wholesale basis "Universal
Converse" sandals since 1962 and "Universal Converse" rubber shoes since 196
as supported with Invoices.
The Director of Patents dismissed the opposition of the petitioner and gave due
course to respondent's application, stating that it will be futile for it to establish
that "CONVERSE" as part of petitioners corporate name Identifies its rubber
shoes
A motion for reconsideration having been denied by the respondent Director of
Patents, petitioner instituted the instant petition for review.
ISSUE
Whether the respondent's partial appropriation of petitioner's corporate name is
of such character that it is calculated to deceive or confuse the public to the injury
of the petitioner to which the name belongs.

464

RULING
The decision of the Director of Patents is hereby set aside and denying
Respondent Universal Rubber Products, Inc.'s application for registration of the
trademark "UNIVERSAL CONVERSE AND DEVICE" on its rubber shoes and
slippers.
A trade name is any individual name or surname, firm name, device or word used
by manufacturers, industrialists, merchants and others to Identify their
businesses, vocations or occupations.
From a cursory appreciation of the petitioner's corporate name "CONVERSE
RUBBER CORPORATION,' it is evident that the word "CONVERSE" is the
dominant word which Identifies petitioner from other corporations engaged in
similar business. Respondent, in the stipulation of facts, admitted petitioner's
existence since 1946 as a duly organized foreign corporation engaged in the
manufacture of rubber shoes. This admission necessarily betrays its knowledge
of the reputation and business of petitioner even before it applied for registration
of the trademark in question. Knowing, therefore, that the word "CONVERSE"
belongs to and is being used by petitioner, and is in fact the dominant word in
petitioner's corporate name, respondent has no right to appropriate the same for
use on its products which are similar to those being produced by petitioner.
It is a corollary logical deduction that while Converse Rubber Corporation is not
licensed to do business in the country and is not actually doing business here, it
does not mean that its goods are not being sold here or that it has not earned a
reputation or goodwill as regards its products. A foreign corporation which has
never done any business in the Philippines and which is unlicensed and
unregistered to do business here, but is widely and favorably known in the
Philippines through the use therein of its products bearing its corporate and
tradename, has a legal right to maintain an action in the Philippines to restrain
the residents and inhabitants thereof from organizing a corporation therein
bearing the same name as the foreign corporation, when it appears that they
have personal knowledge of the existence of such a foreign corporation, and it is
apparent that the purpose of the proposed domestic corporation is to deal and
trade in the same goods as those of the foreign corporation.
Another factor why respondent's applications should be denied is the confusing
similarity between its trademark is the imprint of logo in a circular manner as
such like the petitioner would likely cause confusion or mistake on the part of the
buying public.
By appropriating the word "CONVERSE," respondent's products are likely to be
mistaken as having been produced by petitioner. "The risk of damage is not
limited to a possible confusion of goods but also includes confusion of reputation
if the public could reasonably assume that the goods of the parties originated
from the same source.

465

Wang Laboratories, Inc. vs. Mendoza,


156 SCRA 44 (1987)
PARAS, J.:
Facts: Petitioner is a corporation duly organized under the laws of the United
States with principal address at One Industrial Avenue, Lowell, Massachusetts,
U.S.A., engaged in the business of manufacturing and selling computers
worldwide. In the Philippines, petitioner sells its products to EXXBYTE
TECHNOLOGIES CORPORATION, hereinafter referred to as EXXBYTE, its
exclusive distributor. Angara, Concepcion, Regala & Cruz Law Offices
(ACCRALAW) is a duly registered professional partnership.
On June 10, 1981, ACCRALAW and EXXBYTE entered into another contract for
the development of a data processing software program needed to computerize
the ACCRALAW office. Subsequent thereto and for one reason or the other, the
contract for the development of a data processing software program or ISLA was
not implemented. On May 7, 1984, ACCRALAW filed a complaint for breach of
contract with damages, replevin and attachment against herein petitioner. On
May 23, 1984, petitioner filed a Motion to Dismiss the complaint on the ground
that there was improper service of summons, hence, the court below had not
obtained jurisdiction over the person of the petitioner. On March 29, 1985,
ACCRALAW filed an Ex-Abundante Cautela Motion for leave to Effect
Extraterritorial Service of Summons on petitioner.
In an order dated April 24, 1985, respondent Judge Mendoza, among others,
granted the Ex-Abundante Cautela Motion to Effect Extraterritorial Service of
Summons, denied the petitioner's motion to dismiss on the ground that it had
voluntarily submitted itself to the jurisdiction of the court, and thus declined to
consider the legal and factual issues raised in the Motion to Dismiss.
Issue: That the respondent Court has acquired jurisdiction over the person of the
petitioner, a foreign corporation.
Ruling: It is well settled that "A voluntary appearance is a waiver of the necessity
of formal notice." Thus, it has been held that when the appearance is by motion
for the purpose of objecting to the jurisdiction of the court over the person it must
be for the sole and separate purpose of objecting to the jurisdiction of the Court.
If the appearance is for any other purpose, the defendant is deemed to have
466

submitted himself to the jurisdiction of the court. Such an appearance gives the
court jurisdiction over the person (Flores v. Zurbito, 37 Phil. 746 [1918]).
Clarifying further, the Court has likewise ruled that even though the defendant
objects to the jurisdiction of the Court, if at the same time he alleges any nonjurisdictional ground for dismissing the action, the Court acquires jurisdiction over
him (Far East International Import & Export Corporation v. Nankai Kogyo, Co.,
Ltd., 6 SCRA 725 11962]). Hence, the petition is DISMISSED for lack of merit,
with costs against the petitioner.

467

N.V. Reederij Amsterdam vs. Commissioner on International Revenue,


162 SCRA 487 (1988)
Ponente : Justice Gancayco

Facts : Both vessels of petitioner N.V. Reederij Amsterdam called on Philippine


ports to load cargoes for foreign destinations. The freight fees for these
transactions were paid in abroad. In these two transactions, petition Royal
Interocean Lines acted as husbanding agent for a fee or commission on said
vessels. No income tax has been paid by Amsterdam on the freight receipts. As
a result, Commissioner of Internal Revenue filed the corresponding income tax
returns for the petitioner. Commissioner assessed petitioner for deficiency of
income tax, as a non-resident foreign corporation NOT engaged in trade or
business. On the assumption that the said petitioner is a foreign corporation
engaged in trade or business in the Philippines, petitioner Royal Interocean Lines
filed an income tax return of the aforementioned vessels and paid the tax in
pursuant to their supposed classification. On the same date, petitioner Royal
Interocean Lines, as the husbanding agent of Amsterdam, filed a written protest
against

the

abovementioned

assessment

made

by

the

respondent

Commissioner. The protest was denied. On appeal, CTA modified the


assessment by eliminating the 50% fraud compromise penalties imposed upon
petitioners. Petitioner still was not satisfied and decided to appeal to the SC.

Issue : Whether or not N.V. Reederij Amsterdam should be taxed as a foreign


corporation not engaged in trade or business in the Philippines?
Ruling : Petitioner is a foreign corporation not authorized or licensed to do
business in the Philippines. It does not have a branch in the Philippines, and it
only made two calls in Philippine ports, one in 1963 and the other in 1964.
In order that a foreign corporation may be considered engaged in trade or
business, its business transactions must be continuous. A casual business
activity in the Philippines by a foreign corporation does not amount to engaging in
trade

or

business

in

the

Philippines

for

income

tax

purposes.

A foreign corporation doing business in the Philippines is taxable on income


solely from sources within the Philippines. It is permitted to claim deductions from
gross income but only to the extent connected with income earned in the
Philippines. On the other hand, foreign corporations not doing business in the
Philippines are taxable on income from all sources within the Philippines. The tax
468

is 30% (now 35% for non-resident foreign corporation which is also known as
foreign corporation not engaged in trade or business) of such gross income.
(*take note that in a resident foreign corporation, what is being taxed is the
taxable income, which is with deductions, as compared to a non-resident foreign
corporation

which

the

tax

base

is

gross

income)

Petitioner Amsterdam is a non-resident foreign corporation, organized and


existing under the laws of the Netherlands with principal office in Amsterdam and
not licensed to do business in the Philippines.

469

Schmid & Oberly Inc. vs. RJL Martinez Fishing Corporation,


166 SCRA 493 (1988)
Facts:
RJL Martinez Fishing Corporation is engaged in deep-sea fishing, and in the
course of its business, needed electrical generators for the operation of its
business; that the defendant sells electrical generators with the brand of
"Nagata", a Japanese product; that the supplier is the manufacturer, the D.
Nagata Co. Ltd., of Japan, that the defendant Schmid & Oberly Inc. advertised
the 12 Nagata generators for sale; that the plaintiff purchased 12 brand new
Nagata generators, as advertised by herein defendant; that through an
irrevocable line of credit, the D. Nagata Co., Ltd., shipped to the plaintiff 12
electric generators, and the latter paid the amount of the purchase price; that the
12 generators were found to be factory defective; that the plaintiff informed the
defendant herein that it shall return the 12 generators as in fact three of the 12
were actually returned to the defendant; that the plaintiff sued the defendant on
the warranty; asking for rescission of the contract; that the defendant be ordered
to accept the generators and be ordered to pay back the purchase money; and
that the plaintiff asked for damages. the Court of Appeals affirmed the decision
of the trial court ordering petitioner to refund to private respondent the purchase
price for the twelve (12) generators and to accept delivery of the same and to pay
s and attorney's fees, with a slight modification as to the amount to be refunded.
In its resolution of the motion for reconsideration, the Court of Appeals further
modified the trial courts decision as to the award of consequential damages.
Issues:
Whether Schmid is not liable for the reimbursement claimed by RJL Martinez nor
for the latter's unsubstantiated claim of PI 10.33 operational losses a day nor for
exemplary damages, attorney's fees and costs
Ruling:
The Civil Code defines a contract of sale, thus ART. 458. By the contract of sale
one of the contracting parties obligates himself to transfer the ownership of and
to deliver a determinate thing, and the other to pay therefor a price certain in
money or its equivalent. It has been said that the essence of the contract of sale
is transfer of title or agreement to transfer it for a price paid or promised If such
transfer puts the transferee in the attitude or position of an owner and makes him
liable to the transferor as a debtor for the agreed price, and not merely as an
agent who must account for the proceeds of a resale, the transaction is, a sale.
The chief feature of a commercial broker and a commercial merchant is that in
effecting a sale, they are merely intermediaries or middle-men, and act in a
certain sense as the agent of both parties to the transaction. The Supreme Court
reversed the decision of Court of Appeals.

470

Marubeni Nederland B.V. vs. Tensuan 190 SCRA 105 (1990)


Ponente : Fernan, C. J. :

FACTS:
Marubeni Nederland BV and Teodoro Development Corporation (DBT) entered
into a contract in Tokyo, Japan whereby the parties agreed that petitioner will
supply the necessary equipment for the construction of DBTs lime plant at
Guimaras Islands in Iloilo. DBT also obtained a financial loan from Marubeni
guaranteed by National Investment and Development Corporation (NIDC). When
the loan amount fell due, DBT wrote to NIDC claiming that Marubeni had not
delivered its contractual commitments and sought for re-schedule of payment
hence, the Settlement Agreement. Later on, DBT rejected the lime plant on the
ground that it was not constructed based on the agreement and demanded
indemnification. However, Marubeni refused the unilateral rejection and so, DBT
sued Marubeni but the latter sought for its dismissal on the ground that it is a
foreign corporation neither doing nor licensed to do business in the Philippines.
Said motion was denied by the lower court hence this petition for certiorari.
ISSUE: Whether Marubeni can be considered doing business in the Philippines.
RULING:
The Supreme Court (SC) ruled that Marubeni was doing business in the
Philippines. Since under RA 5455, soliciting orders, purchases or service
contracts by a foreign firm constitute doing business in the Philippines even if the
enterprise has no office or fixed place of business in the Philippines.
Notwithstanding Marubenis lack of license, the Philippine court has jurisdiction
over the case at bar hence, the petition was dismissed.

471

Merrill Lynch Futures, Inc. vs. Court of Appeals,


211 SCRA 824 (1992)
FACTS: Merrill Lynch Futures, Inc. (ML FUTURES) stated that it is a nonresident foreign corporation organized in the USA, not doing business in the
Philippines and that it is duly licensed to act as a broker by executing orders to
buy and sell future contracts received from its customers on U.S futures
exchanges.
ML FUTURES alleged that it entered into a Futures Customer Agreement
with the defendant spouses, thus acted as the latter's broker. Pursuant to the
contract, orders were transmitted to ML FUTURES by the Lara spouses through
Merrill Lynch Philippines, Inc. (MLPI) , a Philippine corporation and a company
servicing plaintiffs customers. The Lara spouses were duly advised by MLPI that
the latter was not a broker and that it did not have a license from the SEC to
operate as a commodity trading advisor but the Lara spouses still actively traded
in futures contracts for four years or so, incurring a balace loss of US$84,836.27,
which ML FUTURES asked them to pay.
The Lara spouses refused to pay alleging that the transactions were null
and void because MLPI had no license to operate. Hence, on November 23,
1987, ML FUTURES filed a complaint with the Regional Trial Court at Quezon
City against the Spouses Pedro M. Lara and Elisa G. Lara for the recovery of a
debt and interest thereon, damages, and attorney's fees. The trial court and
subsequently the Court of Appeals ruled in favor of the spouses holding that no
foreign corporation transacting business in the Philippines without a license, or its
successors or assigns, shall be permitted to maintain or intervene in any action,
suit or proceeding in any court or administrative agency in the Philippines in
pursuant to the provisions of Section 133 of the Corporation Code. Hence this
petition.
ISSUE
Whether or not ML FUTURES has the capacity to sue in the Philippines.
RULING
Yes. The Court is satisfied that the facts on record adequately establish
that ML FUTURES, operating in the United States, had indeed done business
with the Lara Spouses in the Philippines over several years, had done so at all
times through Merrill Lynch Philippines, Inc. (MLPI), a corporation organized in
this country, and had executed all these transactions without ML FUTURES
being licensed to so transact business here, and without MLPI being authorized
to operate as a commodity futures trading advisor. These are the factual findings
of both the Trial Court and the Court of Appeals. These, too, are the conclusions
of the Securities Exchange Commission which denied MLPIs application to
operate as a commodity futures trading advisor, a denial subsequently affirmed
by the Court of Appeals. Prescinding from the proposition that factual findings of
the Court of Appeals are generally conclusive this Court has been cited to no
circumstance of substance to warrant reversal of said Appellate Courts findings
or conclusions in this case.
472

A foreign corporation doing business in the Philippines may sue in


Philippine courts although not authorized to do business here against a
Philippine citizen who had contracted with and been benefited by said
corporation. There would seem to be no question that the Laras received benefits
generated by their business relations with ML FUTURES. Those business
relations, according to the Laras themselves, spanned a period of seven (7)
years; and they evidently found those relations to be of such profitability as
warranted their maintaining them for that no insignificant period of time;
otherwise, it is reasonably certain that they would have terminated their dealings
with ML FUTURES much, much earlier. In fact, even as regards their last
transaction, in which the Laras allegedly suffered a loss in the sum of
US$160,749.69, the Laras nonetheless still received some monetary advantage,
for ML FUTURES credited them with the amount of US$75,913.42 then due to
them, thus reducing their debt to US$84,836.27. Given these facts, and
assuming that the Lara Spouses were aware from the outset that ML FUTURES
had no license to do business in this country and MLPI, no authority to act as
broker for it, it would appear quite inequitable for the Laras to evade payment of
an otherwise legitimate indebtedness due and owing to ML FUTURES upon the
plea that it should not have done business in this country in the first place, or that
its agent in this country, MLPI, had no license either to operate as a commodity
and/ or financial futures broker.
The rule is that a party is estopped to challenge the personality of a
corporation after having acknowledged the same by entering into a contract with
it. And the "doctrine of estoppel to deny corporate existence applies to foreign as
well as to domestic corporations;" "one who has dealt with a corporation of
foreign origin as a corporate entity is estopped to deny its corporate existence
and capacity." The principle "will be applied to prevent a person contracting with
a foreign corporation from later taking advantage of its noncompliance with the
statutes, chiefly in cases where such person has received the benefits of the
contract (Sherwood v. Alvis, 83 Ala 115, 3 So 307, limited and distinguished in
Dudley v. Collier, 87 Ala 431, 6 So 304; Spinney v. Miller, 114 Iowa 210, 86 NW
317), where such person has acted as agent for the corporation and has violated
his fiduciary obligations as such, and where the statute does not provide that the
contract shall be void, but merely fixes a special penalty for violation of the
statute. . . ."

473

Philipp Morris, Inc. vs. Court of Appeals, 224 SCRA 576 (1993)
Melo, J.
FACTS OF THE CASE

Philip Morris, Incorporated is a corporation organized under the laws of the


State of Virginia, United States of America, and does business at 100 Park
Avenue, New York, New York, United States of America. The two other plaintiff
foreign corporations, which are wholly-owned subsidiaries of Philip Morris, Inc.,
are similarly not doing business in the Philippines but are suing on an isolated
transaction. As registered owners "MARK VII", "MARK TEN", and "LARK" per
certificates of registration issued by the Philippine Patent Office on April 26,
1973, May 28, 1964, and March 25, 1964, plaintiffs-petitioners asserted that
defendant Fortune Tobacco Corporation has no right to manufacture and sell
cigarettes bearing the allegedly identical or confusingly similar trademark
"MARK" in contravention of Section 22 of the Trademark Law, and should,
therefore, be precluded during the pendency of the case from performing the acts
complained of via a preliminary injunction.

For its part, Fortune Tobacco Corporation admitted petitioners' certificates


of registration with the Philippine Patent Office subject to the affirmative and
special defense on misjoinder of party plaintiffs. Private respondent alleged
further that it has been authorized by the Bureau of Internal Revenue to
manufacture and sell cigarettes bearing the trademark "MARK", and that "MARK"
is a common word which cannot be exclusively appropriated (p.158, Court of
Appeals Rollo in A.C.-G.R. SP No. 13132). On March 28, 1983, petitioners'
prayer for preliminary injunction was denied by the Presiding Judge of Branch
166 of the Regional Trial Court of the National Capital Judicial Region stationed
at Pasig.

ISSUE

Whether Philipp Morris, Inc., a foreign corporation not doing business in


the Philippines, can file a civil action for trademark infringement under Philippine
law.

474

RULING
Yes. Section 21-A of the Trademark Law reads as follows:
Sec. 21-A. Any foreign corporation or juristic person to which
a mark or trade-name has been registered or assigned under
this act may bring an action hereunder for infringement, for
unfair competition, or false designation of origin and false
description, whether or not it has been licensed to do
business in the Philippines under Act Numbered Fourteen
hundred and fifty-nine, as amended, otherwise known as the
Corporation Law, at the time it brings complaint: Provided,
That the country of which the said foreign corporation or
juristic person is a citizen or in which it is domiciled, by treaty,
convention or law, grants a similar privilege to corporate or
juristic persons of the Philippines. (As inserted by Sec.7 of
Republic Act No. 638.).

However, on May, 21, 1984, Section 21-A, the provision under


consideration, was qualified by this Court in La Chemise Lacoste S.A. vs.
Fernandez, to the effect that a foreign corporation not doing business in the
Philippines may have the right to sue before Philippine Courts, but existing
adjective axioms require that qualifying circumstances necessary for the
assertion of such right should first be affirmatively pleaded. Indeed, it is not
sufficient for a foreign corporation suing under Section 21-A to simply allege its
alien origin. Rather, it must additionally allege its personality to sue. Relative to
this condition precedent, it may be observed that petitioners were not remiss in
averring their personality to lodge a complaint for infringement (p. 75,Rollo in ACG.R. SP No. 13132) especially so when they asserted that the main action for
infringement is anchored on an isolated transaction.

475

Signetics Corporation vs. Court of Appeals, 225 SCRa 737 (1993)


Vitug, J.:
FACTS:
The petitioner, Signetics was organized under the laws of the United
States of America. Through Signetics Filipinas Corporation (SigFil), a whollyowned subsidiary, Signetics entered into lease contract over a piece of land with
Fruehauf Electronics Phils., Inc. (Freuhauf).

Freuhauf sued Signetics for damages, accounting or return of certain


machinery, equipment and accessories, as well as the transfer of title and
surrender of possession of the buildings, installations and improvements on the
leased land, before the RTC of Pasig (Civil Case No. 59264). Claiming that
Signetics caused SigFil to insert in the lease contract the words "machineries,
equipment and accessories," the defendants were able to withdraw these assets
from the cost-free transfer provision of the contract.
Service of summons was made on Signetics through TEAM Pacific Corp.
on the basis of the allegation that Signetics is a "subsidiary of US PHILIPS
CORPORATION, and may be served summons at Philips Electrical Lamps, Inc.,
Las Pias, Metro Manila and/or c/o Technology Electronics Assembly &
Management (TEAM) Pacific Corporation, Electronics Avenue, FTI Complex,
Taguig, Metro Manila," service of summons was made on Signetics through
TEAM Pacific Corporation.
Petitioner filed a motion to dismiss the complaint on the ground of lack of
jurisdiction over its person. Invoking Section 14, Rule 14, of the Rules of Court
and the rule laid down in Pacific Micronisian Line, Inc., v. Del Rosario and
Pelington to the effect that the fact of doing business in the Philippines should
first be established in order that summons could be validly made and jurisdiction
acquired by the court over a foreign corporation.
The RTC denied the Motion to dismiss. While the CA affirmed RTC. Hence
this petition. The petitioner argues that what was effectively alleged in the
complaint as an activity of doing business was "the mere equity investment" of
petitioner in SigFil, which the petitioner insists, had theretofore been transferred
to TEAM holdings, Ltd.

476

ISSUE:
Whether the lower court, had correctly assumed jurisdiction over the
petitioner, a foreign corporation, on its claim in a motion to dismiss, that it had
since ceased to do business in the Philippines.

RULLING:
YES. Signetics cannot, at least in this early stage, assail, on the one hand,
the veracity and correctness of the allegations in the complaint and proceed, on
the other hand, to prove its own, in order to hasten a peremptory escape. As
explained by the Court in Pacific Micronisian, summons may be served upon an
agent of the defendant who may not necessarily be its "resident agent
designated in accordance with law." The term "agent", in the context it is used in
Section 14, refers to its general meaning, i.e., one who acts on behalf of a
principal.
The allegations in the complaint have thus been able to amply convey that
not only is TEAM Pacific the business conduit of the petitioner in the Philippines
but that, also, by the charge of fraud, is none other than the petitioner itself.
The rule is that, a foreign corporation, although not engaged in business in
the Philippines, may still look up to our courts for relief; reciprocally, such
corporation may likewise be "sued in Philippine courts for acts done against a
person or persons in the Philippines" (Facilities Management Corporation v. De
la Osa), provided that, in the latter case, it would not be impossible for court
processes to reach the foreign corporation, a matter that can later be
consequential in the proper execution of judgment. Hence, a State may not
exercise jurisdiction in the absence of some good basis (and not offensive to
traditional notions of fair play and substantial justice) for effectively exercising it,
whether the proceedings are in rem, quasi in rem or in personam.

477

George Grotjahn GBBH & Co. vs. Isnani 235 SCRA 216 (1994)
FACTS:
Petitioner is a multi-national company organized and existing under the
laws of Germany. It applied with the Securities and Exchange Commission the
establishment of a regional headquarters in the Philippines to which was granted.
Private respondent Lanchinebre was a sales representative of the petitioner
company form 1983-1992. On March of 1992, she secured a loan of P25k and
cash advances of P10k. In total, P12, 170 remained unpaid. Despite repeated
demands for payments, Lanchinebre failed to settle her obligation with the
respondent.
In July of 1992, Lanchinebre filed with the NLRC a case for illegal
suspension, illegal dismissal, and non-payment of commission against petitioner.
Petitioner countered by filing a petition for damages against the private
respondent. Both cases were consolidated and were raffled to respondent judge.
Lanchinebre filed a motion to dismiss. The respondent judge granted the motion
to dismiss on grounds that trial court has no jurisdiction over the case as it was a
controversy between employer and an employee. According to the ruling, NLRC
has the original and exclusive jurisdiction of the case.
Hence, this petition.

ISSUE

WON the ruling of the judge in granting the motion to dismiss is correct.

RULINGS
The judge is incorrect in dismissing the case.
The claim to the principal relief sought is to be resolved not by reference to
the Labor Code or other labor relations statute or a collective bargaining
agreement but by the general civil law, the jurisdiction over the dispute belongs
to the regular courts of justice and not to the Labor Arbiter and the NLRC. In such
situations, resolutions of the dispute requires expertise, not in labor management
relations nor in wage structures and other terms and conditions of employment,
but rather in the application of the general civil law. Clearly, such claims fall
outside the area of competence or expertise ordinarily ascribed to Labor Arbiters
and the NLRC and the rationale for granting jurisdiction over such claims to these
agencies disappears.
The trial court should not have held itself without jurisdiction over case. It is
true that the loan and cash advances sought to be recovered by petitioner were
contracted by private respondent Romana Lanchinebre while she was still in the
478

employ of petitioner. Nonetheless, it does not follow that Article 217 of the Labor
Code covers their relationship. Not every dispute between an employer and
employee involves matters that only labor arbiters and the NLRC can resolve in
the exercise of their adjudicatory or quasi-judicial powers. The jurisdiction of
labor arbiters and the NLRC under Article 217 of the Labor Code is limited to
disputes arising from an employer-employee relationship which can only be
resolved by reference to the Labor Code, other labor statutes, or their collective
bargaining agreement.
Secondly, the trial court erred in holding that petitioner does not have
capacity to sue in the Philippines. It is clear that petitioner is a foreign corporation
doing business in the Philippines. Petitioner is covered by the Omnibus
Investment Code of 1987. Said law defines "doing business," as follows:
. . . shall include soliciting orders, purchases, service contracts, opening offices,
whether called "liaison" offices or branches; appointing representatives or
distributors who are domiciled in the Philippines or who in any calendar year stay
in the Philippines for a period or periods totalling one hundred eighty (180) days
or more; participating in the management, supervision or control of any domestic
business firm, entity or corporation in the Philippines, and any other act or acts
that imply a continuity of commercial dealings or arrangements and contemplate
to that extent the performance of acts or works, or the exercise of some of the
functions normally incident to, and in progressive prosecution of, commercial
gain or of the purpose and object of the business organization.
There is no general rule or governing principle as to what constitutes
"doing" or "engaging in" or "transacting" business in the Philippines. Each case
must be judged in the light of its peculiar circumstances. In the case at bench,
petitioner does not engage in commercial dealings or activities in the country
because it is precluded from doing so by P.D. No. 218, under which it was
established. Nonetheless, it has been continuously, since 1983, acting as
supervision, communications and coordination center for its home office's
affiliates in Singapore, and in the process has named its local agent and has
employed Philippine nationals like private respondent Romana Lanchinebre.
From this uninterrupted performance by petitioner of acts pursuant to its primary
purposes and functions as a regional/area headquarters for its home office, it is
clear that petitioner is doing business in the country. Moreover, private
respondents are estopped from assailing the personality of petitioner.

479

Litton Mill, Inc. vs. Court of Appeals, 256 SCRA 696 (1996)
FACTS:

RTC denied private respondent's plea that it is a foreign corporation not doing
business in the Philippines and therefore not subject to the jurisdiction of
Philippine courts. CA: annulled RTC SC: this petition to review Petitioner Litton
entered into an agreement with Empire Sales Philippines Corporation (Empire),
as local agent of private respondent Gelhaar, a corporation organized under the
laws of the US, whereby Litton agreed to supply Gelhaar 7,770 dozens of soccer
jerseys. The agreement stipulated that before it could collect from the bank on
the letter of credit (which was due to expire on February 14, 1984), Litton must
present an inspection certificate issued by Gelhaar's agent in the Philippines,
Empire Sales that the goods were in satisfactory condition. Litton sent four
shipments totalling 4,770 and a fifth shipment, consisting of 2,110 dozens of the
jerseys, was inspected by Empire, but Empire refused to issue the required
certificate of inspection. Alleging that Empire's refusal to issue a certificate was
without valid reason, Litton filed a complaint with the RTC of Pasig (Branch 158)
for specific performance. Litton sought the issuance of a writ of preliminary
mandatory injunction to compel Empire to issue the inspection certificate
covering the 2,110 dozen jerseys and the recovery of compensatory and
exemplary damages, costs, attorney's fees and other just and equitable relief.
The trial court issued the writ. The next day, Empire issued the inspection
certificate, so that the cargo was shipped on time. On February 8, 1984, Atty.
Noval filed in behalf of the defendants a "Motion For Extension of Time To File
An Answer/Responsive Pleading, which were (filed on different times man)
granted by the court, with the exception of the last, which the Court denied. On
his motion, the court later reconsidered its order of denial. On January 29, 1985,
the law firm of Sycip et al entered a special appearance for the purpose of
objecting to the jurisdiction of the court over Gelhaar. On February 4, 1985, it
moved to dismiss the case and to quash the summons on the ground that
Gelhaar was a foreign corporation not doing business in the Philippines, and as
such, was beyond the reach of the local courts. It contended that Litton failed to
allege and prove that Gelhaar was doing business in the Philippines, which they
argued was required by the ruling in Pacific Micronisian Lines, Inc. v. Del
Rosario, 1 before summons could be served under Rule 14, Sec. 14. It likewise
denied the authority of Atty. Noval to appear for Gelhaar and contended that the
answer filed by Atty. Noval did not amount to Gelhaar's submission to the
jurisdiction of the court. Of course, Litton opposed the motion. Empire moved to
dismiss on the ground of failure of the complaint to state a cause of action since
the complaint alleged that Empire only acted as agent of Gelhaar; that it was
made partydefendant only for the purpose of securing the issuance of an
inspection certificate; and that it had already issued such certificate and the
shipment had already been shipped on time. For his part, Atty. Noval claimed
that he had been authorized by Gelhaar to appear for it in the case; that he had
in fact given legal advice to Empire and his advice had been transmitted to
Gelhaar; that Gelhaar had been furnished a copy of the answer; that Gelhaar
480

denied his authority only on December of 1984; and that the belated repudiation
of his authority could be only an afterthought because of problems which had
developed between Gelhaar and Empire. (Gelhaar refused to pay Empire for its
services as agent man gud). Nevertheless, Atty. Noval withdrew his appearance
with respect to Gelhaar. RTC: issued an order denying for lack of merit Gelhaar's
motion to dismiss and to quash the summons. It held that Gelhaar was doing
business in the Philippines, and that the service of summons on Gelhaar was
therefore valid. Gelhaar filed a motion for reconsideration, but its motion was
denied. CA: reversed RTC. It held that proof that Gelhaar was doing business in
the Philippines should have been presented because, under the doctrine of
Pacific Micronisian, this is a condition sine qua non for the service of summons
under Rule 14, Sec. 14 of the Rules of Court, and that it was error for the RTC to
rely on the mere allegations of the complaint. Further, it held that neither did the
RTC acquire jurisdiction over Gelhaar through voluntary submission because the
authority of Atty. Noval to represent Gelhaar had been questioned. Pursuant to
Rule 138, Sec. 21, the trial court should have required Atty. Noval to prove his
authority. It ordered pa gyud RTC to issue anew summons to be served on
Empire Sales Philippines Corporation. Hence this petition.

ISSUE:

Whether the jurisdiction over Gelhaar was acquired by the trial court by the
service of summons through Gelhaar's agent (si Empire) (YES!) and, at any rate,
by the voluntary appearance of Atty. Noval as counsel of Gelhaar (NO!).

RULING:

We sustain petitioner's contention based on the first ground, namely, that the trial
court acquired jurisdiction over Gelhaar by service of summons upon its agent
pursuant to Rule 14, Sec. 14 (pero I think Sec12 nani sa revised ROC). First. The
appellate court invoked the ruling in Pacific Micronisian, in which it was stated
that the fact of doing business must first be established before summons can be
served in accordance with Rule 14, Sec. 14. This section provides for three
modes of effecting service upon a private corporation, namely: (a) service may
be made on its resident agent designated in accordance with law for that
purpose, or, (b) if there be no such agent, on the government official designated
by law to that effect, or (c) on any of its officers or agents within the Philippines.
But, it should be noted, in order that service may be effected, said section also
requires that the foreign corporation be one which is doing business in the
Philippines. This is a sine qua non requirement. This fact must first be
established in order that summons can be made and jurisdiction acquired. In the
later case of Signetics Corporation v. CA however, we clarified the holding in
Pacific Micronisian, thus: The petitioner opines that the phrase, "(the) fact (of
doing business in the Philippines) must first be established in order that
481

summons be made and jurisdiction acquired," used in the above pronouncement,


would indicate that a mere allegation to that effect in the complaint is not enough
there must instead be proof of doing business. In any case, the petitioner
points out, the allegations themselves did not sufficiently show the fact of its
doing business in the Philippines. It should be recalled that jurisdiction and venue
of actions are, as they should so be, initially determined by the allegations of the
complaint. Jurisdiction cannot be made to depend on independent pleas set up in
a mere motion to dismiss, otherwise jurisdiction would become dependent almost
entirely upon the defendant. The fact of doing business must then, in the first
place, be established by appropriate allegations in the complaint. This is what the
Court should be seen to have meant in the Pacific Micronisian case. The
complaint, it is true, may have been vaguely structured but, taken correlatively,
not disjunctively as the petitioner would rather suggest, it is not really so weak as
to be fatally deficient in the above requirement. . . . REMEDIAL LAW; CIVIL
PROCEDURE; SUMMONS; A COURT NEED NOT GO BEYOND THE
ALLEGATIONS IN THE COMPLAINT TO DETERMINE WHETHER OR NOT A
DEFENDANT FOREIGN CORPORATION IS DOING BUSINESS FOR THE
PURPOSE OF RULE 14, SECTION 14; CASE AT BAR. A court need not go
beyond the allegations in the complaint to determine whether or not a defendant
foreign corporation is doing business for the purpose of Rule 14, 14. In the case
at bar, the allegation that Empire, for and in behalf of Gelhaar, ordered 7,770
dozens of soccer jerseys from Litton and for this purpose Gelhaar caused the
opening of an irrevocable letter of credit in favor of Litton is a sufficient allegation
that Gelhaar was doing business in the Philippines. Second. Gelhaar contends
that the contract with Litton was a single, isolated transaction and that it did not
constitute "doing business." Reference is made to Pacific Micronisian in which
the only act done by the foreign company was to employ a Filipino as a member
of the crew on one of its ships. This court held that the act was an isolated,
incidental or casual transaction, not sufficient to indicate a purpose to engage in
business. It is not really the fact that there is only a single act done that is
material. The other circumstances of the case must be considered. Thus, in
Wang Laboratories, Inc. v. Mendoza, it was held that where a single act or
transaction of a foreign corporation is not merely incidental or casual but is of
such character as distinctly to indicate a purpose on the part of the foreign
corporation to do other business in the state, such act will be considered as
constituting doing business. This Court referred to acts which were in the
ordinary course of business of the foreign corporation. In the case at bar, the trial
court was certainly correct in holding that Gelhaar's act in purchasing soccer
jerseys to be within the ordinary course of business of the company considering
that it was engaged in the manufacture of uniforms. The acts noted above are of
such a character as to indicate a purpose to do business.
In accordance with Rule 14, 14, service upon Gelhaar could be made in three
ways: (1) by serving upon the agent designated in accordance with law to accept
service of summons; (2) if there is no resident agent, by service on the
government official designated by law to that effect; and (3) by serving on any
officer or agent of said corporation within the Philippines. Here, service was
made through Gelhaar's agent, the Empire Sales Philippines Corp. There was,
therefore, a valid service of summons on Gelhaar, sufficient to confer on the trial
482

court jurisdiction over the person of Gelhaar. Third. On the question, however, of
whether the appearance of Atty. Noval in behalf of Gelhaar was binding on the
latter, we hold that the CA correctly ruled that it was not. Atty. Noval admits that
he was not appointed by Gelhaar as its counsel. What he claims is simply that
Gelhaar knew of the filing of the case in the trial court and of his representation
but Gelhaar did not object. Atty. Noval contends that there was thus a tacit
confirmation of his authority. Gelhaar claims, however, that it was only sometime
in December, 1994 when it found out that the answer which Atty. Noval had filed
in June was also made in its behalf. Gelhaar in fact sent a telex message dated
January 15, 1985 to its counsel, the Sycip law firm, stating WE NEVER
AUTHORIZED THE RETENTION OF MR. NOVAL ON OUR BEHALF. WE HAVE
NEVER EXCHANGED CORRESPONDENCE NOR HAD ANY TELEPHONE
CONVERSATIONS WITH HIM RE ANY ASPECT OF THIS CASE, INCL. HIS
FEES. WE ARE TOLD THAT HE HAS FILED AN ANSWER TO LTN'S (Litton's)
COMPLT. PURPORTEDLY ON OUR BEHALF BUT HE HAS NEVER
DISCUSSED THAT ANSWER WITH US NOR EVEN SENT US A DRAFT OR
THE FINAL VERSION OF SUCH ANSWER. WE ARE SENDING SWORN
AFFIDAVITS TO THIS EFFECT BY COURIER. Atty. Noval has not denied any of
these statements. Noval does not claim that he ever directly conferred with
Gelhaar regarding the case. There is no evidence to show that he notified
Gelhaar of his appearance in its behalf, or that he furnished Gelhaar with copies
of pleadings or the answer which he filed in its behalf. No voluntary appearance
by Gelhaar can, therefore, be inferred from the acts of Atty. Noval. Nor can Atty.
Noval's representations in the answer he considered binding on Gelhaar.
Gelhaar should be allowed a new period for filing its own answer. WHEREFORE,
the decision of the Court of Appeals is REVERSED. The order of the trial court
denying the motion to dismiss is hereby REINSTATED, with the MODIFICATION
that Gelhaar is given a new period of ten (10) days for the purpose of filing its
answer. SO ORDERED.

483

Communication Materials and Design, Inc. vs. Court of Appeals, 260


SCRA 673, (1996)
PONENTE: TORRES, JR., J.:

Facts:
Petitioner Communication Materials and Design, Inc. and ASPAC Multi
trade, Inc. also known as ASPAC, domestic corporations entered into a
representative agreement with ITEC International, Inc. (ITEC), a foreign
corporation under State of Alabama, US whereby ASPAC will be the exclusive
representative of ITEC in the distribution and selling of the latters products to the
PLDT. Thus, a License Agreement was entered into by the same parties in
which ASPAC corporation may use and incorporate the name of ITEC in its own
name. Thus, ASPAC became legally and publicly known as ASPAC ITEC
Philippines.

By virtue of this, ASPAC Philippines sold electronic products to

PLDT. However, one year of the 2nd term of their contract with the ITEC, the
latter terminated their agreement on the ground that ASPAC Phil. Violated the
contractual commitment in their agreement. Hence, respondent ITEC filed a civil
case before the RTC of Makati City against the herein petitioners together with
ASPAC and Digital Base Communication (DIGITAL). However, ASPAC filed a
Motion to Dismiss the Complaint on the ground that the ITEC, being a foreign
corporation does not have the capacity to sue before the court in the Philippines.
But it was denied by the RTC. Thus, they elevated it to the CA which the latter
affirmed the decision of RTC denying the motion to dismiss. Hence, this case.

Issue:

Whether respondent ITEC is an unlicensed corporation doing business in


the Philippines cannot sue in the Philippine Court being a foreign corporation

Ruling:

No, it can sue in the Philippine Court even if it is a foreign corporation.

A foreign corporation doing business in the Philippines, although not


authorized to do business in the Philippines against a Philippine citizen or entity
who had contracted with and benefited by said corporation. A party is estopped
to challenge the personality of a corporation after having acknowledged the same
484

by entering into a contract.


In this case, by entering into a representative Agreement with ITEC,
petitioner is charged with knowledge that ITEC was not licensed to engage in
business activities in the country and thus, is estopped from raising in defective
such incapacity of ITEC to sue.

485

Hahn vs. Court of Appeals, 266 SCRA 537 (1997)


Ponente: Justice Mendoza
Facts:
Petitioner Alfred Hahn executed a "Deed of Assignment with Special Power of
Attorney," in favor of BMW on March 7, 1967 and continued doing business
thereon. On February 24, 1993, Alfred received confirmation of the information
from BMW which, in a letter, expressed dissatisfaction with various aspects of
petitioner's business, mentioning among other things, decline in sales,
deteriorating services, and inadequate showroom and warehouse facilities, and
petitioner's alleged failure to comply with the standards for an exclusive BMW
dealer. Even with the letter, they still continued doing business with each other
but BMW will terminate Alfreds exclusive dealership effective June 30, 1993 and
replace it with a standard BMW importer contract.
Because of Hahn's insistence on the former business relation, BMW withdrew on
March 26, 1993 its offer of a "standard importer contract" and terminated the
exclusive dealer relationship effective June 30, 1993. At a conference of BMW
Regional Importers held on April 26, 1993 in Singapore, Hahn was surprised to
find Alvarez (the president of Columbia Motors Corporation (CMC)in which BMW
was arranging the exclusive partnership after Hahn) among those invited from
the Asian region. On April 29, 1993, BMW proposed that Hahn and CMC jointly
import and distribute BMW cars and parts.
Hahn declined the proposal and filed a petition for specific damages.
Issue:
1. Whether private respondent BMW is doing business in the Philippines
2. Whether Hahn is considered as an agent of BMW
Ruling:
1. Yes. Section 1(f)(1) of the Rules and Regulations implementing (IRR) the
Omnibus Investment Code of 1987 (E.O. No. 226) provided:
(f) "Doing business" shall be any act or combination of acts, enumerated in
Article 44 of the Code. In particular, "doing business" includes:
(1).... A foreign firm which does business through middlemen acting in their own
names, such as indentors, commercial brokers or commission merchants, shall
not be deemed doing business in the Philippines. But such indentors, commercial
brokers or commission merchants shall be the ones deemed to be doing
business in the Philippines.
2. Yes. The fact that Hahn invested his own money to put up these service
centers and showrooms does not necessarily prove that he is not an agent of
BMW. For as already noted, there are facts in the record which suggest that
BMW exercised control over Hahn's activities as a dealer and made regular

486

inspections of Hahn's premises to enforce compliance with BMW standards and


specifications.

487

Avon Insurance PLC. Vs. Court of Appeals, 278 SCRa 312 (1997)
Torres, Jr., J.;

FACTS:

Yupangco Cotton Mills engaged to secure with Worldwide Security and


Insurance Co. Inc., a foreign corporation not doing business in the Philippines
with no office, place of business or agents in the Philippines, several of its
properties for the periods July 6, 1979 to July 6, 1980 and from October 1, 1980
to October 1, 1981, under separate insurance policies for the same amount. Both
contracts were covered by reinsurance treaties between Worldwide Surety and
Insurance and several foreign reinsurance companies, including the petitioners.
On December 16, 1979 and May 2, 1981, within the respective effectivity periods
of the policies, the properties therein insured were razed by fire, thereby giving
rise to the obligation of the insurer to indemnify the Yupangco Cotton Mills.
Partial payments were made by Worldwide Surety and Insurance and some of
the reinsurance companies.
On May 2, 1983, Worldwide Surety and Insurance, in a Deed of Assignment,
acknowledged a remaining balance of P19,444,447.75 still due Yupangco Cotton
Mills, and assigned to the latter all reinsurance proceeds still collectible from all
the foreign reinsurance companies. Thus, in its interest as assignee and original
insured, Yupangco Cotton Mills instituted this collection suit against the
petitioners.

ISSUE:

Whether a foreign corporation has rights under Philippine law.

RULING:

YES. A foreign corporation, is one which owes its existence to the laws of
another state, and generally, has no legal existence within the state in which it is
foreign. It was also held that corporations have no legal status beyond the
bounds of the sovereignty by which they are created.
Nevertheless, it is widely accepted that foreign corporations are, by reason of
state comity, allowed to transact business in other states and to sue in the courts
of such fora. In the Philippines foreign corporations are allowed such privileges,
subject to certain restrictions, arising from the state's sovereign right of
488

regulation. Before a foreign corporation can transact business in the country, it


must first obtain a license to transact business here and secure the proper
authorizations under existing law.
If a foreign corporation engages in business activities without the necessary
requirements, it opens itself to court actions against it, but it shall not be allowed
to maintain or intervene in an action, suit or proceeding for its own account in any
court or tribunal or agency in the Philippines.
The purpose of the law in requiring that foreign corporations doing business in
the country be licensed to do so, is to subject them to the jurisdiction of the
Philippine courts, otherwise, a foreign corporation illegally doing business here
because of its refusal or neglect to obtain the required license and authority to do
business may successfully though unfairly plead such neglect or illegal act so as
to avoid service and thereby impugn the jurisdiction of the local courts.
The same danger does not exist among foreign corporations that are indubitably
not doing business in the Philippines. Indeed, if a foreign corporation does not do
business here, there would be no reason for it to be subject to the State's
regulation.

489

Hutchison Ports Philippine Limited vs. Subic Bay Metropolitan


Authority, 339 SCRa 34 (2000)
YNARES-SANTIAGO, J.:

FACTS:
In 1996, Hutchison Ports Philippines Limited (HPPL)won a public bidding made
by the Subic Bay Metropolitan Authority (SBMA). The project was to develop and
operate a modern marine container terminal within the Subic Bay Freeport Zone.
The SBMA Board of Directors already declared HPPL as the winner but later on,
the Office of the President reversed the decision of the Board and ordered a
rebidding. In the rebidding however, HPPL was no longer among the qualified
bidders. Eventually, HPPL filed a petition for injunction to enjoin SBMA from
conducting the rebidding.
ISSUE:
Whether or not Hutchison has the right to file an injunction case against SBMA.
HELD:
No. The declaration made by the SBMA Board declaring HPPL as the winning
bidder was neither final nor unassailable. Under LOI No. 620, all projects
undertaken by the SBMA are subject to the approval of the Office of the
President. Hence, the Board of SBMA is under the control and supervision of the
President of the Philippines. Therefore, the declaration made by the Board did
not vest any right in favor of HPPL.
Further, HPPL cannot sue in the Philippines. It is a foreign corporation registered
under the laws of the British Virgin Islands. It did not register here in the
Philippines.
HPPL cannot invoke that it was suing only on an isolated transaction. The
conduct of bidding is not an isolated transaction. It is doing business here in the
Philippines. The Supreme Court emphasized that as a general rule, doing or
engaging in or transacting business in the Philippines is a case to case basis.
It has often been held that a single act or transaction may be considered as
doing business when a corporation performs acts for which it was created or
exercises some of the functions for which it was organized. The amount or
volume of the business is of no moment, for even a singular act cannot be merely
490

incidental or casual if it indicates the foreign corporations intention to do


business.
Participating in the bidding process constitutes doing business because it
shows the foreign corporations intention to engage in business here. The bidding
for the concession contract is but an exercise of the corporations reason for
creation or existence. Therefore, HPPL has done business here without license.
It cannot now sue in the Philippines without license because its participation in
the bidding is not merely an isolated transaction.
The primary purpose of the license requirement is to compel a foreign
corporation desiring to do business within the Philippines to submit itself to the
jurisdiction of the courts of the state and to enable the government to exercise
jurisdiction over them for the regulation of their activities in this country.

491

MR. Hondings, Ltd. vs. Bajar, 380 SCRA 617 (2002)


Ponente: Justice Villarama Jr.
Facts:
Marcopper Mining Corporation was unable to pay its loans from the Asian
Development Bank (ADB). Later, ADB transferred all its rights to collect from
Marcopper to MR Holdings, Ltd. In order to pay MR Holdings, Marcopper
assigned all its assets to MR Holdings and executed therefor a Deed of
Assignment in MR Holdings favor. Meanwhile, another creditor of Marcopper,
Solidbank Corporation, won a case against Marcopper. The court then issued a
writ of execution directing Sheriff Carlos Bajar to levy Marcoppers assets. MR
Holdings then filed an opposition asserting that it is now the owner of
Marcoppers assets hence, Bajar cannot levy them. The lower court denied MR
Holdings on the ground that the Deed of Assignment was made in bad faith and
that MR Holdings was a foreign corporation doing business without a license in
the Philippines (by virtue of the Deed of Assignment) and as such cannot sue in
the Philippines.
Issue/s:

Whether or not MR Holdings may sue on this particular transaction.

Ruling:
The Supreme Court held in the affirmative. The Supreme Court emphasized the
following rules when it comes to foreign corporations doing business here in the
Philippines:
1. if a foreign corporation does business in the Philippines without a
license, it cannot sue before the Philippine courts;
2. if a foreign corporation is not doing business in the Philippines,
it needs no license to sue before Philippine courts on an isolated
transaction or on a cause of action entirely independent of any
business transaction;
3. if a foreign corporation does business in the Philippines with the
required license, it can sue before Philippine courts on any
transaction.
Being a mere assignee does not constitute doing business in the Philippines.
MR Holdings, a foreign corporation, cannot be said to be doing business simply
because it became an assignee of Marcopper. MR Holdings was not doing
anything else other than being a mere assignee. The only time that MR Holdings
492

is considered to be doing business here is that if it continues the business of


Marcopper , which it did not. Since it is not doing business here, pursuant to the
rules above, it can sue without any license before Philippine courts on an isolated
transaction or on a cause of action entirely independent of any business
transaction. Anent the issue of bad faith, the same was not proven. It appears
that the deed of assignment was an earlier agreement incidental to the loan
agreement between ADB and Marcopper which precedes the action brought by
Solidbank against Marcopper.

493

Lorenzo Shipping Corp. vs. Chubb and Sons, 431 SCRA 266 (2004)
PUNO, J.:

FACTS:
On November 21, 1987, Mayer Steel Pipe Corporation of Binondo, Manila,
loaded 581 bundles of ERW black steel pipes on board the vessel owned by
petitioner Lorenzo Shipping for shipment to Davao City. Petitioner Lorenzo
Shipping issued a clean bill of lading for the account of the consignee, Sumitomo
Corporation of San Francisco, California, USA, which in turn, insured the goods
with respondent Chubb and Sons, Inc.
The M/V Lorcon IV arrived at the Sasa Wharf in Davao City on December 2,
1987. Respondent Transmarine Carriers received the subject shipment which
was discharged on December 4, 1987. The consignee Sumitomo hired the
services of R.J. Del Pan Surveyors to inspect the shipment prior to and
subsequent to discharge. Del Pans Survey Report showed that the subject
shipment was no longer in good condition.
After the survey, respondent Gearbulk loaded the shipment on board its
vessel M/V San Mateo Victory, for carriage to the United States. It issued two
Bills of Lading to be discharged at Vancouver,Washington, U.S.A. All bills of
lading were marked ALL UNITS HEAVILY RUSTED.
Due to its heavily rusted condition, the consignee Sumitomo rejected the
damaged steel pipes and declared them unfit for the purpose they were intended.
It then filed a marine insurance claim with respondent Chubb and Sons, Inc.
which the latter settled in the amount of US$104,151.00.
On December 2, 1988, respondent Chubb and Sons, Inc. filed a complaint for
collection of a sum of money, against respondents Lorenzo Shipping, Gearbulk,
and Transmarine. Respondent Chubb and Sons, Inc. alleged that it is not doing
business in the Philippines, and that it is suing under an isolated transaction.
The Regional Trial Court ruled in favor of the respondent Chubb and Sons,
Inc., finding that: (1) respondent Chubb and Sons, Inc. has the right to institute
this action; and, (2) petitioner Lorenzo Shipping was negligent in the performance
of its obligations as a carrier.
The appellate court denied the appeal and affirmed the decision of the trial
court.
494

The Court of Appeals likewise denied petitioner Lorenzo Shippings Motion for
Reconsideration.

ISSUES:
(1) Whether respondent Chubb and Sons has capacity to sue before the
Philippine courts; and,
(2) Whether petitioner Lorenzo Shipping is negligent in carrying the subject
cargo.

RULING:
Art. 133 of the Corporation Code states:
Doing business without a license. No foreign corporation transacting business
in the Philippines without a license, or its successors or assigns, shall be
permitted to maintain or intervene in any action, suit or proceeding in any court or
administrative agency of the Philippines; but such corporation may be sued or
proceeded against before Philippine courts or administrative tribunals on any
valid cause of action recognized under Philippine laws.
The law does not prohibit foreign corporations from performing single acts of
business. A foreign corporation needs no license to sue before Philippine courts
on an isolated transaction. As held by this Court in the case of Marshall-Wells
Company vs. Elser & Company:
The object of the statute (Secs. 68 and 69, Corporation Law) was not to prevent
the foreign corporation from performing single acts, but to prevent it from
acquiring a domicile for the purpose of business without taking the steps
necessary to render it amenable to suit in the local courts . . . the implication of
the law (being) that it was never the purpose of the legislature to exclude a
foreign corporation which happens to obtain an isolated order for business for the
Philippines, from seeking redress in the Philippine courts.
On the second issue, the Court affirmed the findings of the lower courts that
petitioner Lorenzo Shipping was negligent in its care and custody of the
consignees goods.

495

A bill of lading which has no notation of any defect or damage in the goods is
called a clean bill of lading. A clean bill of lading constitutes prima facie evidence
of the receipt by the carrier of the goods as therein described.

496

Expertravel & Tour, Inc. vs. CA. 459 SCRA 147 (2005)
CALLEJO, SR., J.:
Facts: KAL, through Atty. Aguinaldo, filed a Complaint2 against ETI with the
Regional Trial Court (RTC) of Manila, for the collection of the principal amount
of P260,150.00, plus attorneys fees and exemplary damages.
The verification and certification against forum shopping was signed by Atty.
Aguinaldo, who indicated therein that he was the resident agent and legal
counsel of KAL and had caused the preparation of the complaint.
ETI filed a motion to dismiss the complaint on the ground that Atty. Aguinaldo
was not authorized to execute the verification and certificate of non-forum
shopping as required by Section 5, Rule 7 of the Rules of Court.
KAL opposed the motion, contending that Atty. Aguinaldo was its resident agent
and was registered as such with the Securities and Exchange Commission
(SEC) as required by the Corporation Code of the Philippines.
KAL submitted on March 6, 2000 an Affidavit of even date, executed by its
general manager Suk Kyoo Kim, alleging that the board of directors conducted a
special teleconference on June 25, 1999, which he and Atty. Aguinaldo attended.
It was also averred that in that same teleconference, the board of directors
approved a resolution authorizing Atty. Aguinaldo to execute the certificate of
non-forum shopping and to file the complaint. Suk Kyoo Kim also alleged,
however, that the corporation had no written copy of the aforesaid resolution.
The trial court issued an Order denying the motion to dismiss. ETI filed a motion
for the reconsideration of the Order, contending that it was inappropriate for the
court to take judicial notice of the said teleconference without any prior hearing,
which was also denied.
ETI then filed a petition for certiorari and mandamus in the CA which rendered
judgment dismissing the petition, ruling that the verification and certificate of nonforum shopping executed by Atty. Aguinaldo was sufficient compliance with the
Rules of Court. The motion for reconsideration was likewise denied.
Petitioner now comes to this Court.
Issues: a) Is Atty. Aguinaldo specifically authorized by KAL to sign the
verification and the certificate of non-forum shopping?
497

b) Can the court take judicial notice of the teleconference held?


Ruling: a)Section 5, Rule 7 of the Rules of Court provides:
SEC. 5. Certification against forum shopping. The plaintiff or principal
party shall certify under oath in the complaint or other initiatory pleading
asserting a claim for relief, or in a sworn certification annexed thereto and
simultaneously filed therewith: (a) that he has not theretofore commenced
any action or filed any claim involving the same issues in any court,
tribunal or quasi-judicial agency and, to the best of his knowledge, no such
other action or claim is pending therein; (b) if there is such other pending
action or claim, a complete statement of the present status thereof; and (c)
if he should thereafter learn that the same or similar action or claim has
been filed or is pending, he shall report that fact within five (5) days
therefrom to the court wherein his aforesaid complaint or initiatory pleading
has been filed.
Failure to comply with the foregoing requirements shall not be curable by
mere amendment of the complaint or other initiatory pleading but shall be
cause for the dismissal of the case without prejudice, unless otherwise
provided, upon motion and after hearing. The submission of a false
certification or non-compliance with any of the undertakings therein shall
constitute

indirect

contempt

of

court,

without

prejudice

to

the

corresponding administrative and criminal actions. If the acts of the party


or his counsel clearly constitute willful and deliberate forum shopping, the
same shall be ground for summary dismissal with prejudice and shall
constitute direct contempt, as well as a cause for administrative sanctions.
xxxx.
In a case where the plaintiff is a private corporation, the certification may be
signed, for and on behalf of the said corporation, by a specifically authorized
person, including its retained counsel, who has personal knowledge of the facts
required to be established by the documents. The reason was explained by the
Court in National Steel Corporation v. Court of Appeals, as follows:
Unlike natural persons, corporations may perform physical actions only
through properly delegated individuals; namely, its officers and/or agents.

498

The corporation, such as the petitioner, has no powers except those


expressly conferred on it by the Corporation Code and those that are
implied by or are incidental to its existence. In turn, a corporation exercises
said powers through its board of directors and/or its duly-authorized
officers and agents. Physical acts, like the signing of documents, can be
performed only by natural persons duly-authorized for the purpose by
corporate by-laws or by specific act of the board of directors. "All acts
within the powers of a corporation may be performed by agents of its
selection; and except so far as limitations or restrictions which may be
imposed by special charter, by-law, or statutory provisions, the same
general principles of law which govern the relation of agency for a natural
person govern the officer or agent of a corporation, of whatever status or
rank, in respect to his power to act for the corporation; and agents once
appointed, or members acting in their stead, are subject to the same rules,
liabilities and incapacities as are agents of individuals and private
persons."

For who else knows of the circumstances required in the Certificate but
its own retained counsel. Its regular officers, like its board chairman and
president, may not even know the details required therein.

Indeed, the certificate of non-forum shopping may be incorporated in the


complaint or appended thereto as an integral part of the complaint.xxxx.
In this case, the petitioner, as the defendant in the RTC, assailed the authority of
Atty. Aguinaldo to execute the requisite verification and certificate of non-forum
shopping as the resident agent and counsel of the respondent. It was, thus,
incumbent upon the respondent, as the plaintiff, to allege and establish that Atty.
Aguinaldo had such authority to execute the requisite verification and certification
for and in its behalf. The respondent, however, failed to do so.
xxxx.
While Atty. Aguinaldo is the resident agent of the respondent in the Philippines,
this does not mean that he is authorized to execute the requisite certification
against forum shopping. Under Section 127, in relation to Section 128 of the
499

Corporation Code, the authority of the resident agent of a foreign corporation with
license to do business in the Philippines is to receive, for and in behalf of the
foreign corporation, services and other legal processes in all actions and other
legal proceedings against such corporation, thus:
SEC. 127. Who may be a resident agent. A resident agent may either be
an individual residing in the Philippines or a domestic corporation lawfully
transacting business in the Philippines: Provided, That in the case of an
individual, he must be of good moral character and of sound financial
standing.
SEC. 128. Resident agent; service of process. The Securities and Exchange
Commission shall require as a condition precedent to the issuance of the license
to transact business in the Philippines by any foreign corporation that such
corporation file with the Securities and Exchange Commission a written power of
attorney designating some persons who must be a resident of the Philippines, on
whom any summons and other legal processes may be served in all actions or
other legal proceedings against such corporation, and consenting that service
upon such resident agent shall be admitted and held as valid as if served upon
the duly-authorized officers of the foreign corporation as its home office.
Under the law, Atty. Aguinaldo was not specifically authorized to execute a
certificate of non-forum shopping as required by Section 5, Rule 7 of the Rules of
Court. This is because while a resident agent may be aware of actions filed
against his principal (a foreign corporation doing business in the Philippines),
such resident may not be aware of actions initiated by its principal, whether in the
Philippines against a domestic corporation or private individual, or in the country
where such corporation was organized and registered, against a Philippine
registered corporation or a Filipino citizen.
The respondent knew that its counsel, Atty. Aguinaldo, as its resident agent, was
not specifically authorized to execute the said certification. xxxx.
b) Generally speaking, matters of judicial notice have three material requisites:
(1) the matter must be one of common and general knowledge; (2) it must be
well and authoritatively settled and not doubtful or uncertain; and (3) it must be
known to be within the limits of the jurisdiction of the court. The principal guide in
determining what facts may be assumed to be judicially known is that of
notoriety. Hence, it can be said that judicial notice is limited to facts evidenced by
public records and facts of general notoriety.[15] Moreover, a judicially noticed fact
500

must be one not subject to a reasonable dispute in that it is either: (1) generally
known within the territorial jurisdiction of the trial court; or (2) capable of accurate
and ready determination by resorting to sources whose accuracy cannot
reasonably be questionable.
xxxx. . But a court cannot take judicial notice of any fact which, in part, is
dependent on the existence or non-existence of a fact of which the court has no
constructive knowledge.
In this age of modern technology, the courts may take judicial notice that
business transactions may be made by individuals through teleconferencing.
Teleconferencing is interactive group communication (three or more people in
two or more locations) through an electronic medium. In general terms,
teleconferencing can bring people together under one roof even though they are
separated by hundreds of miles.18 This type of group communication may be
used in a number of ways, and have three basic types: (1) video conferencing television-like communication augmented with sound; (2) computer conferencing
- printed communication through keyboard terminals, and (3) audio-conferencingverbal communication via the telephone with optional capacity for telewriting or
telecopying.
xxxx.
In the Philippines, teleconferencing and videoconferencing of members of board
of directors of private corporations is a reality, in light of Republic Act No. 8792.
The Securities and Exchange Commission issued SEC Memorandum Circular
No. 15, on November 30, 2001, providing the guidelines to be complied with
related to such conferences. Thus, the Court agrees with the RTC that persons in
the Philippines may have a teleconference with a group of persons in South
Korea relating to business transactions or corporate governance.
Even given the possibility that Atty. Aguinaldo and Suk Kyoo Kim participated in
a teleconference along with the respondents Board of Directors, the Court is not
convinced that one was conducted; even if there had been one, the Court is not
inclined to believe that a board resolution was duly passed specifically
authorizing Atty. Aguinaldo to file the complaint and execute the required
certification against forum shopping.
xxxx.

501

The

respondents

allegation

that

its

board

of

directors

conducted

teleconference on June 25, 1999 and approved the said resolution (with Atty.
Aguinaldo in attendance) is incredible, given the additional fact that no such
allegation was made in the complaint. If the resolution had indeed been approved
on June 25, 1999, long before the complaint was filed, the respondent should
have incorporated it in its complaint, or at least appended a copy thereof. The
respondent failed to do so. It was only on January 28, 2000 that the respondent
claimed, for the first time, that there was such a meeting of the Board of Directors
held on June 25, 1999; it even represented to the Court that a copy of its
resolution was with its main office in Korea, only to allege later that no written
copy existed. It was only on March 6, 2000 that the respondent alleged, for the
first time, that the meeting of the Board of Directors where the resolution was
approved was held via teleconference.
xxxx.
The Court is, thus, more inclined to believe that the alleged teleconference
on June 25, 1999 never took place, and that the resolution allegedly
approved by the respondents Board of Directors during the said
teleconference was a mere concoction purposefully foisted on the
RTC, the CA and this Court, to avert the dismissal of its complaint
against the petitioner.

502

Van Zuiden Bros Ltd. vs. GTVL Manufacturing Industries 523 SCRA
233 (2007)
CARPIO, J.:

FACTS:
ZUIDEN is a corporation, incorporated under the laws of Hong Kong engaged in
the importation and exportation of several products, including lace products.
GTVL purchased lace products from ZUIDEN and as per their given instruction to
Zuiden, the products purchased by GTVL should be delivered to a certain Hong
Kong corporation and the products are then considered as sold, upon receipt by
KENZAR of the goods purchased by GTVL. In turn, KENZAR has the obligation
to deliver the products to the Philippines and follow whatever instructions GTVL
had on the matter.

However, conflict arose when GTVL failed and refused to pay the agreed
purchase price for several deliveries ordered by it and delivered by ZUIDEN
hence a petition of complaint for sum of money against respondent which
respondent countered with a Motion to Dismiss on the ground that petitioner has
no legal capacity to sue.
Respondent alleged that petitioner is doing business in the Philippines without
securing the required license. Accordingly, petitioner cannot sue before
Philippine courts

RTC rendered a decision dismissing the complaint; CA affirmed said decision

ISSUE:
Whether petitioner, an unlicensed foreign corporation, has legal capacity to sue
before Philippine courts

HELD:
YES petitioner is a foreign corporation which claims that it is not doing business
in the Philippines. As such, it needs no license to institute a collection suit against
respondent before Philippine courts

Section 133 of the Corporation Code provides:


Doing business without license. No foreign corporation transacting business in
the Philippines without a license, or itssuccessors or assigns, shall be permitted
to maintain or intervene in any action, suit or proceeding in any court
oradministrative agency of the Philippines; but such corporation may be sued or
503

proceeded against before Philippine courts oradministrative tribunals on any valid


cause of action recognized under Philippine laws

The law is clear that an unlicensed foreign corporation doing business in the
Philippines cannot sue before Philippine courts. On the other hand, an
unlicensed foreign corporation not doing business in the Philippines can sue
before Philippine courts

To be considered as doing business in the Philippines for purposes of Section


133 of the Corporation Code,the foreign corporation must actually transact
business in the Philippines, that is, perform specific business transactions within
the Philippine territory on a continuing basis in its own name and for its own
account.

Actual transaction of business within the Philippine territory is an essential


requisite for the Philippines to acquire jurisdiction over a foreign corporation and
thus require the foreign corporation to secure a Philippine business license. If a
foreign corporation does not transact such kind of business in the Philippines,
even if it exports its products to the Philippines, the Philippines has no jurisdiction
to require such foreign corporation to secure a Philippine business license

In the present case, the series of transactions between petitioner and respondent
cannot be classified as "doing business" in the Philippines
An essential condition to be considered as "doing business" in the Philippines is
the actual performance of specific commercial acts within the territory of the
Philippines for the plain reason that the Philippines has no jurisdiction over
commercial acts performed in foreign territories

There is no showing that petitioner performed within the Philippine territory the
specific acts of doing business. Petitioner did not also open an office here in the
Philippines, appoint a representative or distributor, or manage, supervise or
control a local business. While petitioner and respondent entered into a series of
transactions implying a continuity of commercial dealings, the perfection and
consummation of these transactions were done in Hongkong.

504

Aboitiz Shipping Corp. vs. Insurance Co. of North America 561 SCRA
262 (2008)
REYES, R.T., J.:

FACTS:In June 20, 1993, MSAS Cargo International Limited and/or Associated
and/or Subsidiary Companies (MSAS) procured an "all-risk" marine insurance
policy from ICNA UK Limited of London for wooden work tools and workbenches
purchased by consignee Science Teaching Improvement Project (STIP), Ecotech
Center, SudlonLahug, Cebu City.

It was shipped, then in the Stripping Report, checker noted that the crates
were slightly broken or cracked at the bottom. Despite such, it was stille delivered
to Don Bosco Technical High School, Punta Princesa, Cebu City. Perez, Head of
Aboiti received a call from the receiver Mr. Bernhard Willig that the cargo
sustained water damage so he checked the other cargo but they were dry

In a letter dated August 15, 1993, Willig informed Aboitiz that the damage
was caused by water entering through the broken bottom parts of the crate.

ICNA paid the amount of P280,176.92 to consignee and a subrogation


receipt was duly signed by Willig.

ICNA then advised Aboitiz of the receipt

signed in its favor but received no reply so it filed for collection at the RTC.

RTC ruled against ICNA - subrogation Form is self-serving and has no


probative value since Willig was not presented to the witness stand. CA reversed
RTC ruling - right of subrogation accrues simply upon payment by the insurance
company of the insurance claim even assuming that it is an unlicensed foreign
corporation

ISSUE: Whether or not ICNA can claim under the right of subrogation

HELD: YES.
505

Only when that foreign corporation is "transacting" or "doing business" in


the country will a license be necessary before it can institute suits.

It may,

however, bring suits on isolated business transactions, which is not prohibited


under Philippine law

The policy benefits any subsequent assignee, or holder, including the


consignee, who may file claims on behalf of the assured.

This right of subrogation, however, has its limitations. First, both the
insurer and the consignee are bound by the contractual stipulations under the bill
of lading. Second, the insurer can be subrogated only to the rights as the insured
may have against the wrongdoer. If by its own acts after receiving payment from
the insurer, the insured releases the wrongdoer who caused the loss from
liability, the insurer loses its claim against the latter.

Article 366 of the Civil Code provides that within twenty four hours
following the receipt of the merchandise, the claim against the carrier for
damages or average which may be found therein upon opening the packages,
may be made, provided that the indications of the damage or average which give
rise to the claim cannot be ascertained from the outside part of such packages, in
which case the claim shall be admitted only at the time of receipt.
After the periods mentioned have elapsed, or the transportation charges
have been paid, no claim shall be admitted against the carrier with regard to the
condition in which the goods transported were delivered.

The call was made 2 from delivery, a reasonable period considering that
the goods could not have corroded instantly overnight such that it could only
have sustained the damage during transit.

Art. 1735. In all cases other than those mentioned in Nos. 1, 2, 3, 4, and 5 of the
preceding article, if the goods are lost, destroyed or deteriorated, common
506

carriers are presumed to have been at fault or to have acted negligently, unless
they prove that they observed extraordinary diligence as required in Article 1733.
the shipment delivered to the consignee sustained water damage. We agree with
the findings of the CA that petitioner failed to overturn this presumption

507

J.G. Summit Holdings, Inc., vs. Court of Appeals 450 SCRA 169 (2005)
Puno, J
Facts:
On January 27, 1997, the National Investment and Development Corporation
(NIDC), a government corporation, entered into a Joint Venture Agreement (JVA)
with Kawasaki Heavy Industries, Ltd. of Kobe, Japan (KAWASAKI) for the
construction, operation and management of the Subic National Shipyard, Inc.
(SNS) which subsequently became the Philippine Shipyard and Engineering
Corporation (PHILSECO). Under the JVA, the NIDC and KAWASAKI will
contribute P330 million for the capitalization of PHILSECO in the proportion of
60%-40% respectively. One of its salient features is the grant to the parties of
the right of first refusal should either of them decide to sell, assign or transfer
its interest in the joint venture.
On November 25, 1986, NIDC transferred all its rights, title and interest in
PHILSECO to the Philippine National Bank (PNB). Such interests were
subsequently transferred to the National Government pursuant to Administrative
Order No. 14. On December 8, 1986, President Corazon C. Aquino issued
Proclamation No. 50 establishing the Committee on Privatization (COP) and the
Asset Privatization Trust (APT) to take title to, and possession of, conserve,
manage and dispose of non-performing assets of the National Government.
Thereafter, on February 27, 1987, a trust agreement was entered into between
the National Government and the APT wherein the latter was named the trustee
of the National Government's share in PHILSECO. In 1989, as a result of a
quasi-reorganization of PHILSECO to settle its huge obligations to PNB, the
National Government's shareholdings in PHILSECO increased to 97.41%
thereby reducing KAWASAKI's shareholdings to 2.59%.
In the interest of the national economy and the government, the COP and the
APT deemed it best to sell the National Government's share in PHILSECO to
private entities. After a series of negotiations between the APT and KAWASAKI,
they agreed that the latter's right of first refusal under the JVA be "exchanged" for
the right to top by five percent (5%) the highest bid for the said shares. They
further agreed that KAWASAKI would be entitled to name a company in which it
was a stockholder, which could exercise the right to top. On September 7, 1990,
KAWASAKI informed APT that Philyards Holdings, Inc. (PHI)1 would exercise its
right to top.
508

On 29 December 1993, JGSMI informed the APT that it was protesting the offer
of PHI to top its bid on the grounds that: (a) the Kawasaki/PHI consortium
composed of Kawasaki, Philyards, Mitsui, Keppel, SM Group, ICTSI and Insular
Life violated the ASBR because the last four (4) companies were the losing
bidders (for P1.528 billion) thereby circumventing the law and prejudicing the
weak winning bidder; (b) only Kawasaki could exercise the right to top; (c) giving
the same option to top to PHI constituted unwarranted benefit to a third party; (d)
no right of first refusal can be exercised in a public bidding or auction sale, and
(e) the JG Summit Consortium was not estopped from questioning the
proceedings. On 2 February 1994, JGSMI was notified that PHI had fully paid the
balance of the purchase price of the subject bidding. On 7 February 1994, the
APT notified JGSMI that PHI had exercised its option to top the highest bid and
that the COP had approved the same on 6 January 1994. On 24 February 1994,
the APT and PHI executed a Stock Purchase Agreement. Consequently, JGSMI
filed with the Supreme Court a petition for mandamus under GR 114057. On 11
May 1994, said petition was referred to the Court of Appeals. On 18 July 1995,
the Court of Appeals "denied" for lack of merit the petition for mandamus. JGSMI
filed a motion for the reconsideration of said Decision which was denied on 15
March 1996. JGSMI filed the petition for review on certiorari.

Issue:
Whether PHILSECO, as a shipyard, is a public utility and, hence, could be
operated only by a corporation at least 60% of whose capital is owned by Filipino
citizens, in accordance with Article XII, Section 10 of the Constitution.

Ruling:
A shipyard such as PHILSECO being a public utility as provided by law, Section
11 of the Article XII of the Constitution applies. The provision states that "No
franchise, certificate, or any other form of authorization for the operation of a
public utility shall be granted except to citizens of the Philippines or to
corporations or associations organized under the laws of the Philippines at least
sixty per centum of whose capital is owned by such citizens, nor shall such
franchise, certificate, or authorization be exclusive in character or for a longer
period than fifty years. Neither shall any such franchise or right be granted except
509

under the condition that it shall be subject to amendment, alteration, or repeal by


the Congress when the common good so requires. The State shall encourage
equity participation in public utilities by the general public. The participation of
foreign investors in the governing body of any public utility enterprise shall be
limited to their proportionate share in its capital, and all the executive and
managing officers of such corporation or association shall be citizens of the
Philippines." The progenitor of this constitutional provision, Article XIV, Section 5
of the 1973 Constitution, required the same proportion of 60% - 40%
capitalization. The JVA between NIDC and Kawasaki entered into on 27 January
1977 manifests the intention of the parties to abide by the constitutional mandate
on capitalization of public utilities. The joint venture created between NIDC and
Kawasaki falls within the purview of an "association" pursuant to Section 5 of
Article XIV of the 1973 Constitution and Section 11 of Article XII of the 1987
Constitution. Consequently, a joint venture that would engage in the business of
operating a public utility, such as a shipyard, must observe the proportion of
60%-40% Filipino-foreign capitalization. Further, paragraph 1.4 of the JVA
accorded the parties the right of first refusal "under the same terms." This phrase
implies that when either party exercises the right of first refusal under paragraph
1.4, they can only do so to the extent allowed them by paragraphs 1.2 and 1.3 of
the JVA or under the proportion of 60%-40% of the shares of stock. Thus, should
the NIDC opt to sell its shares of stock to a third party, Kawasaki could only
exercise its right of first refusal to the extent that its total shares of stock would
not exceed 40% of the entire shares of stock of SNS or PHILSECO. The NIDC,
on the other hand, may purchase even beyond 60% of the total shares. As a
government corporation and necessarily a 100% Filipino-owned corporation,
there is nothing to prevent its purchase of stocks even beyond 60% of the
capitalization as the Constitution clearly limits only foreign capitalization.
Kawasaki was bound by its contractual obligation under the JVA that limits its
right of first refusal to 40% of the total capitalization of PHILSECO. Thus,
Kawasaki cannot purchase beyond 40% of the capitalization of the joint venture
on account of both constitutional and contractual proscriptions. From the facts on
record, it appears that at the outset, the APT and Kawasaki respected the 60%40% capitalization proportion in PHILSECO. However, APT subsequently
encouraged Kawasaki to participate in the public bidding of the National
Government's shareholdings of 87.67% of the total PHILSECO shares, definitely
over and above the 40% limit of its shareholdings. In so doing, the APT went
beyond the ambit of its authorit
510

Yamane vs. BA Lepanto Condominium Corporation 474 SCRA 258


(2005)
FACTS:
Respondent BA-Lepanto Condominium Corporation is a duly organized
condominium corporation constituted in accordance with the Condominium Act,
which owns and holds title to the common and limited common areas of the BALepanto Condominium situated in Paseo de Roxas, Makati City. The Corporation
is authorized, by its Amendmended By-Laws, to collect regular assessments
from its members for operating expenses, capital expenditures on the common
areas, and other special assessments as provided for in the Master Deed with
Declaration of Restrictions of the Condominium.
On December 15, 1998 the Corporation received a Notice of Assessment
from the City of Makati, signed by City Treasurer, Ms. Luz Yamane. The Notice
of Assessment ordered the Corporation to pay the correct city business taxes,
fees and other charges totaling P 1, 601, 1013.77 for the years 1995 to 1997.
The Corporation, thru counsel responded with a written tax protest, addressed to
the City Treasurer, alleging that Makati Revenue Code and the Local
Government Code do not contain any provision on which the Assessment could
be based. The Corporation argue that under both the Makati Code and the Local
Government Code business is defined as trade or commercial activity regularly
engaged as a means of live hood or with a view to profit.
The Condominium Corporation was organized not for profit, but hold title
over the common areas of the Condominium for the unit owners, and to hold the
parcel of land which the Condominium was located.
The protest was rejected by the City Treasurer, and insisted that the
collection of dues from the unit owners was effected primarily to sustain and
maintain the expenses to the common areas, with the end view of getting full
appreciative living values, and to command better marketable prices for those
occupants who might in the future sell their respective units. Hence, the
corporation activity is a profit venture making.
The corporation filed an appeal with the RTC; the latter dismissed the
appeal for lack of merit. Thereby the Corporation filed a Petition for Review
Under Rule 42 of the Rules of Civil Procedures with the Court of Appeals. The
Court of appeals reversed the Decision of the RTC in favor of the Corporation.
Upon denial of her Motion for Reconsideration, the City Treasurer elevated the
present Petition for Review under Rule 45 of the Rules of Court.

ISSUE
Whether the City of Makati may collect business taxes on respondent
condominium corporation.

511

RULING

The Supreme Court held that Condominium Corporation is generally


exempt from local business taxation under the Local Government Code,
irrespective of any ordinance that seeks to declare otherwise.
The City Treasurer failed to cite specific statutory basis of the tax imposed
on the Condominium Corporation. With the careful examination of the Court with
the applicable laws and statutes which may give light to the issue in question, the
Court could not find any law, statute, or ordinance in the Local Government
Code, the Makati Revenue Code, as well as in the local City Ordinance which
subject the condominium Corporation to pay business tax.
Sec.143 of the Local Government Code specifically enumerates several
types of businesses on which municipalities and cities may impose taxes. Local
Sanggunian is also authorized to impose taxes on any business not otherwise
specified in Sec.143 of the Local Government Code. Chapter III of the Makati
Revenue Code is also specific as to particular business which is covered by
business taxes. In all these sources, none of them specifically provides
imposition of business tax to Condominium Corporation.
The creation of Condominium Corporation is sanctioned by R.A 4726
otherwise known as the Condominium Act. The Condominium Act imposes
several limitations on the Condominium Corporation which prove crucial to the
disposition of the case. A condominium corporation is precluded under the Act, to
engage in corporate activities other than the holding of the common areas, the
administration of the condominium project and other acts necessary, incidental or
convenient to the accomplishment of such purposed. Even though the
corporation is empowered to collect dues from the unit owners, these amounts
are not intended for the incurrence of profit by the Corporation or its members,
but to shoulder the multitude of necessary expenses that may arise from the
maintenance of the Condominium Project. Hence petition is denied.

512

Philex Mining Corporation vs. Reyes, 118 SCRA 602 (1982)


FACTS:
Private respondent Huenefeld is a stockholder of petitioner Philex Mining
Corporation (Philex). He originally owned 800,000 shares of stock. On February
15, 1979, Philex declared a 10% stock dividend. Stock Certificate No. 190579 for
80,000 shares was issued by Philex in favor of Huenefeld. Then, Philex sent the
stock certificate to Huenefeld through its transfer agent First Asian, Stock
Transfer, Inc. Huenefeld claims that he never received the stock certificate.
First Asian wrote Huenefeld informing him that the stock certificate has
been delivered to his address, and that if the certificate could not be located, that
Huenefeld execute an Affidavit of Loss with notice of loss to be published once a
week for three consecutive weeks in a newspaper of general circulation in
accordance with RA No. 201 (now Section 73, Corporation Code).
Huenefeld replied that RA 201 is not applicable because the stock
certificate was not lost in the possession of the stockholder, assuming it was, the
expenses of publication and premiums for the bond should be at Philexs
expense.
Huenefeld commenced a suit for specific performance with damages
against Philex to compel the issuance of a replacement for the stock certificate.
Philex filed a motion to dismiss on the ground that the CFI has no jurisdiction
over the case, the issue being one of intra-corporate relationship between a
stockholder and a corporation which under PD No. 902-A, falls within the original
and exclusive jurisdiction of the SEC. Huenefeld opposed, claiming that the
refusal of Philex to issue a replacement certificate resulted in actual damages to
him, thus it is civil or tortious in nature.

ISSUE

Whether the SEC has jurisdiction over the present controversy, and whether it is
an intra-corporate issue

RULING
Section 5 of PD No. 902-A provides that the SEC has original and
exclusive jurisdiction to hear and decide cases involving intra-corporate or
partnership relations. Evident from the foregoing is that an intra-corporate
controversy is one which arises between a stockholder and the corporation.
There is no distinction, qualification, nor any exemption whatsoever. The
provision is broad and covers all kinds of controversies between stockholders
and corporations. The issue of whether or not a corporation is bound to replace a
stockholder's lost certificate of stock is a matter purely between a stockholder
513

and the corporation. It is a typical intra-corporate dispute. The question of


damages raised is merely incidental to that main issue. Hence, SEC has proper
jurisdiction over the case.

514

Union Glass & Container Corporation vs. Securities & Exchange


Commission, 126 SCRA 21 (1983). See also Magalad vs. Premier
Financing Corporation, 209 SCRA 260 (1992)
Ponente: Justice Venicio T. Escolin

Facts:

Carolina Hofilea is a stockholder at Pioneer Glass Manufacturing


Corporation which obtained various loan accommodations from the Development
Bank of the Philippines (DBP). As security for loan accommodations, Pioneer
Glass mortgaged and/or assigned its assets to DBP. When Pioneer Glass
suffered serious liquidity problems such that it could no longer meet its financial
obligations with DBP, it entered into a dacion en pago agreement with the latter,
whereby all its assets mortgaged to DBP were ceded to the latter in full
satisfaction of the corporation's obligations. Hofilea filed a complaint before the
Securities and Exchange Commission (SEC) against DBP, Union Glass and
Pioneer Glass. The respondents moved for the dismissal of the case on the
ground that the SEC had no jurisdiction over the subject matter or nature of the
suit. The SEC Hearing Officer granted the motion to dismiss for lack of
jurisdiction but he later reversed his original order.

Issue:

Is it the regular court or the SEC that has jurisdiction over the case?

Ruling:

The regular court has jurisdiction over the case. The jurisdiction of the SEC
is delineated by Sec. 5 of PD No. 902-A which gives SEC original and exclusive
jurisdiction to hear and decide cases involving controversies arising out of intracorporate or partnership relations. This grant of jurisdiction must be viewed in the
light of Sec. 3 of PD No. 902-A which confers upon SEC absolute jurisdiction,
supervision, and control over all corporations, partnerships or associations, who
are grantees of primary franchise and/or license or permit issued by the
government to operate in the Philippines.

515

The law explicitly specified and delimited its jurisdiction to matters


intrinsically connected with the regulation of corporations, partnerships and
associations and those dealing with the internal affairs of such corporations,
partnerships or associations. Otherwise stated, in order that the SEC can take
cognizance of a case, the controversy must pertain to any of the following
relationships: (a) between the corporation, partnership or association and the
public; (b) between the corporation, partnership or association and its
stockholders, partners, members, or officers; (c) between the corporation,
partnership or association and the state in so far as its franchise, permit or
license to operate is concerned; and (d) among the stockholders, partners or
associates themselves.

The fact that the controversy at bar involves the rights of petitioner Union
Glass who has no intra-corporate relation either with complainant or DBP, places
the suit beyond the jurisdiction of SEC. The case should be tried and decided by
the court of general jurisdiction, the Regional Trial Court. This view is in accord
with the rudimentary principle that administrative agencies, like the SEC, are
tribunals of limited jurisdiction and, as such, could wield only such powers as are
specifically granted to them by their enabling statutes.

516

Philippine School of Business Administration vs. Leano,


127 SCRA 778 (1984)
FACTS:
A stabbing incident caused the death of Carlitos Bautista, a third year student,
while on the second-floor premises of the Philippine School of Business
Administration (PSBA). The parents of the deceased filed a suit for damages
against the PSBA and its corporate officers for their alleged negligence,
recklessness and lack of security precautions, means and methods before,
during and after the attack on the victim. The assailants were not members of the
schools academic community but were elements from outside the school. The
suit impleaded the PSBA and the following school authorities: Juan D. Lim
(President),
Benjamin
P.
Paulino
(Vice-President),
Antonio
M.
Magtalas(Treasurer/Cashier), Col. Pedro Sacro (Chief of Security) and a Lt. M.
Soriano(Assistant Chief of Security).Defendants sought to have the suit
dismissed, alleging that since they are presumably sued under Article 2180 of the
Civil Code, the complaint states no cause of action against them, as
jurisprudence on the subject is to the effect that academic institutions, such as
the PSBA, are beyond the ambit of the rule in the afore-stated article. The trial
court denied the motion to dismiss; the Court of Appeals affirmed. The CA
primarily anchored its decision on the law of quasi-delicts, as enunciated in
Articles 2176 and 2180 of the Civil Code. According to the CA, the law holds the
teachers and heads of the school staff liable unless they relieve themselves of
such liability pursuant to the last paragraph of Article2180 by 'proving that they
observed all the diligence to prevent damage.

ISSUE

Could PSBA and its officers be held liable for Arts? 2176 and 2180? Could PSBA
and its officers be held liable for Arts. 2176 and 2180?
RULING

No. But they could be held liable for breach of contractual obligation and for tort,
in conjunction with Art. 21 of the Civil Code, even if there is a contractual
obligation.Ratio:1.Article 2180, in conjunction with Article 2176 of the Civil Code,
establishes the rule in in loco parentis. This Court discussed this doctrine in the
cases of Exconde, Mendoza, Palisoc and, more recently, in Amadora vs. Court of
Appeals. 6 In all such cases, it had been stressed that the law (Article 2180)
plainly provides that the damage should have been caused or inflicted by pupils
or students of the educational institution sought to be held liable for the acts of its
pupils or students while in its custody. However, this material situation does not
exist in the present case for, as earlier indicated, the assailants of Carlitos were
not students of the PSBA, for whose acts the school could be made liable.
517

However, does the appellate court's failure to consider such material facts mean
the exculpation of the petitioners from liability? It does not necessarily follow.
Because the circumstances of the present case evince a contractual relation
between the PSBA and Carlitos Bautista, the rules on quasi-delict do not really
govern. A perusal of Article 2176 shows that obligations arising from quasi-delicts
or tort, also known as extra-contractual obligations, arise only between parties
not otherwise bound by contract, whether express or implied. When an academic
institution accepts students for enrolment, there is established a contract
between them, resulting in bilateral obligations which both parties are bound to
comply with. For its part, the school undertakes to provide the student with an
education that would presumably suffice to equip him with the necessary tools
and skills to pursue higher education or a profession. On the other hand, the
student covenants to abide by the school's academic requirements and observe
its rules and regulations.

518

DMRC Enterprises vs. Este Del Sol Mountain Reserve, Inc.,


132 SCRA 293 (1984)

FACTS: Petitioner is a partnership engaged in the business of general


construction and leasing heavy equipment and other allied transactions. On May
12, 1978, the petitioner made an offer in writing to the private respondent for the
lease to it of three (3) units of heavy equipment. As further conditions of the
agreement, respondent was to advance the sum of P5,000.00 per unit to be
deducted from the first collection to be made by the petitioner; that the payments
due to the petitioner shall be made every 15th and 30th of each calendar month
and that an amount equivalent to 30% of the collection shall be invested in the
purchase of shares of stock of the defendant corporation at the market value of
P37,000.00 per share. The offer was accepted.
As a result of the agreement between petitioner and private respondent, the
former proceeded to perform what was incumbent upon it. However, despite
repeated demands made by the petitioner to the respondent for the payment of
outstanding obligations, the respondent refused to comply with its obligations to
the petitioner. Thus, on March 25, 1980, the petitioner filed a complaint before
the court of First Instance of Rizal, Quezon City, Branch III, against the private
respondent Este del Sol Mountain Resort. The respondent interposed a motion to
dismiss on the sole ground that the respondent court has no jurisdiction over the
nature of the action or suit. The motion was dismissed. Hence, this petition.
ISSUE: Whether the court has jurisdiction over the case.
RULING: The Supreme Court agrees with the contention of the petitioner that e
complaint is simply an action for the collection of money and delivery of personal
property representing unpaid obligations within the competence of the regular
courts. urisdiction of a court is conferred by the Constitution and by the laws in
force at the time of the commencement of the action. (People v. Mariano, 71
SCRA 600; Villamayor v. Luciano, 88 SCRA 156). however, whether or not a
court has jurisdiction over the subject matter of a case is determined from the
allegations of the complaint Magay v. Estandan 69 SCRA 456; Republic v.
Sebastian, 72 SCRA 222). Therefore, to resolve the issue raised to us, an
interpretation and application of the law on jurisdiction, must be made vis-a-vis
the avernments of the petitioner's complaint. It must be stressed that the plaintiffpetitioner submitted himself to the jurisdiction of the lower court as creditor and
the respondent did so as debtor. The fact that the case involves shares of stock
to be used as payment for lease rentals does not convert it into an intracorporate controversy. In fact, the greater of the petitioner's claim is in terms of
cash or money. or pass upon a money claim under a lease contract would be
beyond the competence of the Securities and Exchange Commission and to
separate the claim for money from the claim for shares of stock would be splitting
a single cause of action resulting in a multiplicity of suits. Respondent cites the
case of PAIC Securities v. Securities and Exchange Commission and Pedro
Ong (G.R. No. 53981/ June 11, 1980) claiming that with an almost Identical
factual background to the case at bar, the jurisdiction of the Securities and
Exchange Commission was upheld. The respondent overlooks a singular fact
which distinguishes it from the present controversy. The case of Pedro Ong was
not merely a simple money claim and action for specific performance arising from
a contractual obligation. It emanated from stock transactions between a
stockholder PAIC client client (Pedro Ong). Definitely, such an action arising from
the stock operations of a stockbroker comes within the jurisdiction and control of
519

the Commission. Said case may not be invoked to support the respondent's
contention.
WHEREFORE, in view of the foregoing, the petition is hereby GRANTED and the
order of the respondent Presiding Judge of the Court of First Instance of Rizal,
Quezon City, Branch LII dated April 23, 1981 in Civil Case No. Q-29585
REVERSED and SET ASIDE.

520

Development Bank of the Philippines vs. Ilustre, Jr.,


138 SCRA 11 (1985)
ESCOLIN, J.:

FACTS:
This is a petition for certiorari with preliminary injunction, filed by the
Development Bank of the Philippines (DBP), which seeks to annul and set aside
the restraining order issued by the respondent Judge Joaquin Ilustre, Jr. in Civil
Case No. 6599 of the then Court of First Instance of Albay, entitled "Isarog Pulp
and Paper Co., et al. v. DBP, et al." Contending that said civil case falls within the
exclusive jurisdiction of the Security and Exchange Commission.
ISAROG was originally a family corporation owned or controlled by respondents
Bernardo Silverio, Lourdes Silverio, Miguel Angelo Silverio, Elizabeth Ann
Silverio, Jose Bernardo Silverio and Roberto Noel Silverio.
In1973, ISAROG entered into a contract with the French firm Creusot Loire
Enterprise (CLE) for the construction of an abaca pulp and paper mill. To finance
the purchase of the plant, ISAROG had applied for financial assistance with the
DBP.
ISAROG and DBP, entered into a memoramdum of agremment whereby the
latter accets to conduct a joint re-study of the project, extend financial and other
assistance to ISAROG as may be determined in the joint re-study and to convert
portion of ISAROG's liability into equity upon agreement of the parties.
Soon after the completion of the plant in 1975, a controversy arose between
ISAROG and CLE As a result of this dispute CLE abandoned the project.
As likewise stipulated in said agreement, ISAROG's past due obligation of P30
million was converted into preferred shares of DBP in ISAROG while ISAROG's
arrearages in various industrial and agricultural loans, with total of P45 million,
were converted into common shares of DBP. These transactions resulted in DBP
obtaining approximately 91% of the total outstanding shares of ISAROG, thereby
enabling it to elect a substantial majority in the board of directors
In 1981, the Silverios sent DBP a letter, charging the latter with having violated
the memorandum agreement and giving notice that they were declaring the
agreement rescinded. The Silverios further demanded the return of the
ownership, management and control of ISAROG.
Meanwhile, notices were sent out for the holding of the annual meeting of
stockholders of ISAROG. At said meeting, a new set of board of directors was
elected by the majority stockholders representing 91% of the equity of the
corporation, on which the Silverios were no longer art of the executive officers.
The Silverios instituted Civil Case against DBP and PHINMA in the Court of First
Instance of Albay, praying inter alia that the memorandum agreement of March
18, 1977 be declared rescinded, that they be restored to the positions they were
occupying before the signing of said agreement, and that defendants DBP and
521

PHINMA be ordered to pay actual, moral and exemplary damages as well as


attorney's fees. They also sought the issuance of a writ of preliminary injunction
or temporary restraining order to enjoin and prohibit defendants, their officers,
attorneys, agents and all persons acting on their behalf from ousting the Silverio
family from ISAROG
Respondent Judge Ilustre rendered a decision in favor of the Silverios and issued
a restraining order.
DBP filed a motion to dismiss the complaint on the ground, among others, that
the case falls within the exclusive jurisdiction of the SEC. The motion to lift was
denied for lack of merit. DBP moved for reconsideration of the order, which
respondent judge denied.
Hence, this recourse.
ISSUE

Whether the Securities and Exchange Commission has the jurisdiction over the
case as alleged by DBP, thereby challenging the competence and jurisdiction of
the CFI.

RULING
The petition is granted. CFI is dismissed for lack of jurisdiction.
Under Section 5 of PD No. 902-A, the Securities and Exchange Commission has
original and exclusive jurisdiction to hear and decide cases involving
controversies arising out of intra-corporate or partnership relations, between and
among stockholders, members or associates, between any or all of them and the
corporation, partnership, or association of which they are stockholders, members
or associates and in controversies in the election or appointments of directors,
trustees, officers or managers of such corporations, partnerships or associations.
It is evident that there exists an intra-corporate relationship between the parties:
both the Silverios and the DBP are stockholders of ISAROG, while PHINMA acts
as manager thereof. Obviously, therefore, jurisdiction over the case at bar
pertains to the SEC and petitioner DBP is correct in assailing the competence of
the CFI.

522

Banez vs. Dimensional Construction Trade and Development


Corporation, 140 SCRA 249 (1985)
ALAMPAY, J.:
Facts: Petitioners filed with the RTC of Urdaneta a complaint praying that the
defendant be ordered to pay them the sums of money which were already due to
them under the various promissory notes issued by said defendant. Defendant
was served copy of the summons and complaint on November 18, 1980, but it
failed to file any responsive pleading. Thereafter, petitioners moved to have
private respondents declared in default. The trial court declared private
respondent in default and set the date for the reception of plaintiff's evidence.
Before the court renders its judgment, defendant filed an omnibus motion to: (a)
annul all proceedings taken in the court below; (b) lift and dissolve all
attachments, levies or executions and (c) to dismiss the case. It was defendant's
contention that it is the Securities and Exchange Commission and not the Court
of First Instance that has exclusive jurisdiction over the subject matter of the case
because of Section 5 (a) of Presidential Decree No. 902-A, which vests in the
Securities and Exchange Commission original and exclusive jurisdiction to hear
and decide cases involving the devices or schemes employed by or any acts of
the Board of Directors, business associates, its officers or partners, amounting to
fraud and misrepresentation which may be detrimental to the interest of the
public and/or of the stockholders, partners, members of associations or
organizations registered with the Commission.
Issue: That the Securities and Exchange Commission, and not the Court of First
Instance, has the jurisdiction to decide on the case.
Ruling: The petition is impressed with merit as there appears to be a grave
abuse of discretion committed by the court below in dismissing petitioners'
complaint.
The recitals of the complaint in Civil Case No. 3569 disclose that plaintiff's cause
of action is merely for the collection of the various sums of money that have
already become payable to petitioners due to the promissory notes executed by
defendant corporation which have already matured. There is neither allegation
nor any mention whatsoever in plaintiff's complaint that a device or scheme was
resorted to by private respondent corporation amounting to fraud and
misrepresentation. It is, therefore, difficult to consider that petitioners' case falls
within the jurisdiction of the Securities and Exchange Commission pursuant to
523

PD 902-A. Paradoxically, despite the absence of imputation of fraud and


misrepresentation being alleged by plaintiff, it is the defendant corporation itself
which insinuates the existence of fraud and misrepresentation on its part.
Evidently, the defendant's challenge to the jurisdiction of the court below is
principally intended to negate the effects of the order of default earlier issued
against it as well as the evidence already adduced by petitioners in the court
below. The tactical step resorted by the private respondent in the trial court
appears to be its deliberate attempt to unduly delay the satisfaction of the reliefs
claimed for by petitioners and to avoid the effects of its failure to file any answer
to the complaint and controvert the evidence already adduced against it.

524

Dy vs. The National Labor Relations Commission,


145 SCRA 211 (1986)
FACTS:
Petitioners assail in this Court the resolution of the National Labor Relations
Commission (NLRC) dismissing their appeal from the decision of the Executive
Labor Arbiter 1 in Cebu City which found private respondent to have been illegally
dismissed by them.
Said private respondent, Carlito H. Vailoces, was the manager of the Rural Bank
of Ayungon (Negros Oriental), a banking institution duly organized under
Philippine laws. He was also a director and stockholder of the bank.
On June 4, 1983, a special stockholders' meeting was called for the purpose of
electing the members of the bank's Board of Directors. Immediately after the
election the new Board proceeded to elect the bank's executive officers.
Pursuant to Article IV of the bank's by-laws, 2 providing for the election by the
entire membership of the Board of the executive officers of the bank, i.e., the
president, vice-president, secretary, cashier and bank manager, in that board
meeting of June 4, 1983, petitioners Lorenzo Dy, William Ibero and Ricardo
Garcia were elected president, vice-president and corporate secretary,
respectively. Vailoces was not re-elected as bank manager, 3 Because of this
development, the Board, on July 2, 1983, passed Resolution No. 5, series of
1983, relieving him as bank manager.
On August 3, 1983, Vailoces filed a complaint for illegal dismissal and damages
with the Ministry of Labor and Employment against Lorenzo Dy and Zosimo Dy,
Sr. The complaint was amended on September 22, 1983 to include additional
respondents-William Ibero, Ricardo Garcia and the Rural Bank of Ayungon, and
additional causes of action for underpayment of salary and non-payment of living
allowance.
In his complaint and position paper, Vailoces asserted that Lorenzo Dy, after
obtaining control of the majority stock of the bank by buying the shares of
Marcelino Maximo, called an illegal stockholders' meeting and elected a Board of
Directors controlled by him; that after its illegal constitution, said Board convened
on July 2, 1983 and passed a resolution dismissing him as manager, without
giving him the opportunity to be heard first; that his dismissal was motivated by
Lorenzo Dy's desire to take over the management and control of the bank, not to
mention the fact that he (Dy) harbored ill feelings against Vailoces on account of
the latter's filing of a complaint for violation of the corporation code against him
and another complaint for compulsory recognition of natural child with damages
against Zosimo Dy, Sr. 4
In their answer, Lorenzo Dy, et al. denied the charge of illegal dismissal. They
pointed out that Vailoces' position was an elective one, and he was not re-elected
as bank manager because of the Board's loss of confidence in him brought about
by his absenteeism and negligence in the performance of his duties; and that the
525

Board's action was taken to protect the interest of the bank and was "designed
as an internal control measure to secure the check and balance of authority
within the organization." 5
Lorenzo Dy, et al. appealed to the NLRC, assigning error to the decision of the
Labor Arbiter on various grounds, among them: that Vailoces was not entitled to
notice of the Board meeting of July 2, 1983 which decreed his relief because he
was no longer a member of the Board on said date; that he nonetheless had the
opportunity to refute the charges against him and seek a formal investigation
because he received a copy of the minutes of said meeting while he was still the
bank manager (his removal was to take effect only on August 15, 1983), instead
of which he simply abandoned the work he was supposed to perform up to the
effective date of his relief; and that the matter of his relief was within the
adjudicatory powers of the Securities and Exchange Commission. 7
The NLRC, however bypassed the issues raised and simply dismissed the
appeal for having been filed late.
ISSUES

Whether intracorporate disputes fall within the jurisdiction of SEC and whether
the petitioner is estopped for not raising the issue on jurisdictions timely.

RULINGS
Presidential Decree No. 902-A vests in the Securities and Exchange Commission
for its original and exclusive jurisdiction to hear and decide cases involving:
a) Devices or schemes employed by or any acts, of the board of directors,
business associates, its officers or partners, amounting to fraud and
misrepresentation) which may be detrimental to the interest of the public and/or
of the stockholders, partners, members of associations or organizations
registered with the Commission.
b) Controversies arising out of intracorporate or partnership relations, between
and among stockholders, members or associates; between any of all of them and
the corporation, partnership or association of which they are stockholders,
members or associates, respectively; and between such corporation, partnership
or association and the state insofar as it concerns their individual franchise or
right to exist as such entity;
c) Controversies in the election or appointments of directors, trustees, officers or
managers of such corporations, partnership or associations.
Although the petitioners failed to raise the issue of jurisdiction in their petition
before this Court. But this, too, is no hindrance to the Court's considering said
issue.
526

The failure of the appellees to invoke anew the aforementioned solid ground of
want of jurisdiction of the lower court in this appeal should not prevent this
Tribunal to take up that issue as the lack of jurisdiction of the lower court is
apparent upon the face of the record and it is fundamental that a court of justice
could only validly act upon a cause of action or subject matter of a case over
which it has jurisdiction and said jurisdiction is one conferred only by law; and
cannot be acquired through, or waived by, any act or omission of the parties
(Lagman vs. CA, 44 SCRA 234 [1972]); hence may be considered by this court
motu proprio (Gov't. vs. American Surety Co., 11 Phil. 203 [1908])...14
These considerations make inevitable the conclusion that the judgment of the
Labor Arbiter and the resolution of the NLRC are void for lack of cause of
jurisdiction, and this Court must set matters aright in the exercise of its judicial
power. It is of no moment that Vailoces, in his amended complaint, seeks other
relief which would seemingly fan under the jurisdiction of the Labor Arbiter,
because a closer look at these-underpayment of salary and non-payment of
living allowance-shows that they are actually part of the perquisites of his elective
position, hence, intimately linked with his relations with the corporation. The
question of remuneration, involving as it does, a person who is not a mere
employee but a stockholder and officer, an integral part, it might be said, of the
corporation, is not a simple labor problem but a matter that comes within the area
of corporate affairs and management, and is in fact a corporate controversy in
contemplation of the Corporation Code.
WHEREFORE, the questioned decision of the Labor Arbiter and the Resolution
of the NLRC dismissing petitioners' appeal from said decision are hereby set
aside because rendered without jurisdiction. The amended complaint for illegal
dismissal, etc., basis of said decision and Resolution, is ordered dismissed,
without prejudice to private respondent's seeking recourse in the appropriate
forum.

527

Abejo vs. Dela Cruz,


149 SCRA 654 (1987)
Facts:
These two cases, jointly heard, are jointly herein decided. They involve the
question of who, between the Regional Trial Court and the Securities and
Exchange Commission (SEC), has original and exclusive jurisdiction over the
dispute between the principal stockholders of the corporation Pocket Bell
Philippines, Inc. (Pocket Bell), a "tone and voice paging corporation," namely, the
spouses Jose Abejo and Aurora Abejo (hereinafter referred to as the Abejos) and
the purchaser, Telectronic Systems, Inc. (hereinafter referred to as Telectronics)
of their 133,000 minority shareholdings (for P5 million) and of 63,000 shares
registered in the name of Virginia Braga and covered by five stock certificates
endorsed in blank by her (for P1,674,450.00), and the spouses Agapito Braga
and Virginia Braga (hereinafter referred to as the Bragas), erstwhile majority
stockholders. With the said purchases, Telectronics would become the majority
stockholder, holding 56% of the outstanding stock and voting power of the
corporation Pocket Bell. With the said purchases in 1982, Telectronics
requested the corporate secretary of the corporation, Norberto Braga, to register
and transfer to its name, and those of its nominees the total 196,000 Pocket Bell
shares in the corporation's transfer book, cancel the surrendered certificates of
stock and issue the corresponding new certificates of stock in its name and those
of its nominees. Norberto Braga, the corporate secretary and son of the Bragas,
refused to register the aforesaid transfer of shares in t e corporate oo s, asserting
that the Bragas claim preemptive rights over the 133,000 Abejo shares and that
Virginia Braga never transferred her 63,000 shares to Telectronics but had lost
the five stock certificates representing those shares. This triggered off the series
of intertwined actions between the protagonists, all centered on the question of
jurisdiction over the dispute, which were to culminate in the filing of the two cases
at bar. The Bragas assert that the regular civil court has original and exclusive
jurisdiction as against the Securities and Exchange Commission, while the
Abejos claim the contrary. The dispute between the contending parties for
control of the corporation manifestly fans within the primary and exclusive
jurisdiction of the SEC in whom the law has reserved such jurisdiction as an
administrative agency of special competence to deal promptly and expeditiously
therewith.
Issues:
Whether the Securities and Exchange Commision has the jurisdiction to decide
on the case presented.
Ruling:
The Corporation Code (B.P. No. 178) enacted on May 1, 1980 specifically vests
the SEC with the Rule-making power in the discharge of its task of implementing
the provisions of the Code and particularly charges it with the duty of preventing
fraud and abuses on the part of controlling stockholders, directors and officers,
as follows: SEC. 143. Rule-making power of the Securities and Exchange
528

Commission. The Securities and Exchange Commission shall have the power
and authority to implement the provisions of this Code, and to promulgate rules
and regulations reasonably necessary to enable it to perform its duties
hereunder, particularly in the prevention of fraud and abuses on the part of the
controlling stockholders, members, directors, trustees or officers.. The dispute
between the contending parties for control of the corporation manifestly fans
within the primary and exclusive jurisdiction of the SEC in whom the law has
reserved such jurisdiction as an administrative agency of special competence to
deal promptly and expeditiously therewith. On the decision of Supreme Court it
directed the SEC through its Hearing Committee to proceed immediately with the
implementation of its receivership or management committee.

529

Reyes vs. RTC of Makati


561 SCRA 593 (2008)
Ponente: Justice Brion
Facts:
Petitioner and private respondent were siblings together with two others, namely
Pedro and Anastacia, in a family business established as Zenith Insurance
Corporation (Zenith), from which they owned shares of stocks. The Pedro and
Anastacia subsequently died. The former had his estate judicially partitioned
among his heirs, but the latter had not made the same in her shareholding in
Zenith. Zenith and Rodrigo filed a complaint with the Securities and Exchange
Commission (SEC) against petitioner (1) a derivative suit to obtain accounting of
funds and assets of Zenith, and (2) to determine the shares of stock of deceased
Pedro and Anastacia that were arbitrarily and fraudulently appropriated [by
Oscar, and were unaccounted for]. In his answer with counterclaim, petitioner
denied the illegality of the acquisition of shares of Anastacia and questioned the
jurisdiction of SEC to entertain the complaint because it pertains to settlement of
[Anastacias] estate. The case was transferred to. Petitioner filed Motion to
Declare Complaint as Nuisance or Harassment Suit and must be dismissed. RTC
denied the motion. The motion was elevated to the Court of Appeals by way of
petition for certiorari, prohibition and mandamus, but was again denied.
Issue/s:
(1) Whether or not Rodrigo may be considered a stockholder of Zenith with
respect to the shareholdings originally belonging to Anastacia.
(2) Whether or not there is an intra-corporate relationship between the parties
that would characterize the case as an intra-corporate dispute?

Ruling:
The Supreme Court held in the negative. It ruled that Rodrigo must hurdle
two obstacles before he can be considered a stockholder of Zenith with respect
to the shareholdings originally belonging to Anastacia. First, he must prove that
there are shareholdings that will be left to him and his co-heirs, and this can be
determined only in a settlement of the decedents estate. No such proceeding
530

has been commenced to date. Second, he must register the transfer of the
shares allotted to him to make it binding against the corporation. He cannot
demand that this be done unless and until he has established his specific
allotment (and prima facie ownership) of the shares. Without the settlement of
Anastacias estate, there can be no definite partition and distribution of the estate
to the heirs. Without the partition and distribution, there can be no registration of
the transfer. And without the registration, we cannot consider the transferee-heir
a stockholder who may invoke the existence of an intra-corporate relationship as
premise for an intra-corporate controversy within the jurisdiction of a special
commercial court. The subject shares of stock (i.e., Anastacias shares) are
concerned Rodrigo cannot be considered a stockholder of Zenith.
As to the second issue, the Supreme Court held that courts cannot declare
that an intra-corporate relationship exists that would serve as basis to bring this
case within the special commercial courts jurisdiction under Section 5(b) of PD
902-A, as amended because Rodrigos complaint failed the relationship test
above.

531

Boman Environmental Corp. vs. Court of Appeals,


167 SCRA 540 (1988)
GRIO-AQUINO, J.:

Facts: Respondent Nilcar Y. Fajilan offered in writing to resign as President and


Member of the Board of Directors of petitioner, Boman Environmental
Development Corporation (BEDECO), and to sell to the company all his shares,
rights, and interests therein for P300,000 plus the transfer to him of the
companys Isuzu pick-up truck which he had been using. At a meeting of the
Board of Directors of BEDECO on June 14, 1984, Fajilans resignation as
president was accepted and new officers were elected, the Board promising to
pay for them on a staggered basis from July 15, 1984 to December 15, 1984.
Boman paid the first two P50,000.00 installments but defaulted in paying the
remaining P200,000.00. Fajilan then sued Boman in the RTC of Makati.

Issue: That a suit brought by a withdrawing stockholder against the corporation


to enforce payment of the balance due on the consideration for the surrender of
his shares of stock and interests in the corporation, involves an intra-corporate
dispute. The resolution of that issue will determine whether the Securities and
Exchange Commission (SEC) or a regular court has jurisdiction over the action

Ruling: This case involves an intra-corporate controversy because the parties


are a stockholder and the corporation. As correctly observed by the trial court,
the perfection of the agreement to sell Fajilans participation and interests in
BEDECO and the execution of the promissory note for payment of the price of
the sale did not remove the dispute from the coverage of Section 5(b) of P.D. No.
902, as amended, for both the said agreement (Annex C) and the promissory
note (Annex D) arose from intra-corporate relations. Indeed, all the signatories of
both documents were stockholders of the corporation at the time of signing the
same. It was an intra-corporate transaction; hence, this suit is an intra-corporate
controversy. Fajilans suit against the corporation to enforce the latters
promissory note or compel the corporation to pay for his shareholdings is
cognizable by the SEC alone which shall determine whether such payment will
not constitute a distribution of corporate assets to a stockholder in preference
over creditors of the corporation. The SEC has exclusive supervision, control and
regulatory jurisdiction to investigate whether the corporation has unrestricted
retained earnings to cover the payment for the shares, and whether the purchase
is for a legitimate corporate purpose as provided in Sections 41 and 122 of the
Corporation Code.
532

Andaya vs. Abadia, 228 SCRA 705 (1993)


Bellosillo, J.

FACTS OF THE CASE


Andaya was the President and general manager of the Armed Forces and
Police Savings and Loans Association, Inc that. Petitioner alleged that the
respondents, as directors of the AFPSLAI, acted in concerts and pursuant to an
illegal and nefarious scheme to oust the petitioner in his then position in
AFPSLAI, with grave abuse of authority and in gross and deliberate violation of
the norms of human relations and of petitioners right to due process. This led the
petitioner to file a civil case against the respondent Abadia. Respondents, as
charged, convened a meeting of the AFPSLAI Board of Directors and illegally
reorganized the management of AFPSLAI by ousting and removing the
petitioner, without just and lawful cause. Petitioner prayed for the rewarding of
moral and exemplary damages because of this incident. The petitioner also
applied for a temporary restraining order to prevent respondents Punzalan and
Tangan to assume the petitioners office in AFPSLAI.
Respondents, in their counter, contend that the issue is within the exclusive
jurisdiction of the Securities and Exchange Commission, it being an intracorporate controversy and filed a motion to dismiss. Before the trial court ruled
on the motion, the petitioner amended the complaint impleading as additional
defendants Central bank Governor Cuisia, and Central Bank SRDC Managing
Director, Lirio, and SES Acting Director Guererro. This move is to correct the
petitioners claim on the jurisdiction of the Trial Court as the case now cannot be
considered an intra-corporate controversy. Nevertheless, the motion to dismiss
has been granted by the trial court for lack of jurisdiction. Hence, this petition.

ISSUE

Whether the case was incorrectly dismissed for lack of jurisdiction.

RULING

The trial court is correct in dismissing the case for lack of jurisdiction even if other
parties not part of AFPSLAI were impleaded as respondents. The allegations
533

against herein respondents in the amended complaint unquestionably reveal


intra-corporate controversies cleverly concealed, although unsuccessfully, by use
of civil law terms and phrases. The amended complaint impleads herein
respondents who, in their capacity as directors of AFPSLAI, allegedly convened
an illegal meeting and voted for the reorganization of management resulting in
petitioner's ouster as corporate officer. While it may be said that the same
corporate acts also give rise to civil liability for damages, it does not follow that
the case is necessarily taken out of the jurisdiction of the SEC as it may award
damages which can be considered consequential in the exercise of its
adjudicative powers.
The determination of the rights of petitioner arising from the alleged illegal
convening of the meeting of AFPSLAI Board of Directors and his subsequent
ouster from corporate offices as a result of the voting for the reorganization of
management are obviously intra-corporate controversies subject to the
jurisdiction of SEC as provided in P.D. No. 902-A which states: Sec. 5. In
addition to the regulatory and adjudicative functions of the Securities and
Exchange Commission over corporations . . . it shall have original and exclusive
jurisdiction to hear and decide cases involving . . . . (b) Controversies arising out
of intra-corporate . . . relations . . . . (c) Controversies in the election or
appointment of directors, trustees, officers or managers of such corporations
The same may also be said of petitioner's prayer for damages, considering
that his right thereto either depends on, or is inextricably linked with, the
resolution of the corporate controversies.
Jurisdiction over subject matter is essential in the sense that erroneous
assumption thereof may put at naught whatever proceedings the court might
have had. Hence, even on appeal, and even if the parties do not raise the issue
of jurisdiction, the reviewing court is not precluded from ruling that it has no
jurisdiction over the case For this matter, the ground of lack of jurisdiction in
dismissing a case is not waivable. Hence, the last sentence of Sec. 2, Rule 9,
Rules of Court, expressly states: "Whenever it appears that the court has no
jurisdiction over the subject matter, it shall dismiss the action."
We note that Sec. 2, Rule 9 uses the word "shall," leaving the court no
choice under the given situation but to dismiss the case. The same Rule also
uses the phrase "whenever it appears," which means at any time after the
complaint or amended complaint is filed, because the lack of jurisdiction may be
apparent from the allegations therein. Hence, from the foregoing, even if no
534

answer or motion to dismiss is filed, the court may dismiss the case for want of
jurisdiction. In this sense, dismissal for lack jurisdiction may be ordered by the
court motu propio. Applying this notion to the case at bar, with the dismissal of
the case against respondents for lack of jurisdiction, it then becomes
inconsequential whether the court acted on the Urgent Motion to Dismiss or on
the Omnibus Motion without the requisite notice as provided in Secs. 4 and 6 of
Rule 15 of the Rules of Court. The determination of lack of jurisdiction over
respondents being apparent from the face of the amended complaint, the defect
of want of prior notice and hearing of the Omnibus Motion could not by itself
confer jurisdiction upon the court a quo.

535

Pearson & George, (S.E. Asia), Inc. vs. National Labor Relations
Commission, 253 SCRA 136 (1996)
FACTS:

The petitioner insists that the Labor Arbiter and the NLRC do not have jurisdiction
over the private respondents complaint for illegal dismissal arising out of his
removal as Managing Director of the petitioner due to his non-reelection and the
abolition of the said position. It claims that the matter is intra-corporate and thus
falls within the exclusive jurisdiction of the Securities and Exchange Commission
pursuant to Section 5(c) of P.D. No. 902-A.
Private respondent Leopoldo Llorente was a member of the Board of Directors of
the petitioner and was elected as Vice-Chairman of the Board and as Managing
Director for a term of one year and until his successor should have been duly
elected pursuant to the petitioners by-laws. On 29 January 1990, Llorente was
preventively suspended, with pay, by reason of alleged anomalous transactions
entered by him, which were prejudicial to the interest of the petitioner. Llorente,
protested his suspension and requested an examination of the supporting
documents to enable him to explain the accusations leveled against him, but to
no avail.
At the regular stockholders meeting on 5 March 1990, the stockholders of the
petitioner elected a new set of directors. Llorente was not reelected. On the
same day, the new Board of Directors held a meeting wherein it elected a new
set of officers and abolished the position of Managing Director. The petitioners
counsel informed Llorente of his non-reelection, the abolition of the position of
Managing Director, and his termination for cause. Llorente filed with the Labor
Arbiter a complaint for unfair labor practice, illegal dismissal, and illegal
suspension alleging therein that he was dismissed without due process of law.

ISSUE:

Whether the NLRC which has jurisdiction over the complaint for illegal dismissal
which the private respondent had filed with the NLRC.

RULING:

NO.

The removal of Llorente as Managing Director is purely an intra-corporate


dispute which falls within the exclusive jurisdiction of the SEC and not of the
NLRC. In reality, Llorente was not dismissed. If he lost the position of Managing
Director, it was primarily because he was not reelected as Director during the
536

regular stockholders meeting. The office of Managing Director presupposes that


its occupant is a Director; hence, one who is not a Director of the petitioner or
who has ceased to be a Director cannot be elected or appointed as a Managing
Director. The holding of the position of Director is a prerequisite for the election,
appointment, or designation of Managing Director. If a Managing Director should
lose his position because he ceased to be a Director for any reason, such as
non-reelection as in the case of Liorente, such loss is not dismissal but failure to
qualify or to maintain a prerequisite for that position. Then too, the position of
Managing Director was abolished.
Any question relating or incident to the election of the new Board of Directors, the
non-reelection of Liorente as a Director, his loss of the position of Managing
Director, or the abolition of the said office are intra-corporate matters. Disputes
arising therefrom are intra-corporate disputes which, if unresolved within the
corporate structure of the petitioner, may be resolved in an appropriate action
only by the SEC pursuant to its authority under paragraphs (b) and (c), Section 5
of P.D. No. 902-A.

537

Ongkingco vs. National Labor Relations Commission, 270 SCRA 613


(1997)
Ponente : KAPUNAN, J.
Facts:

Petitioner Galeria De Magallanes Condominium Associated, Inc.


(GALERIA), a non stock, non profit corporation represented by Bienvenido
Ongkingco as President. By virtue of Board of Directors, the private respondent
Federico Guilas was appointed as Administrator/Superintendent receiving a
salary of Php 10,000.00 per month. However, Guilas wa not re appointed as
Administrator. He then filed a Complaint for Illegal Dismissal and non payment
of salaries before the NLRC. However, petitioner filed a Motion to Dismiss
alleging that it is the SEC not the labor arbiter which has jurisdiction over the
subject matter. Labor Arbiter however, granted such motion to dismiss.
However, the NLRC reversed the resolution of the Labor Arbiter in which it fell
under its own jurisdiction because the termination of the complainant was
terminated by a resolution of Board of Directors. Petitioner Galeria filed a Motion
for Reconsideration but the same was denied. Then it was elevated to SC.
Issue:
Whether private respondent acted without or in excess of its jurisdiction or
committed grave abuse of discretion in taking cognizance within the original and
exclusive jurisdiction of SEC.
Ruling:
No, it does not.
As it is specifically delineated in P.D. 902-A are the cases over which the
SEC exercises exclusive jurisdiction:
(a)

Devices or schemes employed by or any acts of the board of directors,


business associates, its officers or partners, amounting to fraud and
misrepresentation which may be detrimental to the interest of the
public and/or of the stockholders, partners, members of associations
or organizations registered with the Commission.

(b)

Controversies arising out of intra-corporate or partnership relations,


between and among stockholders, members, or associates; between
any or all of them and the corporation, partnership or association of
which they are stockholders, members or associates, respectively; and
between such corporation, partnership or association and the State
insofar as it concerns their individual franchise or right to exist as such
entity;

(c)

Controversies in the election or appointment of directors, trustees,


officers, or managers of such corporations, partnerships or
associations.
538

(d) Petitions of corporations, partnerships or associations to be declared in


the state of suspension of payments in cases where the corporation, partnership
or association possesses property to cover all of its debts but foresees the
impossibility of meeting them when they respectively fall due or in cases where
the corporation, partnership or association has no sufficient assets to cover its
liabilities, but is under the Management Committee created pursuant to this
Decree.
Moreover, as provided in the by-laws of the Galeria de Magallanes
Condominium Association specifically includes the Superintendent/Administrator
in its roster of corporate officers, which was appointed by the Board of Directors.
Well-settled is the rule that a corporate officer's dismissal is always a
corporate act and/or an intra-corporate controversy and that nature is not altered
by the reason or wisdom which the Board of Directors may have in taking such
action.

539

Calleja vs. Panday 483 SCRA 689 (2006)


Ponente: Justice Austria-Martinez
Facts:
Respondents had been members of the board of directors and officers of St.
John Hospital Incorporated from 1985 up to the time of the complaint. Sometime
in May 20015, petitioners, who are also among the incorporators and
stockholders of said corporation, forcibly and with the aid of armed men usurped
the powers which supposedly belonged to Respondents. On May 16, 2005,
respondents filed a petition with the Regional Trial Court of San Jose, Camarines
Sur for quo warranto with Damages and Prayer for Mandatory and Prohibitory
Injunction, Damages and Issuance of Temporary Restraining Order against
herein petitioners.
The RTC-Br. 58 then proceeded to issue and serve summons on Panday et. al.
(respondents in this case). Petitioner Tabora filed his Answer dated June 8,
2005, raising therein the affirmative defenses of (1) improper venue, (2) lack of
jurisdiction, and (3) wrong remedy of quo warranto. Thereafter, the other
petitioners also filed their Answer, also raising the same affirmative defenses.
PD 902-A, which was approved on March 11, 1976, provides that cause of action
involves controversies arising out of intra-corporate relations is within the original
and exclusive jurisdiction of the Securities and Exchange Commission to try and
decide in addition to its regulatory and adjudicated functions (Section 5, PD 902A). on January 23, 2001, supplemental Administrative Circular No. 8-01 which
took effect on March 1, 2001 was issued by the Supreme Court which directed
that "all SEC cases originally assigned or transmitted to the regular Regional
Trial Court shall be transferred to branches of the Regional Trial Court specially
designated to hear such cases in accordance with A.M. No. 00-11-03-SC.
Issue:
Whether the RTC Br. 58 has jurisdiction to try cases arising out of intra-corporate
relations
Ruling:
No. actions of quo warranto against persons who usurp an office in a corporation,
which were formerly cognizable by the Securities and Exchange Commission
under PD 902-A, have been transferred to the courts of general jurisdiction. But,
this does not change the fact that Rule 66 of the 1997 Rules of Civil Procedure
does not apply to quo warranto cases against persons who usurp an office in a
private corporation. Note, further, that respondents petition for quo warranto was
filed as late as 2005. A.M. No. 03-03-03-SC took effect as early as July 1, 2003
and it was clearly provided therein that such petitions shall be filed in the Office
of the Clerk of Court in the official station of the designated Special Commercial
Court.

540

Yujuico vs. Quiambao 513 SCRA 243 (2007)


Sandoval-Gutierrez, J.:

FACTS:

On July 27, 1998, the Securities and Exchange Commission (SEC) approved the
amendment of Strategic Alliance Development Corporation

(STRADEC)

Articles of Incorporation authorizing the change of its principal office from Pasig
City Pangasinan.
On March 1, 2004 STRADEC held its annual stockholders meeting in Pasig City
its office as indicated in the notices sent to the stockholders. Herein petitioners
and respondent were elected members of the Board of Directors.
Five months thereafter, respondents filed with the RTC in Pangasinan a
complaint againsts STRADEC. The complaint seeks for the nullification of the
election on the ground of improper venue, pursuant to Section 51 of the
Corporation Code, next is the nullification of all subsequent transactions
conducted by the elected directors and lastly that a special stockholders meeting
be held once again. The RTC under pairing Judge Emuslan issued an Order for
granting respondents application for preliminary injunction ordering the holding of
a special stockholders meeting of STRADEC on December , in the principal
office of the corporation in Bayambang, Pangasinan; and the turn-over by
petitioner Bonifacio Sumbilla to the court of the duplicate key of the safety
deposit box in Export Industry Bank, Shaw Boulevard, Pasig City where the
original Stock and Transfer Book of STRADEC was deposited. The plaintiff filed
with the Court of Appeals as Petition for Certiorari, CA dismissed such petition
and upheld the jurisdiction of the RTC.

541

ISSUE:
Whether the RTC has the power to call a special stockholders meeting
involving an intra-corporate controcersy?

Ruling:

Yes.
Upon the enactment of RA No, 8799, otherwise known as The Securities
Regulation Code which took effect on August 8, 2000, the jurisdiction of the SEC
over intra-corporate controversies and other cases enumerated in Section 5 of
PD No, 902-A has been transferred to the courts of general jurisdiction, or the
appropriate RTC. Section 5-2 of RA 8799 provides: 5.2 The Commissions
jurisdiction over all cases enumerated in Section 5 of Presidential Decree No.
902-A is hereby transferred to the Courts of general jurisdiction or the
appropriate RTC, Provided, that the Supreme Court in the exercise of its
authority may designate the RTC Branches that shall exercise jurisdiction over
these cases. The Commission shall retain jurisdiction over pending cases
involving intracorporate disputes submitted for final resolution which should be
resolved within one year from the enactment of this Code. The Commission shall
retain jurisdiction over pending suspension of payments/rehabilitation cases filed
as of June 30, 2000 until finally disposed.
The RTC has the power to hear and decide the intra-corporate controversy of the
parties herein. Concomitant to said power is the authority to issue orders
necessary or incidental to the carrying out of the powers expressly granted to it.
Thus, the RTC may, in appropriate cases, order the holding of a special meeting
of stockholders or members of a corporation involving an intra-corporate dispute
under its supervision.

542

Reyes vs. RTC of Makati 561 SCRA 593 (2008)


BRION, J.:

FACTS:

Petitioner and private respondent were siblings together with two others, namely
Pedro and Anastacia, in a family business established as Zenith Insurance
Corporation (Zenith), from which they owned shares of stocks. The Pedro and
Anastacia subsequently died. The former had his estate judicially partitioned
among his heirs, but the latter had not made the same in her shareholding in
Zenith. Zenith and Rodrigo filed a complaint with the Securities and Exchange
Commission (SEC) against petitioner (1) a derivative suit to obtain accounting of
funds and assets of Zenith, and (2) to determine the shares of stock of deceased
Pedro and Anastacia that were arbitrarily and fraudulently appropriated [by
Oscar, and were unaccounted for]. In his answer with counterclaim, petitioner
denied the illegality of the acquisition of shares of Anastacia and questioned the
jurisdiction of SEC to entertain the complaint because it pertains to settlement of
[Anastacias] estate. The case was transferred to. Petitioner filed Motion to
Declare Complaint as Nuisance or Harassment Suit and must be dismissed. RTC
denied the motion. The motion was elevated to the Court of Appeals by way of
petition for certiorari, prohibition and mandamus, but was again denied.

ISSUES:

(1) Whether or not Rodrigo may be considered a stockholder of Zenith with


respect to the shareholdings originally belonging to Anastacia.

(2) Whether or not there is an intra-corporate relationship between the parties


that would characterize the case as an intra-corporate dispute?
HELD:

(1)

No. Rodrigo must, hurdle two obstacles before he can be considered a

stockholder of Zenith with respect to the shareholdings originally belonging to


Anastacia. First, he must prove that there are shareholdings that will be left to
him and his co-heirs, and this can be determined only in a settlement of the
decedents estate. No such proceeding has been commenced to date. Second,
he must register the transfer of the shares allotted to him to make it binding
543

against the corporation. He cannot demand that this be done unless and until he
has established his specific allotment (and prima facie ownership) of the shares.
Without the settlement of Anastacias estate, there can be no definite partition
and distribution of the estate to the heirs. Without the partition and distribution,
there can be no registration of the transfer. And without the registration, we
cannot consider the transferee-heir a stockholder who may invoke the existence
of an intra-corporate relationship as premise for an intra-corporate controversy
within the jurisdiction of a special commercial court. The subject shares of stock
(i.e., Anastacias shares) are concerned Rodrigo cannot be considered a
stockholder of Zenith.

(2)

No. Court cannot declare that an intra-corporate relationship exists that

would serve as basis to bring this case within the special commercial courts
jurisdiction under Section 5(b) of PD 902-A, as amended because Rodrigos
complaint failed the relationship test above.

544

Unlad Resources Devt., Corp., et al. vs. Renato P. Dragon,


560 SCRA 63 (2008)
FACTS:

On December 29, 1981, the Plaintiffs (herein respondents) and defendant (herein
petitioner) Unlad Resources, through its Chairman [,] Helena Z. Benitez [,]
entered into a Memorandum of Agreement wherein it is provided that
[respondents], as controlling stockholders of the Rural Bank [of Noveleta] shall
allow Unlad Resources to invest four million eight hundred thousand pesos (P4,
800,000.00) in the Rural Bank in the form of additional equity. On the other hand,
[petitioner] Unlad Resources bound itself to invest the said amount of 4.8 million
pesos in the Rural Bank; upon signing, it was, likewise, agreed that [petitioner]
Unlad Resources shall subscribe to a minimum of four hundred eighty thousand
pesos (P480,000.00) (sic) common or preferred non-voting shares of stock with a
total par value of four million eight hundred thousand pesos (P4,800,000.00) and
pay up immediately one million two hundred thousand pesos (P1,200,000.00) for
said subscription; that the [respondents], upon the signing of the said agreement
shall transfer control and management over the Rural Bank to Unlad Resources.
According to the [respondents], immediately after the signing of the agreement,
they complied with their obligation and transferred control of the Rural Bank to
Unlad Resources and its nominees and the Bank was renamed the Unlad Rural
Bank of Noveleta, Inc. However, [respondents] claim that despite repeated
demands, Unlad Resources has failed and refused to comply with their obligation
under the said Memorandum of Agreement when it did not invest four million
eight hundred thousand pesos (P4, 800,000.00) in the Rural Bank in the form of
additional equity and, likewise, it failed to immediately infuse one million two
hundred thousand pesos (P1, 200,000.00) as paid in capital upon signing of the
Memorandum of Agreement.

On July 3, 1987, herein respondents filed before the Regional Trial Court
(RTC) of Makati City, Branch 61 a Complaint [4] for rescission of the agreement
and the return of control and management of the Rural Bank from petitioners to
respondents, plus damages. After trial, the RTC favored herein defendant, which
the CA affirmed.

ISSUES

Whether disputes on contractual obligations involving Corporation fall under the


jurisdiction of SEC.

RULING
545

The Petition is bereft of merit. We uphold the Decision of the CA affirming


that of the RTC.

First, the subject of jurisdiction. The main issue in this case is the
rescission of the Memorandum of Agreement. This is to be distinguished from
respondents allegation of the alleged mismanagement and dissipation of
corporate assets by the petitioner which is based on the prayer for receivership
over the bank. The two issues, albeit related, are obviously separate, as they
pertain to different acts of the parties involved. The issue of receivership does
not arise from the parties obligations under the Memorandum of Agreement, but
rather from specific acts attributed to petitioners as members of the Board of
Directors of the Bank. Clearly, the rescission of the Memorandum of Agreement
is a cause of action within the jurisdiction of the trial courts, notwithstanding the
fact that the parties involved are all directors of the same corporation.

Still, the petitioners insist that the trial court had no jurisdiction over the
complaint because the issues involved are intra-corporate in nature.

This argument miserably fails to persuade. The law in force at the time of
the filing of the case was Presidential Decree (P.D.) 902-A, Section 5(b) of which
vested the Securities and Exchange Commission with original and exclusive
jurisdiction to hear and decide cases involving controversies arising out of intracorporate relations.[8] Interpreting this statutorily conferred jurisdiction on the
SEC, this Court had occasion to state:

Nowhere in said decree do we find even so much as an [intimation]


that absolute jurisdiction and control is vested in the Securities and
Exchange Commission in all matters affecting corporations. To
uphold the respondents arguments would remove without legal
imprimatur from the regular courts all conflicts over matters involving
or affecting corporations, regardless of the nature of the transactions
which give rise to such disputes. The courts would then be divested
of jurisdiction not by reason of the nature of the dispute submitted to
them for adjudication, but solely for the reason that the dispute
involves a corporation. This cannot be done.[9]

It is well to remember that the respondents had actually filed with the SEC
a case against the petitioners which, however, was dismissed for lack of
546

jurisdiction due to the pendency of the case before the RTC.[10] The SECs Order
dismissing the respondents complaint is instructive:

From the foregoing allegations, it is apparent that the present


action involves two separate causes of action which are interrelated,
and the resolution of which hinges on the very document sought to
be rescinded. The assertion that the defendants failed to comply
with their contractual undertaking and the claim for rescission of the
contract by the plaintiffs has, in effect, put in issue the very status of
the herein defendants as stockholders of the Rural Bank. The issue
as to whether or not the defendants are stockholders of the Rural
Bank is a pivotal issue to be determined on the basis of the
Memorandum of Agreement. It is a prejudicial question and a logical
antecedent to confer jurisdiction to this Commission.

It is to be noted, however, that determination of the contractual


undertaking of the parties under a contract lies with the Regional
Trial Courts and not with this Commission. x x x[11]

Be that as it may, this point has been rendered moot by Republic Act
(R.A.) No. 8799, also known as the Securities Regulation Code. This law, which
took effect in 2000, has transferred jurisdiction over such disputes to the RTC.
Specifically, R.A. 8799 provides:

Sec. 5. Powers and Functions of the Commission

xxxx

5.2. The Commissions jurisdiction over all cases enumerated under


Section 5 of Presidential Decree No. 902-A is hereby transferred to
the Courts of general jurisdiction or the appropriate Regional Trial
Court: Provided, That the Supreme Court in the exercise of its
authority may designate the Regional Trial Court branches that shall
exercise jurisdiction over these cases. The Commission shall retain
jurisdiction over pending cases involving intra-corporate disputes
submitted for final resolution which should be resolved within one (1)
year from the enactment of this Code. The Commission shall retain
jurisdiction over pending suspension of payments/rehabilitation
cases filed as of 30 June 2000 until finally disposed.
547

Section 5 of P.D. No. 902-A reads, thus:

Sec.5. In addition to the regulatory and adjudicative functions


of the Securities and Exchange Commission over corporations,
partnerships and other forms of associations registered with it as
expressly granted under existing laws and decrees, it shall have
original and exclusive jurisdiction to hear and decide cases
involving:

a) Devices and schemes employed by or any acts of the board


of directors, business associates, its officers or partnership,
amounting to fraud and misrepresentation which may be detrimental
to the interest of the public and/or of the stockholder, partners,
members of associations or organizations registered with the
Commission;

b) Controversies arising out of intra-corporate or partnership


relations, between and among stockholders, members, or
associates; between any or all of them and the corporation,
partnership or association of which they are stockholders, members
or associates, respectively; and between such corporation,
partnership or association and the state insofar as it concerns their
individual franchise or right to exist as such entity;

c) Controversies in the election or appointment of directors,


trustees, officers or managers of such corporations, partnerships or
associations.

Consequently, whether the cause of action stems from a contractual


dispute or one that involves intra-corporate matters, the RTC already has
jurisdiction over this case.
Hence, the award of exemplary damages is in itself sufficient justification
for the award of attorneys fees.[26]

WHEREFORE, the foregoing premises considered, the petition is


hereby DENIED. The assailed Decision and Resolution of the Court of Appeals
in CA-G.R. CV No. 54226 are AFFIRMED.
548

Peneyra vs. Intermediate Appellate Court, 181 SCRA 244 (1990)


FERNAN, C.J.:
FACTS:
The Board of Trustees of the Corregidor College Inc. awarded the
management and operation of its canteen at a monthly rental of P80.00 to
petitioners herein who are stockholders of the said College. Subsequently, upon
instructions of Eulogio Dizon, Chairman of the Board of Trustees of Corregidor
College, Inc., the rental payments of petitioners were refused, and on August 6,
1980, partial demolition of the canteen was effected. Consequently, petitioners
filed in the then Court of First Instance of Nueva Ecija an action against Eulogio
R. Dizon for damages with preliminary mandatory injunction.
While the case is pending, Eulogio Dizon died. By reason thereof, his counsel
moved to dismiss the complaint.The trial court dismissed petitioners' complaint
on the ground that the action for damages did not survive the death of Eulogio
Dizon. They then filed a special civil action of certiorari and mandamus against
respondent judge before the IAC.
On September 28, 1984, the Appellate Court dismissed the petition holding that
the Securities and Exchange Commission (SEC) has jurisdiction over the case,
the same being an intracorporate dispute, that the amendment to include
Corregidor College, Inc. cannot be allowed and that the action for damages
against Eulogio Dizon was extinguished by his death.
Hence, this recourse.
ISSUES:
Does jurisdiction over this case pertain to the Securities and Exchange
Commission?
RULING:
Under Section 3 of Presidential Decree 902-A, the jurisdiction of the SEC is
limited to matters intrinsically connected with the regulation of corporations,
partnerships and associations and those dealing with the internal affairs of such
entities. P.D. 902-A does not confer in the SEC absolute jurisdiction and control
over all matters affecting corporations.
549

While it is true that petitioners herein are stockholders of Corregidor College,


Inc., the. complaint did not stem directly from such relationship, but rather from
the award to petitioners of the management and operation of its canteen at a
monthly rental of P80.00. The management of a canteen, even if awarded to a
stockholder, is outside or merely incidental to the central operations of an
educational institution. Petitioners thus convincingly argue that "the controversy
is not one where petitioners are bringing the action as stockholders but rather as
operators of the canteen under an agreement with said Board. In short, the
cause of action here is for damages arising from a violation of a contract of
management operation of the College canteen by defendant Dizon. Certainly, the
present controversy cannot qualify as an intra-controversy, its root being a
contractual breach separate and distinct from the corporate relationship between
petitioners and Corregidor College, Inc., which, it must be noted, was not even
named as a defendant in the original complaint. It was therefore patent error for
the Court of Appeals to immediately rule that the present case belongs to the
SEC just because petitioners alleged that they are stockholders of Corregidor
College, Inc.

550

Securities and Exchange Commission vs. Court of appeals,


246 SCRA 738 (1995)
VITUG, J.:
Facts:

Cualoping Securities Corporation (CUALOPING for brevity) is a

stockbroker, Fidelity Stock Transfer, Inc. (FIDELITY for brevity), on the other
hand, is the stock transfer agent of Philex Mining Corporation (PHILEX for brevity
On or about the first half of 1988, certificates of stock of PHILEX representing
one million four hundred [thousand] (1,400,000) shares were stolen from the
premises of FIDELITY.
Agustin Lopez, a messenger of an entirely different stock brokerage firm, Lopez
brought the stolen stock certificates to CUALOPING for trading and sale with the
stock exchange.
CUALOPING stamped each and every certificate with the words "Indorsement
Guaranteed," and thereafter traded the same with the stock exchange.
After the sale of the stocks represented by said certificates to different buyers,
the same were delivered to FIDELITY for the cancellation of the stocks
certificates and for issuance of new certificates in the name of the new buyers.
FIDELITY rejected the issuance of new certificates in favor of the buyers for
reasons that the signatures of the owners of the certificates were allegedly forged
and thus the cancellation and new issuance thereof cannot be effected.
FIDELITY sought an opinion on the matter from SEC which summoned
FIDELITY and CUALOPING to a conference.
The Brokers and Exchange Department ("BED") of the SEC disposed of the
matter ordering FIDELITY to replace all the subject shares and CUALOPING to
pay a fine of P50,000.00 within 5 days for violation of Section 29 a(3) of the
Revised Securities Act.
CUALOPING and FIDELITY appealed to the Commission En Banc which
ordered them to jointly replace the shares of stock and pay the fine.
The decision was appealed to the Court of Appeals which reversed and set aside
SEC order.
The Commission has brought the instant petition for review.
551

Issues: a) Is the SEC correct in exercising its adjudicatory power upon the
subject matter which started from FIDELITYs Request for its opinion?
b) Did FIDELITY and CUALOPING violated Section 29 a(3) of the Revised
Securities Act?
Ruling: a) xxxx. This case, it might be recalled, has started only on the basis of
a request by FIDELITY for an opinion from the SEC. The stockholders who have
been deprived of their certificates of stock or the persons to whom the forged
certificates have ultimately been transferred by the supposed indorsee thereof
are yet to initiate, if minded, an appropriate adversarial action. Neither have they
been made parties to the proceedings now at bench. A justiciable controversy
such as can occasion an exercise of SEC's exclusive jurisdiction would require
an assertion of a right by a proper party against another who, in turn, contests
it. 5 It is one instituted by and against parties having interest in the subject matter
appropriate for judicial determination predicated on a given state of facts. That
controversy must be raised by the party entitled to maintain the action. He is the
person to whom the right to seek judicial redress or relief belongs which can be
enforced against the party correspondingly charged with having been responsible
for, or to have given rise to, the cause of action. A person or entity tasked with
the power to adjudicate stands neutral and impartial and acts on the basis of the
admissible representations of the contending parties.
In the case at bench, the proper parties that can bring the controversy and can
cause an exercise by the SEC of its original and exclusive jurisdiction would be
all or any of those who are adversely affected by the transfer of the pilfered
certificates of stock. Any peremptory judgment by the SEC, without such
proceedings having first been initiated, would be precipitate. xxxx.
b) Here, the SEC has aptly invoked the provisions of Section 29, in relation to
Section 46, of the Revised Securities Act. This law provides:
Sec. 29. Fraudulent transactions. (a) It shall be unlawful for any
person, directly or indirectly, in connection with the purchase or sale
of any securities
xxx xxx xxx

552

(3) To engage in any act, transaction practice, or course of business


which operates or would operate as a fraud or deceit upon any
person.
Sec. 46. Administrative sanctions. If, after proper notice and
hearing, the Commission finds that there is a violation of this Act, its
rules, or its orders or that any registrant has, in a registration
statement and its supporting papers and other reports required by
law or rules to be filed with the Commission, made any untrue
statement of a material fact, or omitted to state any material fact
required to be stated therein or necessary to make the statements
therein not misleading, or refused to permit any unlawful
examination into its affairs, it shall, in its discretion, impose any or all
of the following sanctions:
(a) Suspension, or revocation of its certificate of registration and
permit to offer securities;
(b) A fine of no less than two hundred (P200.00) pesos nor more
than fifty thousand (P50,000.00) pesos plus not more than five
hundred (P500.00) pesos for each day of continuing violation.
(Emphasis supplied.)
There is, to our mind, no question that both FIDELITY and CUALOPING have
been guilty of negligence in the conduct of their affairs involving the questioned
certificates of stock. To constitute, however, a violation of the Revised Securities
Act that can warrant an imposition of a fine under Section 29(3), in relation to
Section 46 of the Act, fraud or deceit, not mere negligence, on the part of the
offender must be established. Fraud here is akin to bad faith which implies a
conscious and intentional design to do a wrongful act for a dishonest purpose or
moral obliquity; it is unlike that of the negative idea of negligence in that fraud or
bad faith contemplates a state of mind affirmatively operating with furtive
objectives. Given the factual circumstances found by the appellate court, neither
FIDELITY nor CUALOPING, albeit indeed remiss in the observance of due
diligence, can be held liable under the above provisions of the Revised Securities
Act.
Our attention is called by the Solicitor General on the violation by FIDELITY of
SEC-BED Memorandum Circular No. 9, series of 1987, which reads:
553

To expedite the release of Certificates of Securities to the buyers,


the Commission reiterates the following rules in delivery of stock
certificates:
1. Deadlines for Delivery of Documents All requirements must be
complied with the certificates of stock, as well as necessary
documents required for the transfer of shares shall be delivered
within the following periods:
xxx xxx xxx
d. From transfer agent back to clearing house and/or broker not
longer than ten (10) days from receipt of documents provided there
is a "good delivery," where there is no "good delivery," the certificate
and the accompanying documents shall be returned to the clearing
house or broker not later than two (2) days after receipt
thereof, except when defects can be readily remedied, in which case
the clearing house or the broker shall instead be notified of the
requirements within the same period. The notice to the clearing
house or broker shall indicate that the Securities and Exchange
Commission has been notified of such defective delivery.
FIDELITY is candid enough to admit that it has truly failed to
promptly notify CUALOPING and the clearing house of the pilferage
of the certificates of stock. Xxxx.
In this case at bench, particularly, it is not a new issue that is being
raised but a memorandum-circular having the force and effect of law
that has been cited to support a position that relates to the very
subject matter of the controversy. On this point, accordingly, we
must rule in favor of petitioner SEC.

554

Philippine Stock Exchange vs. Securities & Exchange Commission


281 SCRA 232 (1997)
TORRES, JR., J.:

FACTS:
Puerto Azul Land, Inc. (PALI) is a corporation engaged in the real estate
business. PALI was granted permission by the Securities and Exchange
Commission (SEC) to sell its shares to the public in order for PALI to develop its
properties.
PALI then asked the Philippine Stock Exchange (PSE) to list PALIs
stocks/shares to facilitate exchange. The PSE Board of Governors denied PALIs
application on the ground that there were multiple claims on the assets of PALI.
Apparently, the Marcoses, Rebecco Panlilio (trustee of the Marcoses), and some
other corporations were claiming assets if not ownership over PALI.
PALI then wrote a letter to the SEC asking the latter to review PSEs decision.
The SEC reversed PSEs decisions and ordered the latter to cause the listing of
PALI shares in the Exchange.

ISSUE:
Whether or not it is within the power of the SEC to reverse actions done by the
PSE.

RULING:
Yes. The SEC has both jurisdiction and authority to look into the decision of PSE
pursuant to the Revised Securities Act and for the purpose of ensuring fair
administration of the exchange. PSE, as a corporation itself and as a stock
exchange is subject to SECs jurisdiction, regulation, and control. In order to
insure fair dealing of securities and a fair administration of exchanges in the PSE,
the SEC has the authority to look into the rulings issued by the PSE. The SEC is
the entity with the primary say as to whether or not securities, including shares of
stock of a corporation, may be traded or not in the stock exchange.

HOWEVER, in the case at bar, the Supreme Court emphasized that the SEC
may only reverse decisions issued by the PSE if such are tainted with bad faith.
In this case, there was no showing that PSE acted with bad faith when it denied
the application of PALI.
555

Based on the multiple adverse claims against the assets of PALI, PSE deemed
that granting PALIs application will only be contrary to the best interest of the
general public. It was reasonable for the PSE to exercise its judgment in the
manner it deems appropriate for its business identity, as long as no rights are
trampled upon, and public welfare is safeguarded.

556

Philippine National Construction Corporation vs. Pabion,


320 SCRA 188 (1999)
Facts: Private respondents Ernesto Pabion and Lovella Ramiro, claiming to be
stockholders of the PNCC filed with SEC a verified petition, therein alleging that
since 1982 or for a period of 12 years, there has been no stockholders meeting
of the PNCC to elect the corporations BOD, thus enabling the incumbent
directors to hold on to their position beyond their 1-yr term, in violation of PNCCs
By-Laws and the Corporation Code. Private respondents, therefore prayed the
SEC to issue an order ordering the officers of PNCC or, in the alternative,
authorizing petitioners, to call and hold a meeting of the stockholders for the
purpose of electing new directors. The care was assigned to SEC Hearing Officer
Manuel Perea.

The Commission en banc held that PNCC being incorporated under the
Corporation Code is therefore, subject to Section 50 of the CorporationCode
which requires the holding of regular stockholders meeting for the purpose of
selecting PNCCs BOD.

Issues:
1) Can SEC determine the corporate status of PNCC?
2) Does SEC have jurisdiction over GOCCs? Does it have the authority to
compel PNCC to hold a stockholders meeting for the purpose of electing
members of a BOD?
3) Is PNCC an acquired-asset corporation?

Held:
1) Yes. It is certainly absurd to say that SEC is without jurisdiction to determine if
PNCC is a GOCC simply because the latter claims to be one. The President
does not determine whether a corporation is a GOCC or not. It is the law that
does. PNCCs status as a GOCC can be ruled upon by SEC based on law.

557

2) Yes. GOCCs may either be (1) with original charter or created by special law;
or (2) incorporated under general law, via either the Old Corporation Code or the
New Corporation Code. SEC has no jurisdictionover corporations of the first type
primarily because they are governed by their charters. But even this is not
absolute, since the corporationCode may apply suppletorily, either by operation
of law or through express provision in the charter.

On the other hand, over GOCCs established or organized under Corporation


Code, the SEC can exercise jurisdiction. These GOCCs are regarded as private
corporations despite common misconception. That the government may own the
controlling shares in the corporation does not diminish the fact that the latter
owes its existence to the Corporation Code. Prescinding from such premises, it
necessarily follows that SEC can compel PNCC to hold a stockholders meeting
for the purpose of electing members of the latters BOD as clearly provided for by
Section 50 of the Corporation Code.

3) Yes. PNCC is indeed an acquired asset corporation as defined in Section 2 (a)


of A.O. 59, to wit: a corporation under private ownership, the voting or
outstanding share of which (i) were conveyed to the government financial
institutions in satisfaction of debts.

Moreover, there is no inconsistency between AO 59 and EO 292 otherwise


known as Revised Administrative Code. AO 59 does not purport to have
established a new kind of corporation that supersedes EO 292. Neither does the
former seek to revise the definition of GOCC given in the latter. What AO 59 in
fact does is to distinguish GOCCs in general from those that are sought to be
privatized. In fact, the definition given in EO 292 itself stated that the GOCCs
may be further categorized. This caveat suggests that the definition is broad
enough to admit distinctions as to the kinds of GOCCs defined under AC 59.

Hence, PNCC is as a GOCC under EO 292. However, for purposes of AO 59,


particularly in the application of Section 16 thereof, PNCC is anacquired asset
corporation. In this light, the alleged inconsistency is more apparent than real.
558

TCL Sales Corporation vs. Court Appeals, 349 SCRA 35 (2001


QUISUMBING, J.:

FACTS: Ting Ping Lay, not one of the original subscribers of the shares of stock
of TCL Sales Corporation, acquired his shares by purchasing those of some of
the original subscribers. In order to protect his shareholdings with TCL, Lay
requested Anna Teng, TCL Corporate Secretary to enter the transfer of shares of
stock for proper recording of his acquisitions in the Stock & Transfer Book of
TCL. He too demanded issuance of new certificates of stock in his favor.
TCL, however, even after repeated demands, refused. Lay filed a case with the
SEC for mandamus against TCL and Teng. This was in turn granted by the SEC
denying a later MR as well. The CA dismissed TCLs petition as well for being
filed out of time.

ISSUES:
1.Whether or not SEC has jurisdiction over the petition for mandamus filed
by Lay.
2. Whether or not the alleged transfer of shares in favor of Lay are valid
and can be ordered recorded.

HELD: Denied and CA decision affirmed.

Even if Lay were not a Share Holder, he is still a member of the public
whose investment in the corporate the law seeks to protect and encourage, as
his purchase of shares of stock has been established. Principal function of SEC
is supervision and control of corps, partnerships, assoc with the view of
protecting and encouraging investments for the protection of economic
development.

SEC has power of control & supervision over all corps to

encourage active public participation in the affairs of private corps through


investments.

559

Jurisdiction over an action for mandamus lies with the SEC even if the
proponent is not yet a SH of record, as in the case of Abejo v. de la Cruz. SEC
by express mandate has absolute jurisdiction to enforce the provisions of the
Corp Code among which is the stock purchasers right to secure the
corresponding certificate of stock in his name.

Determination of whether or not a Share Holder is entitled to exercise the


rights of a Share Holder is within jurisdiction of the SEC. The SEC en banc
found that TCL did not refute the validity of the transfers of the shares of stock
they conceded that they could not assail the documents evincing the transfer of
the shares to Lay. Lay was able to establish prima facie ownership through the
deeds of transfer of shares of stock of TCL. A listing of TCLs Share Holders &
their respective shares before & after the execution of a certain deed of
assignment shows that Lay is indeed listed as a Share Holder of TCL.

The

dispute is an intra-corp controversy involving Share Holders of TCL.

As held in Lim Tay v. CA, the duty of the corporate secretary to record
transfers of stocks is ministerial. It however, cannot be compelled when the
transferees title has no prima facie validity or is uncertain. Mandamus will not
issue to establish a right but only to enforce one already established.
Although during the trial before the SEC, TCL admitted that they ignored
Lays request was based simply on the fact that they did not want to grant it.
Having been capricious, whimsical & unwarranted, it constitutes bad faith.
However, the SEC en banc modified & deleted the said award for damages
imposed on the corp. The matter of damages now concerns only Teng, the
corporate secretary.

It was Tengs refusal as corp secretary to record the

transfer of the shares, without evidence that such refusal was authorized by
TCLs BOD, that caused damage. No error was committed by the respondent
court in refusing to disturb the SECs findings.

560

Pilipinas Loan Company, Inc. vs. Securities and Exchange


Commission, 356 SCRA 193 (2001)
Gonzaga-Reyes, J.
Facts:

Private respondent Filipinas Pawnshop, Inc is a duly organized corporation


registered with the Securities and Exchange Commission on February 9, 1959
with its principal place of business located along Pedro Gil St Paco, Metro
Manila. The articles of incorporation states that its primary purpose is to extend
loans at legal interest on the security of either personal properties or on the
security of real properties, and to finance installment sales of motor vehicles,
home appliances and other chattels.
Petitioner is a lending corporation duly registered with the SEC on July 27, 1989
with some of its places of business located along Pedro Gil, Sta. Ana, Manila and
Onyx St., cor. Augusto Francisco St., San Andres, Paco, Manila. The primary
purpose of petitioner is to act as a lending investor or, otherwise, to engage in
the practice of lending money or extending loans on the security of real or
personal, tangible or intangible properties whether as pledge, real or chattel
mortgage or otherwise, without however, engaging in pawnbroking as defined
under PD 114.
On September 11, 1990, private respondent filed a complaint against petitioner
with the Prosecution and Enforcement Department (PED) of the SEC. The
complaint alleged that: (1) petitioner, contrary to the restriction set by the
Commission, has been operating and doing business as a pawnbroker,
pawnshop or sanglaan in the same neighborhood where private respondent has
had its own pawnshop for 30 years in violation of its primary purpose and without
the imprimatur of the Central Bank to engage in the pawnshop business thereby
causing unjust and unfair competition with private respondent; and (2) the
business name of petitioner, PILIPINAS Loan, bears similarity in spelling and
phonetics with the corporate name of private respondent, FILIPINAS Pawnshop,
creating constant confusion in the minds of the public and the customers of
private respondent.

Issue:
Whether the SEC or the Central Bank has the jurisdiction over the case of
violation of PD 114.
561

Ruling:
Basic is the rule that it is the allegations in the complaint that vests
jurisdiction. A case in point is Philippine Womans Christian Temperance Union,
Inc. vs. Abiertas House of Friendship, Inc. wherein we held that when the thrust
of a complaint is on the ultra vires act of a corporation, that is the complained act
of a corporation is contrary to its declared corporate purposes, the SEC has
jurisdiction to entertain the complaint before it.
It must be recalled that the complaint of private respondent alleged that the
articles of incorporation of petitioner contained this prohibition: without, however,
engaging in pawnbroking as defined in PD 114 and despite this restriction,
petitioner allegedly continued to actually operate and do business as a
pawnshop. The complaint thus treats of a violation of petitioners primary
franchise. Section 5 of PD 114, the same law invoked by petitioner, mandates
that a corporation desiring to engage in the pawnshop business must first
register with the SEC. Without question, the complaint filed by private respondent
against petitioner called upon the SEC to exercise its adjudicatory and
supervisory powers. By law, the SEC has absolute jurisdiction, supervision and
control over all corporations that are enfranchised to act as corporate entities. A
violation by a corporation of its franchise is properly within the jurisdiction of the
SEC.

562

Philippine Association of Stock Transfer and Registry Agencies, Inc.


v. CA et al. 539 SCRA 61 (2007)
Ponente: Justice Leonardo A. Quisumbing

Facts:

Philippine Association of Stock Transfer and Registry Agencies, Inc.


(PASTRA) is an association of stock transfer agents principally engaged in the
registration of stock transfers in the stock-and-transfer book of corporations. Its
Board of Directors unanimously approved a resolution allowing its members to
increase the transfer processing fee they charge their clients from PHP45 to
PHP75 per certificate and eventually to PHP100. The resolution also authorized
the imposition of a processing fee for the cancellation of stock certificates at P20
per

certificate.

According

to

PASTRA,

the

rates

had

to be increased since it had been over 5 years since the old rates were fixed and
an increase of its fees was needed to sustain the financial viability of the
association and to upgrade facilities and services. Philippine Association of
Securities Brokers and Dealers, Inc. registered its objection to the measure and
requested SEC to defer its implementation, which later advised PASTRA to hold
in abeyance the implementation of the increases until the matter was cleared.
PASTRA nonetheless proceeded with the implementation of the increased fees.

SEC declared PASTRA as having defied a lawful order of the Commission


and was imposed with fines. The Court of Appeals ruled against PASTRA.

Issue:

Whether SEC has the power to regulate fees

Ruling:

Yes, SEC has the power to regulate fees. Before its repeal, Section 47 of
The Revised Securities Act clearly gave the SEC the power to enjoin the acts or
practices of securities-related organizations even without first conducting a
hearing if, upon proper investigation or verification, SEC is of the opinion that
there exists the possibility that the act or practice may cause grave or irreparable
injury to the investing public, if left unrestrained. Said section enforces the power
of general supervision of the SEC under Section 40 of the then Revised
563

Securities Act. As a securities-related organization under the jurisdiction and


supervision of the SEC by virtue of Section 40 of The Revised Securities Act and
Section 3 of Presidential Decree No. 902-A, petitioner was under the obligation to
comply with the SEC Order. Defiance of the order was subject to administrative
sanctions provided in the said Act.

The regulatory and supervisory powers of the Commission under Section


40 of the then Revised Securities Act were broad enough to include the power to
regulate petitioners fees. The intentional omission in the law of any qualification
as to what acts or practices are subject to the control and supervision of the SEC
under Section 47 confirms the broad extent of the SECs regulatory powers over
the operations of securities-related organizations like petitioner.

564

Eustacio Atwel, et al. vs. Conception Progressive Association, Inc.


551 SCRA 272 (2008)

FACTS:
Assemblyman Emiliano Melgazo founded and organized Concepcion
Progressive Association (CPA), an organization aimed to provide livelihood to
and generate income for his supporters. After his election as CPA president,
Emiliano Melgazo bought a parcel of land in behalf of the association. The
property was later on converted into a wet market where agricultural, livestock
and other farm products were sold. It also housed a cockpit and an area for
various forms of amusement. The income generated from the property, mostly
rentals from the wet market, was paid to CPA. When Emiliano Melgazo died, his
son, petitioner Manuel Melgazo, succeeded him as CPA president and
administrator of the property. On the other hand, petitioners Atwel and Pilpil were
elected as CPA vice-president and treasurer, respectively.
While CPA was in the process of registering as a stock corporation, its
other elected officers and members formed their own group and registered
themselves in the Securities and Exchange Commission (SEC) as officers and
members of respondent Concepcion Progressive Association, Inc. (CPAI).
Petitioners were not listed either as officers or members of CPAI. Later, CPAI
objected to petitioners' collection of rentals from the wet market vendor and filed
a case in the SEC for mandatory injunction. With the passage of RA 8799, the
case was transferred to Branch 24 of the Southern Leyte RTC and subsequently,
to Branch 8 of the Tacloban City RTC, both special commercial courts. CPAI
alleged that it was the owner of the property and petitioners, without authority,
were collecting rentals from the wet market vendors. Petitioners refuted CPAI's
claim saying that it was preposterous and impossible for the latter to have
acquired ownership over the property in 1968 when it was only in 1997 that it
was incorporated and registered with the SEC.
The special commercial court ruled that the deed of sale covering the
property was in the name of CPA, not Emiliano Melgazo. In the dispositive
portion of the decision, the court, however, considered CPA to be one and the
same as CPAI. Aggrieved, petitioners went to the CA and contested the
jurisdiction of the special commercial court over the case alleging that the case
did not involve an intra-corporate dispute "between and among members" so as
to warrant the special commercial court's jurisdiction over it. The CA found that
the special commercial court should not have tried the case since there was no
intra-corporate dispute among CPAI members or officers, nonetheless held that
petitioners were already barred from questioning the court's jurisdiction based on
the doctrine of estoppel. Petitioners filed a motion for reconsideration but it was
denied by the CA. Hence, this petition under Rule 45 of the Rules of Court.

565

ISSUE

Whether or not the controversy falls under the jurisdiction of the Securities and
Exchange Commission.

RULINGS
We agree.
Originally, Section 5 of Presidential Decree (PD) 902-A conferred on the SEC
original and exclusive jurisdiction over: (2) Controversies arising out of intracorporate, partnership, or association relations, between and among
stockholders, members, or associates; or association of which they are
stockholders, members, or associates, respectively;
Upon the enactment of RA 8799 in 2000, the jurisdiction of the SEC over
intra-corporate controversies and other cases enumerated in Section 5 of PD
902-A was transferred to the courts of general jurisdiction.
To determine whether a case involves an intra-corporate controversy to be heard
and decided by the RTC, two elements must concur:
(1) The status or relationship of the parties and
(2) The nature of the question that is subject of their controversy.
The first element requires that the controversy must arise out of intracorporate or partnership relations: (a) between any or all of the parties and the
corporation, partnership or association of which they are stockholders, members
or associates; (b) between any or all of them and the corporation, partnership or
association of which they are stockholders, members or associates and (c)
between such corporation, partnership or association and the State insofar as it
concerns their individual franchises. On the other hand, the second element
requires that the dispute among the parties be intrinsically connected with the
regulation of the corporation. If the nature of the controversy involves matters
that are purely civil in character, necessarily, the case does not involve an intracorporate controversy.
In the case at bar, these elements are not present. The records reveal that
petitioners were never officers or members of CPAI. CPAI itself admitted this in
its pleadings. In fact, petitioners were the only remaining members of CPA which,
obviously, was not the CPAI that was registered in the SEC.
Moreover, the issue in this case does not concern the regulation of CPAI
(or even CPA). The determination as to who is the true owner of the disputed
property entitled to the income generated therefrom is civil in nature and should
be threshed out in a regular court. Cases of this nature are cognizable by the
RTC under BP 129. Therefore, the conflict among the parties here was outside
the jurisdiction of the special commercial court.
Consequently, CPAI cannot be permitted to wrest from petitioners (as the
remaining CPA officers) the administration of the disputed property until after the
parties' rights are clearly adjudicated in the proper courts. It is neither fair nor
566

legal to bind a party to the result of a suit or proceeding in a court with no


jurisdiction. The decision of a tribunal not vested with the appropriate jurisdiction
is null and void.
Wherefore, the petition is granted. The assailed decision of the Court of
Appeals is reversed and set aside. Accordingly, SEC Case No. 2001-07-110 is
dismissed for lack of jurisdiction.

567

Baviera vs. Standard Chartered Bank, et al.


515 SCRA 170 (2007)

FACTS: Manuel Baviera, petitioner in these cases, was the former head of the
HR Service Delivery and Industrial Relations of Standard Chartered BankPhilippines. SCB did not comply with the conditions set forth by the BSP.
Although unregistered with the SEC, SCB was able to sell securities worth
around P6 billion to some 645 investors. Petitioner entered into an Investment
Trust Agreement with SCB wherein he purchased US$8,000.00 worth of
securities upon the banks promise of 40% return on his investment and a
guarantee that his money is safe. After six (6) months, however, petitioner
learned that the value of his investment went down to US$7,000.00. He tried to
withdraw his investment but was persuaded by Antonette de los Reyes of SCB to
hold on to it for another six (6) months in view of the possibility that the market
would pick up. The trend in the securities market, however, was bearish and the
worth of petitioners investment went down further to only US$3,000.00. On
October 26, 2001, Petitioner then filed with the BSP a letter-complaint
demanding compensation for his lost investment. But SCB denied his demand on
the ground that his investment is "regular." On July 15, 2003, petitioner filed with
the Department of Justice (DOJ), represented herein by its prosecutors, public
respondents, a complaint charging the above-named officers and members of
the SCB Board of Directors and other SCB officials, private respondents, with
syndicated estafa. For their part, private respondents filed the following as
counter-charges against petitioner: (1) blackmail and extortion and blackmail and
perjury. On September 29, 2003, petitioner also filed a complaint for perjury
against private . On February 7, 2004, petitioner also filed with the DOJ a
complaint for violation of Section 8.19 of the Securities Regulation Code against
private respondents, On February 23, 2004, the DOJ rendered its Joint
Resolution dismissing all the complaints and counter-charges filed the herein
parties. Petitioner filed with the Court of Appeals a petition for certiorari alleging
that the DOJ acted with grave abuse of discretion amounting to lack or excess of
jurisdiction in dismissing his complaint for syndicated estafa and a separate
petition for certiorari assailing the DOJ Resolution dismissing the case for
violation of the Securities Regulation Code. Petitioner claimed that the DOJ acted
with grave abuse of discretion tantamount to lack or excess of jurisdiction in
holding that the complaint should have been filed with the SEC. On January 7,
2005, the Court of Appeals promulgated its Decision dismissing the petition. It
sustained the ruling of the DOJ that the case should have been filed initially with
the SEC. Meanwhile, on February 21, 2005, the Court of Appeals rendered its
Decision involving petitioners charges and respondents counter charges
dismissing the petitions on the ground that the purpose of a petition for certiorari
is not to evaluate and weigh the parties evidence but to determine whether the
assailed Resolution of the DOJ was issued with grave abuse of discretion
tantamount to lack of jurisdiction. Petitioner moved for a reconsideration but it
was denied . Hence, the instant petitions for review on certiorari.

568

ISSUE: 1) Whether or not the Court of Appeals erred in concluding that the DOJ
did not commit grave abuse of discretion in dismissing petitioners complaint for;
2) violation of Securities Regulation Code and for syndicated estafa.
RULIN : 1) NO. The Court of Appeals held that under Section 53.1 of the said
Code provides, a criminal complaint for violation of any law or rule administered
by the SEC must first be filed with the latter. If the Commission finds that there is
probable cause, then it should refer the case to the DOJ. Since petitioner failed
to comply with the foregoing procedural requirement, the DOJ did not gravely
abuse its discretion in dismissing his complaint. Under the doctrine of primary
jurisdiction, courts will not determine a controversy involving a question within the
jurisdiction of the administrative tribunal, where the question demands the
exercise of sound administrative discretion requiring the specialized knowledge
and expertise of said administrative tribunal to determine technical and intricate
matters of fact
2) NO. Section 5, Rule 110 of the 2000 Rules of Criminal Procedure, as
amended, provides that all criminal actions, commenced by either a complaint or
an information, shall be prosecuted under the direction and control of a public
prosecutor. This mandate is founded on the theory that a crime is a breach of the
security and peace of the people at large, an outrage against the very
sovereignty of the State. It follows that a representative of the State shall direct
and control the prosecution of the offense. A public prosecutor is in a peculiar
and very definite sense a servant of the law, the twofold aim of which is that guilt
shall not escape or innocence suffers.

569

Provident International Resources Corp.. et al. vs. Joaquin T. Venus,


et al. 554 SCRA 540 (2008)
QUISUMBING, J.:
Facts: Petitioner is a corporation duly organized under Philippine law which was
registered with the SEC on September 20, 1979. Edward T. Marcelo et al.,
collectively known as the Marcelo group, were its incorporators, original
stockholders, and directors. However, the group of the respondents (Asistio
group) claimed that the Marcelo group acquired shares in PIRC as mere
trustees. The Marcelo group allegedly executed a waiver of pre-emptive right,
blank deeds of assignment, and blank deeds of transfer; endorsed in blank their
respective stock certificates over all of the outstanding capital stock registered in
their names; and completed the blank deeds in 2002 to effect transfers to the
Asistio group. Hence, the Asistio group re-registered the corporation in the year
2002. The SEC issued a letter recalling the certification it had issued on August
6, 2002 and canceling the 2002-registered STB. The Asistio group appealed to
the SEC Board of Commissioners. They claimed that the issue of which of the
two STBs is valid is intra-corporate in nature; hence, the RTC, not the SEC, has
jurisdiction.
Issue: That the SEC has the jurisdiction to recall and cancel a stock and transfer
book which it issued in 2002 because of its mistaken assumption that no stock
and transfer book had been previously issued in 1979.
Ruling: The Court agrees to the above ruling proper and within the SEC's
jurisdiction to make. Per Republic Act No. 8799 (The Securities Regulation
Code), the following are SECs powers and functions:
(a) Have jurisdiction and supervision over all corporations, partnerships or
associations who are the grantees of primary franchises and /or a license
or permit issued by the Government;
(b) Formulate policies and recommendations in issues concerning the
securities market, advise Congress and other government agencies on all
aspects of the securities market and propose legislation and amendments
thereto;
(d) Regulate, investigate or supervise the activities of persons to ensure
compliance; and

570

(n) Exercise such other powers as may be provided by law as well as


those which may be implied from, or which are necessary or incidental to
the carrying out of, the express powers granted the Commission to achieve
the objectives and purposes of these laws.
As the regulatory body, it is the SEC's duty to ensure that there is only one set of
STB for each corporation. The determination of whether or not the 1979registered STB is valid and of whether to cancel and revoke the August 6, 2002
certification and the registration of the 2002 STB on the ground that there already
is an existing STB is impliedly and necessarily within the regulatory jurisdiction of
the SEC.

571

Securities and Exchange Commission vs. Interport Resources


Corporation 567 SCRA 354 (2008)

Ponente : Justice Chico-Nazario


Facts : Under the MoA, IRC acquired 100% or the entire capital stock of Ganda
Energy Holdings, Inc. (GEHI), which would own and operate a 102 megawatt gas
turbine power-generating barge. b. Also stipulated is that GEHI would assume a
five-year power purchase contract with National Power Corp. At that time, GEHIs
power-generating barge was 97% complete and would go on-line by mid-Sept
1994. c. In exchange, IRC will issue to GHB 55% of the expanded capital stock
of IRC (amounting to 40.88 billion shares total par value of P488.44 million) d.
On the side, IRC would acquire 67% of the entire capital stock of Philippine
Racing Club, Inc. (PRCI). PRCI owns 25.724 hectares of real estate property in
Makati. e. Under the Agreement, GHB, a member of the Westmont Group of
Companies in Malaysia, shall extend or arrange a loan required to pay for the
proposed acquisition by IRC of PRCI. IRC alleged that a press release
announcing the approval of the agreement was sent through fax to Philippine
Stock Exchange (PSE) and the SEC, but that the fax machine of SEC could not
receive it. Upon the advice of SEC, IRC sent the press release on the morning of
9 Aug 1994. SEC averred that it received reports that IRC failed to make timely
public disclosures of its negotiations with GHB and that some of its directors
heavily traded IRC shares utilizing this material insider information. SEC
Chairman issued a directive requiring IRC to submit to SEC a copy of its
aforesaid MoA with GHB and further directed all principal officers of IRC to
appear at a hearing before the Brokers and Exchanges Dept (BED) of SEC to
explain IRCs as amended by PD 175. CA promulgated a Decision a. jurisdiction
was conferred upon the Prosecution and Enforcement Dept (PED) of SEC. IRC
also claimed that SEC violated their right to due process when it ordered that the
respondents appear before SEC and show cause why no administrative. Failure
to immediately disclose the information as required by the Rules on Disclosure of
Material Facts by Corporations Whose Securities are Listed in Any Stock
Exchange or Registered/Licensed Under the Securities Act. IRC sent a letter to
SEC. SEC Chairman issued an Order finding that IRC violated the Rules on
Disclosure when it failed to make timely disclosure. No formal hearings were
conducted in connection with the Motions. SEC. Determined that there were no
implementing rules and regulations regarding disclosure. which effectively
enjoined SEC from filing any criminal. or any of the provisions of the Revised
572

Securities Acts which respondents allegedly violated. c. Further decided that the
Rules of Practice and Procedure before the PED did not comply with the
statutory requirements contained in the Administrative Code of 1997. Since
under Sec 8 of PD 902-A. It found no statutory authority for SEC to initiate and
file any suit for civil liability under Sec 8. SEC issued an Omnibus Order: creating
a special investigating panel to hear and decide the case in accordance with
Rules of Practice and Procedure before the PED. shifted the burden of proof to
the respondents. insider trading. and thus. 30 and 36 of the Revised Securities
Act. Section 9. criminal or administrative proceedings may possibly be held
against the respondents without violating their rights to due process and equal
protection. in relation to Sec 36 of the Revised Securities Act. to recall the show
cause orders. Respondents filed a petition before the CA questioning the
Omnibus Orders and filed a Supplemental Motion wherein they prayed for the
issuance of a writ of preliminary injunction and to deny the Motion for
Continuance for lack of merit. IRC filed an Omnibus Motion (later an Amended
Omnibus Motion) alleging that SEC had no authority to investigate the subject
matter. They filed a Motion for Continuance of Proceedings and that some of the
officers and directors of IRC entered into transactions involving IRC shares in
violation of Sec 30. It ruled that no civil. Thus, attaching copies of MoA and its
directors appeared to explain IRCs alleged failure to immediately disclose
material information as required under the Rules on Disclosure of Material Facts.
civil or administrative case against the respondents. civil or criminal sanctions
should be imposed on them. CA granted their motion and issued a writ of
preliminary injunction.
Issue : Whether the Court of Appeals erred in when it denied petitioners motion
for leave to quash assailed SEC omnibus orders.
Ruling : The petition is impressed with merit.
The provision explains in simple terms that the insider's misuse of
nonpublic and undisclosed information is the gravamen of illegal conduct. The
intent of the law is the protection of investors against fraud, committed when an
insider, using secret information, takes advantage of an uninformed investor.
Insiders are obligated to disclose material information to the other party or
abstain from trading the shares of his corporation. This duty to disclose or
abstain is based on two factors: first, the existence of a relationship giving
access, directly or indirectly, to information intended to be available only for a
corporate purpose and not for the personal benefit of anyone; and second, the
573

inherent unfairness involved when a party takes advantage of such information


knowing it is unavailable to those with whom he is dealing. Respondents further
aver that under Section 30 of the Revised Securities Act, the SEC still needed to
define the following terms: material fact, reasonable person, nature and
reliability and generally available. In determining whether or not these terms are
vague, these terms must be evaluated in the context of Section 30 of the
Revised Securties Act. To fully understand how the terms were used in the
aforementioned provision, a discussion of what the law recognizes as afact of
special significance is required, since the duty to disclose such fact or to abstain
from any transaction is imposed on the insider only in connection with a fact of
special significance.

Under the law, what is required to be disclosed is a fact of special


significance which may be (a) a material fact which would be likely, on being
made generally available, to affect the market price of a security to a significant
extent, or (b) one which a reasonable person would consider especially important
in determining his course of action with regard to the shares of stock.

574

Tysons Super Concrete, Inc., v. Court of Appeals 461 SCRA 69 (2005)

Facts:
Romana Dela Cruz is the registered owner of several parcels of land located at
P. Dela Cruz St., Sta. Quiteria, Caloocan City and covered by Transfer
Certificates of Title Nos. T-176211, T-176206, T-176207, T-176208, T-176209
and T-176210, all of the Registry of Deeds of Caloocan City. Sometime in
October 1992, Dela Cruz entered into a contract of lease with Tysons Super
Concrete, Inc. (Tysons for brevity) where it was agreed that the latter shall
occupy the property as lessee for a period of twenty (20) years beginning
January 1, 1993 until December 31, 2012.3 Under the contract, the lease
payments were graduated and spread over the entire twenty-year period with an
initial monthly rental of P36,444.00 per month for the first year to a maximum
of P151,529.00 a month for the last year. Tysons introduced various permanent
improvements over the property to be turned over to Dela Cruz after the lapse of
the twenty-year period of lease. Sometime in March 1995, the two major blocs of
stockholders of Tysons comprising of Elsa and Francis Chua, on one hand, and
Nancy, William, Genaro and Lydia, all surnamed Hao, on the other, due to
internal squabbling, filed a joint motion with the Securities and Exchange
Commission (SEC) praying for the appointment of a receiver to oversee the
functions of the corporation. On April 11, 1995, the SEC issued an order creating
a Management Committee (Committee, for brevity) to undertake the
management of Tysons, to take custody of and control over all the existing
assets, funds and records of the corporation, and to determine the best way to
protect the interest of the stockholders and creditors. the SEC appointed the
following as members of the Committee: Francis Chua, as the representative of
their bloc, and Genaro Hao, also as the representative of their group. The
accounting firm of Punong Bayan and Araullo was appointed as Chairman of the
Committee. A complaint for ejectment was filed by Dela Cruz against Tysons
with the Metropolitan Trial Court of Caloocan City for the alleged failure of
Tysons to pay its rentals despite repeated written demands for such payment .
The METC ordered to vacate the leased premises located at N.P. dela Cruz
Compound, P. dela Cruz Street, Sta. Quiteria, Caloocan City and to surrender
possession peacefully to the plaintiff. Dela Cruz filed a Motion for Immediate
Execution of the MeTC judgment. Tysons, on the other hand, filed a motion
praying for the stay of execution of the MeTC decision contending that the MeTC
did not acquire jurisdiction over the defendant corporation on the ground that said
corporation. Tysons then filed with the Regional Trial Court (RTC) of Caloocan
City a petition for certiorari and prohibition with application for the issuance of a
writ of preliminary injunction and temporary restraining order seeking to stop the
judgment of the MeTC. Tysons elevated the case to the CA via a special civil
action for certiorari. Court of Appeals ordered the decision of RTC as null and
void.

575

Issue:
Whether or not the Court of Appeals erred in deciding that the service summons
are valid.
Ruling:
The resolution of whether or not Francis Chua employed extrinsic fraud to
deprive petitioners of their day in court entails determination of factual issues
which is beyond the province of this Court. The resolution of factual issues is the
function of trial courts whose findings on these matters are received with respect
and are, as a rule, binding on this Court unless it is shown that they are grounded
on speculations, surmises or conjectures. well-settled is the rule that factual
matters cannot be inquired into by this Court in an appeal bycertiorari.32 This
Court, at this stage, is limited to reviewing errors of law that may have been
committed by the lower courts. . Petitioners line of reasoning is flawed. The
management committee created by the SEC is composed of the accounting firm
of Punong Bayan and Araullo represented by petitioner Gregorio S. Navarro as
the chairman, with Nancy Hao and Francis Chua as members. Hence, even if we
are to follow petitioners premise that the Committee is the only body authorized
to receive summons, we still find no basis to conclude that only its chairman is
authorized to receive summons. Like the chairman of the Committee, its
members are also authorized to receive summons since they are also considered
"responsible officers" as contemplated by the Rules of Court in effect at the time
the ejectment case against Tysons was filed. In the present case, since it is not
disputed that Francis Chua is a member of the management committee, he is
therefore authorized to receive summons for and in behalf of Tysons. The
Supreme Court denied the petition.

576

Spouses Sobrejuanite v. ASB Development Corporation 471 SCRA


763 (2005)
Ponente: Ynares-Santiago, J:

FACTS:
Sps Sobrejuanite entered into a contract to sell with ASB Development
Corporation over a condominium unit and a parking space but ASB failed to
deliver said property despite full payment of the spouses. Consequently, herein
petitioners filed a complaint for refund of payment and rescission of contract
before Housing and Land Use Regulatory Board (HLURB). ASB, moved for its
dismissal on the ground that SEC approve its rehabilitation plan. However,
HLURB denied the motion and continued with the proceedings finding ASB liable
for the delivery of said property hence rescission of the contract plus damages
was proper. ASB filed an appeal before the Office of the President but was
denied. Nevertheless, on appeal before CA, the appellate court favored ASB
stating that SEC approval of the rehabilitation plan suspended the HLURB
Proceedings. Hence, this petition.

ISSUE:

Whether the approval of Corporate Rehabilitation plan suspended the HLURB


proceedings.

RULING:

The Supreme Court (SC) decided in favor of ASB. SC explained that the purpose
for the suspension of the proceedings is to give enough breathing space for the
corporation to make the business viable again and to put all creditors on equal
footing. It added that HLURB should have suspended the proceedings to allow
ASB to devote its time and effort to the rehabilitation and restructuring of the
distressed corporation, to extend the period of delivery on account of causes
beyond its control, such as financial reverses.
Petition denied.

577

Sy Chim vs. Sy Siy Ho & Sons, Inc. 480 SCRA 465 (2006)
FACTS:
The Sy Siy Ho & Sons, Inc. (hereinafter referred to as the corporation) is a
domestic corporation which was doing business under the name and style Guan
Yiac Hardware. The corporation was owned and controlled by Sy Chim and his
children. In 1993 a complaint for accounting and damages against the spouses
Sy Chim was filed by the corporation alleging that Felicidad Chan Sy, as
custodian of all cash collections, had been depositing amounts less than those
appearing in the financial statements which are in the defendants' custody and
that no deposits were made in the corporation's account from November 1, 2002
to January 31, 2003. Based on the accountant's report, Felicidad Chan Sy failed
to account for P67,117,230.30. Defendants averred, that any unaccounted cash
account and irregularities in the management of the corporation, if any, were the
full responsibility of stockholder Sy Tiong Shiou since he has direct and actual
management of the corporation under the by-laws. Sy Chim, as corporate
president, was a mere figurehead, who only had general supervision over the
corporation's officers. Juanita Tan Sy, as corporate treasurer, had custody of the
corporation's funds and should have kept a complete and accurate record of
receipts, disbursements, and other commercial transactions of the corporation.
Felicidad Chan Sy merely performed clerical work and acted as Corporate
Treasurer only in the absence of Juanita Tan Sy and under the latter's close
supervision. Later on, defendants filed a "Motion for the Appointment of a
Management Committee . They allege that since the plaintiff itself has alleged
that there has been a massive dissipation and loss of its corporate assets and
funds, and this Court is still in the process of determining whether the General
Manager, Sy Tiong Shiou, and Treasurer, Juanita Tan, are the parties
responsible for such dissipation and loss, the control and management of the
Corporation must be transferred to an independent party to ensure the
preservation of the corporate assets. until this Honorable Court resolves with
finality that Sy Tiong Shiou and his wife, Juanita Tan, are not responsible for the
dissipation and loss. That while Sy Tiong Shiou and Juanita Tan remain in
control of the management of the corporation, there is imminent danger of further
dissipation, loss, wastage or destruction of the corporate funds and assets. Thus,
there exists an urgent need for the immediate appointment of a management
committee to administer, manage and preserve the assets, funds, properties and
records of Sy Siy Ho & Sons, Inc. in order to prevent any further dissipation,
wastage and loss. The RTC granted said motion for the creation of management
committee however the CA annulled it. Thus this petition
ISSUE
Whether the creation of a management committee in this case is proper

RULING
No. Section 1, Rule 9 of the Interim Rules provides:
578

SECTION 1. Creation of a management committee. - As an incident to any of the


cases filed under these Rules or the Interim Rules on Corporate Rehabilitation, a
party may apply for the appointment of a management committee for the
corporation, partnership or association, when there is imminent danger of:
(1) Dissipation, loss, wastage or destruction of assets or other properties;
andcralawlibrary
(2) Paralyzation of its business operations which may be prejudicial to the
interest of the minority stockholders, parties-litigants or the general public
The rationale for the need to establish the confluence of the two (2) requisites
under Section 1, Rule 9 by an applicant for the appointment of a management
committee is primarily based upon the fact that such committee and receiver
appointed by the court will immediately take over the management of the
corporation, partnership or association, including such power as it may deem
appropriate, and any of the powers specified in Section 5 of the Rule. Indeed,
upon the appointment of a receiver, the duly elected/appointed officers of the
corporation are divested of the management of such corporation in favour of the
management committee/receiver. Such transference of the corporation's
management will certainly have a negative, if not crippling effect, on the
operations/affairs of the corporation not only with banks and other business
institutions including those abroad which it deals business with. A wall of
uncertainty is erected; the short and long-term plans of the management of the
corporation are disrupted, if not derailed
In the present case, petitioners failed to make a strong showing that there was an
imminent danger of dissipation, loss, wastage or destruction of assets or other
properties of respondent corporation and paralysis of its business operations
which may be prejudicial to the interest of the parties-litigants, petitioners, or the
general public. Petitioners failed to adduce a shred of evidence during the
hearing of their motion to prove their claim that there was imminent danger of
dissipation, loss, wastage or destruction of the assets or other properties of
respondent ever since Sy Tiong Shiou became president and Juanita Tan Sy
continued discharging her duties as corporate treasurer; nor is there proof that
there was imminent danger of paralyzing the business operations of the
corporation. While the allegation that Sy Tiong Shiou and Juanita Tan Sy abused
their positions and mismanaged the affairs of respondent corporation is a distinct
possibility, petitioners failed to adduce proof thereon. Mere possibility without
proof of abusing corporate positions and dissipation of assets and properties of
the corporation is not a valid ground for the appointment of a management
committee/receiver.

579

Ao-As vs. CA 491 SCRA 339 (2006)


FACTS:
The Lutheran Church in the Philippines (LCP) is a religious organization
registered with the Securities and Exchange Commission in 1967. Members are
composed of clergy men and the local congregation in the Philippines which was
then divided into three districts- North Luzon District (NLD), South Luzon District
(SLD), and Mindanao District (MDD). Board of directors is composed of 2
representatives per district and a President who is elected at-large. By 1976, in
LCP national convention,,a resolution was passed to divide NLD into two
districts; the NLD Highland district (NLHD) and the NL Lowland District (NLLD).
Again in 1984, the Visayas Island District (VID) was created. The creation of 3
new districts increased the number of directors to 11. The creation of new
districts is provided in the by-laws as when LCP sees the creation needed.
There was no challenge from the members regarding the creation of the
new district, and the addition of representatives to the Board. However, when
LCP business manager and Treasurer Eclesio Hipe was terminated in his office
by t he Board, controversies arose. A number of cases were filed with the SEC,
among which is the LCP vs Hipe which sought to recover from Hipe the
corporate records still in his possession. Batong group, an internal faction in LCP
involved in the controversy of this instant case, were the duly elected board of
directors while another faction, Ao-as group, served in various capacities as
directors or officers of the LCP.
In 1990, the Ao-as group filed with SEC for the accounting and damages
with prayer for preliminary injunction and appointment of a management
committee. Their allegations were the non-accounting of Batong on a La Trinidad
land transaction, the non-liquidated/non-accounting of cash advances made by
Batong, a 4.8M unaccounted LCP general fund, non-registration of a Leyte land
purchased with LCP funds, severance of the relationship with Lutheran ChurchMissouri synod- LCPs benefactor, and the transfer of the corporate books from
Sta. Mesa office to Caloocan office. SEC, in its decision, declared the election of
the new Board of Trustees, the Batong group, is null and void, and created a
management committee until new Board of Directors are duly elected under the
Management Committees facilitation.
CA reversed the decision of SEC and favored the Batong group; hence,
this petition.

ISSUE

WON the creation of a management committee, as ordered by SEC, is correct.

RULING
580

No, the creation of the management committee to handle the affairs of the
Lutheran Church is not the proper remedy.
All of the grounds relied upon by [the Ao-As group] pertain to past
delinquencies for which there are other available remedies such as accounting
and reconveyance. The [Ao-As group] did not allege, much less prove, any
present or imminent loss or destruction of LCP properties and assets. At best, it
expresses merely a general apprehension for possible mismanagement by
respondent on the basis of the aforementioned past transactions.
It must be stressed that the appointment of a management committee
inevitably results in the drastic summary removal of all directors and officers of
LCP. Clearly, the appointment of a management committee is not justified due to
the failure of only two (2) of the LCP Board members to liquidate past cash
advances and other transactions involving corporate property and funds. Where
the corporation is solvent, a receiver will not be appointed because of past
misconduct and a subsequent mere apprehension of a future misdoing, where
the present situation and the prospects for the future are not such as to warrant a
receivership.

581

New Frontier Sugar Corporation vs. RTC of Ilo-ilo and Equitable PCI
Bank 513 SCRA 601 (2007)

Austria-Martinez, J.

FACTS OF THE CASE

New Frontier Sugar Corporation (NFSC) is a domestic corporation


engaged in the business of raw sugar milling. It filed a petition for declaration of
suspension of payments with approval of proposed rehabilitation plan.

The RTC issued a stay order, and Manuel B. Clemente was appointed as
rehabilitation receiver. NFSC was also ordered to put up a bond.
Equitable PCI Bank as creditor filed a Comment/Opposition to the petition
with a Motion to exclude property. It alleged that NFSC is not qualified for
corporate rehabilitation as it cannot operate anymore. It has no assets left. It
claims that the financial statements of the company were misleading and
inaccurate because its properties have already been foreclosed and transferred
to EPCIB before the rehabilitation was filed. The properties were foreclosed
before the filing of the petition for rehabilitation.
The company still owes EPCIB deficiency liability.
The case was dismissed by the RTC. However the foreclosure of the
properties were questioned in another proceedings.

ISSUES

Whether the RTC is correct in its dismissal of the corporate rehabilitation


proceedings of the NFSB.

RULING

Petition is denied for lack of merit.

582

Rehabilitation contemplates a continuance of corporate life and activities in


an effort to restore and reinstate the corporation to its former position of
successful operation and solvency. Presently, the applicable law on rehabilitation
petitions

filed

by

corporations,

partnerships

or

associations,

including

rehabilitation cases transferred from the Securities and Exchange Commission to


the RTCs pursuant to Republic Act No. 8799 or the Securities Regulation Code,
is the Interim Rules of Procedure on Corporate Rehabilitation (2000).
The fact that there is a pending case for the annulment of the foreclosure
proceedings and auction sales is of no moment. Until a court of competent
jurisdiction, which in this case is the RTC of Dumangas, Iloilo, Branch 68, annuls
the foreclosure sale of the properties involved, petitioner is bereft of a valid title
over the properties. In fact, it is the trial courts ministerial duty to grant a
possessory writ over the properties. Consequently, the CA was correct in
upholding the RTCs dismissal of the petition for rehabilitation in view of the fact
that the titles to petitioners properties have already passed on to respondent
bank and petitioner has no more assets to speak of, specially since petitioner
does not dispute the fact that the properties which were foreclosed by
respondent bank comprise the bulk, if not the entirety, of its assets.

A.M. No. 00-8-10-SC promulgated by the Court on September 4, 2001 provides


that a petition for rehabilitation is considered a special proceeding given that it
seeks to establish the status of a party or a particular fact. Accordingly, the
period of appeal provided in paragraph 19 (b) of the Interim Rules Relative to the
Implementation of Batas Pambansa Blg. 129 for special proceedings shall apply.
Under said paragraph 19 (b), the period of appeal shall be thirty (30) days, a
record of appeal being required. However, it should be noted that the Court
issued A.M. No. 04-9-07-SC on September 14, 2004, clarifying the proper mode
of appeal in cases involving corporate rehabilitation and intra-corporate
controversies. It is provided therein that all decisions and final orders in cases
falling under the Interim Rules of Corporate Rehabilitation and the Interim Rules
of Procedure Governing Intra-Corporate Controversies under Republic Act No.
8799 shall be appealed to the CA through a petition for review under Rule 43 of
the Rules of Court to be filed within fifteen (15) days from notice of the decision
or final order of the RTC.

583

Philippine Airlines vs. Zamora 514 SCRA 584 (2007)

Facts:

On 1 February 2005, the Court of Appeals promulgated an Amended Decision


modifying its 13 August 2004 Decision but at the same time resolving petitioner
PAL's Motion for Reconsideration in this wise: WHEREFORE, this Court's August
13, 2004 decision is hereby AMENDED, the dispositive portion to read as
follows:
WHEREFORE, in view of the foregoing, the petition is GRANTED. The
NLRC resolution dated April 27, 2001 is MODIFIED. Considering that
petitioner is a detention prisoner making reinstatement impossible, PAL is
hereby ordered to pay petitioner Zamora his separation pay, in lieu of
reinstatement, to be computed at one month salary for every year of
service from February 9, 1981 and backwages to be computed from
December 19, 1995, both up to October 1, 2000, the date of his
incarceration.
Considering that PAL is still under receivership, the monetary claims of petitioner
Zamora must be presented to the PAL Rehabilitation Receiver, subject to the
rules on preference of credits. The Court of Appeals took into account
respondent Zamora's incarceration when it recalled its order of reinstatement.
Anent its earlier pronouncement against the suspension of the proceedings of the
case owing to the present rehabilitation of petitioner PAL, the appellate court only
had this to say:
However, since PAL is still under receivership, the provisions of PD 902-A,
should apply. The enforcement of the monetary claims of petitioner should be
brought before the PAL Rehabilitation Receiver for proper disposition.

Issue:
Whether the respondent Zamoras monetary claim should be presented to the
PAL rehabilitation receiver, subject to the rules on preference of credits.

RULING:
No. The relevant law dealing with the suspension of actions for claims against
corporations is Presidential Decree No. 902-A, 52 as amended. The term "claim,"
as contemplated in Sec. 6 (c) of Presidential Decree No. 902-A, refers "to debts
or demands of a pecuniary nature. It means 'the assertion of a right to have
money paid.
It is plain from the foregoing provisions of law that "upon the appointment [by the
SEC] of a management committee or a rehabilitation receiver," all actions for
584

claims against the corporation pending before any court, tribunal or board shall
ipso jure be suspended
The law is clear: upon the creation of a management committee or the
appointment of a rehabilitation receiver, all claims for actions "shall be
suspended accordingly." No exception in favor of labor claims is mentioned in the
law. Since the law makes no distinction or exemptions, neither should this Court.
Otherwise stated, no other action may be taken in, including the rendition of
judgment during the state of suspension what are automatically stayed or
suspended are the proceedings of an action or suit and not just the payment of
claims during the execution stage after the case had become final and executory.
The suspension of action for claims against a corporation under rehabilitation
receiver or management committee embraces all phases of the suit, be it before
the trial court or any tribunal or before this Court. Furthermore, the actions that
are suspended cover all claims against a distressed corporation whether for
damages founded on a breach of contract of carriage, labor cases, collection
suits or any other claims of a pecuniary nature. As to the appellate court's
amended directive that "the monetary claims of petitioner Zamora must be
presented to the PAL Rehabilitation Receiver, subject to the rules on preference
of credits," the same is erroneous for there has been no declaration of
bankruptcy or judicial liquidation. Thus, the rules on preference of credits do not
apply.

585

Union Bank of the Phil. vs. ABS Devt. Corp. G.R. No. 172895 560
SCRA 578 (2008)
Ponente: CHICO NAZARIO, J.

Facts:

ASB Development Corporation (ASBDC), a domestic corporation entered


into an agreement with RCBC, PNB, UCPB and Union Bank of the Philippines
(UBP) in which the former executed a Mortgage Trust Indenture (MTI) in favor of
the creditor banks in consideration for the loan extended to them. However,
SBDC filed before Securities and Exchange Commission (SEC) Securities and
Clearing Department (SICD) a Petition for Rehabilitation with prayer for the
Suspension of Actions and Payments Proceeding. UBP opposed to said petition
but the SEC hearing panel granted such petition as well as the suspension of
payment and created a management committee or appointment of a
rehabilitation receiver. Thus, all claims against ASBDC were suspended. Later,
Petition of Rehabilitation was approved and consequently, rehabilitation plan of
the ASB Group of Companies was approved. The UBP and PNB assailed the
Order suspending the payments before the SEC hearing panel, but then it was
dismissed. Thus, they elevated it to SC under Petition for Certiorari. During the
pendency, UBP filed with RTC of Mandaluyong a Notice of Extra judicial Sale
of Properties, the subject of the mortgage property by ASBDC.

UBP, after

foreclosure, secured a Certificate of Sale. Upon knowledge of the ASBC of the


sale, they immediately filed a Motion praying for the nullification of the Extra
judicial sale of its property made by UBP. This was granted by the SEC Hearing
Panel. The UBP then filed a Petition for Review on Certiorari on the annulment
of said sale but it was denied for lack of merit. UBP elevated the case before CA
arguing that ASBDC should not have filed a petition for Rehabilitation because it
admitted already that it possessed sufficient properties to cover its obligation but
foresaw its inability to pay its obligations within a period of one year.

CA

dismissed the petition. Hence, this petition before SC.

Issue:

Whether the Court of Appeals erred when it applied the Rules of


Procedure on corporate Recovery
586

Whether Petition for Rehabilitation with Prayer for Suspension of Actions


before SEC suffers from fatal and jurisdictional defects.

Ruling:

1. No, CA did not err in applying the Rules of Procedure on Corporate


Recovery.
A Corporation has two (2) distinct remedies whenever it suffers financially
distressed corporation: (1) suspension of payments; and (2) rehabilitation
proceedings.

A debtor or petitioning corporation may have a sufficient assets to cover its


liabilities, it also alleged that it had foreseen its inability to pay its obligations
within a period of one year

2. No, it does not.


The UBP petitioner was wrong when it asserted that respondent ASBDC was
technically insolvent.

A Petition for Rehabilitation must comply with the jurisdictional


requirements under Rule IV of the Rules of Procedure on Corporate
Recovery.Section 4-1 of the said Rules provides that any of the following:
(1) an actually insolvent debtor;
(b) a technically insolvent debtor; or
(3) a creditor or stockholder of the debtor, can file a petition for
rehabilitation.

ASBDC Although admitted in its Petition that it had sufficient assets to cover its
liabilities, it also alleged that it had foreseen its inability to pay its obligations
within a period of one year. This is the very definition of technical
insolvency: the inability

of

the

petitioning

corporation

to

pay,

although

temporarily, for a period longer than one year from the filing of the petition.
Hence, it is a very definition of technical insolvency.

587

Malayan Insurance Company, Inc. vs. Victorias Milling Company, Inc.


586 SCRA 45 (2009)
FACTS:
On July 8, 1997, the SEC issued a Stay Order, suspending all actions for claims
against respondent pending before any court, tribunal, office, board, body or
commission. On August 8, 1997, the SEC constituted a Management Committee.
On May 31, 1999, the Labor Arbiter rendered a decision in Abelido v. Victorias
Milling, ordering respondent to pay Abelido the sum of P6, 605,275.24. On July
16, 1999, respondent procured from the petitioner a surety bond as a requisite to
the filing of an appeal with the NLRC from the Labor Arbiters decision. On
September 7, 2000, the NLRC affirmed the decision of the Labor Arbiter, and a
writ of execution was issued on April 4, 2001.
The Executive Labor Arbiter issued three orders (dated April 10, 2001,
April 17, 2001, and May 7, 2001, respectively) directing the petitioner to turn over
to the NLRC the amount of P6.605,275.24, on pain of contempt. On May 11,
2001, petitioner served a demand upon BPI for the release of the bank deposits
that respondent had assigned in its favor, but BPI refused. On May 16, 2001,
respondent advised petitioner that the enforcement of the writ of execution was
premature and without legal basis. The following day, petitioner replied that the
NLRC was bent on enforcing the writ, and sought from the respondent a copy of
a TRO, if any, issued by the Court of Appeals. On May 18, 2001, petitioner
released the amount to the NLRC.
Failing to obtain reimbursement from the respondent despite a series of
demands, petitioner, on January 15, 2003, filed a complaint for sum of money
with the RTC. On January 27, 2003, SEC issued an order appointing a
rehabilitation receiver for respondent. On July 2, 2003, the RTC suspended the
proceedings against respondent, and subsequently denied the petitioners motion
for reconsideration.
Petitioner then went to the CA on a petition for certiorari which the CA
dismissed on May 21, 2004, concurring with the RTC that the SEC Stay Order
covered petitioners claim. On April 11, 2005, the CA denied the petitioners
motion for reconsideration.
Meanwhile, on June 5, 2003, the CA resolved the petition
for certiorari filed by the respondent assailing the NLRC decision. The appellate
court, while affirming the NLRC decision, set aside the latters resolution on the
respondents motion for reconsideration, and remanded the case to the NLRC for
suspension of the proceedings, ruling that the NLRC decision cannot be enforced
while [the respondent] is under a management committee.
Petitioner now comes to us, insisting that since its claim (for
reimbursement of the amount it released to NLRC to satisfy the judgment on the
labor claims of Abelido) arose after the respondent was placed under a
management committee, such claim should not be suspended nor covered by
the SEC Stay Order.
588

ISSUE

Whether the petitioners claim is covered by the SEC Stay Order.

RULING

Yes. We have consistently held in Rubberworld (Phils.) Inc. v. NLRC that


the suspension of proceedings referred to in Section 6 (c) of Presidential Decree
No. 902-A uniformly applies to all actions for claims filed against a corporation,
partnership or association under management or receivership, without distinction.
The suspension of action for claims against a corporation under rehabilitation
receiver or management committee embraces all phases of the suit, be it before
the trial court or any tribunal or before this Court. Otherwise stated, what are
automatically stayed or suspended are the proceedings of an action or suit and
not just the payment of claims. Furthermore, the actions that is suspended cover
all claims against a distressed corporation whether for damages founded on a
breach of contract of carriage, labor cases, collection suits or any other claims of
a pecuniary nature.
The indiscriminate suspension of actions for claims is intended to expedite
the rehabilitation of the distressed corporation. As this Court held
in Rubberworld, the automatic stay of actions is designed to enable the
management committee or the rehabilitation receiver to effectively exercise
its/his powers free from any judicial or extrajudicial interference that might unduly
hinder or prevent the rescue of the debtor company. To allow such other actions
to continue would only add to the burden of the management committee or
rehabilitation receiver, whose time, effort and resources would be wasted in
defending claims against the corporation instead of being directed toward its
restructuring and rehabilitation.
Given these premises, it is not difficult to understand why actions for
claims against the ailing enterprise have to be suspended. It then becomes easy
to accept the hypothesis that the date when the claim arose, or when the action
is filed, is of no moment. As long as the corporation is under a management
committee or a rehabilitation receiver, all actions for claims against it --- for
money or otherwise --- must yield to the greater imperative of corporate
rehabilitation, excepting only, as already mentioned, claims for payment of
obligations incurred by the corporation in the ordinary course of business.
Enforcement of writs of execution issued by judicial or quasi-judicial tribunals,
since such writs emanate from actions for claims, must, likewise, be
suspended.
If we allow the reimbursement action to proceed, and if petitioners claim is
granted, it would be in a position to assert a preference over other creditors.
Worse, respondent would be compelled to dispose of its properties in order to
satisfy the claim of petitioner. It would in effect be a clear defiance of the
proscription set forth in the Interim Rules on selling, encumbering, transferring,
589

or disposing in any manner any of its (respondents) properties except in the


ordinary course of business. Certainly, petitioners claim for reimbursement did
not arise from the usual operations of respondents business. Neither can we
consider it as an ordinary expense for the conduct of its operations.

590

Union Bank of the Philippines vs. Securities and Exchange


Commission 358 SCRA 479 (2001)
PANGANIBAN, J.:

FACTS:

Union Bank sought the opinion of SEC as to the applicability and coverage of the
Full Material Disclosure Rule on banks, contending that said rules, in effect,
amend Section 5 (a) (3) of the Revised Securities Act which exempts securities
issued or guaranteed by banking institutions from the registration requirement.
Because its securities are exempt from the registration requirements under
Section 5(a)(3) of the Revised Securities Act, petitioner argues that it is not
covered by RSA Implementing Rulels:

Rule 11(a)-1, which requires the filing of annual, quarterly, current


predecessor and successor reports;

Rule 34(a)-1, which mandates the filing of proxy statements and forms of
proxy;

Rule 34(c)-1, which obligates the submission of information statements.

According to SEC, while the requirements of registration do not apply to


securities of banks, banks with a class of securities listed for trading on the
Philippine Stock Exchange, Inc. are covered by certain Revised Securities Act
Rules governing the filing of various reports.

Union bank was fined for failure for failure to file SEC Form 11-A. CA affirmed the
decision of SEC.

ISSUE:

Whether the RSA Implementing Rules 11(a)-1, 34(a)-1 and 34(c)-1 applies to
Union Bank
HELD:

YES.

Sec 5. Exempt Securities. (a) Except as expressly provided, the requirement of


registration under subsection (a) of Section four of this Act shall not apply to any
591

of the following classes of securities:

xxxxxxxxx

(3) Any security issued or guaranteed by any banking institution authorized to do


business in the Philippines, the business of which is substantially confined to
banking, or a financial institution licensed to engage in quasi-banking, and is
supervised by the Central Bank.

This provision exempts from registration the securities issued by banking or


financial institutions mentioned in the law. Nowhere does it state or even imply
that petitioner, as a listed corporation, is exempt from complying with the reports
required by the assailed RSA Implementing Rules. Worth repeating is the CAs
disquisition on the matter, which we quote:

However, the exemption from the registration requirement enjoyed by petitioner


does not necessarily connote that [it is] exempted from the other reportorial
requirements. Having confined the exemption enjoyed by petitioner merely to the
initial requirement of registration of securities for public offering, and not [to] the
subsequent filing of various periodic reports, respondent Commission, as the
regulatory agency, is able to exercise its power of supervision and control over
corporations and over the securities market as a whole. Otherwise, the objectives
of the `Full Material Disclosure policy would be defeated since petitioner
corporation and its dealings would be totally beyond the reach of respondent
Commission and the investing public.[9]

It must be emphasized that petitioner is a commercial banking corporation listed


in the stock exchange. Thus, it must adhere not only to banking and other allied
special laws, but also to the rules promulgated by Respondent SEC, the
government entity tasked not only with the enforcement of the Revised Securities
Act,[11] but also with the supervision of all corporations, partnerships or
associations which are grantees of government-issued primary franchises and/or
licenses or permits to operate in the Philippines.

RSA Rules 11(a)-1, 34(a)-1 and 34(c)-1 require the submission of certain reports
to ensure full, fair and accurate disclosure of information for the protection of the
investing public. These Rules were issued by respondent pursuant to the
authority conferred upon it by Section 3 of the RSA.[13]
592

The said Rules do not amend Section 5(a)(3) of the Revised Securities Act,
because they do not revoke or amend the exemption from registration of the
securities enumerated thereunder. They are reasonable regulations imposed
upon petitioner as a banking corporation trading its securities in the stock market.

That petitioner is under the supervision of the Bangko Sentral ng Pilipinas (BSP)
and the Philippine Stock Exchange (PSE) does not exempt it from complying with
the continuing disclosure requirements embodied in the assailed Rules.
Petitioner, as a bank, is primarily subject to the control of the BSP; and as a
corporation trading its securities in the stock market, it is under the supervision of
the SEC. It must be pointed out that even the PSE is under the control and
supervision of respondent.[14] There is no over-supervision here. Each
regulating authority operates within the sphere of its powers. That stringent
requirements are imposed is understandable, considering the paramount
importance given to the interests of the investing public.

Otherwise stated, the mere fact that in regard to its banking functions, petitioner
is already subject to the supervision of the BSP does not exempt the former from
reasonable disclosure regulations issued by the SEC. These regulations are
meant to assure full, fair and accurate disclosure of information for the protection
of investors in the stock market. Imposing such regulations is a function within
the jurisdiction of the SEC. Since petitioner opted to trade its shares in the
exchange, then it must abide by the reasonable rules imposed by the SEC.

593

Fabia vs. Court of Appeals, 388 SCRA 574 (2002)


Ponente: Justice Bellosillo
Facts:
Petitioner Hernani N. Fabia was the President of private respondent MTCP, a
domestic corporation engaged in providing maritime courses and seminars to
prospective overseas contract workers and seamen. He was likewise a Director
and stockholder thereof. MTCP through its new President Exequiel B. Tamayo
filed an affidavit-complaint for estafa against Hernani N. Fabia alleging that on
various occasions, Fabia drew cash advances from MTCP, covered by cash
vouchers, amounting to P1,291,376.61 which he failed to liquidate despite
repeated demands. Petitioner Fabia moved to dismiss the case. He admitted
having received the various amounts covered by the cash vouchers but reasoned
that they were in the nature of simple loans that had already been liquidated and
paid as shown by the receipts and vouchers which he had attached to his
pleadings. The Office of the City Prosecutor of Manila dismissed the complaint
for lack of jurisdiction because the controversy pertained to is under the
Securities and Exchange Commission (SEC). MTCP moved to reconsider the
resolution but the same was denied. MTCP filed a petition for review before the
Department of Justice (DOJ) questioning the two (2) resolutions issued by the
Office of the City Prosecutor. The petition was however dismissed by the DOJ as
it found no reversible error committed by the Office of the City Prosecutor. The
motion for reconsideration subsequently filed by the MTCP was likewise denied
by the DOJ. MTCP filed a petition for certiorari before the Court of Appeals
raising as sole issue whether the defense of lack of accounting precludes a
finding of probable cause, with prayer that the DOJ Resolutions be annulled
which was granted by CA.
Issue/s: Whether the CA erred in annulling the DOJ decision on the estafa case
Ruling:
The Supreme Court found the petition meritous. It held that Section 6, PD 902-A
confines the jurisdiction of the SEC to "intra-corporate disputes" defined as any
act or omission of the Board of Directors/Trustees of corporations, or of
partnerships, or of other associations, or of their stockholders, officers, or
partners, including any fraudulent devices, schemes or representations, in
violation of any law or rules and regulations administered and enforced by the
Commission. It further ruled that the charge against petitioner is for estafa, an
594

offense punishable under The Revised Penal Code (RPC), and prosecution for
the offense is presently before the regular courts. However, as correctly pointed
out by private respondent MTCP, jurisdiction is determined not from the law upon
which the cause of action is based, nor the type of proceedings initiated, but
rather, it is gleaned from the allegations stated in the complaint. It is evident from
the complaint that the acts charged are in the nature of an intra-corporate dispute
as they involve fraud committed by virtue of the office assumed by petitioner as
President, Director, and stockholder in MTCP, and committed against the MTCP
corporation. This sufficiently removes the action from the jurisdiction of the
regular courts, and transposes it into an intra-corporate controversy within the
jurisdiction of the SEC. The fact that a complaint for estafa, a felony punishable
under the RPC, has been filed against petitioner does not negate and nullify the
intra-corporate nature of the cause of action, nor does it transform the
controversy from intra-corporate to a criminal one.
Accordingly, as the matter involves an intra-corporate dispute within the
jurisdiction of the SEC, the issue of whether prior non-accounting precludes a
finding of probable cause for the charge of estafa no longer finds relevance.

595

Power Homes Unlimited Corporation v. SEC 546 SCRA 567 (2008)


PUNO, C.J.:

FACTS:
Petitioner is a domestic corporation duly registered with public respondent
SEC engaged in the transaction of promoting, acquiring, managing, leasing,
obtaining options on, development, and improvement of real estate properties for
subdivision and allied purposes, and in the purchase, sale and/or exchange of
said subdivision and properties through network marketing.

On October 27, 2000, respondent Noel Manero requested public respondent


SEC to investigate petitioners business. He claimed that he attended a seminar
conducted by petitioner where the latter claimed to sell properties that were
inexistent and without any brokers license.

On November 21, 2000, one Romulo E. Munsayac, Jr. inquired from public
respondent SEC whether petitioners business involves legitimate network
marketing.

On the bases of the letters of respondent Manero and Munsayac, public


respondent SEC held a conference on December 13, 2000 that was attended by
petitioners incorporators John Lim, Paul Nicolas and Leonito Nicolas.

After SECs examination and investigation of the nature of the


plaintiffs business, it issued a Cease and Desist Order.

On February 5, 2001, petitioner moved for the lifting of the CDO, which
public respondent SEC denied for lack of merit on February 22, 2001.

596

Aggrieved, petitioner went to the Court of Appeals which denied the


petition for certiorari and prohibition filed by petitioner Powerhomes Unlimited
Corporation for lack of merit but granted the temporary restraining order.

The Court of Appeals denied petitioners motion for reconsideration; hence,


this petition for review.
ISSUES:
(1) Whether public respondent SEC followed due process in the issuance
of the assailed CDO; and
(2) Whether petitioners business constitutes an investment contract which
should be registered with public respondent SEC before its sale or offer for
sale or distribution to the public.

RULING:

On the first issue, Sec. 64 of R.A. No. 8799 provides:


Sec. 64. Cease and Desist Order. 64.1. The Commission, after
proper investigation or verification, motu proprio or upon verified
complaint by any aggrieved party, may issue a cease and desist
order without the necessity of a prior hearing if in its judgment the
act or practice, unless restrained, will operate as a fraud on investors
or is otherwise likely to cause grave or irreparable injury or prejudice
to the investing public.

We hold that petitioner was not denied due process. The records reveal
that public respondent SEC properly examined petitioners business operations
when it (1) called into conference three of petitioners incorporators, (2) requested
information from the incorporators regarding the nature of petitioners business
operations, (3) asked them to submit documents pertinent thereto, and (4) visited
petitioners business premises and gathered information thereat. All these were
done before the CDO was issued by the public respondent SEC.

597

The second issue is resolved pursuant to Section 8.1 of R.A. No. 8799 which
states that:

Section8.Requirement

of

Registration

of

Securities. 8.1. Securities shall not be sold or offered for sale or


distribution within the Philippines, without a registration statement
duly filed with and approved by the Commission. Prior to such sale,
information on the securities, in such form and with such substance
as the Commission may prescribe, shall be made available to each
prospective purchaser.

The Court ruled that the business operation or the scheme of petitioner
constitutes an investment contract that is a security under R.A. No. 8799. Thus, it
must be registered with public respondent SEC before its sale or offer for sale or
distribution to the public. As petitioner failed to register the same, its offering to
the public was rightfully enjoined by public respondent SEC. The CDO was
proper even without a finding of fraud.

An investment contract is defined in the Amended Implementing Rules


and Regulations of R.A. No. 8799 as a contract, transaction or scheme
(collectively contract) whereby a person invests his money in a common
enterprise and is led to expect profits primarily from the efforts of others.
Thus, to be a security subject to regulation by the SEC, an investment
contract in our jurisdiction must be proved to be (known as HOWEY TEST):
(1) an investment of money,
(2) in a common enterprise,
(3) with expectation of profits,
(4) primarily from efforts of others.

598

Timeshare Realty Corporation v. Cesar Lao and Cynthia V. Cortez


544 SCRA 254 (2008)
AUSTRIA-MARTINEZ, J.:

Facts: Herein petitioner sold to Ceasar M. Lao and Cynthia V. Cortez


(respondents), one timeshare of Laguna de Boracay for US$7,500.00.
Sometime in February 1998, the SEC issued a resolution to the effect that
petitioner was without authority to sell securities, like timeshares, prior to
February 11, 1998. . It also held that the 30 days within which a purchaser may
exercise the option to unilaterally rescind the purchase agreement and receive
the refund of money paid applies to all purchase agreements entered into by
petitioner prior to the effectivity of the Registration Statement.
Petitioner sought a reconsideration of the aforesaid order but the SEC denied the
same.
Despite respondents repeated demands to cancel their contract, petitioner failed
and refused to refund or pay respondents.
Respondents directly filed with SEC En Banc6 a Complaint7 against petitioner
and the Members of its Board of Directors for violation of Section 4 of Batas
Pambansa Bilang (B.P. Blg.) 178. An Answer to the Complaint was expunged
from the records due to tardiness.
SEC En Banc rendered a Decision in favor of respondents.
Petitioner received a copy SEC En Banc Order on July 4, 2002. On July 10,
2002, petitioner sought from the CA an extension of 30 days, counted from July
19, 2002, or until August 19, 2002. The CA partially granted the motion giving
petitioner an extension of 15 days counted from July 10 or until JULY 25.
Petitioner filed a Petition for Review with the CA on August 19, 2002.
The CA dismissed the petition for review. The motion for reconsideration was
also denied.
Hence, this present petition.
Issues: a) Is the CA correct in dismissing the petition?
599

b) Does the eventual approval or issuance of license has retroactive effect


and therefore ratifies all earlier transactions?
Ruling: a) Section 4 of Rule 43 is restrictive in its treatment of the period within
which a petition may be filed:
Section 4. Period of appeal. - The appeal shall be taken within fifteen (15)
days from notice of the award, judgment, final order or resolution, or from
the date of its last publication, if publication is required by law for its
effectivity, or of the denial of petitioners motion for new trial or
reconsideration duly filed in accordance with the governing law of the court
or agency a quo. Only one (1) motion for reconsideration shall be
allowed. Upon proper motion and the payment of the full amount of
the docket fee before the expiration of the reglementary period, the
Court of Appeals may grant an additional period of fifteen (15) days
only within which to file the petition for review. No further extension
shall be granted except for the most compelling reason and in no
case to exceed fifteen (15) days.(Emphasis supplied.)
xxxx.
Its motion for extension being inherently flawed, petitioner should not have
presumed that the CA would fully grant the same. Instead, it should have
exercised due diligence by filing the proper petition within the allowable period, or
at the very least, ascertaining from the CA whether its motion for extension had
been acted upon. As it were, petitioners counsel left the country, unmindful of
the possibility that his clients period to appeal was about to lapse - as it indeed
lapsed on July 25, 1999, after the CA allowed them a 15-day extension only, in
view of the restriction under Section 4, Rule 43. Thus, petitioner has only itself to
blame that the Petition for Review it filed on August 19, 1999 was late by 25
days. The CA cannot be faulted for dismissing it.
b) As cited by the SEC En Banc in its March 25, 2002 Decision, as early as
February 13, 1998, the SEC, through Director Linda A. Daoang, already
rendered a ruling on the effectivity of the registration statement of petitioner,viz:
This has reference to your registration statement which was rendered effective
11 February 1998. The 30 days within which a purchaser may exercise the
option to unilaterally rescind the purchase agreement and receive the refund of
money paid, applies to all purchase agreements entered into by the
600

registrantprior to the effectivity of the registration statement. The 30-day


rescission period for contracts signed before the Registration Statement
was rendered effective shall commence on 11 February 1998. The
rescission period for contracts after 11 February 1998 shall commence on
the date of purchase agreement. (Emphasis supplied.)
xxxx.
Finally, the provisions of B.P. Blg. 178 do not support the contention of petitioner
that its mere registration as a corporation already authorizes it to deal with
unregistered timeshares. Corporate registration is just one of several
requirements before it may deal with timeshares:
Section 8. Procedure for registration. - (a) All securities required to be
registered under subsection (a) of Section four of this Act shall be
registered through the filing by the issuer or by any dealer or underwriter
interested in the sale thereof, in the office of the Commission, of a sworn
registration statement with respect to such securities, containing or having
attached thereto, the following:
xxxx
(36) Unless previously filed and registered with the Commission and
brought up to date:
(a) A copy of its articles of incorporation with all amendments thereof and
its existing by-laws or instruments corresponding thereto, whatever the
name, if the issuer be a corporation.
Prior to fulfillment of all the other requirements of Section 8, petitioner
is absolutely proscribed under Section 4 from dealing with unregistered
timeshares, thus:
Section 4. Requirement of registration of securities. - (a) No securities,
except of a class exempt under any of the provisions of Section five hereof
or unless sold in any transaction exempt under any of the provisions of
Section six hereof, shall be sold or offered for sale or distribution to the
public within the Philippinesunless such securities shall have been
registered and permitted to be sold as hereinafter provided.(Emphasis
supplied.)
601

602

Vous aimerez peut-être aussi